You are on page 1of 161

San Beda College of Law – Alabang

Constitutional Law 2 Case Digests

INTRODUCTION TO CONSTITUTIONAL LAW 2


THE NATURE OF THE CONSTITUTION AND ITS RELATION WITH THE COURTS

FRANCISCO VS. HOUSE OF REPRESENTATIVES


[415 SCRA 44; G.R. No. 160261; 10 Nov 2003]

Facts:

Impeachment proceedings were filed against Supreme Court Chief Justice Hilario Davide. The justiciable
controversy poised in front of the Court was the constitutionality of the subsequent filing of a second complaint to
controvert the rules of impeachment provided for by law.

Issue:

Whether or Not the filing of the second impeachment complaint against Chief Justice Hilario G. Davide, Jr. with
the House of Representatives falls within the one year bar provided in the Constitution and whether the resolution
thereof is a political question – has resulted in a political crisis.

Held:

In any event, it is with the absolute certainty that our Constitution is sufficient to address all the issues which this
controversy spawns that this Court unequivocally pronounces, at the first instance, that the feared resort to extra-
constitutional methods of resolving it is neither necessary nor legally permissible. Both its resolution and
protection of the public interest lie in adherence to, not departure from, the Constitution.
In passing over the complex issues arising from the controversy, this Court is ever mindful of the essential truth
that the inviolate doctrine of separation of powers among the legislative, executive or judicial branches of
government by no means prescribes for absolute autonomy in the discharge by each of that part of the
governmental power assigned to it by the sovereign people.

At the same time, the corollary doctrine of checks and balances which has been carefully calibrated by the
Constitution to temper the official acts of each of these three branches must be given effect without destroying
their indispensable co-equality. There exists no constitutional basis for the contention that the exercise of judicial
review over impeachment proceedings would upset the system of checks and balances. Verily, the Constitution is
to be interpreted as a whole and "one section is not to be allowed to defeat another." Both are integral
components of the calibrated system of independence and interdependence that insures that no branch of
government act beyond the powers assigned to it by the Constitution.

When suing as a citizen, the interest of the petitioner assailing the constitutionality of a statute must be direct and
personal. He must be able to show, not only that the law or any government act is invalid, but also that he
sustained or is in imminent danger of sustaining some direct injury as a result of its enforcement, and not merely
that he suffers thereby in some indefinite way. It must appear that the person complaining has been or is about to
be denied some right or privilege to which he is lawfully entitled or that he is about to be subjected to some
burdens or penalties by reason of the statute or act complained of. In fine, when the proceeding involves the
assertion of a public right, the mere fact that he is a citizen satisfies the requirement of personal interest.

In the case of a taxpayer, he is allowed to sue where there is a claim that public funds are illegally disbursed, or
that public money is being deflected to any improper purpose, or that there is a wastage of public funds through
the enforcement of an invalid or unconstitutional law. Before he can invoke the power of judicial review, however,
he must specifically prove that he has sufficient interest in preventing the illegal expenditure of money raised by
taxation and that he would sustain a direct injury as a result of the enforcement of the questioned statute or
contract. It is not sufficient that he has merely a general interest common to all members of the public.

At all events, courts are vested with discretion as to whether or not a taxpayer's suit should be entertained. This
Court opts to grant standing to most of the petitioners, given their allegation that any impending transmittal to the
Senate of the Articles of Impeachment and the ensuing trial of the Chief Justice will necessarily involve the
expenditure of public funds.

Page 1
Section 1-C, SY ’06-‘07
San Beda College of Law – Alabang
Constitutional Law 2 Case Digests

As for a legislator, he is allowed to sue to question the validity of any official action which he claims infringes his
prerogatives as a legislator. Indeed, a member of the House of Representatives has standing to maintain inviolate
the prerogatives, powers and privileges vested by the Constitution in his office. 83

The framers of the Constitution also understood initiation in its ordinary meaning. Thus when a proposal reached
the floor proposing that "A vote of at least one-third of all the Members of the House shall be necessary… to
initiate impeachment proceedings," this was met by a proposal to delete the line on the ground that the vote of the
House does not initiate impeachment proceeding but rather the filing of a complaint does.

To the argument that only the House of Representatives as a body can initiate impeachment proceedings
because Section 3 (1) says "The House of Representatives shall have the exclusive power to initiate all cases of
impeachment," This is a misreading of said provision and is contrary to the principle of reddendo singula singulis
by equating "impeachment cases" with "impeachment proceeding."

Having concluded that the initiation takes place by the act of filing and referral or endorsement of the
impeachment complaint to the House Committee on Justice or, by the filing by at least one-third of the members
of the House of Representatives with the Secretary General of the House, the meaning of Section 3 (5) of Article
XI becomes clear. Once an impeachment complaint has been initiated, another impeachment complaint may not
be filed against the same official within a one year period.

The Court in the present petitions subjected to judicial scrutiny and resolved on the merits only the main issue of
whether the impeachment proceedings initiated against the Chief Justice transgressed the constitutionally
imposed one-year time bar rule. Beyond this, it did not go about assuming jurisdiction where it had none, nor
indiscriminately turn justiciable issues out of decidedly political questions. Because it is not at all the business of
this Court to assert judicial dominance over the other two great branches of the government.

No one is above the law or the Constitution. This is a basic precept in any legal system which recognizes equality
of all men before the law as essential to the law's moral authority and that of its agents to secure respect for and
obedience to its commands. Perhaps, there is no other government branch or instrumentality that is most zealous
in protecting that principle of legal equality other than the Supreme Court which has discerned its real meaning
and ramifications through its application to numerous cases especially of the high-profile kind in the annals of
jurisprudence. The Chief Justice is not above the law and neither is any other member of this Court. But just
because he is the Chief Justice does not imply that he gets to have less in law than anybody else. The law is
solicitous of every individual's rights irrespective of his station in life.

Thus, the Rules of Procedure in Impeachment Proceedings which were approved by the House of
Representatives on November 28, 2001 are unconstitutional. Consequently, the second impeachment complaint
against Chief Justice Hilario G. Davide, Jr is barred under paragraph 5, section 3 of Article XI of the Constitution.

M ANILA PRINCE HOTEL VS. GSIS


[267 SCRA 408; G.R. No. 122156; 3 Feb 1997]

Facts:

The controversy arose when respondent Government Service Insurance System (GSIS), pursuant to the
privatization program of the Philippine Government under Proclamation No. 50 dated 8 December 1986, decided
to sell through public bidding 30% to 51% of the issued and outstanding shares of respondent Manila Hotel
Corporation. In a close bidding held on 18 September 1995 only two (2) bidders participated: petitioner Manila
Prince Hotel Corporation, a Filipino corporation, which offered to buy 51% of the MHC or 15,300,000 shares at
P41.58 per share, and Renong Berhad, a Malaysian firm, with ITT-Sheraton as its hotel operator, which bid for
the same number of shares at P44.00 per share, or P2.42 more than the bid of petitioner.

Pending the declaration of Renong Berhad as the winning bidder/strategic partner and the execution of the
necessary contracts, matched the bid price of P44.00 per share tendered by Renong Berhad.

On 17 October 1995, perhaps apprehensive that respondent GSIS has disregarded the tender of the matching bid
and that the sale of 51% of the MHC may be hastened by respondent GSIS and consummated with Renong
Berhad, petitioner came to this Court on prohibition and mandamus.

In the main, petitioner invokes Sec. 10, second par., Art. XII, of the 1987 Constitution and submits that the Manila
Hotel has been identified with the Filipino nation and has practically become a historical monument which reflects
the vibrancy of Philippine heritage and culture. It is a proud legacy of an earlier generation of Filipinos who
believed in the nobility and sacredness of independence and its power and capacity to release the full potential of
the Filipino people. To all intents and purposes, it has become a part of the national patrimony. 6 Petitioner also
argues that since 51% of the shares of the MHC carries with it the ownership of the business of the hotel which is

Page 2
Section 1-C, SY ’06-‘07
San Beda College of Law – Alabang
Constitutional Law 2 Case Digests

owned by respondent GSIS, a government-owned and controlled corporation, the hotel business of respondent
GSIS being a part of the tourism industry is unquestionably a part of the national economy.

Issue:

Whether or Not the sale of Manila Hotel to Renong Berhad is violative of the Constitutional provision of Filipino
First policy and is therefore null and void.

Held:

The Manila Hotel or, for that matter, 51% of the MHC, is not just any commodity to be sold to the highest bidder
solely for the sake of privatization. The Manila Hotel has played and continues to play a significant role as an
authentic repository of twentieth century Philippine history and culture. This is the plain and simple meaning of the
Filipino First Policy provision of the Philippine Constitution. And this Court, heeding the clarion call of the
Constitution and accepting the duty of being the elderly watchman of the nation, will continue to respect and
protect the sanctity of the Constitution. It was thus ordered that GSIS accepts the matching bid of petitioner
MANILA PRINCE HOTEL CORPORATION to purchase the subject 51% of the shares of the Manila Hotel
Corporation at P44.00 per share and thereafter to execute the necessary clearances and to do such other acts
and deeds as may be necessary for purpose.

PEOPLE VS. POMAR


[46 Phil 126; G.R. No. L-22008; 3 Nov 1924]

Facts:

Macaria Fajardo was an employee of La Flor de la Isabela, a Tobacco factory. She was granted a vacation leave,
by reason of her pregnancy, which commenced on the 16th of July 1923. According to Fajardo, during that time,
she was not given the salary due her in violation of the provisions of Act No. 3071. Fajardo filed a criminal
complaint based on Section 13 and 15 of said Act against the manager of the tobacco Factory, Julio Pomar,
herein defendant. The latter, on the other hand, claims that the facts in the complaint did not constitute an offense
and further alleges that the aforementioned provisions of Act No. 3071 was unconstitutional. Section 13, Act No.
3071 provides that, “Every person, firm or corporation owning or managing a factory, shop or place of labor of any
description shall be obliged to grant to any woman employed by it as laborer who may be pregnant, thirty days
vacation with pay before and another thirty days after confinement: Provided, That the employer shall not
discharge such laborer without just cause, under the penalty of being required to pay to her wages equivalent to
the total of two months counting from the day of her discharge.” Section 15 of the same Act provides for the
penalty of any violation of section 13. The latter was enacted by the legislature in the exercise of its supposed
Police Power with the purpose of safeguarding the health of pregnant women laborers in "factory, shop or place of
labor of any description," and of insuring to them, to a certain extent, reasonable support for one month before
and one month after their delivery. The trial court rendered a decision in favor of plaintiff, sentencing the
defendant to pay the fine of fifty pesos and in case of insolvency, to suffer subsidiary imprisonment. Hence, the
case was raised to the Court of Appeals which affirmed the former decision.

Issue:

Whether or not Section 13 of Act No. 3071 is unconstitutional.

Whether or not the promulgation of the questioned provision was a valid exercise of Police Power.

Held:

The Supreme Court declared Section 13 of Act No. 3071 to be unconstitutional for being violative or restrictive of
the right of the people to freely enter into contracts for their affairs. It has been decided several times, that the
right to contract about one's affairs is a part of the liberty of the individual, protected by the "due process of law"
clause of the constitution. The contracting parties may establish any agreements, terms, and conditions they may
deem advisable, provided they are not contrary to law, morals or public policy

The police power of the state is a very broad and expanding power. The police power may encompass every law
for the restraint and punishment of crimes, for the preservation of the public peace, health, and morals. But that
power cannot grow faster than the fundamental law of the state, nor transcend or violate the express inhibition of
the constitution. The Police Power is subject to and is controlled by the paramount authority of the constitution of
the state, and will not be permitted to violate rights secured or guaranteed by the latter.

LAMBINO VS. COMELEC


[G.R. No. 174153; 25 Oct 2006]

Page 3
Section 1-C, SY ’06-‘07
San Beda College of Law – Alabang
Constitutional Law 2 Case Digests

Facts:

Petitioners (Lambino group) commenced gathering signatures for an initiative petition to change the 1987
constitution, they filed a petition with the COMELEC to hold a plebiscite that will ratify their initiative petition under
RA 6735. Lambino group alleged that the petition had the support of 6M individuals fulfilling what was provided
by art 17 of the constitution. Their petition changes the 1987 constitution by modifying sections 1-7 of Art 6 and
sections 1-4 of Art 7 and by adding Art 18. the proposed changes will shift the present bicameral- presidential
form of government to unicameral- parliamentary. COMELEC denied the petition due to lack of enabling law
governing initiative petitions and invoked the Santiago Vs. Comelec ruling that RA 6735 is inadequate to
implement the initiative petitions.

Issue:

Whether or Not the Lambino Group’s initiative petition complies with Section 2, Article XVII of the Constitution on
amendments to the Constitution through a people’s initiative.

Whether or Not this Court should revisit its ruling in Santiago declaring RA 6735 “incomplete, inadequate or
wanting in essential terms and conditions” to implement the initiative clause on proposals to amend the
Constitution.

Whether or Not the COMELEC committed grave abuse of discretion in denying due course to the Lambino
Group’s petition.

Held:

According to the SC the Lambino group failed to comply with the basic requirements for conducting a people’s
initiative. The Court held that the COMELEC did not grave abuse of discretion on dismissing the Lambino petition.

1. The Initiative Petition Does Not Comply with Section 2, Article XVII of the Constitution on Direct Proposal by
the People
The petitioners failed to show the court that the initiative signer must be informed at the time of the signing of
the nature and effect, failure to do so is “deceptive and misleading” which renders the initiative void.

2. The Initiative Violates Section 2, Article XVII of the Constitution Disallowing Revision through Initiatives
The framers of the constitution intended a clear distinction between “amendment” and “revision, it is intended
that the third mode of stated in sec 2 art 17 of the constitution may propose only amendments to the
constitution. Merging of the legislative and the executive is a radical change, therefore a constitutes a
revision.

3. A Revisit of Santiago v. COMELEC is Not Necessary


Even assuming that RA 6735 is valid, it will not change the result because the present petition violated Sec 2
Art 17 to be a valid initiative, must first comply with the constitution before complying with RA 6735

Petition is dismissed.

SANTIAGO VS. COMELEC


[270 SCRA 106; G.R. No.127325; 19 Mar 1997]

Facts:

Private respondent Atty. Jesus Delfin, president of People’s Initiative for Reforms, Modernization and Action
(PIRMA), filed with COMELEC a petition to amend the constitution to lift the term limits of elective officials,
through People’s Initiative. He based this petition on Article XVII, Sec. 2 of the 1987 Constitution, which provides
for the right of the people to exercise the power to directly propose amendments to the Constitution. Subsequently
the COMELEC issued an order directing the publication of the petition and of the notice of hearing and thereafter
set the case for hearing. At the hearing, Senator Roco, the IBP, Demokrasya-Ipagtanggol ang Konstitusyon,
Public Interest Law Center, and Laban ng Demokratikong Pilipino appeared as intervenors-oppositors. Senator
Roco filed a motion to dismiss the Delfin petition on the ground that one which is cognizable by the COMELEC.
The petitioners herein Senator Santiago, Alexander Padilla, and Isabel Ongpin filed this civil action for prohibition
under Rule 65 of the Rules of Court against COMELEC and the Delfin petition rising the several arguments, such
as the following: (1) The constitutional provision on people’s initiative to amend the constitution can only be
implemented by law to be passed by Congress. No such law has been passed; (2) The people’s initiative is
limited to amendments to the Constitution, not to revision thereof. Lifting of the term limits constitutes a revision,
therefore it is outside the power of people’s initiative. The Supreme Court granted the Motions for Intervention.

Page 4
Section 1-C, SY ’06-‘07
San Beda College of Law – Alabang
Constitutional Law 2 Case Digests

Issue:

Whether or not Sec. 2, Art. XVII of the 1987 Constitution is a self-executing provision.

Whether or not COMELEC Resolution No. 2300 regarding the conduct of initiative on amendments to the
Constitution is valid, considering the absence in the law of specific provisions on the conduct of such initiative.

Whether the lifting of term limits of elective officials would constitute a revision or an amendment of the
Constitution.

Held:

Sec. 2, Art XVII of the Constitution is not self executory, thus, without implementing legislation the same cannot
operate. Although the Constitution has recognized or granted the right, the people cannot exercise it if Congress
does not provide for its implementation.

The portion of COMELEC Resolution No. 2300 which prescribes rules and regulations on the conduct of initiative
on amendments to the Constitution, is void. It has been an established rule that what has been delegated, cannot
be delegated (potestas delegata non delegari potest). The delegation of the power to the COMELEC being
invalid, the latter cannot validly promulgate rules and regulations to implement the exercise of the right to people’s
initiative.

The lifting of the term limits was held to be that of a revision, as it would affect other provisions of the Constitution
such as the synchronization of elections, the constitutional guarantee of equal access to opportunities for public
service, and prohibiting political dynasties. A revision cannot be done by initiative. However, considering the
Court’s decision in the above Issue, the issue of whether or not the petition is a revision or amendment has
become academic.

GONZALES VS. COMELEC


[21 SCRA 774; G.R. No. L-28196; 9 Nov 1967]

Facts:

The case is an original action for prohibition, with preliminary injunction.

The main facts are not disputed. On March 16, 1967, the Senate and the House of Representatives passed the
following resolutions:

1. R. B. H. (Resolution of Both Houses) No. 1, proposing that Section 5, Article VI, of the Constitution of the
Philippines, be amended so as to increase the membership of the House of Representatives from a maximum of
120, as provided in the present Constitution, to a maximum of 180, to be apportioned among the several
provinces as nearly as may be according to the number of their respective inhabitants, although each province
shall have, at least, one (1) member;

2. R. B. H. No. 2, calling a convention to propose amendments to said Constitution, the convention to be


composed of two (2) elective delegates from each representative district, to be "elected in the general elections to
be held on the second Tuesday of November, 1971;" and

3. R. B. H. No. 3, proposing that Section 16, Article VI, of the same Constitution, be amended so as to authorize
Senators and members of the House of Representatives to become delegates to the aforementioned
constitutional convention, without forfeiting their respective seats in Congress.

Subsequently, Congress passed a bill, which, upon approval by the President, on June 17, 1967, became
Republic Act No. 4913, providing that the amendments to the Constitution proposed in the aforementioned
Resolutions No. 1 and 3 be submitted, for approval by the people, at the general elections which shall be held on
November 14, 1967.

Issue:

Whether or Not a Resolution of Congress, acting as a constituent assembly, violates the Constitution.

Held:

Inasmuch as there are less than eight (8) votes in favor of declaring Republic Act 4913 and R. B. H. Nos. 1 and 3
unconstitutional and invalid, the petitions in these two (2) cases must be, as they are hereby, dismiss and the
writs therein prayed for denied, without special pronouncement as to costs. It is so ordered.

Page 5
Section 1-C, SY ’06-‘07
San Beda College of Law – Alabang
Constitutional Law 2 Case Digests

As a consequence, the title of a de facto officer cannot be assailed collaterally. It may not be contested except
directly, by quo warranto proceedings. Neither may the validity of his acts be questioned upon the ground that he
is merely a de facto officer. And the reasons are obvious: (1) it would be an indirect inquiry into the title to the
office; and (2) the acts of a de facto officer, if within the competence of his office, are valid, insofar as the public is
concerned.

"The judicial department is the only constitutional organ which can be called upon to determine the proper
allocation of powers between the several departments and among the integral or constituent units thereof."

Article XV of the Constitution provides:

. . . The Congress in joint session assembled, by a vote of three-fourths of all the Members of the
Senate and of the House of Representatives voting separately, may propose amendments to this
Constitution or call a contention for that purpose. Such amendments shall be valid as part of this
Constitution when approved by a majority of the votes cast at an election at which the
amendments are submitted to the people for their ratification.

From our viewpoint, the provisions of Article XV of the Constitution are satisfied so long as the electorate knows
that R. B. H. No. 3 permits Congressmen to retain their seats as legislators, even if they should run for and
assume the functions of delegates to the Convention.

SANIDAD VS. COMELEC


[78 SCRA 333; G.R. No. 90878; 29 Jan 1990]

Facts:

This is a petition for certiorari assailing the constitutionality of Section 19 of Comelec Resolution No. 2167 on the
ground that it violates the constitutional guarantees of the freedom of expression and of the press. On October 23,
1989, Republic Act No. 6766, entitled "AN ACT PROVIDING FOR AN ORGANIC ACT FOR THE CORDILLERA
AUTONOMOUS REGION" was enacted into law. Pursuant to said law, the City of Baguio and the Cordilleras
which consist of the provinces of Benguet, Mountain Province, Ifugao, Abra and Kalinga-Apayao, all comprising
the Cordillera Autonomous Region, shall take part in a plebiscite for the ratification of said Organic Act originally
scheduled last December 27, 1989 which was, however, reset to January 30, 1990 by virtue of Comelec
Resolution No. 2226 dated December 27, 1989. The Commission on Elections, by virtue of the power vested by
the 1987 Constitution, the Omnibus Election Code (BP 881), said R.A. 6766 and other pertinent election laws,
promulgated Resolution No. 2167, to govern the conduct of the plebiscite on the said Organic Act for the
Cordillera Autonomous Region. In a petition dated November 20, 1989, herein petitioner Pablito V. Sanidad, who
claims to be a newspaper columnist of the "OVERVIEW" for the BAGUIO MIDLAND COURIER, a weekly
newspaper circulated in the City of Baguio and the Cordilleras, assailed the constitutionality of Section 19 of
Comelec Resolution No. 2167, which provides:

Section 19. Prohibition on columnists, commentators or announcers. — During the plebiscite


campaign period, on the day before and on the plebiscite day, no mass media columnist,
commentator, announcer or personality shall use his column or radio or television time to
campaign for or against the plebiscite Issue.

It is alleged by petitioner that said provision is void and unconstitutional because it violates the
constitutional guarantees of the freedom of expression and of the press enshrined in the Constitution.
Unlike a regular news reporter or news correspondent who merely reports the news, petitioner maintains
that as a columnist, his column obviously and necessarily contains and reflects his opinions, views and
beliefs on any issue or subject about which he writes. Petitioner likewise maintains that if media
practitioners were to express their views, beliefs and opinions on the issue submitted to a plebiscite, it
would in fact help in the government drive and desire to disseminate information, and hear, as well as
ventilate, all sides of the issue.

Issue:

Whether or not Section 19 of Comelec Resolution No. 2167 is unconstitutional.

Page 6
Section 1-C, SY ’06-‘07
San Beda College of Law – Alabang
Constitutional Law 2 Case Digests

Held:

The Supreme Court ruled that Section 19 of Comelec Resolution No. 2167 is unconstitutional. It is clear from Art.
IX-C of the 1987 Constitution that what was granted to the Comelec was the power to supervise and regulate the
use and enjoyment of franchises, permits or other grants issued for the operation of transportation or other public
utilities, media of communication or information to the end that equal opportunity, time and space, and the right to
reply, including reasonable, equal rates therefor, for public information campaigns and forums among candidates
are ensured. The evil sought to be prevented by this provision is the possibility that a franchise holder may favor
or give any undue advantage to a candidate in terms of advertising space or radio or television time. This is also
the reason why a "columnist, commentator, announcer or personality, who is a candidate for any elective office is
required to take a leave of absence from his work during the campaign period (2nd par. Section 11(b) R.A. 6646).
It cannot be gainsaid that a columnist or commentator who is also a candidate would be more exposed to the
voters to the prejudice of other candidates unless required to take a leave of absence.

However, neither Article IX-C of the Constitution nor Section 11 (b), 2nd par. of R.A. 6646 can be construed to
mean that the Comelec has also been granted the right to supervise and regulate the exercise by media
practitioners themselves of their right to expression during plebiscite periods. Media practitioners exercising their
freedom of expression during plebiscite periods are neither the franchise holders nor the candidates. In fact, there
are no candidates involved in a plebiscite. Therefore, Section 19 of Comelec Resolution No. 2167 has no
statutory basis.

Plebiscite Issue are matters of public concern and importance. The people's right to be informed and to be able to
freely and intelligently make a decision would be better served by access to an unabridged discussion of the
Issue, including the forum. The people affected by the Issue presented in a plebiscite should not be unduly
burdened by restrictions on the forum where the right to expression may be exercised. Comelec spaces and
Comelec radio time may provide a forum for expression but they do not guarantee full dissemination of
information to the public concerned because they are limited to either specific portions in newspapers or to
specific radio or television times.

The instant petition is GRANTED. Section 19 of Comelec Resolution No. 2167 is declared null and void and
unconstitutional.

BONDOC VS. PINEDA


[201 SCRA 792; G.R. No. 97710; 26 Sep 1991]

Facts:

In the elections held on May 11, 1987, Marciano Pineda of the LDP and Emigdio Bondoc of the NP were
candidates for the position of Representative for the Fourth District of Pampanga. Pineda was proclaimed winner.
Bondoc filed a protest in the House of Representatives Electoral Tribunal (HRET), which is composed of 9
members, 3 of whom are Justices of the SC and the remaining 6 are members of the House of Representatives
(5 members belong to the LDP and 1 member is from the NP). Thereafter, a decision had been reached in which
Bondoc won over Pineda. Congressman Camasura of the LDP voted with the SC Justices and Congressman
Cerilles of the NP to proclaim Bondoc the winner of the contest.

On the eve of the promulgation of the Bondoc decision, Congressman Camasura


received a letter informing him that he was already expelled from the LDP for allegedly helping to organize the
Partido Pilipino of Eduardo Cojuangco and for allegedly inviting LDP members in Davao Del Sur to join said
political party. On the day of the promulgation of the decision, the Chairman of HRET received a letter informing
the Tribunal that on the basis of the letter from the LDP, the House of Representatives decided to withdraw the
nomination and rescind the election of Congressman Camasura to the HRET.

Issue:

Whether or not the House of Representatives, at the request of the dominant political party therein, may change
that party’s representation in the HRET to thwart the promulgation of a decision freely reached by the tribunal in
an election contest pending therein

Held:

The purpose of the constitutional convention creating the Electoral Commission was to provide an independent
and impartial tribunal for the determination of contests to legislative office, devoid of partisan consideration.

As judges, the members of the tribunal must be non-partisan. They must discharge their functions with complete
detachment, impartiality and independence even independence from the political party to which they belong.

Page 7
Section 1-C, SY ’06-‘07
San Beda College of Law – Alabang
Constitutional Law 2 Case Digests

Hence, disloyalty to party and breach of party discipline are not valid grounds for the expulsion of a member of the
tribunal. In expelling Congressman Camasura from the HRET for having cast a “conscience vote” in favor of
Bondoc, based strictly on the result of the examination and appreciation of the ballots and the recount of the votes
by the tribunal, the House of Representatives committed a grave abuse of discretion, an injustice and a violation
of the Constitution. Its resolution of expulsion against Congressman Camasura is, therefore, null and void.

Another reason for the nullity of the expulsion resolution of the House of Representatives is that it violates
Congressman Camasura’s right to security of tenure. Members of the HRET, as sole judge of congressional
election contests, are entitled to security of tenure just as members of the Judiciary enjoy security of tenure under
the Constitution. Therefore, membership in the HRET may not be terminated except for a just cause, such as, the
expiration of the member’s congressional term of office, his death, permanent disability, resignation from the
political party he represents in the tribunal, formal affiliation with another political party or removal for other valid
cause. A member may not be expelled by the House of Representatives for party disloyalty, short of proof that he
has formally affiliated with another

MIRASOL VS CA
[351 SCRA 44; G.R. No. 128448; 1 Feb 2001]

Facts:

The Mirasols are sugarland owners and planters. Philippine National Bank (PNB) financed the Mirasols' sugar
production venture FROM 1973-1975 under a crop loan financing scheme. The Mirasols signed Credit
Agreements, a Chattel Mortgage on Standing Crops, and a Real Estate Mortgage in favor of PNB. The Chattel
Mortgage empowered PNB to negotiate and sell the latter's sugar and to apply the proceeds to the payment of
their obligations to it.

President Marcos issued PD 579 in November, 1974 authorizing Philippine Exchange Co., Inc. (PHILEX) to
purchase sugar allocated for export and authorized PNB to finance PHILEX's purchases. The decree directed that
whatever profit PHILEX might realize was to be remitted to the government. Believing that the proceeds were
more than enough to pay their obligations, petitioners asked PNB for an accounting of the proceeds which it
ignored. Petitioners continued to avail of other loans from PNB and to make unfunded withdrawals from their
accounts with said bank. PNB asked petitioners to settle their due and demandable accounts. As a result,
petitioners, conveyed to PNB real properties by way of dacion en pago still leaving an unpaid amount. PNB
proceeded to extrajudicially foreclose the mortgaged properties. PNB still had a deficiency claim.

Petitioners continued to ask PNB to account for the proceeds, insisting that said proceeds, if properly liquidated,
could offset their outstanding obligations. PNB remained adamant in its stance that under P.D. No. 579, there was
nothing to account since under said law, all earnings from the export sales of sugar pertained to the National
Government.

On August 9, 1979, the Mirasols filed a suit for accounting, specific performance, and damages against PNB.

Issue:

Whether or not the Trial Court has jurisdiction to declare a statute unconstitutional without notice to the Solicitor
General where the parties have agreed to submit such issue for the resolution of the Trial Court.

Whether PD 579 and subsequent issuances thereof are unconstitutional.

Whether or not said PD is subject to judicial review.


Held:

It is settled that Regional Trial Courts have the authority and jurisdiction to consider the constitutionality of a
statute, presidential decree, or executive order. The Constitution vests the power of judicial review or the power to
declare a law, treaty, international or executive agreement, presidential decree, order, instruction, ordinance, or
regulation not only in this Court, but in all Regional Trial Courts.

The purpose of the mandatory notice in Rule 64, Section 3 is to enable the Solicitor General to decide whether or
not his intervention in the action assailing the validity of a law or treaty is necessary. To deny the Solicitor General
such notice would be tantamount to depriving him of his day in court. We must stress that, contrary to petitioners'
stand, the mandatory notice requirement is not limited to actions involving declaratory relief and similar remedies.
The rule itself provides that such notice is required in "any action" and not just actions involving declaratory relief.
Where there is no ambiguity in the words used in the rule, there is no room for construction. 15 In all actions
assailing the validity of a statute, treaty, presidential decree, order, or proclamation, notice to the Solicitor General
is mandatory.

Page 8
Section 1-C, SY ’06-‘07
San Beda College of Law – Alabang
Constitutional Law 2 Case Digests

Petitioners contend that P.D. No. 579 and its implementing issuances are void for violating the due process
clause and the prohibition against the taking of private property without just compensation. Petitioners now ask
this Court to exercise its power of judicial review.

Jurisprudence has laid down the following requisites for the exercise of this power: First, there must be before the
Court an actual case calling for the exercise of judicial review. Second, the question before the Court must be ripe
for adjudication. Third, the person challenging the validity of the act must have standing to challenge. Fourth, the
question of constitutionality must have been raised at the earliest opportunity, and lastly, the issue of
constitutionality must be the very lis mota of the case.

DUMLAO VS. COMELEC


[95 SCRA 392; G.R. No.L-52245; 22 Jan 1980]

Facts:

Petitioner Dumlao questions the constitutionality of Sec. 4 of Batas Pambansa Blg 52 as discriminatory and
contrary to equal protection and due process guarantees of the Constitution. Sec. 4 provides that any retired
elective provincial or municipal official who has received payments of retirement benefits and shall have been 65
years of age at the commencement of the term of office to which he seeks to be elected, shall not be qualified to
run for the same elective local office from which he has retired. According to Dumlao, the provision amounts to
class legislation. Petitioners Igot and Salapantan Jr. also assail the validity of Sec. 4 of Batas Pambansa Blg 52,
which states that any person who has committed any act of disloyalty to the State, including those amounting to
subversion, insurrection, rebellion, or other similar crimes, shall not be qualified for any of the offices covered by
the act, or to participate in any partisan activity therein: provided that a judgment of conviction of those crimes
shall be conclusive evidence of such fact and the filing of charges for the commission of such crimes before a civil
court or military tribunal after preliminary investigation shall be prima facie evidence of such fact.

Issue:

Whether or Not the aforementioned statutory provisions violate the Constitution and thus, should be declared null
and void

Whether or not the requisites of judicial review are complied with

Held:

No constitutional question will be heard and decided by the Court unless there is compliance with the requisites of
a judicial inquiry, which are: 1) There must be an actual case or controversy; 2) The question of constitutionality
must be raised by the proper party; 3) The constitutional question must be raised at the earliest possible
opportunity; and 4) The decision of the constitutional question must be necessary to the determination of the case
itself.

As to (1), Dumlao has not been adversely affected by the application of the provision. His question is posed
merely in the abstract, and without the benefit of a detailed factual record. As to (2), neither Igot nor Salapantan
has been charged with acts of loyalty to the State, nor disqualified from being candidates for local elective
positions. They have no personal nor substantial interest at stake. Igot and Salapantan have institute the case as
a taxpayer’s suit, but the institution of a taxpayer’s suit per se is no assurance of judicial review. As to (4), there is
no cause of action in this particular case. Therefore, the necessity for resolving the issue of constitutionality is
absent.

In regards to the unconstitutionality of the provisions, Sec. 4 of BP Blg 52 remains constitutional and valid. The
constitutional guarantee of equal protection of the laws is subject to rational classification. One class can be
treated differently from another class. In this case, employees 65 years of age are classified differently from
younger employees. The purpose of the provision is to satisfy the “need for new blood” in the workplace. In
regards to the second paragraph of Sec. 4, it should be declared null and void for being violative of the
constitutional presumption of innocence guaranteed to an accused.

LACSON VS. PEREZ


[357 SCRA 756; G.R. No. 147780 ;10 May 2001]

Facts:

President Macapagal-Arroyo declared a State of Rebellion (Proclamation No. 38) on May 1, 2001 as well as
General Order No. 1 ordering the AFP and the PNP to suppress the rebellion in the NCR. Warrantless arrests of

Page 9
Section 1-C, SY ’06-‘07
San Beda College of Law – Alabang
Constitutional Law 2 Case Digests

several alleged leaders and promoters of the “rebellion” were thereafter effected. Petitioner filed for prohibition,
injunction, mandamus and habeas corpus with an application for the issuance of temporary restraining order
and/or writ of preliminary injunction. Petitioners assail the declaration of Proc. No. 38 and the warrantless arrests
allegedly effected by virtue thereof. Petitioners furthermore pray that the appropriate court, wherein the
information against them were filed, would desist arraignment and trial until this instant petition is resolved. They
also contend that they are allegedly faced with impending warrantless arrests and unlawful restraint being that
hold departure orders were issued against them.

Issue:

Whether or Not Proclamation No. 38 is valid, along with the warrantless arrests and hold departure orders
allegedly effected by the same.

Held:

President Macapagal-Arroyo ordered the lifting of Proc. No. 38 on May 6, 2006, accordingly the instant petition
has been rendered moot and academic. Respondents have declared that the Justice Department and the police
authorities intend to obtain regular warrants of arrests from the courts for all acts committed prior to and until May
1, 2001. Under Section 5, Rule 113 of the Rules of Court, authorities may only resort to warrantless arrests of
persons suspected of rebellion in suppressing the rebellion if the circumstances so warrant, thus the warrantless
arrests are not based on Proc. No. 38. Petitioner’s prayer for mandamus and prohibition is improper at this time
because an individual warrantlessly arrested has adequate remedies in law: Rule 112 of the Rules of Court,
providing for preliminary investigation, Article 125 of the Revised Penal Code, providing for the period in which a
warrantlessly arrested person must be delivered to the proper judicial authorities, otherwise the officer responsible
for such may be penalized for the delay of the same. If the detention should have no legal ground, the arresting
officer can be charged with arbitrary detention, not prejudicial to claim of damages under Article 32 of the Civil
Code. Petitioners were neither assailing the validity of the subject hold departure orders, nor were they
expressing any intention to leave the country in the near future. To declare the hold departure orders null and
void ab initio must be made in the proper proceedings initiated for that purpose. Petitioners’ prayer for relief
regarding their alleged impending warrantless arrests is premature being that no complaints have been filed
against them for any crime, furthermore, the writ of habeas corpus is uncalled for since its purpose is to relieve
unlawful restraint which Petitioners are not subjected to.

Petition is dismissed. Respondents, consistent and congruent with their undertaking earlier adverted to, together
with their agents, representatives, and all persons acting in their behalf, are hereby enjoined from arresting
Petitioners without the required judicial warrants for all acts committed in relation to or in connection with the May
1, 2001 siege of Malacañang.

SANLAKAS VS. EXECUTIVE SECRETARY


[421 SCRA 656; G.R. No. 159085; 3 Feb 2004]

Facts:

During the wee hours of July 27, 2003, some three-hundred junior officers and enlisted men of the AFP, acting
upon instigation, command and direction of known and unknown leaders have seized the Oakwood Building in
Makati. Publicly, they complained of the corruption in the AFP and declared their withdrawal of support for the
government, demanding the resignation of the President, Secretary of Defense and the PNP Chief. These acts
constitute a violation of Article 134 of the Revised Penal Code, and by virtue of Proclamation No. 427 and
General Order No. 4, the Philippines was declared under the State of Rebellion. Negotiations took place and the
officers went back to their barracks in the evening of the same day. On August 1, 2003, both the Proclamation
and General Orders were lifted, and Proclamation No. 435, declaring the Cessation of the State of Rebellion was
issued.

In the interim, however, the following petitions were filed: (1) SANLAKAS AND PARTIDO NG MANGGAGAWA
VS. EXECUTIVE SECRETARY, petitioners contending that Sec. 18 Article VII of the Constitution does not require
the declaration of a state of rebellion to call out the AFP, and that there is no factual basis for such proclamation.
(2)SJS Officers/Members v. Hon. Executive Secretary, et al, petitioners contending that the proclamation is a
circumvention of the report requirement under the same Section 18, Article VII, commanding the President to
submit a report to Congress within 48 hours from the proclamation of martial law. Finally, they contend that the
presidential issuances cannot be construed as an exercise of emergency powers as Congress has not delegated
any such power to the President. (3) Rep. Suplico et al. v. President Macapagal-Arroyo and Executive Secretary
Romulo, petitioners contending that there was usurpation of the power of Congress granted by Section 23 (2),
Article VI of the Constitution. (4) Pimentel v. Romulo, et al, petitioner fears that the declaration of a state of
rebellion "opens the door to the unconstitutional implementation of warrantless arrests" for the crime of rebellion.

Issue:

Page 10
Section 1-C, SY ’06-‘07
San Beda College of Law – Alabang
Constitutional Law 2 Case Digests

Whether or Not Proclamation No. 427 and General Order No. 4 are constitutional?

Whether or Not the petitioners have a legal standing or locus standi to bring suit?

Held:

The Court rendered that the both the Proclamation No. 427 and General Order No. 4 are constitutional. Section
18, Article VII does not expressly prohibit declaring state or rebellion. The President in addition to its Commander-
in-Chief Powers is conferred by the Constitution executive powers. It is not disputed that the President has full
discretionary power to call out the armed forces and to determine the necessity for the exercise of such power.
While the Court may examine whether the power was exercised within constitutional limits or in a manner
constituting grave abuse of discretion, none of the petitioners here have, by way of proof, supported their
assertion that the President acted without factual basis. The issue of the circumvention of the report is of no merit
as there was no indication that military tribunals have replaced civil courts or that military authorities have taken
over the functions of Civil Courts. The issue of usurpation of the legislative power of the Congress is of no
moment since the President, in declaring a state of rebellion and in calling out the armed forces, was merely
exercising a wedding of her Chief Executive and Commander-in-Chief powers. These are purely executive
powers, vested on the President by Sections 1 and 18, Article VII, as opposed to the delegated legislative powers
contemplated by Section 23 (2), Article VI. The fear on warrantless arrest is unreasonable, since any person may
be subject to this whether there is rebellion or not as this is a crime punishable under the Revised Penal Code,
and as long as a valid warrantless arrest is present.

Legal standing or locus standi has been defined as a personal and substantial interest in the case such that the
party has sustained or will sustain direct injury as a result of the governmental act that is being challenged. The
gist of the question of standing is whether a party alleges "such personal stake in the outcome of the controversy
as to assure that concrete adverseness which sharpens the presentation of Issue upon which the court depends
for illumination of difficult constitutional questions. Based on the foregoing, petitioners Sanlakas and PM, and SJS
Officers/Members have no legal standing to sue. Only petitioners Rep. Suplico et al. and Sen. Pimentel, as
Members of Congress, have standing to challenge the subject issuances. It sustained its decision in Philippine
Constitution Association v. Enriquez, that the extent the powers of Congress are impaired, so is the power of each
member thereof, since his office confers a right to participate in the exercise of the powers of that institution.

JOYA VS. PCGG


[225 SCRA 568; G.R. No. 96541; 24 Aug 1993]

Facts:

On 9 August 1990, Mateo A.T. Caparas, then Chairman of PCGG, wrote then President Corazon C. Aquino,
requesting her for authority to sign the proposed Consignment Agreement between the Republic of the
Philippines through PCGG and Christie, Manson and Woods International, Inc concerning the scheduled sale on
11 January 1991 of eighty-two) Old Masters Paintings and antique silverware seized from Malacañang and the
Metropolitan Museum of Manila alleged to be part of the ill-gotten wealth of the late President Marcos, his
relatives and cronies. On 14 August 1990, then President Aquino, through former Executive Secretary Catalino
Macaraig, Jr., authorized Chairman Caparas to sign the Consignment Agreement allowing Christie's of New York
to auction off the subject art pieces for and in behalf of the Republic of the Philippines. On 15 August 1990,
PCGG, through Chairman Caparas, representing the Government of the Republic of the Philippines, signed the
Consignment Agreement with Christie's of New York. According to the agreement, PCGG shall consign to
CHRISTIE'S for sale at public auction the eighty-two Old Masters Paintings then found at the Metropolitan
Museum of Manila as well as the silverware contained in seventy-one cartons in the custody of the Central Bank
of the Philippines, and such other property as may subsequently be identified by PCGG and accepted by
CHRISTIE'S to be subject to the provisions of the agreement.

On 26 October 1990, the Commission on Audit through then Chairman Eufemio C. Domingo submitted to
President Aquino the audit findings and observations of COA on the Consignment Agreement of 15 August 1990
to the effect that: the authority of former PCGG Chairman Caparas to enter into the Consignment Agreement was
of doubtful legality; the contract was highly disadvantageous to the government; PCGG had a poor track record in
asset disposal by auction in the U.S.; and, the assets subject of auction were historical relics and had cultural
significance, hence, their disposal was prohibited by law.

After the oral arguments of the parties on 9 January 1991, we issued immediately our resolution denying the
application for preliminary injunction to restrain the scheduled sale of the artworks on the ground that petitioners
had not presented a clear legal right to a restraining order and that proper parties had not been impleaded.

On 11 January 1991, the sale at public auction proceeded as scheduled and the proceeds of $13,302,604.86
were turned over to the Bureau of Treasury.

Page 11
Section 1-C, SY ’06-‘07
San Beda College of Law – Alabang
Constitutional Law 2 Case Digests

Issue:

Whether or not petitioners have legal standing.

Whether or not the Old Masters Paintings and antique silverware are embraced in the phrase "cultural treasure of
the nation".

Whether or not the paintings and silverware are properties of public dominion on which can be disposed of
through the joint concurrence of the President and Congress.

Whether or not PCGG has complied with the due process clause and other statutory requirements for the
exportation and sale of the subject items.

Whether or not the petition has become moot and academic, and if so, whether the above Issue warrant
resolution from this Court.

Held:

This is premised on Sec. 2, Rule 3, of the Rules of Court which provides that every action must be prosecuted
and defended in the name of the real party-in-interest, and that all persons having interest in the subject of the
action and in obtaining the relief demanded shall be joined as plaintiffs. The Court will exercise its power of
judicial review only if the case is brought before it by a party who has the legal standing to raise the constitutional
or legal question. "Legal standing" means a personal and substantial interest in the case such that the party has
sustained or will sustain direct injury as a result of the governmental act that is being challenged. The term
"interest" is material interest, an interest in issue and to be affected by the decree, as distinguished from mere
interest in the question involved, or a mere incidental interest. Moreover, the interest of the party plaintiff must be
personal and not one based on a desire to vindicate the constitutional right of some third and related party.

There are certain instances however when this Court has allowed exceptions to the rule on legal standing, as
when a citizen brings a case for mandamus to procure the enforcement of a public duty for the fulfillment of a
public right recognized by the Constitution, and when a taxpayer questions the validity of a governmental act
authorizing the disbursement of public funds.

Petitioners' arguments are devoid of merit. They lack basis in fact and in law. The ownership of these paintings
legally belongs to the foundation or corporation or the members thereof, although the public has been given the
opportunity to view and appreciate these paintings when they were placed on exhibit.

The confiscation of these properties by the Aquino administration however should not be understood to mean that
the ownership of these paintings has automatically passed on the government without complying with
constitutional and statutory requirements of due process and just compensation. If these properties were already
acquired by the government, any constitutional or statutory defect in their acquisition and their subsequent
disposition must be raised only by the proper parties the true owners thereof whose authority to recover emanates
from their proprietary rights which are protected by statutes and the Constitution. Having failed to show that they
are the legal owners of the artworks or that the valued pieces have become publicly owned, petitioners do not
possess any clear legal right whatsoever to question their alleged unauthorized disposition.

Neither can this petition be allowed as a taxpayer's suit. Obviously, petitioners are not challenging any
expenditure involving public funds but the disposition of what they allege to be public properties. It is worthy to
note that petitioners admit that the paintings and antique silverware were acquired from private sources and not
with public money.
Anent the second requisite of actual controversy, petitioners argue that this case should be resolved by this Court
as an exception to the rule on moot and academic cases; that although the sale of the paintings and silver has
long been consummated and the possibility of retrieving the treasure trove is nil, yet the novelty and importance of
the Issue raised by the petition deserve this Court's attention. They submit that the resolution by the Court of the
Issue in this case will establish future guiding principles and doctrines on the preservation of the nation's priceless
artistic and cultural possessions for the benefit of the public as a whole.

For a court to exercise its power of adjudication, there must be an actual case of controversy — one which
involves a conflict of legal rights, an assertion of opposite legal claims susceptible of judicial resolution; the case
must not be moot or academic or based on extra-legal or other similar considerations not cognizable by a court of
justice. A case becomes moot and academic when its purpose has become stale, such as the case before us.
Since the purpose of this petition for prohibition is to enjoin respondent public officials from holding the auction
sale of the artworks on a particular date — 11 January 1991 — which is long past, the Issue raised in the petition
have become moot and academic.

Page 12
Section 1-C, SY ’06-‘07
San Beda College of Law – Alabang
Constitutional Law 2 Case Digests

The cultural properties of the nation which shall be under the protection of the state are classified as the
"important cultural properties" and the "national cultural treasures." On the other hand, a "national cultural
treasures" is a unique object found locally, possessing outstanding historical, cultural, artistic and/or scientific
value which is highly significant and important to this country and nation. This Court takes note of the certification
issued by the Director of the Museum that the Italian paintings and silverware subject of this petition do not
constitute protected cultural properties and are not among those listed in the Cultural Properties Register of the
National Museum.

WHEREFORE, for lack of merit, the petition for prohibition and mandamus is DISMISSED.

OPOSA VS. FACTORAN, JR.


[224 SCRA 792; G.R. No. 101083; 30 Jul 1993]

Facts:

Principal petitioners, are all minors duly represented and joined by their respective parents. Impleaded as an
additional plaintiff is the Philippine Ecological Network, Inc. (PENI), a domestic, non-stock and non-profit
corporation organized for the purpose of, inter alia, engaging in concerted action geared for the protection of our
environment and natural resources. The original defendant was the Honorable Fulgencio S. Factoran, Jr., then
Secretary of the Department of Environment and Natural Resources (DENR). His substitution in this petition by
the new Secretary, the Honorable Angel C. Alcala, was subsequently ordered upon proper motion by the
petitioners. The complaint was instituted as a taxpayers' class suit and alleges that the plaintiffs "are all citizens of
the Republic of the Philippines, taxpayers, and entitled to the full benefit, use and enjoyment of the natural
resource treasure that is the country's virgin tropical forests." The same was filed for themselves and others who
are equally concerned about the preservation of said resource but are "so numerous that it is impracticable to
bring them all before the Court."

On 22 June 1990, the original defendant, Secretary Factoran, Jr., filed a Motion to Dismiss the complaint based
on two grounds, namely: the plaintiffs have no cause of action against him and, the issue raised by the plaintiffs
is a political question which properly pertains to the legislative or executive branches of Government. In their 12
July 1990 Opposition to the Motion, the petitioners maintain that, the complaint shows a clear and unmistakable
cause of action, the motion is dilatory and the action presents a justiciable question as it involves the defendant's
abuse of discretion.

On 18 July 1991, respondent Judge issued an order granting the aforementioned motion to dismiss. In the said
order, not only was the defendant's claim that the complaint states no cause of action against him and that it
raises a political question sustained, the respondent Judge further ruled that the granting of the relief prayed for
would result in the impairment of contracts which is prohibited by the fundamental law of the land.
Plaintiffs thus filed the instant special civil action for certiorari under Rule 65 of the Revised Rules of Court and
ask this Court to rescind and set aside the dismissal order on the ground that the respondent Judge gravely
abused his discretion in dismissing the action. Again, the parents of the plaintiffs-minors not only represent their
children, but have also joined the latter in this case.

Petitioners contend that the complaint clearly and unmistakably states a cause of action as it contains sufficient
allegations concerning their right to a sound environment based on Articles 19, 20 and 21 of the Civil Code
(Human Relations), Section 4 of Executive Order (E.O.) No. 192 creating the DENR, Section 3 of Presidential
Decree (P.D.) No. 1151 (Philippine Environmental Policy), Section 16, Article II of the 1987 Constitution
recognizing the right of the people to a balanced and healthful ecology, the concept of generational genocide in
Criminal Law and the concept of man's inalienable right to self-preservation and self-perpetuation embodied in
natural law. Petitioners likewise rely on the respondent's correlative obligation per Section 4 of E.O. No. 192, to
safeguard the people's right to a healthful environment.

It is further claimed that the issue of the respondent Secretary's alleged grave abuse of discretion in granting
Timber License Agreements (TLAs) to cover more areas for logging than what is available involves a judicial
question.

Anent the invocation by the respondent Judge of the Constitution's non-impairment clause, petitioners maintain
that the same does not apply in this case because TLAs are not contracts. They likewise submit that even if TLAs
may be considered protected by the said clause, it is well settled that they may still be revoked by the State when
the public interest so requires.

Issue:

Whether or not the petitioners have locus standi.

Page 13
Section 1-C, SY ’06-‘07
San Beda College of Law – Alabang
Constitutional Law 2 Case Digests

Whether or not the petiton is in a form of a class suit.

Whether or not the TLA’s can be out rightly cancelled.

Whether or not the petition should be dismissed.

Held:

As to the matter of the cancellation of the TLAs, respondents submit that the same cannot be done by the State
without due process of law. Once issued, a TLA remains effective for a certain period of time — usually for
twenty-five (25) years. During its effectivity, the same can neither be revised nor cancelled unless the holder has
been found, after due notice and hearing, to have violated the terms of the agreement or other forestry laws and
regulations. Petitioners' proposition to have all the TLAs indiscriminately cancelled without the requisite hearing
would be violative of the requirements of due process.

The subject matter of the complaint is of common and general interest not just to several, but to all citizens of the
Philippines. Consequently, since the parties are so numerous, it, becomes impracticable, if not totally impossible,
to bring all of them before the court. The plaintiffs therein are numerous and representative enough to ensure the
full protection of all concerned interests. Hence, all the requisites for the filing of a valid class suit under Section
12, Rule 3 of the Revised Rules of Court are present both in the said civil case and in the instant petition, the
latter being but an incident to the former.
Petitioners minors assert that they represent their generation as well as generations yet unborn. Their personality
to sue in behalf of the succeeding generations can only be based on the concept of intergenerational
responsibility insofar as the right to a balanced and healthful ecology is concerned. Nature means the created
world in its entirety. Every generation has a responsibility to the next to preserve that rhythm and harmony for the
full enjoyment of a balanced and healthful ecology. The minors' assertion of their right to a sound environment
constitutes, at the same time, the performance of their obligation to ensure the protection of that right for the
generations to come.

The complaint focuses on one specific fundamental legal right the right to a balanced and healthful ecology which,
for the first time in our nation's constitutional history, is solemnly incorporated in the fundamental law. Section 16,
Article II of the 1987 Constitution.

While the right to a balanced and healthful ecology is to be found under the Declaration of Principles and State
Policies and not under the Bill of Rights, it does not follow that it is less important than any of the civil and political
rights enumerated in the latter. Such a right belongs to a different category of rights altogether for it concerns
nothing less than self-preservation and self-perpetuation — aptly and fittingly stressed by the petitioners the
advancement of which may even be said to predate all governments and constitutions. As a matter of fact, these
basic rights need not even be written in the Constitution for they are assumed to exist from the inception of
humankind. If they are now explicitly mentioned in the fundamental charter, it is because of the well-founded fear
of its framers that unless the rights to a balanced and healthful ecology and to health are mandated as state
policies by the Constitution itself, thereby highlighting their continuing importance and imposing upon the state a
solemn obligation to preserve the first and protect and advance the second, the day would not be too far when all
else would be lost not only for the present generation, but also for those to come generations which stand to
inherit nothing but parched earth incapable of sustaining life.

Conformably with the enunciated right to a balanced and healthful ecology and the right to health, as well as the
other related provisions of the Constitution concerning the conservation, development and utilization of the
country's natural resources, then President Corazon C. Aquino promulgated on 10 June 1987 E.O. No. 192,
Section 4 of which expressly mandates that the Department of Environment and Natural Resources "shall be the
primary government agency responsible for the conservation, management, development and proper use of the
country's environment and natural resources, specifically forest and grazing lands, mineral, resources, including
those in reservation and watershed areas, and lands of the public domain, as well as the licensing and regulation
of all natural resources as may be provided for by law in order to ensure equitable sharing of the benefits derived
therefrom for the welfare of the present and future generations of Filipinos." Section 3 thereof makes the following
statement of policy:

The above provision stresses "the necessity of maintaining a sound ecological balance and protecting and
enhancing the quality of the environment." Section 2 of the same Title, on the other hand, specifically speaks of
the mandate of the DENR; however, it makes particular reference to the fact of the agency's being subject to law
and higher authority.

It may, however, be recalled that even before the ratification of the 1987 Constitution, specific statutes already
paid special attention to the "environmental right" of the present and future generations. On 6 June 1977, P.D. No.
1151 and P.D. No. 1152 were issued. Thus, the right of the petitioners to a balanced and healthful ecology is as
clear as the DENR's duty under its mandate and by virtue of its powers and functions under E.O. No. 192 and the
Administrative Code of 1987 to protect and advance the said right.

Page 14
Section 1-C, SY ’06-‘07
San Beda College of Law – Alabang
Constitutional Law 2 Case Digests

A denial or violation of that right by the other who has the correlative duty or obligation to respect or protect the
same gives rise to a cause of action. Petitioners maintain that the granting of the TLAs, which they claim was
done with grave abuse of discretion, violated their right to a balanced and healthful ecology; hence, the full
protection thereof requires that no further TLAs should be renewed or granted.

It is settled in this jurisdiction that in a motion to dismiss based on the ground that the complaint fails to state a
cause of action; the question submitted to the court for resolution involves the sufficiency of the facts alleged in
the complaint itself. No other matter should be considered; furthermore, the truth of falsity of the said allegations
is beside the point for the truth thereof is deemed hypothetically admitted. Policy formulation or determination by
the executive or legislative branches of Government is not squarely put in issue. What is principally involved is the
enforcement of a right vis-a-vis policies already formulated and expressed in legislation. It must, nonetheless, be
emphasized that the political question doctrine is no longer, the insurmountable obstacle to the exercise of judicial
power or the impenetrable shield that protects executive and legislative actions from judicial inquiry or review.

In the second place, even if it is to be assumed that the same are contracts, the instant case does not involve a
law or even an executive issuance declaring the cancellation or modification of existing timber licenses. Hence,
the non-impairment clause cannot as yet be invoked. Nevertheless, granting further that a law has actually been
passed mandating cancellations or modifications, the same cannot still be stigmatized as a violation of the non-
impairment clause. This is because by its very nature and purpose, such as law could have only been passed in
the exercise of the police power of the state for the purpose of advancing the right of the people to a balanced
and healthful ecology, promoting their health and enhancing the general welfare.

Finally, it is difficult to imagine, as the trial court did, how the non-impairment clause could apply with respect to
the prayer to enjoin the respondent Secretary from receiving, accepting, processing, renewing or approving new
timber licenses for, save in cases of renewal, no contract would have as of yet existed in the other instances.
Moreover, with respect to renewal, the holder is not entitled to it as a matter of right.

Petition is hereby GRANTED, and the challenged Order of respondent Judge of 18 July 1991 dismissing Civil
Case No. 90-777 is hereby set aside. The petitioners may therefore amend their complaint to implead as
defendants the holders or grantees of the questioned timber license agreements.

AGAN JR. VS. PIATCO


[402 SCRA 612; G.R. No. 155001; 5 May 2003]

Facts:

Some time in 1993, six business leaders, explored the possibility of investing in the new NAIA airport terminal, so
they formed Asians Emerging Dragon Corp. They submitted proposals to the government for the development of
NAIA Intl. Passenger Terminal III (NAIA IPT III). The NEDA approved the NAIA IPT III project. Bidders were
invited, and among the proposal Peoples Air Cargo (Paircargo) was chosen. AEDC protested alleging that
preference was given to Paircargo, but still the project was awarded to Paircargo. Because of that, it incorporated
into, Phil. Intl. Airport Terminals Co. (PIATCO). The DOTC and PIATCO entered into a concession agreement in
1997 to franchise and operate the said terminal for 21years. In Nov. 1998 it was amended in the matters of
pertaining to the definition of the obligations given to the concessionaire, development of facilities and proceeds,
fees and charges, and the termination of contract. Since MIAA is charged with the maintenance and operations of
NAIA terminals I and II, it has a contract with several service providers. The workers filed the petition for
prohibition claiming that they would lose their job, and the service providers joined them, filed a motion for
intervention. Likewise several employees of the MIAA filed a petition assailing the legality of arrangements. A
group of congressmen filed similar petitions. Pres. Arroyo declared in her speech that she will not honor PIATCO
contracts which the Exec. Branch's legal office concluded null and void.

Issue:

Whether or Not the 1997 concession agreement is void, together with its amendments for being contrary to the
constitution.

Held:

The 1997 concession agreement is void for being contrary to public policy. The amendments have the effect of
changing it into and entirely different agreement from the contract bidded upon. The amendments present new
terms and conditions which provide financial benefit to PIATCO which may have the altered the technical and
financial parameters of other bidders had they know that such terms were available. The 1997 concession
agreement, the amendments and supplements thereto are set aside for being null and void.

Page 15
Section 1-C, SY ’06-‘07
San Beda College of Law – Alabang
Constitutional Law 2 Case Digests

The petitioners have local standi. They are prejudiced by the concession agreement as their livelihood is to be
taken away from them.

UMALI VS. GUINGONA


[305 SCRA 533; G.R. No. 131124; 21 Mar 1999]

Facts:

Osmundo Umali the petitioner was appointed Regional Director of the Bureau of Internal Revenue by Pres Fidel
V. Ramos. He assigned him in Manila, November 29, 1993 to March 15, 1994 and Makati, March 16, 1994 to
August 4, 1994. On August 1, 1994, President Ramos received a confidential memorandum against the petitioner
for alleged violations of internal revenue laws, rules and regulations during his incumbency as Regional Director,
more particularly the following malfeasance, misfeasance and nonfeasance. upon receipt of the said confidential
memorandum, former President authorized the issuance of an Order for the preventive suspension of the
petitioner and immediately referred the Complaint against the latter to the Presidential Commission on Anti-Graft
and Corruption (PCAGC), for investigation. Petitioner was duly informed of the charges against him. And was
directed him to send in his answer, copies of his Statement of Assets, and Liabilities for the past three years (3),
and Personal Data Sheet. Initial hearing was set on August 25, 1994, at 2:00 p.m., at the PCAGC Office. On
August 23, the petitioner filed his required answer. After evaluating the evidence on record, the PCAGC issued its
Resolution of September 23, 1994, finding a prima facie evidence to support six (6) of the twelve (12) charges
against petitioner. On October 6, 1994, acting upon the recommendation of the PCAGC, then President Ramos
issued Administrative Order No. 152 dismissing petitioner from the service, with forfeiture of retirement and all
benefits under the law.

Issue:

Whether or Not AO No. 152 violated petitioner's Right to Security of Tenure.

Whether or Not Petitioner was denied due process of law

Whether or Not the PCAGC is a validly Constituted government agency and whether the petitioner can raise the
issue of constitutionality belatedly in its motion for reconsideration of the trial courts decision.

Whether or Not the ombudsman's resolution dismissing the charges against the petitioner is still basis for the
petitioner's dismissal with forfeiture of benefits as ruled in AO No. 152

Held:

Petitioner maintains that as a career executive service officer, he can only be removed for cause and under the
Administrative Code of 1987, 6 loss of confidence is not one of the legal causes or grounds for removal.
Consequently, his dismissal from office on the ground of loss confidence violated his right to security of tenure,
petitioner theorized. After a careful study, we are of the irresistible conclusion that the Court of Appeals ruled
correctly on the first three Issue. To be sure, petitioner was not denied the right to due process before the
PCAGC. Records show that the petitioner filed his answer and other pleadings with respect to his alleged
violation of internal revenue laws and regulations, and he attended the hearings before the investigatory body. It
is thus decisively clear that his protestation of non-observance of due process is devoid of any factual or legal
basis. Neither can it be said that there was a violation of what petitioner asserts as his security of tenure.
According to petitioner, as a Regional Director of Bureau of Internal Revenue, he is CESO eligible entitled to
security of tenure. However, petitioner's claim of CESO eligibility is anemic of evidentiary support. It was
incumbent upon him to prove that he is a CESO eligible but unfortunately, he failed to adduce sufficient evidence
on the matter. His failure to do so is fatal. As regards the issue of constitutionality of the PCAGC, it was only
posed by petitioner in his motion for reconsideration before the Regional Trial Court of Makati. It was certainly too
late to raise for the first time at such late stage of the proceedings. As to last issue, It is worthy to note that in the
case under consideration, the administrative action against the petitioner was taken prior to the institution of the
criminal case. The charges included in Administrative Order No. 152 were based on the results of investigation
conducted by the PCAGC and not on the criminal charges before the Ombudsman. In sum, the petition is
dismissable on the ground that the Issue posited by the petitioner do not constitute a valid legal basis for
overturning the finding and conclusion arrived at by the Court of Appeals. However, taking into account the
antecedent facts and circumstances aforementioned, the Court, in the exercise of its equity powers, has decided
to consider the dismissal of the charges against petitioner before the Ombudsman, the succinct and unmistakable
manifestation by the Commissioner of the Bureau of Internal Revenue that his office is no longer interested in
pursuing the case, and the position taken by the Solicitor General, that there is no more basis for Administrative
Order No. 152, as effective and substantive supervening events that cannot be overlooked.

IN RE CUNANAN

Page 16
Section 1-C, SY ’06-‘07
San Beda College of Law – Alabang
Constitutional Law 2 Case Digests

[94 Phil 534; Resolution; 18 Mar 1954]

Facts:

Congress passed Republic Act Number 972, commonly known as the “Bar Flunkers’ Act of 1953.” In accordance
with the said law, the Supreme Court then passed and admitted to the bar those candidates who had obtained an
average of 72 per cent by raising it to 75 percent.

After its approval, many of the unsuccessful postwar candidates filed petitions for admission to the bar invoking its
provisions, while other motions for the revision of their examination papers were still pending also invoked the
aforesaid law as an additional ground for admission. There are also others who have sought simply the
reconsideration of their grades without, however, invoking the law in question. To avoid injustice to individual
petitioners, the court first reviewed the motions for reconsideration, irrespective of whether or not they had
invoked Republic Act No. 972.

Issue:

Whether or Not RA No. 972 is constitutional and valid.

Held:

RA No. 972 has for its object, according to its author, to admit to the Bar, those candidates who suffered from
insufficiency of reading materials and inadequate preparation.

In the judicial system from which ours has been evolved, the admission, suspension, disbarment and
reinstatement of attorneys at law in the practice of the profession and their supervision have been indisputably a
judicial function and responsibility. We have said that in the judicial system from which ours has been derived, the
admission, suspension, disbarment or reinstatement of attorneys at law in the practice of the profession is
concededly judicial.

On this matter, there is certainly a clear distinction between the functions of the judicial and legislative
departments of the government.

It is obvious, therefore, that the ultimate power to grant license for the practice of law belongs exclusively to this
Court, and the law passed by Congress on the matter is of permissive character, or as other authorities may say,
merely to fix the minimum conditions for the license.

Republic Act Number 972 is held to be unconstitutional.

REPUBLIC ACT 6735, INITIATIVE AND REFERENDUM ACT

R.A. No. 6735 was, as its history reveals, intended to cover initiative to propose amendments to the Constitution.
The Act is a consolidation of House Bill No. 21505 and Senate Bill No. 17. The former was prepared by the
committee on Suffrage and Electoral Reforms of Representatives on the basis of two House Bills referred to it,
viz., (a) House Bill No. 497, which dealt with the initiative and referendum mentioned in Sections 1 and 32 of
Article VI of the Constitution; and (b) House Bill No. 988, which dealt with the subject matter of House Bill No.
497, as well as with initiative and referendum under Section 3 of Article XVII of the Constitution. Senate Bill No.
17 solely, dealt with initiative and referendum concerning ordinances or resolutions of local government units. The
Bicameral Conference Committee consolidated Senate Bill No. 17 and House Bill No. 21505 into a draft bill,
which was subsequently approved on 8 June 1989 by the Senate and by the House of Representatives. This
approved bill is now R.A. No. 6735.

Page 17
Section 1-C, SY ’06-‘07
San Beda College of Law – Alabang
Constitutional Law 2 Case Digests

THE FUNDAMENTAL POWERS OF


THE STATE
THE POLICE POWER

AGUSTIN VS. EDU


[88 SCRA 195; G.R. No. L-49112; 2 Feb 1979]

Facts:

President Marcos issued the Letter of Instruction No. 229 which states that all owners, users or drivers shall have
at all times one pair of early warning devise (EWD) in their cars acquire from any source depending on the
owner’s choice. The Letter of Instruction was assailed by petitioner Leovillo Agustin to have violated the
constitution guarantee of due process against Hon Edu, Land Transportation Commissioner, Hon. Juan Ponce
Enrile, Minister of national Defense, Hon. Juinio, Minister of Public Works, Transportation and Communication
and Hon. Aquino, Minister of Public Highways. Because of such contentions, the Implementing Rules and
Regulation was ordered to be suspended for a period of 6 months. Petitioner alleges that EWD are not necessary
because vehicles already have hazard lights (blinking lights) that can be use as a warning device. Also petitioner
contest that the letter of instruction violates the delegation of police power because it is deemed harsh,
oppressive and unreasonable for the motorists and those dealers of EWD will become instant millionaires
because of such law.

Issue:

Whether or not Petitioner’s contentions possess merit.

Held:

Petitioner’s contentions are without merit because the exercise of police power may interfere with personal liberty
or property to ensure and promote the safety, health and prosperity of the State. Also, such letter of instruction is
intended to promote public safety and it is indeed a rare occurrence that such contention was alleged in a
instruction with such noble purpose. Petitioner also failed to present the factual foundation that is necessary to
invalidate the said letter of instruction. In cases where there is absence in the factual foundation, it should be
presumed that constitutionality shall prevail. Pres. Marcos on the other hand possesses vital statistics that will
justify the need for the implementation of this instruction. As signatory to the 1968 Vienna Conventions on Road
Signs and Signals, our country must abide with the standards given as stated in our Constitution that “the
Philippines adopts the generally accepted principles of International Law as part of the law of the land. In the case
at bar, the Vienna Convention also requires the use of EWD. Vehicle owners are not obliged to buy an EDW.
They can personally create a EWD provided that it is in accordance to the specifications provided by law.
Petitioner’s allegation against the manufacturers of EDW being millionaires is deemed to be an unfounded
speculation. Wherefore, the petition is dismissed. The restraining order regarding the implementation of the
Reflector Law is lifted making the said law immediately executory.

ICHONG VS. HERNANDEZ


[101 Phil 1117; G.R. No. L-7995; 31 May 1957]

Facts:

Republic Act 1180 or commonly known as “An Act to Regulate the Retail Business” was passed. The said law
provides for a prohibition against foreigners as well as corporations owned by foreigners from engaging from retail
trade in our country. This was protested by the petitioner in this case. According to him, the said law violates the
international and treaty of the Philippines therefore it is unconstitutional. Specifically, the Treaty of Amity between
the Philippines and China was violated according to him.

Issue:

Whether or Not Republic Act 1180 is a valid exercise of police power.

Held:

Page 18
Section 1-C, SY ’06-‘07
San Beda College of Law – Alabang
Constitutional Law 2 Case Digests

According to the Court, RA 1180 is a valid exercise of police power. It was also then provided that police power
can not be bargained away through the medium of a treaty or a contract. The Court also provided that RA 1180
was enacted to remedy a real and actual danger to national economy posed by alien dominance and control. If
ever the law infringes upon the said treaty, the latter is always subject to qualification or amendment by a
subsequent law and the same may never curtain or restrict the scope of the police power of the state.

LUTZ VS. ARANETA


[98 Phil 148; G.R. No. L-7859; 22 Dec 1955]

Facts:

Walter Lutz, as the Judicial Administrator of the Intestate Estate of Antonio Jayme Ledesma, seeks to recover
from J. Antonio Araneta, the Collector of Internal Revenue, the sum of money paid by the estate as taxes,
pursuant to the Sugar Adjustment Act. Under Section 3 of said Act, taxes are levied on the owners or persons in
control of the lands devoted to the cultivation of sugar cane. Furthermore, Section 6 states all the collections
made under said Act shall be for aid and support of the sugar industry exclusively. Lutz contends that such
purpose is not a matter of public concern hence making the tax levied for that cause unconstitutional and void.
The Court of First Instance dismissed his petition, thus this appeal before the Supreme Court.

Issue:

Whether or Not the tax levied under the Sugar Adjustment Act ( Commonwealth Act 567) is unconstitutional.

Held:

The tax levied under the Sugar Adjustment Act is constitutional. The tax under said Act is levied with a regulatory
purpose, to provide means for the rehabilitation and stabilization of the threatened sugar industry. Since sugar
production is one of the great industries of our nation, its promotion, protection, and advancement, therefore
redounds greatly to the general welfare. Hence, said objectives of the Act is a public concern and is therefore
constitutional. It follows that the Legislature may determine within reasonable bounds what is necessary for its
protection and expedient for its promotion. If objectives and methods are alike constitutionally valid, no reason is
seen why the state may not levy taxes to raise funds for their prosecution and attainment. Taxation may be made
with the implement of the state’s police power. In addition, it is only rational that the taxes be obtained from those
that will directly benefit from it. Therefore, the tax levied under the Sugar Adjustment Act is held to be
constitutional.

TIO VS. VIDEOGRAM REGULATORY BOARD


[151 SCRA 208; G.R. No. L-75697; 18 Jun 1987]

Facts:

The case is a petition filed by petitioner on behalf of videogram operators adversely affected by Presidential
Decree No. 1987, “An Act Creating the Videogram Regulatory Board" with broad powers to regulate and
supervise the videogram industry.

A month after the promulgation of the said Presidential Decree, the amended the National Internal Revenue
Code provided that:

"SEC. 134. Video Tapes. — There shall be collected on each processed video-tape cassette,
ready for playback, regardless of length, an annual tax of five pesos; Provided, That locally
manufactured or imported blank video tapes shall be subject to sales tax."

"Section 10. Tax on Sale, Lease or Disposition of Videograms. — Notwithstanding any


provision of law to the contrary, the province shall collect a tax of thirty percent (30%) of the
purchase price or rental rate, as the case may be, for every sale, lease or disposition of a
videogram containing a reproduction of any motion picture or audiovisual program.”

“Fifty percent (50%) of the proceeds of the tax collected shall accrue to the province, and the
other fifty percent (50%) shall accrue to the municipality where the tax is collected; PROVIDED,
That in Metropolitan Manila, the tax shall be shared equally by the City/Municipality and the
Metropolitan Manila Commission.”

Page 19
Section 1-C, SY ’06-‘07
San Beda College of Law – Alabang
Constitutional Law 2 Case Digests

The rationale behind the tax provision is to curb the proliferation and unregulated circulation of videograms
including, among others, videotapes, discs, cassettes or any technical improvement or variation thereof, have
greatly prejudiced the operations of movie houses and theaters. Such unregulated circulation have caused a
sharp decline in theatrical attendance by at least forty percent (40%) and a tremendous drop in the collection of
sales, contractor's specific, amusement and other taxes, thereby resulting in substantial losses estimated at P450
Million annually in government revenues.

Videogram(s) establishments collectively earn around P600 Million per annum from rentals, sales and disposition
of videograms, and these earnings have not been subjected to tax, thereby depriving the Government of
approximately P180 Million in taxes each year.

The unregulated activities of videogram establishments have also affected the viability of the movie industry.

Issue:

Whether or not tax imposed by the DECREE is a valid exercise of police power.

Whether or nor the DECREE is constitutional .

Held:

Taxation has been made the implement of the state's police power. The levy of the 30% tax is for a public
purpose. It was imposed primarily to answer the need for regulating the video industry, particularly because of the
rampant film piracy, the flagrant violation of intellectual property rights, and the proliferation of pornographic video
tapes. And while it was also an objective of the DECREE to protect the movie industry, the tax remains a valid
imposition.

We find no clear violation of the Constitution which would justify us in pronouncing Presidential Decree No. 1987
as unconstitutional and void. While the underlying objective of the DECREE is to protect the moribund movie
industry, there is no question that public welfare is at bottom of its enactment, considering "the unfair competition
posed by rampant film piracy; the erosion of the moral fiber of the viewing public brought about by the availability
of unclassified and unreviewed video tapes containing pornographic films and films with brutally violent
sequences; and losses in government revenues due to the drop in theatrical attendance, not to mention the fact
that the activities of video establishments are virtually untaxed since mere payment of Mayor's permit and
municipal license fees are required to engage in business."

WHEREFORE, the instant Petition is hereby dismissed. No costs.

ASSO. OF SMALL LANDOWNERS VS. SEC. OF DAR


[175 SCRA 343; G.R. NO. L-78742; 14 JUL 1989]

Facts:

Several petitions are the root of the case:

a. A petition alleging the constitutionality of PD No. 27, EO 228 and 229 and RA 6657. Subjects of the
petition are a 9-hectare and 5 hectare Riceland worked by four tenants. Tenants were declared full
owners by EO 228 as qualified farmers under PD 27. The petitioners now contend that President
Aquino usurped the legislature’s power.
b. A petition by landowners and sugarplanters in Victoria’s Mill Negros Occidental against Proclamation
131 and EO 229. Proclamation 131 is the creation of Agrarian Reform Fund with initial fund of
P50Billion.
c. A petition by owners of land which was placed by the DAR under the coverage of Operation Land
Transfer.
d. A petition invoking the right of retention under PD 27 to owners of rice and corn lands not exceeding
seven hectares.

Issue:

Whether or Not the aforementioned EO’s, PD, and RA were constitutional.

Held:

The promulgation of PD 27 by President Marcos was valid in exercise of Police power and eminent domain.

Page 20
Section 1-C, SY ’06-‘07
San Beda College of Law – Alabang
Constitutional Law 2 Case Digests

The power of President Aquino to promulgate Proc. 131 and EO 228 and 229 was authorized under Sec. 6 of the
Transitory Provisions of the 1987 Constitution. Therefore it is a valid exercise of Police Power and Eminent
Domain.

RA 6657 is likewise valid. The carrying out of the regulation under CARP becomes necessary to deprive owners
of whatever lands they may own in excess of the maximum area allowed, there is definitely a taking under the
power of eminent domain for which payment of just compensation is imperative. The taking contemplated is not a
mere limitation of the use of the land. What is required is the surrender of the title and the physical possession of
said excess and all beneficial rights accruing to the owner in favour of the farmer.

A statute may be sustained under the police power only if there is concurrence of the lawful subject and the
method.

Subject and purpose of the Agrarian Reform Law is valid, however what is to be determined is the method
employed to achieve it.

LOZANO VS. M ARTINEZ


[146 SCRA 323; G.R. No. L-63419; 18 Dec 1986]

Facts:

A motion to quash the charge against the petitioners for violation of the BP 22 was made, contending that no
offense was committed, as the statute is unconstitutional. Such motion was denied by the RTC. The petitioners
thus elevate the case to the Supreme Court for relief. The Solicitor General, commented that it was premature for
the accused to elevate to the Supreme Court the orders denying their motions to quash. However, the Supreme
Court finds it justifiable to intervene for the review of lower court's denial of a motion to quash.

Issue:

Whether or not BP 22 is constitutional as it is a proper exercise of police power of the State.

Held:

The enactment of BP 22 a valid exercise of the police power and is not repugnant to the constitutional inhibition
against imprisonment for debt.

The offense punished by BP 22 is the act of making and issuing a worthless check or a check that is dishonored
upon its presentation for payment. It is not the non-payment of an obligation which the law punishes. The law is
not intended or designed to coerce a debtor to pay his debt.

The law punishes the act not as an offense against property, but an offense against public order. The thrust of the
law is to prohibit, under pain of penal sanctions, the making of worthless checks and putting them in circulation.
An act may not be considered by society as inherently wrong, hence, not malum in se but because of the harm
that it inflicts on the community, it can be outlawed and criminally punished as malum prohibitum. The state can
do this in the exercise of its police power.

KWONG SING VS. CITY OF MANILA


[41 Phil 103; G.R. No. 15972; 11 Oct 1920]

Facts:

Kwong Sing, in his own behalf and of other Chinese laundrymen who has general and the same interest, filed a
complaint for a preliminary injunction. The Plaintiffs also questioned the validity of enforcing Ordinance No. 532 by
the city of Manila. Ordinance No. 532 requires that the receipt be in duplicate in English and Spanish duly signed
showing the kind and number of articles delivered by laundries and dyeing and cleaning establishments. The
permanent injunction was denied by the trial court. The appellants claim is that Ordinance No. 532 savors of
class legislation; putting in mind that they are Chinese nationals. It unjustly discriminates between persons in
similar circumstances; and that it constitutes an arbitrary infringement of property rights. They also contest that
the enforcement of the legislation is an act beyond the scope of their police power. In view of the foregoing, this
is an appeal with the Supreme Court.

Issue:

Whether or Not the enforcement of Ordinance no, 532 is an act beyond the scope of police power

Page 21
Section 1-C, SY ’06-‘07
San Beda College of Law – Alabang
Constitutional Law 2 Case Digests

Whether or Not the enforcement of the same is a class legislation that infringes property rights.

Held:

Reasonable restraints of a lawful business for such purposes are permissible under the police power. The police
power of the City of Manila to enact Ordinance No. 532 is based on Section 2444, paragraphs (l) and (ee) of the
Administrative Code, as amended by Act No. 2744, authorizes the municipal board of the city of Manila, with the
approval of the mayor of the city:

(l) To regulate and fix the amount of the license fees for the following: xxxx xxxxxlaundries xxxx.

(ee) To enact all ordinances it may deem necessary and proper for the sanitation and safety, the
furtherance of the prosperity, and the promotion of the morality, peace, good order, comfort,
convenience, and general welfare of the city and its inhabitants.

The court held that the obvious purpose of Ordinance No. 532 was to avoid disputes between laundrymen and
their patrons and to protect customers of laundries who are not able to decipher Chinese characters from being
defrauded. (Considering that in the year 1920s, people of Manila are more familiar with Spanish and maybe
English.)

In whether the ordinance is class legislation, the court held that the ordinance invades no fundamental right, and
impairs no personal privilege. Under the guise of police regulation, an attempt is not made to violate personal
property rights. The ordinance is neither discriminatory nor unreasonable in its operation. It applies to all public
laundries without distinction, whether they belong to Americans, Filipinos, Chinese, or any other nationality. All,
without exception, and each every one of them without distinction, must comply with the ordinance. The obvious
objection for the implementation of the ordinance is based in sec2444 (ee) of the Administrative Code. Although,
an additional burden will be imposed on the business and occupation affected by the ordinance such as that of
the appellant by learning even a few words in Spanish or English, but mostly Arabic numbers in order to properly
issue a receipt, it seems that the same burdens are cast upon the them. Yet, even if private rights of person or
property are subjected to restraint, and even if loss will result to individuals from the enforcement of the
ordinance, this is not sufficient ground for failing to uphold the power of the legislative body. The very foundation
of the police power is the control of private interests for the public welfare.

Finding that the ordinance is valid, judgment is affirmed, and the petition for a preliminary injunction is denied,
with costs against the appellants.

TABLARIN VS. GUTIERREZ


[152 SCRA 730; G.R. No. 78164; 31 July 1987]

Facts:

The petitioners sought to enjoin the Secretary of Education, Culture and Sports, the Board of Medical Education
and the Center for Educational Measurement from enforcing Section 5 (a) and (f) of Republic Act No. 2382, as
amended, and MECS Order No. 52, series of 1985, dated 23 August 1985 and from requiring the taking and
passing of the NMAT as a condition for securing certificates of eligibility for admission, from proceeding with
accepting applications for taking the NMAT and from administering the NMAT as scheduled on 26 April 1987 and
in the future. The trial court denied said petition on 20 April 1987. The NMAT was conducted and administered
as previously scheduled.

Republic Act 2382, as amended by Republic Acts Nos. 4224 and 5946, known as the "Medical Act of 1959"
defines its basic objectives in the following manner:

"SECTION 1. Objectives. — This Act provides for and shall govern (a) the standardization and
regulation of medical education; (b) the examination for registration of physicians; and (c) the
supervision, control and regulation of the practice of medicine in the Philippines."

The statute, among other things, created a Board of Medical Education. Its functions as specified in Section 5 of
the statute include the following:

"(a) To determine and prescribe requirements for admission into a recognized college of
medicine;

Page 22
Section 1-C, SY ’06-‘07
San Beda College of Law – Alabang
Constitutional Law 2 Case Digests

x x x
(f) To accept applications for certification for admission to a medical school and keep a
register of those issued said certificate; and to collect from said applicants the amount of twenty-
five pesos each which shall accrue to the operating fund of the Board of Medical Education;”

Section 7 prescribes certain minimum requirements for applicants to medical schools:

"Admission requirements. — The medical college may admit any student who has not been
convicted by any court of competent jurisdiction of any offense involving moral turpitude and who
presents (a) a record of completion of a bachelor's degree in science or arts; (b) a certificate of
eligibility for entrance to a medical school from the Board of Medical Education; (c) a certificate of
good moral character issued by two former professors in the college of liberal arts; and (d) birth
certificate. Nothing in this act shall be construed to inhibit any college of medicine from
establishing, in addition to the preceding, other entrance requirements that may be deemed
admissible.”

MECS Order No. 52, s. 1985, issued by the then Minister of Education, Culture and Sports and dated 23 August
1985, established a uniform admission test called the National Medical Admission Test (NMAT) as an additional
requirement for issuance of a certificate of eligibility for admission into medical schools of the Philippines,
beginning with the school year 1986-1987. This Order goes on to state that: "2. The NMAT, an aptitude test, is
considered as an instrument toward upgrading the selection of applicants for admission into the medical schools
and its calculated to improve the quality of medical education in the country. The cutoff score for the successful
applicants, based on the scores on the NMAT, shall be determined every year by the Board of Medical Education
after consultation with the Association of Philippine Medical Colleges. The NMAT rating of each applicant,
together with the other admission requirements as presently called for under existing rules, shall serve as a basis
for the issuance of the prescribed certificate of eligibility for admission into the medical colleges.

Issue:

Whether or not Section 5 (a) and (f) of Republic Act No. 2382, as amended, and MECS Order No. 52, s. 1985 are
constitutional.

Held:

Yes. We conclude that prescribing the NMAT and requiring certain minimum scores therein as a condition for
admission to medical schools in the Philippines, do not constitute an unconstitutional imposition.

The police power, it is commonplace learning, is the pervasive and non-waivable power and authority of the
sovereign to secure and promote all the important interests and needs — in a word, the public order — of the
general community. An important component of that public order is the health and physical safety and well being
of the population, the securing of which no one can deny is a legitimate objective of governmental effort and
regulation. Perhaps the only issue that needs some consideration is whether there is some reasonable relation
between the prescribing of passing the NMAT as a condition for admission to medical school on the one hand,
and the securing of the health and safety of the general community, on the other hand. This question is perhaps
most usefully approached by recalling that the regulation of the practice of medicine in all its branches has long
been recognized as a reasonable method of protecting the health and safety of the public.

MECS Order No. 52, s. 1985 articulates the rationale of regulation of this type: the improvement of the
professional and technical quality of the graduates of medical schools, by upgrading the quality of those admitted
to the student body of the medical schools. That upgrading is sought by selectivity in the process of admission,
selectivity consisting, among other things, of limiting admission to those who exhibit in the required degree the
aptitude for medical studies and eventually for medical practice. The need to maintain, and the difficulties of
maintaining, high standards in our professional schools in general, and medical schools in particular, in the
current stage of our social and economic development, are widely known. We believe that the government is
entitled to prescribe an admission test like the NMAT as a means for achieving its stated objective of "upgrading
the selection of applicants into [our] medical schools" and of "improv[ing] the quality of medical education in the
country. We are entitled to hold that the NMAT is reasonably related to the securing of the ultimate end of
legislation and regulation in this area. That end, it is useful to recall, is the protection of the public from the
potentially deadly effects of incompetence and ignorance in those who would undertake to treat our bodies and
minds for disease or trauma.

WHEREFORE, the Petition for Certiorari is DISMISSED and the Order of the respondent trial court denying the
petition for a writ of preliminary injunction is AFFIRMED. Costs against petitioners.

Page 23
Section 1-C, SY ’06-‘07
San Beda College of Law – Alabang
Constitutional Law 2 Case Digests

CITY GOVERNMENT OF QUEZON CITY VS. ERICTA


[122 SCRA 759; G.R. No. L-34915; 24 Jun 1983]

Facts:

Section 9 of Ordinance No. 6118, S-64, entitled "Ordinance Regulating The Establishment, Maintenance And
Operation Of Private Memorial Type Cemetery Or Burial Ground Within The Jurisdiction Of Quezon City And
Providing Penalties For The Violation Thereof" provides:

Sec. 9. At least six (6) percent of the total area of the memorial park cemetery shall be set aside
for charity burial of deceased persons who are paupers and have been residents of Quezon City
for at least 5 years prior to their death, to be determined by competent City Authorities. The area
so designated shall immediately be developed and should be open for operation not later than six
months from the date of approval of the application.

For several years, the aforequoted section of the Ordinance was not enforced but seven years after the
enactment of the ordinance, the Quezon City Council passed a resolution to request the City Engineer, Quezon
City, to stop any further selling and/or transaction of memorial park lots in Quezon City where the owners thereof
have failed to donate the required 6% space intended for paupers burial.

The Quezon City Engineer then notified respondent Himlayang Pilipino, Inc. in writing that Section 9 of the
ordinance would be enforced.

Respondent Himlayang Pilipino reacted by filing a petition for declaratory relief, prohibition and mandamus with
preliminary injunction seeking to annul Section 9 of the Ordinance in question. Respondent alleged that the same
is contrary to the Constitution, the Quezon City Charter, the Local Autonomy Act, and the Revised Administrative
Code.

Issue:

Whether or Not Section 9 of the ordinance in question is a valid exercise of police power.

Held:

Section 9 of the City ordinance in question is not a valid exercise of police power. Section 9 cannot be justified
under the power granted to Quezon City to tax, fix the license fee, and regulate such other business, trades, and
occupation as may be established or practiced in the City.

Bill of rights states that 'no person shall be deprived of life, liberty or property without due process of law' (Art. Ill,
Section 1 subparagraph 1, Constitution). On the other hand, there are three inherent powers of government by
which the state interferes with the property rights, namely-. (1) police power, (2) eminent domain, (3) taxation.

The police power of Quezon City is defined in sub-section 00, Sec. 12, Rep. Act 537 that reads as follows:

“To make such further ordinance and regulations not repugnant to law as may be necessary to
carry into effect and discharge the powers and duties conferred by this act and such as it shall
deem necessary and proper to provide for the health and safety, …, and for the protection of
property therein; and enforce obedience thereto with such lawful fines or penalties as the City
Council may prescribe under the provisions of subsection (jj) of this section.”

The power to regulate does not include the power to prohibit. The power to regulate does not include the power to
confiscate. The ordinance in question not only confiscates but also prohibits the operation of a memorial park
cemetery, because under Section 13 of said ordinance, 'Violation of the provision thereof is punishable with a fine
and/or imprisonment and that upon conviction thereof the permit to operate and maintain a private cemetery shall
be revoked or cancelled’. The confiscatory clause and the penal provision in effect deter one from operating a
memorial park cemetery.

Moreover, police power is defined by Freund as 'the power of promoting the public welfare by restraining and
regulating the use of liberty and property'. It is usually exerted in order to merely regulate the use and enjoyment
of property of the owner. If he is deprived of his property outright, it is not taken for public use but rather to destroy
in order to promote the general welfare.

Page 24
Section 1-C, SY ’06-‘07
San Beda College of Law – Alabang
Constitutional Law 2 Case Digests

It seems to the court that Section 9 of Ordinance No. 6118, Series of 1964 of Quezon City is not a mere police
regulation but an outright confiscation. It deprives a person of his private property without due process of law, nay,
even without compensation.

MMDA Vs. Bel-Air Village


[328 SCRA 836; G.R. No. 135962; 27 Mar 2000]

Facts:

Metropolitan Manila Development Authority (MMDA), petitioner herein, is a Government Agency tasked with the
delivery of basic services in Metro Manila. Bel-Air Village Association (BAVA), respondent herein, received a letter
of request from the petitioner to open Neptune Street of Bel-Air Village for the use of the public. The said opening
of Neptune Street will be for the safe and convenient movement of persons and to regulate the flow of traffic in
Makati City. This was pursuant to MMDA law or Republic Act No. 7924. On the same day, the respondent was
appraised that the perimeter wall separating the subdivision and Kalayaan Avenue would be demolished.

The respondent, to stop the opening of the said street and demolition of the wall, filed a preliminary injunction and
a temporary restraining order. Respondent claimed that the MMDA had no authority to do so and the lower court
decided in favor of the Respondent. Petitioner appealed the decision of the lower courts and claimed that it has
the authority to open Neptune Street to public traffic because it is an agent of the State that can practice police
power in the delivery of basic services in Metro Manila.

Issue:

Whether or not the MMDA has the mandate to open Neptune Street to public traffic pursuant to its regulatory and
police powers.

Held:

The Court held that the MMDA does not have the capacity to exercise police power. Police power is primarily
lodged in the National Legislature. However, police power may be delegated to government units. Petitioner
herein is a development authority and not a political government unit. Therefore, the MMDA cannot exercise
police power because it cannot be delegated to them. It is not a legislative unit of the government. Republic Act
No. 7924 does not empower the MMDA to enact ordinances, approve resolutions and appropriate funds for the
general welfare of the inhabitants of Manila. There is no syllable in the said act that grants MMDA police power.

It is an agency created for the purpose of laying down policies and coordinating with various national government
agencies, people’s organizations, non-governmental organizations and the private sector for the efficient and
expeditious delivery of basic services in the vast metropolitan area.

TATEL VS. MUNICIPALITY OF VIRAC


[207 SCRA 157; G.R. No. 40243; 11 Mar 1992]

Facts:

Petitioner Celestino Tatel owns a warehouse in barrio Sta. Elena, Municipality of Virac. Complaints were received
by the municipality concerning the disturbance caused by the operation of the abaca bailing machine inside
petitioner’s warehouse. A committee was then appointed by the municipal council, and it noted from its
investigation on the matter that an accidental fire within the warehouse of the petitioner created a danger to the
lives and properties of the people in the neighborhood. Resolution No. 29 was then passed by the Municipal
council declaring said warehouse as a public nuisance within a purview of Article 694 of the New Civil Code.
According to respondent municipal officials, petitioner’s warehouse was constructed in violation of Ordinance No.
13, series of 1952, prohibiting the construction of warehouses near a block of houses either in the poblacion or
barrios without maintaining the necessary distance of 200 meters from said block of houses to avoid loss of lives
and properties by accidental fire. On the other hand, petitioner contends that Ordinance No. 13 is unconstitutional.

Issue:

Whether or not petitioner’s warehouse is a nuisance within the meaning Article 694 of the Civil Code

Whether or not Ordinance No. 13, series of 1952 of the Municipality of Virac is unconstitutional and void.

Held:

Page 25
Section 1-C, SY ’06-‘07
San Beda College of Law – Alabang
Constitutional Law 2 Case Digests

The storage of abaca and copra in petitioner’s warehouse is a nuisance under the provisions of Article 694 of the
Civil Code. At the same time, Ordinance No. 13 was passed by the Municipal Council of Virac in the exercise of
its police power. It is valid because it meets the criteria for a valid municipal ordinance: 1) must not contravene the
Constitution or any statute, 2) must not be unfair or oppressive, 3) must not be partial or discriminatory, 4) must
not prohibit but may regulate trade, 5) must be general and consistent with public policy, and 6) must not be
unreasonable. The purpose of the said ordinance is to avoid the loss of property and life in case of fire which is
one of the primordial obligation of government. The lower court did not err in its decision.

Page 26
Section 1-C, SY ’06-‘07
San Beda College of Law – Alabang
Constitutional Law 2 Case Digests

THE POWER OF EMINENT DOMAIN

REPUBLIC VS. TAGLE


[299 SCRA 549; G.R. No. 129079; 2 Dec 1998]

Facts:

Private respondent Helena Z. Benitez is the registered owner of two (2) parcels of land located in Barangay
Salawag, Dasmariñas, Cavite containing an area of 483,331 square meters more or less.

The Philippine Government, through the Philippine Human Resources Development Center (PHRDC), negotiated
with the Japanese International Cooperation Agency (JICA) Survey Team on the technicalities of the
establishment of the ASEAN Human Resources Development Project in the Philippines. Among the five (5) main
programs of the proposed project was Program III (Construction Manpower Development) which involved the
establishment of a Construction Manpower Development Center (CMDC). PHRDC and private respondent
Helena Z. Benitez, signed a Memorandum of Agreement which provides, among others, that Benitez undertakes
to lease within the period of twenty (20) years and/or sell a portion of that property (which is no less than ten-
hectares) in favor of PHRDC which likewise agrees to lease within a period of twenty (20) years and/or buy said
property site.

The Philippine Women’s University (PWU) and Benitez granted a permit to PHRDC to occupy and use the land in
question and to undertake land development, electrical and road network installations and other related works
necessary to attain its objectives. Pursuant thereto, the CMDC took possession of the property and erected
buildings and other related facilities necessary for its operations. A deposit made by the plaintiff with the
Philippine National Bank (PNB) in the amount of P708,490.00 which is equivalent to the assessed value of the
property subject matter hereof based on defendant’s 1990 tax declaration, was made.

In view of the agreement on the sale of the land in question, PHRDC prepared a Deed of Absolute Sale with
Benitez, as vendor, and PHRDC and CMDC, as vendees, duly represented by then Undersecretary Gloria M.
Arroyo, for the signature of Benitez. Benitez in her own capacity did not sign the deed of absolute sale.
Failing to acquire the property involved through negotiated sale, petitioner, through the Department of Trade and
Industry, to which CMDC is attached, instituted a complaint for Eminent Domain, pursuant to the provisions of
Executive Order No. 1035, dated June 25, 1985.

A Motion for Issuance of Writ of Possession was granted by the court but quashed it subsequently.

Issue:

Whether or Not the respondent judge may quash a writ of possession on the ground that the expropriating
government agency is already occupying the property sought to be expropriated.

Held:

No. Under Section 7 of EO 1035, when the government or its authorized agent makes the required deposit, the
trial court has a ministerial duty to issue a writ of possession. The expropriation of real property does not include
mere physical entry or occupation of land. Although eminent domain usually involves a taking of title, there may
also be compensable taking of only some, not all, of the property interests in the bundle of rights that constitute
ownership.

In the instant case, it is manifest that the petitioner, in pursuit of an objective beneficial to public interest, seeks to
realize the same through its power of eminent domain. In exercising this power, petitioner intended to acquire not
only physical possession but also the legal right to possess and ultimately to own the subject property. Hence, its
mere physical entry and occupation of the property fall short of the taking of title, which includes all the rights that
may be exercised by an owner over the subject property.

CITY OF M ANILA VS. CHINESE COMMUNITY


[40 Phil 349; No. 14355; 31 Oct 1919]

Facts: The City of Manila, plaintiff herein, prayed for the expropriation of a portion private cemetery for the
conversion into an extension of Rizal Avenue. Plaintiff claims that it is necessary that such public improvement be
made in the said portion of the private cemetery and that the said lands are within their jurisdiction.

Page 27
Section 1-C, SY ’06-‘07
San Beda College of Law – Alabang
Constitutional Law 2 Case Digests

Defendants herein answered that the said expropriation was not necessary because other routes were available.
They further claimed that the expropriation of the cemetery would create irreparable loss and injury to them and to
all those persons owing and interested in the graves and monuments that would have to be destroyed.

The lower court ruled that the said public improvement was not necessary on the particular-strip of land in
question. Plaintiff herein assailed that they have the right to exercise the power of eminent domain and that the
courts have no right to inquire and determine the necessity of the expropriation. Thus, the same filed an appeal.

Issue:

Whether or not the courts may inquire into, and hear proof of the necessity of the expropriation.

Held:

The courts have the power of restricting the exercise of eminent domain to the actual reasonable necessities of
the case and for the purposes designated by the law. The moment the municipal corporation or entity attempts to
exercise the authority conferred, it must comply with the conditions accompanying the authority. The necessity for
conferring the authority upon a municipal corporation to exercise the right of eminent domain is admittedly within
the power of the legislature. But whether or not the municipal corporation or entity is exercising the right in a
particular case under the conditions imposed by the general authority, is a question that the courts have the right
to inquire to.

REPUBLIC VS. PLDT


[26 SCRA 320; G.R. No. L-18841; 27 Jan 1969]

Facts:

The plaintiff Republic of the Philippines is a political entity exercising government powers through one of its
branches, the Bureau of Telecommunication. Herein defendant, PLDT is a public service corporation holding a
franchise to install operates and maintains a telephone system. After its creation, the BOT set up its own
government telephone system by utilizing its own appropriations and other equipment and by renting trunk lines of
the PLDT to enable the govt offices to call privately. BOT entered into an agreement with the RCA
communications for joint overseas telephone service whereby BOT would convey overseas calls received by RCA
to local residents. PLDT complained to the BOT that it was a violation of the condition of their agreement since
the BOT had used trunk lines only for the use of government offices but even to serve private persons or the
general public in competition with the business of PLDT. Subsequently, the plaintiff commenced suit against
PLDT asking the court judgment be rendered ordering the PLDT to execute a contract with the plaintiff, through
the BOT for the use of the facilities of PLDT's telephone system throughout the country under such conditions as
the court may consider reasonable. The CFI rendered judgment stating that it could not compel PLDT to enter into
such agreement. Hence this petition.

Issue:

Whether or Not PLDT may be compelled to enter into such agreement.

Held:

Yes, the state, may, in the interest of national welfare transfer utilities to public ownership upon payment of just
compensation, there is no reason why the state ma not require a public utility to render services in the general
interest provided just compensation is paid.

PEOPLE VS. FAJARDO


[104 Phil 443; G.R. No. L-12172; 29 Aug 1958]

Facts:

The municipal council of baao, camarines sur stating among others that construction of a building, which will
destroy the view of the plaza, shall not be allowed and therefore be destroyed at the expense of the owner,
enacted an ordinance. Herein appellant filed a written request with the incumbent municipal mayor for a permit to
construct a building adjacent to their gasoline station on a parcel of land registered in Fajardo's name, located
along the national highway and separated from the public plaza by a creek. The request was denied, for the
reason among others that the proposed building would destroy the view or beauty of the public plaza. Defendants
reiterated their request for a building permit, but again the mayor turned down the request. Whereupon,

Page 28
Section 1-C, SY ’06-‘07
San Beda College of Law – Alabang
Constitutional Law 2 Case Digests

appellants proceeded with the construction of the building without a permit, because they needed a place of
residence very badly, their former house having been destroyed by a typhoon and hitherto they had been living on
leased property. Thereafter, defendants were charged in violation of the ordinance and subsequently convicted.
Hence this appeal.

Issue:

Whether or Not the ordinance is a valid exercise of police power.

Held:

No. It is not a valid exercise of police power. The ordinance is unreasonable and oppressive, in that it operates to
permanently deprive appellants of the right to use their own property; hence, it oversteps the bounds of police
power, and amounts to a taking of appellant’s property without just compensation. We do not overlook that the
modern tendency is to regard the beautification of neighborhoods as conducive to the comfort and happiness of
residents.

As the case now stands, every structure that may be erected on appellants' land, regardless of its own beauty,
stands condemned under the ordinance in question, because it would interfere with the view of the public plaza
from the highway. The appellants would, in effect, be constrained to let their land remain idle and unused for the
obvious purpose for which it is best suited, being urban in character. To legally achieve that result, the
municipality must give appellants just compensation and an opportunity to be heard.

CITY OF BAGUIO V. NAWASA


[106 Phil; G.R. No. L-12032; 31 Aug 1959]

Facts:

Plaintiff a municipal corporation filed a complaint against defendant a public corporation, created under Act.1383.
It contends that the said act does not include within its purview the Baguio Water Works system, assuming that it
does, is unconstitutional because it deprives the plaintiff ownership, control and operation of said water works
without just compensation and due process of law. The defendant filed a motion to dismiss ion the ground that it
is not a proper exercise of police power and eminent domain. The court denied the motion and ordered the
defendants to file an answer. The court holds that the water works system of Baguio belongs to private property
and cannot be expropriated without just compensation. Sec. 8 of R.A.1383 provides for the exchange of the
NAWASA assets for the value of the water works system of Baguio is unconstitutional for this is not just
compensation. Defendants motion for reconsideration was denied hence this appeal.

Issue:

Whether or Not there is a valid exercise of police power of eminent domain.

Held:

R.A. 1383 does not constitute a valid exercise of police power. The act does not confiscate, destroy or
appropriate property belonging to a municipal corporation. It merely directs that all water works belonging to cities,
municipalities and municipal districts in the Philippines to be transferred to the NAWASA. The purpose is placing
them under the control and supervision of an agency with a view to promoting their efficient management, but in
so doing does not confiscate them because it directs that they be paid with equal value of the assets of NAWASA.

The Baguio water works system is not like a public road, the park, street other public property held in trust by a
municipal corporation for the benefit of the public. But it is a property of a municipal corporation, water works
cannot be taken away except for public use and upon payment of just compensation. Judgment affirmed.

NATIONAL POWER CORP. VS. GUTIERREZ


[193 SCRA 1; G.R. No. 60077; 18 Jan 1991]

Facts:

Petitioner filed an action to acquire a right of way over the land of Respondents for the construction of
transmission lines. Petitioner was adjudged to pay the full market value of land traversed by the transmission
lines. Petitioner argued that it was only asking for a right of way.

Issue:

Page 29
Section 1-C, SY ’06-‘07
San Beda College of Law – Alabang
Constitutional Law 2 Case Digests

Whether or Not the acquisition of the right of way constitutes "taking" and such the case will be entitled just
compensation.

Held:

The acquisition of the right of way constitutes taking. It perpetually deprives Respondents of their proprietary
rights. No plant higher than three meters is allowed below the transmission lines. Because of high tension current
conveyed through the transmission lines, danger to life and limbs cannot be discounted. The owner of the
property is entitled to just compensation.

REPUBLIC VS. CASTELVI


[58 SCRA 336; G.R. No. L-20620; 15 Aug 1974]

Facts:

In 1947, the republic, through the Armed Forces of the Philippines (AFP), entered into a lease agreement with
Castelvi on a year-to-year basis. When Castelvi gave notice to terminate the lease in 1956, the AFP refused. She
then instituted an ejectment proceeding against the AFP. In 1959, however, the republic commenced the
expropriation proceedings for the land in question.

Issue:

Whether or Not the compensation should be determined as of 1947 or 1959.

Held:

The Supreme Court ruled that the “taking” should not be reckoned as of 1947, and that just compensation should
not be determined on the basis of the value of the property as of that year.

The requisites for taking are: 1) the expropriator must enter a private property, 2) the entry must be for more than
a momentary period, 3) it must be under warrant or color of authorities, 4) the property must be devoted for public
use or otherwise informally appropriated or injuriously affected, and 5) the utilization of the property for public use
must be such a way as to oust the owner and deprive him of beneficial enjoyment of the property. Under Sec. 4
Rule 67 of the Rules of Court, “just compensation” is to be determined as of the date of the filing of the complaint.
The Supreme Court has ruled that when the taking of the property sought to be expropriated coincides with the
commencement of the expropriation proceedings, or takes place subsequent to the filing of the complaint for
eminent domain, the just compensation should be determined as of the date of the filing of the complaint. In the
instant case, it is undisputed that the Republic was placed in possession of the Castelvi property, by authority of
court, on August 10, 1959. The “taking” of the Castelvi property for the purposes of determining the just
compensation to be paid must, therefore, be reckoned as of June 26, 1959 when the complaint for eminent
domain was filed. There is no basis to the contention of the Republic that a lease on a year-to-year basis can give
rise to permanent right to occupy since by express provision a lease made for a determinate time, as was the
lease of Castelvi land in the instant case, ceases upon the day fixed, without need of a demand (Art. 1669, New
Civil Code). The Supreme Court, however, did not apply Art. 1250 of the New Civil Code for the adjustment of the
peso rate in times of extraordinary inflation or deflation because in eminent domain cases the obligation to pay
arises from law independent of contract.

EPZA VS. DULAY


[148 SCRA 305; G.R. No. L-59603; 29 Apr 1987]

Facts:

The four parcels of land which are the subject of this case is where the Mactan Export Processing Zone Authority
in Cebu (EPZA) is to be constructed. Private respondent San Antonio Development Corporation (San Antonio, for
brevity), in which these lands are registered under, claimed that the lands were expropriated to the government
without them reaching the agreement as to the compensation. Respondent Judge Dulay then issued an order for
the appointment of the commissioners to determine the just compensation. It was later found out that the payment
of the government to San Antonio would be P15 per square meter, which was objected to by the latter contending
that under PD 1533, the basis of just compensation shall be fair and according to the fair market value declared
by the owner of the property sought to be expropriated, or by the assessor, whichever is lower. Such objection
and the subsequent Motion for Reconsideration were denied and hearing was set for the reception of the
commissioner’s report. EPZA then filed this petition for certiorari and mandamus enjoining the respondent from
further hearing the case.

Issue:

Page 30
Section 1-C, SY ’06-‘07
San Beda College of Law – Alabang
Constitutional Law 2 Case Digests

Whether or Not the exclusive and mandatory mode of determining just compensation in PD 1533 is
unconstitutional.

Held:

The Supreme Court ruled that the mode of determination of just compensation in PD 1533 is unconstitutional.

The method of ascertaining just compensation constitutes impermissible encroachment to judicial prerogatives. It
tends to render the courts inutile in a matter in which under the Constitution is reserved to it for financial
determination. The valuation in the decree may only serve as guiding principle or one of the factors in determining
just compensation, but it may not substitute the court’s own judgment as to what amount should be awarded and
how to arrive at such amount. The determination of just compensation is a judicial function. The executive
department or the legislature may make the initial determination but when a party claims a violation of the
guarantee in the Bill of Rights that the private party may not be taken for public use without just compensation, no
statute, decree, or executive order can mandate that its own determination shall prevail over the court’s findings.
Much less can the courts be precluded from looking into the justness of the decreed compensation.

AMIGABLE VS. CUENCA


[43 SCRA 360; G.R. No. L-26400; 29 Feb. 1972]

Facts:

Victoria Amigable is the registered owner of a particular lot. At the back of her Transfer Certificate of Title (1924),
there was no annotation in favor of the government of any right or interest in the property. Without prior
expropriation or negotiated sale, the government used a portion of the lot for the construction of the Mango and
Gorordo Avenues. On 1958, Amigable’s counsel wrote the President of the Philippines, requesting payment of the
portion of the said lot. It was disallowed by the Auditor General in his 9 th Endorsement. Petitioner then filed in the
court a quo a complaint against the Republic of the Philippines and Nicolas Cuenca, in his capacity as
Commissioner of Public Highways for the recovery of ownership and possession of the lot. According to the
defendants, the action was premature because it was not filed first at the Office of the Auditor General. According
to them, the right of action for the recovery of any amount had already prescribed, that the Government had not
given its consent to be sued, and that plaintiff had no cause of action against the defendants.

Issue:

Whether or Not, under the facts of the case, appellant may properly sue the government.

Held:

In the case of Ministerio v. Court of First Instance of Cebu, it was held that when the government takes away
property from a private landowner for public use without going through the legal process of expropriation or
negotiated sale, the aggrieved party may properly maintain a suit against the government without violating the
doctrine of governmental immunity from suit without its consent. In the case at bar, since no annotation in favor of
the government appears at the back of the certificate of title and plaintiff has not executed any deed of
conveyance of any portion of the lot to the government, then she remains the owner of the lot. She could then
bring an action to recover possession of the land anytime, because possession is one of the attributes of
ownership. However, since such action is not feasible at this time since the lot has been used for other purposes,
the only relief left is for the government to make due compensation—price or value of the lot at the time of the
taking.

PHILIPPINE PRESS INSTITUTE VS. COMELEC


[244 SCRA 272; G.R. No. 119694; 22 May 1995]

Facts:

Respondent Comelec promulgated Resolution No. 2772 directing newspapers to provide free Comelec space of
not less than one-half page for the common use of political parties and candidates. The Comelec space shall be
allocated by the Commission, free of charge, among all candidates to enable them to make known their
qualifications, their stand on public Issue and their platforms of government. The Comelec space shall also be
used by the Commission for dissemination of vital election information.

Page 31
Section 1-C, SY ’06-‘07
San Beda College of Law – Alabang
Constitutional Law 2 Case Digests

Petitioner Philippine Press Institute, Inc. (PPI), a non-profit organization of newspaper and magazine publishers,
asks the Supreme Court to declare Comelec Resolution No. 2772 unconstitutional and void on the ground that it
violates the prohibition imposed by the Constitution upon the government against the taking of private property for
public use without just compensation. On behalf of the respondent Comelec, the Solicitor General claimed that
the Resolution is a permissible exercise of the power of supervision (police power) of the Comelec over the
information operations of print media enterprises during the election period to safeguard and ensure a fair,
impartial and credible election.

Issue:

Whether or not Comelec Resolution No. 2772 is unconstitutional.

Held:

The Supreme Court declared the Resolution as unconstitutional. It held that to compel print media companies to
donate “Comelec space” amounts to “taking” of private personal property without payment of the just
compensation required in expropriation cases. Moreover, the element of necessity for the taking has not been
established by respondent Comelec, considering that the newspapers were not unwilling to sell advertising space.
The taking of private property for public use is authorized by the constitution, but not without payment of just
compensation. Also Resolution No. 2772 does not constitute a valid exercise of the police power of the state. In
the case at bench, there is no showing of existence of a national emergency to take private property of
newspaper or magazine publishers.

REYES VS. NATIONAL HOUSING AUTHORITY


[395 SCRA 494; GR NO. 147511; 20 JAN 2003]

Facts:

Respondent National Housing Authority (NHA) filed complaints for the expropriation of sugarcane lands belonging
to the petitioners. The stated public purpose of the expropriation was the expansion of the Dasmariñas
Resettlement Project to accommodate the squatters who were relocated from the Metropolitan Manila area. The
trial court rendered judgment ordering the expropriation of these lots and the payment of just compensation. The
Supreme Court affirmed the judgment of the lower court.

A few years later, petitioners contended that respondent NHA violated the stated public purpose for the expansion
of the Dasmariñas Resettlement Project when it failed to relocate the squatters from the Metro Manila area, as
borne out by the ocular inspection conducted by the trial court which showed that most of the expropriated
properties remain unoccupied. Petitioners likewise question the public nature of the use by respondent NHA when
it entered into a contract for the construction of low cost housing units, which is allegedly different from the stated
public purpose in the expropriation proceedings. Hence, it is claimed that respondent NHA has forfeited its rights
and interests by virtue of the expropriation judgment and the expropriated properties should now be returned to
herein petitioners.

Issue:

Whether or not the judgment of expropriation was forfeited in the light of the failure of respondent NHA to use the
expropriated property for the intended purpose but for a totally different purpose.

Held:

The Supreme Court held in favor of the respondent NHA. Accordingly, petitioners cannot insist on a restrictive
view of the eminent domain provision of the Constitution by contending that the contract for low cost housing is a
deviation from the stated public use. It is now settled doctrine that the concept of public use is no longer limited to
traditional purposes. The term "public use" has now been held to be synonymous with "public interest," "public
benefit," "public welfare," and "public convenience." Thus, whatever may be beneficially employed for the general
welfare satisfies the requirement of public use."

In addition, the expropriation of private land for slum clearance and urban development is for a public purpose
even if the developed area is later sold to private homeowners, commercials firms, entertainment and service
companies, and other private concerns. Moreover, the Constitution itself allows the State to undertake, for the
common good and in cooperation with the private sector, a continuing program of urban land reform and housing
which will make at affordable cost decent housing and basic services to underprivileged and homeless citizens in

Page 32
Section 1-C, SY ’06-‘07
San Beda College of Law – Alabang
Constitutional Law 2 Case Digests

urban centers and resettlement areas. The expropriation of private property for the purpose of socialized housing
for the marginalized sector is in furtherance of social justice.

MUNICIPALITY OF PARAÑAQUE VS. VM REALTY CORPORATION


[292 SCRA 676; G. R. NO. 127820; 20 JUL 1998]

Facts:

Petitioner sought to exercise its power of eminent domain based on a resolution by the municipal council.
Petitioner cites a previous case wherein a resolution gave authority to exercise eminent domain. Petitioner also
relies on the Implementing Rules, which provides that a resolution authorizes a Local Government Unit to
exercise eminent domain.

Issue:

Whether or Not an LGU can exercise its power of eminent domain pursuant to a resolution by its law-making
body.

Held:

Under Section 19, of the present Local Government Code (RA 7160), it is stated as the first requisite that LGUs
can exercise its power of eminent domain if there is an ordinance enacted by its legislative body enabling the
municipal chief executive. A resolution is not an ordinance, the former is only an opinion of a law-making body,
the latter is a law. The case cited by Petitioner involves BP 337, which was the previous Local Government
Code, which is obviously no longer in effect. RA 7160 prevails over the Implementing Rules, the former being the
law itself and the latter only an administrative rule which cannot amend the former.

ASLP VS. SEC. OF AGRARIAN REFORM


[175 SCRA 343; G.R. NO. 78742; 14 JUL 1989]

Facts:

Several petitions are the root of the case:

e. A petition alleging the constitutionality of PD No. 27, EO 228 and 229 and RA 6657. Subjects of the
petition are a 9-hectare and 5 hectare Riceland worked by four tenants. Tenants were declared full
owners by EO 228 as qualified farmers under PD 27. The petitioners now contend that President
Aquino usurped the legislature’s power.
f. A petition by landowners and sugarplanters in Victoria’s Mill Negros Occidental against Proclamation
131 and EO 229. Proclamation 131 is the creation of Agrarian Reform Fund with initial fund of
P50Billion.
g. A petition by owners of land which was placed by the DAR under the coverage of Operation Land
Transfer.
h. A petition invoking the right of retention under PD 27 to owners of rice and corn lands not exceeding
seven hectares.

Issue:

Whether or Not the aforementioned EO’s, PD, and RA were constitutional.

Held:

The promulgation of PD 27 by President Marcos was valid in exercise of Police power and eminent domain.

The power of President Aquino to promulgate Proc. 131 and EO 228 and 229 was authorized under Sec. 6 of the
Transitory Provisions of the 1987 Constitution. Therefore it is a valid exercise of Police Power and Eminent
Domain.

RA 6657 is likewise valid. The carrying out of the regulation under CARP becomes necessary to deprive owners
of whatever lands they may own in excess of the maximum area allowed, there is definitely a taking under the
power of eminent domain for which payment of just compensation is imperative. The taking contemplated is not a

Page 33
Section 1-C, SY ’06-‘07
San Beda College of Law – Alabang
Constitutional Law 2 Case Digests

mere limitation of the use of the land. What is required is the surrender of the title and the physical possession of
said excess and all beneficial rights accruing to the owner in favour of the farmer.

A statute may be sustained under the police power only if there is concurrence of the lawful subject and the
method.

Subject and purpose of the Agrarian Reform Law is valid, however what is to be determined is the method
employed to achieve it.

ESLABAN VS. ONORIO


[360 SCRA 230; G.R. NO. 146062; 28 JUN 2001]

Facts:

Clarita Vda. De Onorio is the owner of the land in Barangay M. Roxas, Sto. Nino, South Cotabato. Such land is
the subject for the construction of an irrigation canal of the National Irrigation Administration (NIA). Mr. Santiago
Eslaban Jr. is the project manager of NIA. The parties agreed to the construction of the canal provided that the
government will pay for the area that has been taken. A right-of-way agreement was entered into by the parties in
which respondent was paid the amount of P4, 180.00 as right of way damages. Subsequently, respondent
executed an Affidavit of Waiver of Rights and Fees which waives her rights for the damage to the crops due to
construction of the right of way. After which, respondent demands that petitioner pay P111, 299.55 for taking her
property but the petitioner refused. Petitioner states that the government had not consented to be sued and that
the respondent is not entitled for compensation by virtue of the homestead patent under CA no. 141. The RTC
held that the NIA should pay respondent the amount of P107, 517.60 as just compensation for the 24,660 sq
meters that have been used for the construction of the canal. The Court of Appeals also affirmed the decision of
the RTC.

Issue:

Whether or Not the CA erred in affirming the decision of the RTC.

Held:

The CA is correct in affirming the decision of the RTC but modifications shall be made regarding the value of the
just compensation. The following are the points to be considered in arriving in this decision.

First, Rule 7 par 5 of the Rule of Civil Procedure provides that the certification against forum shopping should only
be executed by the plaintiff or the principal. The petition for review was filed by Mr. Eslaban jr. while the
verification or certification were signed by Mr. Cesar Gonzales, an administrator of the agency. Neither of the two
has the authority to sign such certificate for they are not the plaintiff or principal. Such case is a sufficient ground
for dismissing this petition.

Second, PD NO. 1529 provides that the owner is required to recognize in favor of the government the easement
of a “public highway, way, private way established by law, or any government canal where the certificate of title
does not state that the boundaries thereof have been pre-determined. In the case at bar, the irrigation canal was
constructed on Oct 1981 after the property had been registered in May of 1976. In this case, prior expropriation
proceedings must be filed and just compensation shall be paid to the owner before the land could be taken for
public use.

Third, In this case, just compensation is defined as not only the correct amount to be paid but the reasonable time
for the Government to pay the owner. The CA erred in this point by stating that the market value (just
compensation) of the land is determined in the filing of the complaint in 1991.The determination of such value
should be from the time of its taking by the NIA in 1981.

Lastly, the petitioner cannot argue that the Affidavit of waiver of rights and fees executed by the respondent
pertains to the payment of the value of the land therefore exempting NIA to pay the value of the land taken. Such
waiver pertains only to the crops and improvements that were damage due to the construction of the right-of-way
not the value of the land.

Wherefore, decision of CA affirmed with modification regarding the just compensation in the amount of P16,
047.61 per hectare.

KNECHT VS. COURT OF APPEALS


[290 SCRA 223; G.R. NO. 108015, 20 MAY 1998]

Page 34
Section 1-C, SY ’06-‘07
San Beda College of Law – Alabang
Constitutional Law 2 Case Digests

Facts:

The instant case is an unending sequel to several suits commenced almost twenty years ago involving a parcel of
land located at the corner of the south end of EDSA and F.B. Harrison in Pasay City. The land was owned by
petitioners Cristina de Knecht and her son, Rene Knecht. On the land, the Knechts constructed eight houses,
leased out the seven and occupied one of them as their residence. In 1979, the government filed for the
expropriation of Knechts’ property. The government wanted to use the land for the completion of the Manila Flood
Control and Drainage Project and the extension of the EDSA towards Roxas Boulevard. In 1982, the City
Treasurer of Pasay discovered that the Knechts failed to pay real estate taxes on the property from 1980 to 1982.
As a consequence of this deficiency, the City Treasurer sold the property at public auction for the same amount of
their deficiency taxes. The highest bidders were respondent Spouses Anastacio and Felisa Babiera (the
Babieras) and respondent Spouses Alejandro and Flor Sangalang (the Sangalangs). Subsequently, Sangalang
and Babiera sold the land to respondent Salem Investment Corporation. On February 17, 1983, the Batasang
Pambansa passed B.P. Blg. 340 authorizing the national government to expropriate certain properties in Pasay
City for the EDSA Extension. The property of the Knechts was part of those expropriated under B.P. Blg. 340. The
government gave out just compensation for the lands expropriated under B.P. Blg. 340. Salem was included and
received partial payment. Seven of the eight houses of the Knechts were demolished and the government took
possession of the portion of land on which the houses stood. Since the Knechts refused to vacate their one
remaining house, Salem filed a case against them for unlawful detainer. As defense, the Knechts claimed
ownership of the land and building. The Municipal Trial Court however ordered the Knechts' ejectment thus their
residence was demolished.

The Knechts continuously claimed ownership of the property and allege that they must be given just
compensation.

Issue:

Whether or not Knechts are the lawful owners of the land at subject.

Held:

The Supreme Court held that the Knechts were not the owners anymore of the said land. The Knechts' right to the
land had been foreclosed after they failed to redeem it one year after the sale at public auction. Since the petitions
questioning the order of dismissal were likewise dismissed by the Court of Appeals and this Court, the order of
dismissal became final and res judicata on the issue of ownership of the land. Petitioners contended that they did
not receive notice of their tax delinquency. Neither did they receive notice of the auction sale. However, this
question has been previously raised in the cases which have been already set aside. The court is not a trier of
facts. Res judicata has already set it. The Knechts therefore are not the lawful owners of the land and are not any
longer accountable for just compensation given by the government.

Note: Res judicata is a ground for dismissal of an action. It is a rule that precludes parties from relitigating Issue
actually litigated and determined by a prior and final judgment. It pervades every well-regulated system of
jurisprudence, and is based upon two grounds embodied in various maxims of the common law — one, public
policy and necessity, that there should be a limit to litigation; and another, the individual should not be vexed twice
for the same cause. When a right of fact has been judicially tried and determined by a court of competent
jurisdiction, or an opportunity for such trial has been given, the judgment of the court, so long as it remains
unreversed, should be conclusive upon the parties and those in privity with them in law or estate. To follow a
contrary doctrine would subject the public peace and quiet to the will and neglect of individuals and prefer the
gratification of the litigious disposition of the parties to the preservation of the public tranquility.

Res judicata applies when: (1) the former judgment or order is final; (2) the judgment or order is one on the
merits; (3) it was rendered by a court having jurisdiction over the subject matter and the parties; (4) there is
between the first and second actions, identity of parties, of subject matter and of cause of action.

REPUBLIC VS. KER


[383 SCRA 584; G.R. NO. 136171, 2 JULY 2002]

Facts:

Petitioner filed before the Regional Trial Court of Davao City a petition for expropriation of portions of two parcels
of land owned by respondent. Petitioner needed the parcels of land for the widening of the road component of
J.P. Laurel-Buhangin Interchange in Davao City. The Regional trial court rendered decision of a fair just
compensation for defendant Ker Corporation. However, it was challenged by Petitioner Republic of the
Philippines, represented by the Department of Public Works and Highways alleging that just compensation for site
must be reduced. Petitioner alleged that when the petition for expropriation was filed, the tax declaration of the
property indicated its assessed value at a lower price.

Page 35
Section 1-C, SY ’06-‘07
San Beda College of Law – Alabang
Constitutional Law 2 Case Digests

Issue:

Whether or not respondent Ker Company was given a decision for fair just compensation.

Held:

The Supreme Court held that the valuation for the lot Sites are excessive and unreasonable. Just compensation
cannot be measured by the assessed value of the property as stated in the tax declaration and schedule of
market values. For the purpose of appraisal, the fair market value of the property is taken into account and such
value refers to the highest price in terms of money which a property will bring if exposed for sale in the public
market.

In computing just compensation for expropriation proceedings, it is the value of the land at the time of the taking
or at the time of the filing of the complaint not at the time of the rendition of judgment which should be taken into
consideration. 4 Section 4, Rule 67 of the 1997 Rules of Civil Procedure provides that just compensation is to be
determined as of the date of the taking or the filing of the complaint whichever came first. On this matter, the
appellate court is correct in disregarding petitioner's claim.

M ANOSCA VS. COURT OF APPEALS


[252 SCRA 412; G.R. NO. 106440, 29 JAN. 1996]

Facts:

The National Historical Institute declared the parcel of land owned by Petitioners as a national historical landmark,
because it was the site of the birth of Felix Manalo, the founder of Iglesia ni Cristo. The Republic of the
Philippines filed an action to appropriate the land. Petitioners argued that the expropriation was not for a public
purpose.

Issue:

Whether or Not the taking or exercise of eminent domain may be granted.

Held:

Public use should not be restricted to the traditional uses. The taking is for a public use because of the
contribution of Felix Manalo to the culture and history of the Philippines.

Page 36
Section 1-C, SY ’06-‘07
San Beda College of Law – Alabang
Constitutional Law 2 Case Digests

THE POWER OF TAXATION

PASCUAL VS. SEC. OF PUBLIC WORKS


[110 PHIL 331; G.R. NO.L-10405; 29 DEC 1960]

Facts:

Petitioner, the governor of the Province of Rizal, filed an action for declaratory relief with injunction on the ground
that RA 920, Act appropriating funds for public works, providing P85,000 for the construction, reconstruction,
repair, extension and improvement of Pasig feeder road terminals, were nothing but projected and planned
subdivision roads within Antonio Subdivision. Antonio Subdivision is owned by the respondent, Jose Zulueta, a
member of the Senate of the Philippines. Respondent offered to donate the said feeder roads to the municipality
of Pasig and the offer was accepted by the council, subject to a condition that the donor would submit plan of the
roads and an agreement to change the names of two of the street. However, the donation was not executed,
which prompted Zuleta to write a letter to the district engineer calling attention the approval of RA 920. The district
engineer, on the other hand, did not endorse the letter that inasmuch the feeder roads in question were private
property at the time of passage and approval of RA 920, the appropriation for the construction was illegal and
therefore, void ab initio. Petitioner, prayed for RA 920 be declared null and void and the alleged deed of donation
be declared unconstitutional. Lower court dismissed the case and dissolved the writ of preliminary injunction.

Issue:

Whether or Not the deed of donation and the appropriation of funds stipulated in RA 920 are constitutional.

Held:

The ruling case law rules that the legislature is without power to appropriate public revenue for anything but public
purpose. The taxing power must be exercised for public purposes only and the money raised by taxation can be
expended only for public purposes and not for the advantage of private individuals.

In the case at bar, the legality of the appropriation of the feeder roads depend upon whether the said roads were
public or private property when the bill was passed by congress or when it became effective. The land which was
owned by Zulueta, the appropriation sought a private purpose and hence, null and void. The donation did not cure
the nullity of the appropriation; therefore a judicial nullification of a said donation need not precede the declaration
of unconstitutionality of the said appropriation.

The decision appealed from is reversed.

PUNSALAN VS. MUNICIPAL BOARD OF M ANILA


[95 PHIL 46; NO.L-4817; 26 MAY 1954]

Facts:

Petitioners, who are professionals in the city, assail Ordinance No. 3398 together with the law authorizing it
(Section 18 of the Revised Charter of the City of Manila). The ordinance imposes a municipal occupation tax on
persons exercising various professions in the city and penalizes non-payment of the same. The law authorizing
said ordinance empowers the Municipal Board of the city to impose a municipal occupation tax on persons
engaged in various professions. Petitioners, having already paid their occupation tax under section 201 of the
National Internal Revenue Code, paid the tax under protest as imposed by Ordinance No. 3398. The lower court
declared the ordinance invalid and affirmed the validity of the law authorizing it.

Issue:

Whether or Not the ordinance and law authorizing it constitute class legislation, and authorize what amounts to
double taxation.

Held:

The Legislature may, in its discretion, select what occupations shall be taxed, and in its discretion may tax all, or
select classes of occupation for taxation, and leave others untaxed. It is not for the courts to judge which cities or
municipalities should be empowered to impose occupation taxes aside from that imposed by the National
Government. That matter is within the domain of political departments. The argument against double taxation

Page 37
Section 1-C, SY ’06-‘07
San Beda College of Law – Alabang
Constitutional Law 2 Case Digests

may not be invoked if one tax is imposed by the state and the other is imposed by the city. It is widely recognized
that there is nothing inherently terrible in the requirement that taxes be exacted with respect to the same
occupation by both the state and the political subdivisions thereof. Judgment of the lower court is reversed with
regards to the ordinance and affirmed as to the law authorizing it.

OSMEÑA VS. ORBOS


[220 SCRA 703; G.R. NO. 99886; 31 MAR 1993]

Facts:

On October 10, 1984, Pres. Marcos issued P.D. 1956 creating a Special Account in the General Fund, designated
as the Oil Price Stabilization Fund (OPSF). The OPSF was designed to reimburse oil companies for cost
increases in crude oil and imported petroleum products resulting from exchange rate adjustments and from
increases in the world market prices of crude oil.

Subsequently, the OPSF was reclassified into a "trust liability account," in virtue of E.O. 1024, and ordered
released from the National Treasury to the Ministry of Energy.

Pres. Aquino, amended P.D. 1956. She promulgated Executive Order No. 137 on February 27, 1987, expanding
the grounds for reimbursement to oil companies for possible cost underrecovery incurred as a result of the
reduction of domestic prices of petroleum products, the amount of the underrecovery being left for determination
by the Ministry of Finance.

The petition avers that the creation of the trust fund violates
29(3), Article VI of the Constitution, reading as follows:

(3) All money collected on any tax levied for a special purpose shall be treated as a special fund
and paid out for such purposes only. If the purpose for which a special fund was created has
been fulfilled or abandoned, the balance, if any, shall be transferred to the general funds of the
Government.

The petitioner argues that "the monies collected pursuant to . . P.D. 1956, as amended, must be treated as a
'SPECIAL FUND,' not as a 'trust account' or a 'trust fund,' and that "if a special tax is collected for a specific
purpose, the revenue generated therefrom shall 'be treated as a special fund' to be used only for the purpose
indicated, and not channeled to another government objective." Petitioner further points out that since "a 'special
fund' consists of monies collected through the taxing power of a State, such amounts belong to the State,
although the use thereof is limited to the special purpose/objective for which it was created."

He also contends that the "delegation of legislative authority" to the ERB violates 28 (2). Article VI of the
Constitution, viz.:

(2) The Congress may, by law, authorize the President to fix, within specified limits, and subject
to such limitations and restrictions as it may impose, tariff rates, import and export quotas,
tonnage and wharfage dues, and other duties or imposts within the framework of the national
development program of the Government;

and, inasmuch as the delegation relates to the exercise of the power of taxation, "the limits, limitations and
restrictions must be quantitative, that is, the law must not only specify how to tax, who (shall) be taxed (and) what
the tax is for, but also impose a specific limit on how much to tax." 12

Issue:

Whether or Not the invalidity of the "TRUST ACCOUNT" in the books of account of the Ministry of Energy (now,
the Office of Energy Affairs), created pursuant to § 8, paragraph 1, of P.D. No. 1956, as amended, "said creation
of a trust fund being contrary to Section 29 (3), Article VI of the Constitution.

Whether or Not the unconstitutionality of 8, paragraph 1 (c) of P.D. No. 1956, as amended by Executive Order No.
137, for "being an undue and invalid delegation of legislative power to the Energy Regulatory Board.

Held:

The OPSF is a "Trust Account" which was established "for the purpose of minimizing the frequent price changes
brought about by exchange rate adjustment and/or changes in world market prices of crude oil and imported

Page 38
Section 1-C, SY ’06-‘07
San Beda College of Law – Alabang
Constitutional Law 2 Case Digests

petroleum products." Under P.D. No. 1956, as amended by Executive Order No. 137 dated 27 February 1987,
this Trust Account may be funded from any of the following sources:

a) Any increase in the tax collection from ad valorem tax or customs duty imposed on petroleum products
subject to tax under this Decree arising from exchange rate adjustment, as may be determined by the
Minister of Finance in consultation with the Board of Energy;

b) Any increase in the tax collection as a result of the lifting of tax exemptions of government
corporations, as may be determined by the Minister of Finance in consultation with the Board of Energy;

c) Any additional amount to be imposed on petroleum products to augment the resources of the Fund
through an appropriate Order that may be issued by the Board of Energy requiring payment of persons or
companies engaged in the business of importing, manufacturing and/or marketing petroleum products;

d) Any resulting peso cost differentials in case the actual peso costs paid by oil companies in the
importation of crude oil and petroleum products is less than the peso costs computed using the reference
foreign exchange rate as fixed by the Board of Energy.

Hence, it seems clear that while the funds collected may be referred to as taxes, they are exacted in the exercise
of the police power of the State. Moreover, that the OPSF is a special fund is plain from the special treatment
given it by E.O. 137. It is segregated from the general fund; and while it is placed in what the law refers to as a
"trust liability account," the fund nonetheless remains subject to the scrutiny and review of the COA. The Court is
satisfied that these measures comply with the constitutional description of a "special fund." Indeed, the practice is
not without precedent.

With regard to the alleged undue delegation of legislative power, the Court finds that the provision conferring the
authority upon the ERB to impose additional amounts on petroleum products provides a sufficient standard by
which the authority must be exercised. In addition to the general policy of the law to protect the local consumer by
stabilizing and subsidizing domestic pump rates, § 8(c) of P.D. 1956 expressly authorizes the ERB to impose
additional amounts to augment the resources of the Fund.

What petitioner would wish is the fixing of some definite, quantitative restriction, or "a specific limit on how much
to tax." The Court is cited to this requirement by the petitioner on the premise that what is involved here is the
power of taxation; but as already discussed, this is not the case. What is here involved is not so much the power
of taxation as police power. Although the provision authorizing the ERB to impose additional amounts could be
construed to refer to the power of taxation, it cannot be overlooked that the overriding consideration is to enable
the delegate to act with expediency in carrying out the objectives of the law which are embraced by the police
power of the State.

The interplay and constant fluctuation of the various factors involved in the determination of the price of oil and
petroleum products, and the frequently shifting need to either augment or exhaust the Fund, do not conveniently
permit the setting of fixed or rigid parameters in the law as proposed by the petitioner. To do so would render the
ERB unable to respond effectively so as to mitigate or avoid the undesirable consequences of such fluidity. As
such, the standard as it is expressed suffices to guide the delegate in the exercise of the delegated power, taking
account of the circumstances under which it is to be exercised.

LLADOC VS. COMMISSIONER OF INTERNAL REVENUE


[14 SCRA 292; NO.L-19201; 16 JUN 1965]

Facts:

Sometime in 1957, M.B. Estate Inc., of Bacolod City, donated 10,000.00 pesos in cash to Fr. Crispin Ruiz, the
parish priest of Victorias, Negros Occidental, and predecessor of Fr. Lladoc, for the construction of a new Catholic
church in the locality. The donated amount was spent for such purpose.

On March 3, 1958, the donor M.B. Estate filed the donor's gift tax return. Under date of April 29, 1960.
Commissioner of Internal Revenue issued an assessment for the donee's gift tax against the Catholic Parish of
Victorias of which petitioner was the parish priest.

Issue:

Whether or not the imposition of gift tax despite the fact the Fr. Lladoc was not the Parish priest at the time of
donation, Catholic Parish priest of Victorias did not have juridical personality as the constitutional exemption for
religious purpose is valid.

Held:

Page 39
Section 1-C, SY ’06-‘07
San Beda College of Law – Alabang
Constitutional Law 2 Case Digests

Yes, imposition of the gift tax was valid, under Section 22(3) Article VI of the Constitution contemplates
exemption only from payment of taxes assessed on such properties as Property taxes contra distinguished from
Excise taxes The imposition of the gift tax on the property used for religious purpose is not a violation of the
Constitution. A gift tax is not a property by way of gift inter vivos.

The head of the Diocese and not the parish priest is the real party in interest in the imposition of the donee's tax
on the property donated to the church for religious purpose.

CASSANOVAS VS. HORD


[8 Phil 125; No. 3473; 22 Mar 1907]

Facts:

The Spanish Govt. by virtue of a royal decree granted the plaintiff certain mines. The plaintiff is now the owner of
those mines. The Collector of Internal Revenue imposed tax on the properties, contending that they were valid
perfected mine concessions and it falls within the provisions of sec.134 of Act No. 1189 known as Internal
Revenue Act. The plaintiff paid under protest. He brought an action against the defendant Collector of Internal
Revenue to recover the sum of Php. 9, 600 paid by him as taxes. Judgment was rendered in favor of the
defendant, so the plaintiff appealed.

Issue:

Whether or Not Sec. 164 is void or valid.

Held:

The deed constituted a contract between the Spanish Government and the plaintiff. The obligation of which
contract was impaired by the enactment of sec. 134 of the Internal Revenue Law infringing sec. 5 of the Act of
Congress which provides that “no law impairing the obligation of contracts shall be enacted”. Sec. 134 of the
Internal Revenue Law of 1904 is void because it impairs the obligation of contracts contained in the concessions
of mine made by the Spanish Government. Judgment reversed.

Page 40
Section 1-C, SY ’06-‘07
San Beda College of Law – Alabang
Constitutional Law 2 Case Digests

THE BILL
OF RIGHTS

Page 41
Section 1-C, SY ’06-‘07
San Beda College of Law – Alabang
Constitutional Law 2 Case Digests

DUE PROCESS OF LAW

Art 3, Sec. 1. “No person shall be deprived of life, liberty, or property without due process of law…”

ERMITA-M ALATE HOTEL AND MOTEL OPERATORS ASSO. VS. M AYOR OF M ANILA
[20 SCRA 849; G.R. NO.L-24693; 31 JULY 1967]

Facts:

Petitioners Ermita-Malate Hotel and Motel Operators Association with one of its members, Hotel del Mar Inc., and
Go Chiu, the president and general manager of the second petitioner, filed a petition for prohibition against
Ordinance No. 4760 against the respondent Mayor of the City of Manila who was sued in his capacity as such
charged with the general power and duty to enforce ordinances of the City of Manila and to give the necessary
orders for the execution and enforcement of such ordinances. It was alleged that the petitioner non-stock
corporation is dedicated to the promotion and protection of the interest of its eighteen members operating hotels
and motels, characterized as legitimate businesses duly licensed by both national and city authorities and
regularly paying taxes. It was alleged that on June 13, 1963, the Municipal Board of the City of Manila enacted
Ordinance No. 4760, approved on June 14, 1963 by the then acting City Mayor, Vice-Mayor Herminio Astorga.
After which the alleged grievances against the ordinance were set forth in detail. There was the assertion of its
being beyond the powers of the Municipal Board of the City of Manila to enact insofar as it regulate motels, on the
ground that in the revised charter of the City of Manila or in any other law, no reference is made to motels. it also
being provided that the premises and facilities of such hotels, motels and lodging houses would be open for
inspection either by the City Mayor, or the Chief of Police, or their duly authorized representatives. The lower
court on July 6, 1963 issued a writ of preliminary injunction ordering respondent Mayor to refrain from enforcing
said Ordinance No. 4760 from and after July 8, 1963.

Issue:

Whether or Not Ordinance No. 4760 of the City of Manila is unconstitutional, therefore, null and void.

Held:

A decent regard for constitutional doctrines of a fundamental character ought to have admonished the lower court
against such a sweeping condemnation of the challenged ordinance. Its decision cannot be allowed to stand,
consistently with what has been the accepted standards of constitutional adjudication, in both procedural and
substantive aspects.

Primarily what calls for a reversal of such a decision is the absence of any evidence to offset the presumption of
validity that attaches to a challenged statute or ordinance. As was expressed categorically by Justice Malcolm:
"The presumption is all in favor of validity x x x . The action of the elected representatives of the people cannot be
lightly set aside. The councilors must, in the very nature of things, be familiar with the necessities of their
particular municipality and with all the facts and circumstances which surround the subject and necessitate action.
The local legislative body, by enacting the ordinance, has in effect given notice that the regulations are essential
to the well being of the people x x x . The Judiciary should not lightly set aside legislative action when there is not
a clear invasion of personal or property rights under the guise of police regulation.

It admits of no doubt therefore that there being a presumption of validity, the necessity for evidence to rebut it is
unavoidable, unless the statute or ordinance is void on its face which is not the case here. The principle has been
nowhere better expressed than in the leading case of O'Gorman & Young v. Hartford Fire Insurance Co. where
the American Supreme Court through Justice Brandeis tersely and succinctly summed up the matter thus: The
statute here questioned deals with a subject clearly within the scope of the police power. We are asked to declare
it void on the ground that the specific method of regulation prescribed is unreasonable and hence deprives the
plaintiff of due process of law. As underlying questions of fact may condition the constitutionality of legislation of
this character, the resumption of constitutionality must prevail in the absence of some factual foundation of record
for overthrowing the statute." No such factual foundation being laid in the present case, the lower court deciding
the matter on the pleadings and the stipulation of facts, the presumption of validity must prevail and the judgment
against the ordinance set aside.

VILLEGAS VS. HIU CHIONG


[86 SCRA 270; NO.L-29646; 10 NOV 1978]

Facts:

Page 42
Section 1-C, SY ’06-‘07
San Beda College of Law – Alabang
Constitutional Law 2 Case Digests

The controverted Ordinance no. 6537 was passed by the Municipal Board of Manila on February 22, 1968 and
signed by Mayor Villegas. It is an ordinance making it unlawful for any person not a citizen of the Philippines to be
employed in any place of employment or to be engaged in any kind of trade business or occupation within the
city of Manila without securing an employment permit from the Mayor of Manila and for other purposes.

Hiu Chiong Tsai Pao Ho, who was employed in Manila filed a petition praying for the writ of preliminary injunction
and restraining order to stop the enforcement of said ordinance.

Issue:

Whether or Not Ordinance no.6537 violates the due process and equal protection clauses of the Constitution.

Held:

It is a revenue measure. The city ordinance which imposes a fee of 50.00 pesos to enable aliens generally to be
employed in the city of Manila is not only for the purpose of regulation.

While it is true that the first part which requires the alien to secure an employment permit from the Mayor
involves the exercise of discretion and judgment in processing and approval or disapproval of application is
regulatory in character, the second part which requires the payment of a sum of 50.00 pesos is not a
regulatory but a revenue measure.

Ordinance no. 6537 is void and unconstitutional. This is tantamount to denial of the basic human right of the
people in the Philippines to engaged in a means of livelihood. While it is true that the Philippines as a state is not
obliged to admit aliens within it's territory, once an alien is admitted he cannot be deprived of life without due
process of law. This guarantee includes the means of livelihood. Also it does not lay down any standard to guide
the City Mayor in the issuance or denial of an alien employment permit fee.

NAMIL VS. COMELEC


[414 SCRA 553; G.R. NO. 150540; 28 OCT 2003]

Facts:

On May 20, 2001, the Municipal Board of Canvassers of Palimbang, Sultan Kudarat proclaimed the petitioners as
winning candidates for their Sangguniang Bayan. The following day, herein private respondents were proclaimed
winners as well. Private respondents claimed that they should be recognized as the winners, and not the
petitioners. Upon receipt of such letter, the Commissioner-in-charge for Region XII asked the Law Department,
the Regional Election Registrar and the Provincial Elections Supervisor to submit their reports on the matter. All
of them found the second proclamation valid. Hence, the COMELEC issued a Resolution ordering the immediate
installation of the private respondents as the newly elected members of the Sangguniang Bayan, even though
petitioners herein have already taken their oath and have assumed office. Petitioners contend that such
Resolution is null and void because they were not accorded due notice and hearing, hence constituting a violation
of the due process principle.

Issue:

Whether or Not due the COMELEC has the power to suspend a proclamation or the effects thereof without notice
and hearing.

Held:

No. The COMELEC is without power to partially or totally annul a proclamation or suspend the effects of a
proclamation without notice and hearing. The proclamation on May 20, 2001 enjoys the presumption of regularity
and validity since no contest or protest was even filed assailing the same. The petitioners cannot be removed
from office without due process of law. Due process in quasi-judicial proceedings before the COMELEC requires
due notice and hearing. Furthermore, the proclamation of a winning candidate cannot be annulled if he has not
been notified of any motion to set aside his proclamation. Hence, as ruled in Fariñas vs. COMELEC, Reyes vs.
COMELEC and Gallardo vs. COMELEC, the COMELEC is without power to partially or totally annul a
proclamation or suspend the effects of a proclamation without notice and hearing.
ICHONG VS. HERNANDEZ
[101 PHIL 1155; L-7995; 31 MAY 1957]

Facts:

Republic Act 1180 or commonly known as “An Act to Regulate the Retail Business” was passed. The said law
provides for a prohibition against foreigners as well as corporations owned by foreigners from engaging from retail

Page 43
Section 1-C, SY ’06-‘07
San Beda College of Law – Alabang
Constitutional Law 2 Case Digests

trade in our country. This was protested by the petitioner in this case. According to him, the said law violates the
international and treaty of the Philippines therefore it is unconstitutional. Specifically, the Treaty of Amity between
the Philippines and China was violated according to him.

Issue:

Whether or Not Republic Act 1180 is a valid exercise of police power.

Held:

According to the Court, RA 1180 is a valid exercise of police power. It was also then provided that police power
can not be bargained away through the medium of a treaty or a contract. The Court also provided that RA 1180
was enacted to remedy a real and actual danger to national economy posed by alien dominance and control. If
ever the law infringes upon the said treaty, the latter is always subject to qualification or amendment by a
subsequent law and the same may never curtain or restrict the scope of the police power of the state.

PHIL. PHOSPHATE FERTILIZER CORP. VS. TORRES


[231 SCRA 335; G.R. NO.98050; 17 MAR 1994]

Facts:

Philphos Movement for Progress, Inc. (PMPI for brevity), filed with the Department of Labor and Employment a
petition for certification election among the supervisory employees of petitioner, alleging that as a supervisory
union duly registered with the Department of Labor and Employment it was seeking to represent the supervisory
employees of Philippine Phosphate Fertilizer Corporation. Mediator-Arbiter Rodolfo S. Milado issued an order
directing the holding of a certification election among the supervisory employees of petitioner, excluding therefrom
the superintendents and the professional and technical employees. However, the PMPI filed an amended petition
with the Mediator-Arbiter wherein it sought to represent not only the supervisory employees of petitioner but also
its professional/technical and confidential employees. The parties therein agreed to submit their respective
position papers and to consider the amended petition submitted for decision on the basis thereof and related
documents. Mediator-Arbiter Milado issued an order granting the petition and directing the holding of a
certification election among the "supervisory, professional (engineers, analysts, mechanics, accountants, nurses,
midwives, etc.), technical, and confidential employees. PHILPHOS appealed the order to the Secretary of Labor
and Employment who rendered a decision through Undersecretary Bienvenido Laguesma dismissing the appeal.
PHILPHOS moved for reconsideration but the same was denied; hence, the instant petition alleging denial of due
process on the part of the DOLE to which the mediator-arbiter was under.

Issue:

Whether or Not there was denial of due process.

Held:

There was no denial of due process. The essence of due process is simply an opportunity to be heard or, as
applied to administrative proceedings, an opportunity to explain one's side or an opportunity to seek a
reconsideration of the action or ruling complained of petitioner PHILPHOS agreed to file its position paper with the
Mediator-Arbiter and to consider the case submitted for decision on the basis of the position papers filed by the
parties, there was sufficient compliance with the requirement of due process, as petitioner was afforded
reasonable opportunity to present its side. Moreover, petitioner could have, if it so desired, insisted on a hearing
to confront and examine the witnesses of the other party. But it did not; instead it opted to submit its position
paper with the Mediator-Arbiter. Besides, petitioner had all the opportunity to ventilate its arguments in its appeal
to the Secretary of Labor.

RUBI VS. PROVINCIAL BOARD OF MINDORO


[39 PHIL 660; G.R. NO. 14078; 7 MAR 1919]

Facts:

This is an application for habeas corpus in favor of Rubi and other Manguianes of the Province of Mindoro.

The provincial board of Mindoro adopted resolution No. 25 which states that “provincial governor of any province
in which non-Christian inhabitants (uncivilized tribes) are found is authorized, when such a course is deemed
necessary in the interest of law and order, to direct such inhabitants to take up their habitation on sites on
unoccupied public lands to be selected by him and approved by the provincial board”. It is resolved that under
section 2077 of the Administrative Code, 800 hectares of public land in the sitio of Tigbao on Naujan Lake be

Page 44
Section 1-C, SY ’06-‘07
San Beda College of Law – Alabang
Constitutional Law 2 Case Digests

selected as a site for the permanent settlement of Mangyanes in Mindoro. Further, Mangyans may only solicit
homesteads on this reservation providing that said homestead applications are previously recommended by the
provincial governor.

Thereafter, the provincial governor of Mindoro issued executive order No. 2, which says that the provincial
governor has selected a site in the sitio of Tigbao on Naujan Lake for the permanent settlement of Mangyanes in
Mindoro. In that case, pursuant to Section 2145 of the Revised Administrative Code, all the Mangyans in the
townships of Naujan and Pola and the Mangyans east of the Baco River including those in the districts of
Dulangan and Rubi's place in Calapan, were ordered to take up their habitation on the site of Tigbao, Naujan
Lake. Also, that any Mangyan who shall refuse to comply with this order shall upon conviction be imprisoned not
exceed in sixty days, in accordance with section 2759 of the revised Administrative Code.

Said resolution of the provincial board of Mindoro were claimed as necessary measures for the protection of the
Mangyanes of Mindoro as well as the protection of public forests in which they roam, and to introduce civilized
customs among them.

It appeared that Rubi and those living in his rancheria have not fixed their dwelling within the reservation of
Tigbao and are liable to be punished.

It is alleged that the Manguianes are being illegally deprived of their liberty by the provincial officials of that
province. Rubi and his companions are said to be held on the reservation established at Tigbao, Mindoro, against
their will, and one Dabalos is said to be held under the custody of the provincial sheriff in the prison at Calapan for
having run away form the reservation.

Issue:

Whether or Not Section 2145 of the Administrative Code deprive a person of his liberty without due process of
law.

Whether or Not Section 2145 of the Administrative Code of 1917 is constitutional.

Held:

The Court held that section 2145 of the Administrative Code does not deprive a person of his liberty without due
process of law and does not deny to him the equal protection of the laws, and that confinement in reservations in
accordance with said section does not constitute slavery and involuntary servitude. The Court is further of the
opinion that section 2145 of the Administrative Code is a legitimate exertion of the police power, somewhat
analogous to the Indian policy of the United States. Section 2145 of the Administrative Code of 1917 is
constitutional.

The preamble of the resolution of the provincial board of Mindoro which set apart the Tigbao reservation, it will be
read, assigned as reasons fort the action, the following: (1) The failure of former attempts for the advancement of
the non-Christian people of the province; and (2) the only successfully method for educating the Manguianes was
to oblige them to live in a permanent settlement. The Solicitor-General adds the following; (3) The protection of
the Manguianes; (4) the protection of the public forests in which they roam; (5) the necessity of introducing
civilized customs among the Manguianes.

Considered purely as an exercise of the police power, the courts cannot fairly say that the Legislature has
exceeded its rightful authority. It is, indeed, an unusual exercise of that power. But a great malady requires an
equally drastic remedy. One cannot hold that the liberty of the citizen is unduly interfered without when the degree
of civilization of the Manguianes is considered. They are restrained for their own good and the general good of the
Philippines. Nor can one say that due process of law has not been followed.

None of the rights of the citizen can be taken away except by due process of law. To constitute "due process of
law," as has been often held, a judicial proceeding is not always necessary. In some instances, even a hearing
and notice are not requisite a rule which is especially true where much must be left to the discretion of the
administrative officers in applying a law to particular cases.

The idea of the provision in question is to unify the people of the Philippines so that they may approach the
highest conception of nationality. The public policy of the Government of the Philippine Islands is shaped with a
view to benefit the Filipino people as a whole. The Manguianes, in order to fulfill this governmental policy, must be
confined for a time, as we have said, for their own good and the good of the country.

Therefore, petitioners are not unlawfully imprisoned or restrained of their liberty. Habeas corpus can, therefore,
not issue.

Page 45
Section 1-C, SY ’06-‘07
San Beda College of Law – Alabang
Constitutional Law 2 Case Digests

KWONG SING VS. CITY OF MANILA


[41 PHIL 103; G.R. NO. 15972; 11 OCT 1920]

Facts:

Kwong Sing, in his own behalf and of other Chinese laundrymen who has general and the same interest, filed a
complaint for a preliminary injunction. The Plaintiffs also questioned the validity of enforcing Ordinance No. 532 by
the city of Manila. Ordinance No. 532 requires that the receipt be in duplicate in English and Spanish duly signed
showing the kind and number of articles delivered by laundries and dyeing and cleaning establishments. The
permanent injunction was denied by the trial court. The appellants claim is that Ordinance No. 532 savors of
class legislation; putting in mind that they are Chinese nationals. It unjustly discriminates between persons in
similar circumstances; and that it constitutes an arbitrary infringement of property rights. They also contest that
the enforcement of the legislation is an act beyond the scope of their police power. In view of the foregoing, this
is an appeal with the Supreme Court.

Issue:

Whether or Not the enforcement of Ordinance no, 532 is an act beyond the scope of police power

Whether or not the enforcement of the same is a class legislation that infringes property rights.

Held:

Reasonable restraints of a lawful business for such purposes are permissible under the police power. The police
power of the City of Manila to enact Ordinance No. 532 is based on Section 2444, paragraphs (l) and (ee) of the
Administrative Code, as amended by Act No. 2744, authorizes the municipal board of the city of Manila, with the
approval of the mayor of the city:

(l) To regulate and fix the amount of the license fees for the following: xxxx xxxxxlaundries xxxx.
(ee) To enact all ordinances it may deem necessary and proper for the sanitation and safety, the
furtherance of the prosperity, and the promotion of the morality, peace, good order, comfort, convenience,
and general welfare of the city and its inhabitants.

The court held that the obvious purpose of Ordinance No. 532 was to avoid disputes between laundrymen and
their patrons and to protect customers of laundries who are not able to decipher Chinese characters from being
defrauded. (Considering that in the year 1920s, people of Manila are more familiar with Spanish and maybe
English.)

In whether the ordinance is class legislation, the court held that the ordinance invades no fundamental right, and
impairs no personal privilege. Under the guise of police regulation, an attempt is not made to violate personal
property rights. The ordinance is neither discriminatory nor unreasonable in its operation. It applies to all public
laundries without distinction, whether they belong to Americans, Filipinos, Chinese, or any other nationality. All,
without exception, and each every one of them without distinction, must comply with the ordinance. The obvious
objection for the implementation of the ordinance is based in sec2444 (ee) of the Administrative Code. Although,
an additional burden will be imposed on the business and occupation affected by the ordinance such as that of
the appellant by learning even a few words in Spanish or English, but mostly Arabic numbers in order to properly
issue a receipt, it seems that the same burdens are cast upon the them. Yet, even if private rights of person or
property are subjected to restraint, and even if loss will result to individuals from the enforcement of the
ordinance, this is not sufficient ground for failing to uphold the power of the legislative body. The very foundation
of the police power is the control of private interests for the public welfare.

Finding that the ordinance is valid, judgment is affirmed, and the petition for a preliminary injunction is denied,
with costs against the appellants.

YU CONG ENG VS. TRINIDAD


[47 PHIL 385; G.R. NO. 20479; 6 FEB 1925]

Facts:

The petitioner, Yu Cong Eng, was charged by information in the court of first instance of Manila, with a violation of
Act 2972, which provides that (Section 1) it shall be unlawful for any person, company, or partnership or
corporation engaged in commerce, industry or any other activity for the purpose of profit in the Philippine Islands,
in accordance with existing law, to keep its account books in any language other than English, Spanish or any
local dialect. He was arrested, his books were seized, and the trial was about to proceed, when he and the other
petitioner, Co Liam, on their own behalf, and on behalf of all the other Chinese merchants in the Philippines, filed

Page 46
Section 1-C, SY ’06-‘07
San Beda College of Law – Alabang
Constitutional Law 2 Case Digests

the petition against the fiscal, or prosecuting attorney of Manila, and the collector of internal revenue engaged in
the prosecution, and against the judge presiding.

Issue:

Whether or Not Act 2972 is unconstitutional.

Held:

Yes. The Philippine government may make every reasonable requirement of its taxpayers to keep proper records
of their business transactions in English or Spanish or Filipino dialect by which an adequate measure of what is
due from them in meeting the cost of government can be had. But we are clearly of opinion that it is not within the
police power of the Philippine Legislature, because it would be oppressive and arbitrary, to prohibit all Chinese
merchants from maintaining a set of books in the Chinese language, and in the Chinese characters, and thus
prevent them from keeping advised of the status of their business and directing its conduct.

ANIAG VS. COMELEC


[237 SCRA 194; G.R. NO. 104961; 7 OCT 1994]

Facts:

In preparation for the synchronized national and local elections, the COMELEC issued Resolution No. 2323, “Gun
Ban”, promulgating rules and regulations on bearing, carrying and transporting of firearm or other deadly weapons
on security personnel or bodyguards, on bearing arms by members of security agencies or police organizations,
and organization or maintenance of reaction forces during the election period. COMELEC also issued Resolution
No. 2327 providing for the summary disqualification of candidates engaged in gunrunning, using and transporting
of firearms, organizing special strike forces, and establishing spot checkpoints. Pursuant to the “Gun Ban”, Mr.
Serrapio Taccad, Sergeant at Arms of the House of Representatives, wrote petitioner for the return of the two
firearms issued to him by the House of Representatives. Petitioner then instructed his driver, Arellano, to pick up
the firearms from petitioner’s house and return them to Congress. The PNP set up a checkpoint. When the car
driven by Arellano approached the checkpoint, the PNP searched the car and found the firearms. Arellano was
apprehended and detained. He then explained the order of petitioner. Petitioner also explained that Arellano was
only complying with the firearms ban, and that he was not a security officer or a bodyguard. Later, COMELEC
issued Resolution No.92-0829 directing the filing of information against petitioner and Arellano for violation of the
Omnibus Election Code, and for petitioner to show cause why he should not be disqualified from running for an
elective position. Petitioner then questions the constitutionality of Resolution No. 2327. He argues that
“gunrunning, using or transporting firearms or similar weapons” and other acts mentioned in the resolution are not
within the provisions of the Omnibus Election Code. Thus, according to petitioner, Resolution No. 2327 is
unconstitutional. The issue on the disqualification of petitioner from running in the elections was rendered moot
when he lost his bid for a seat in Congress in the elections.

Issue:

Whether or Not petitioner can be validly prosecuted for instructing his driver to return the firearms issued to him
on the basis of the evidence gathered from the warrant less search of his car

Held:

A valid search must be authorized by a search warrant issued by an appropriate authority. However, a
warrantless search is not violative of the Constitution for as long as the vehicle is neither searched nor its
occupants subjected to a body search, and the inspection of the vehicle is merely limited to a visual search. In the
case at bar, the guns were not tucked in Arellano’s waist nor placed within his reach, as they were neatly packed
in gun cases and placed inside a bag at the back of the car. Given these circumstances, the PNP could not have
thoroughly searched the car lawfully as well as the package without violating the constitutional injunction. Absent
any justifying circumstance specifically pointing to the culpability of petitioner and Arellano, the search could not
have been valid. Consequently, the firearms obtained from the warrantless search cannot be admitted for any
purpose in any proceeding. It was also shown in the facts that the PNP had not informed the public of the purpose
of setting up the checkpoint. Petitioner was also not among those charged by the PNP with violation of the
Omnibus Election Code. He was not informed by the City Prosecutor that he was a respondent in the preliminary
investigation. Such constituted a violation of his right to due process. Hence, it cannot be contended that
petitioner was fully given the opportunity to meet the accusation against him as he was not informed that he was
himself a respondent in the case. Thus, the warrantless search conducted by the PNP is declared illegal and the

Page 47
Section 1-C, SY ’06-‘07
San Beda College of Law – Alabang
Constitutional Law 2 Case Digests

firearms seized during the search cannot be used as evidence in any proceeding against the petitioner.
Resolution No. 92-0829 is unconstitutional, and therefore, set aside.

JAVIER VS. COMELEC


[144 SCRA 194; G.R. NOS. L-68379-81; 22 SEPT 1986]

Facts:

The petitioner and the private respondent were candidates in Antique for the Batasang Pambansa in the May
1984 elections. The former appeared to enjoy more popular support but the latter had the advantage of being the
nominee of the KBL with all its perquisites of power. On May 13, 1984, the eve of the elections, the bitter contest
between the two came to a head when several followers of the petitioner were ambushed and killed, allegedly by
the latter's men. Seven suspects, including respondent Pacificador, are now facing trial for these murders. Owing
to what he claimed were attempts to railroad the private respondent's proclamation, the petitioner went to the
Commission on Elections to question the canvass of the election returns. His complaints were dismissed and the
private respondent was proclaimed winner by the Second Division of the said body. The petitioner thereupon
came to this Court, arguing that the proclamation was void because made only by a division and not by the
Commission on Elections en banc as required by the Constitution. Meanwhile, on the strength of his
proclamation, the private respondent took his oath as a member of the Batasang Pambansa.

Issue:

Whether or Not the Second Division of the Commission on Elections authorized to promulgate its decision of July
23, 1984, proclaiming the private respondent the winner in the election.

Held:

This Court has repeatedly and consistently demanded "the cold neutrality of an impartial judge" as the
indispensable imperative of due process. To bolster that requirement, we have held that the judge must not only
be impartial but must also appear to be impartial as an added assurance to the parties that his decision will be
just. The litigants are entitled to no less than that. They should be sure that when their rights are violated they
can go to a judge who shall give them justice. They must trust the judge, otherwise they will not go to him at all.
They must believe in his sense of fairness, otherwise they will not seek his judgment. Without such confidence,
there would be no point in invoking his action for the justice they expect.

Due process is intended to insure that confidence by requiring compliance with what Justice Frankfurter calls the
rudiments of fair play. Fair play cans for equal justice. There cannot be equal justice where a suitor approaches a
court already committed to the other party and with a judgment already made and waiting only to be formalized
after the litigants shall have undergone the charade of a formal hearing. Judicial (and also extra-judicial)
proceedings are not orchestrated plays in which the parties are supposed to make the motions and reach the
denouement according to a prepared script. There is no writer to foreordain the ending. The judge will reach his
conclusions only after all the evidence is in and all the arguments are filed, on the basis of the established facts
and the pertinent law.

YNOT VS. IAC


[148 SCRA 659; G.R. NO. 74457; 20 MAR 1987]

Facts:

Executive Order No. 626-A prohibited the transportation of carabaos and carabeef from one province to another.
The carabaos of petitioner were confiscated for violation of Executive Order No 626-A while he was transporting
them from Masbate to Iloilo. Petitioner challenged the constitutionality of Executive Order No. 626-A. The
government argued that Executive Order No. 626-A was issued in the exercise of police power to conserve the
carabaos that were still fit for farm work or breeding.

Issue:

Whether or Not EO No. 626-A is a violation of Substantive Due Process.

Held:

The challenged measure is an invalid exercise of police power, because it is not reasonably necessary for the
purpose of the law and is unduly oppressive. It is difficult to see how prohibiting the transfer of carabaos from one

Page 48
Section 1-C, SY ’06-‘07
San Beda College of Law – Alabang
Constitutional Law 2 Case Digests

province to another can prevent their indiscriminate killing. Retaining the carabaos in one province will not prevent
their slaughter there. Prohibiting the transfer of carabeef, after the slaughter of the carabaos, will not prevent the
slaughter either.

PHILCOMSAT VS. ALCUAZ


[180 SCRA 218; G.R. NO.84818; 18 DEC 1989]

Facts:

Herein petitioner is engaged in providing for services involving telecommunications. Charging rates for certain
specified lines that were reduced by order of herein respondent Jose Alcuaz Commissioner of the National
Telecommunications Commission. The rates were ordered to be reduced by fifteen percent (15%) due to
Executive Order No. 546 which granted the NTC the power to fix rates. Said order was issued without prior notice
and hearing.

Issue:

Whether or Not E.O. 546 is unconstitutional.

Held:

Yes. Respondents admitted that the application of a policy like the fixing of rates as exercised by administrative
bodies is quasi-judicial rather than quasi-legislative. But respondent’s contention that notice and hearing are not
required since the assailed order is merely incidental to the entire proceedings and temporary in nature is
erroneous. Section 16(c) of the Public Service Act, providing for the proceedings of the Commission, upon notice
and hearing, dictates that a Commission has power to fix rates, upon proper notice and hearing, and, if not
subject to the exceptions, limitations or saving provisions.

It is thus clear that with regard to rate-fixing, respondent has no authority to make such order without first giving
petitioner a hearing, whether the order be temporary or permanent, and it is immaterial whether the same is made
upon a complaint, a summary investigation, or upon the commission's own motion as in the present case.

WHEREFORE, the writ prayed for is GRANTED and the order of respondents is hereby SET ASIDE.

EASTERN BROADCASTING CORP (DYRE) V. DANS JR.


[137 SCRA 628; L-59329; 19 JUL 1985]

Facts:

A petition was filed to reopen the Radio Station DYRE. DYRE was “summarily closed” on grounds of national
security. The radio station was allegedly used to incite people to sedition. Petitioner, DYRE contends that they
were denied due process. There was no hearing to establish factual evidence for the closure. Furthermore, the
closure of the radio station violates freedom of expression. Before the court could even promulgate a decision
upon the Issue raised, Petitioner, through its president Mr. Rene Espina, filed a motion to withdraw the petition.
The rights of the station were sold to a new owner, Manuel Pastrana; who is no longer interested in pursuing the
case. Despite the case becoming moot and academic, (because there are no longer interested parties, thus the
dismissal of the case) the Supreme Court still finds that there is need to pass a “RESOLUTION” for the guidance
of inferior courts and administrative tribunals in matters as this case.

Issue:

Whether or not due process was exercised in the case of DYRE.

Whether or not the closure of DYRE is a violation of the Constitutional Right of Freedom of Expression.

Held:

The court finds that the closure of the Radio Station in 1980 as null and void. The absence of a hearing is a
violation of Constitutional Rights. The primary requirements in administrative proceedings are laid down in the
case of Ang Tibay v. Court of Industrial Relation (69 Phil.635). The Ang Tibay Doctrine should be followed before
any broadcast station may be closed. The Ang Tibay Doctrine provides the following requirements:

(1) The right to hearing, includes the right to present one’s case and submit evidence presented.
(2) The tribunal must consider the evidence presented
(3) The decision must have something to support itself.

Page 49
Section 1-C, SY ’06-‘07
San Beda College of Law – Alabang
Constitutional Law 2 Case Digests

(4) Evidence must be substantial (reasonable evidence that is adequate to support conclusion)
(5) Decision must be based on the evidence presented at hearing
(6) The tribunal body must act on its own independent consideration of law and facts and not
simply accept subordinate’s views
(7) Court must render decision in such a manner that the proceeding can know the various
issued involved and reasons for decisions rendered.

The court stresses that while there is no controlling and precise definition of Due Process, it gives an unavoidable
standard that government actions must conform in order that deprivation of life, liberty and property is valid.

The closure of the radio station is like wise a violation of the constitutional right of freedom of speech and
expression. The court stresses that all forms of media, whether print or broadcast are entitled to this
constitutional right. Although the government still has the right to be protected against broadcasts which incite the
listeners to violently overthrow it. The test for the limitation of freedom of expression is the “clear and present
danger” rule. If in the circumstances that the media is used in such nature as to create this danger that will bring
in such evils, then the law has the right to prevent it. However, Radio and television may not be used to organize
a rebellion or signal a start of widespread uprising. The freedom to comment on public affairs is essential to the
vitality of a representative democracy. The people continues to have the right to be informed on public affairs and
broadcast media continues to have the pervasive influence to the people being the most accessible form of
media. Therefore, broadcast stations deserve the the special protection given to all forms of media by the due
process and freedom of expression clauses of the Constitution.

ANG TIBAY VS. COURT OF INDUSTRIAL RELATIONS (CIR)


[69 PHIL 635; G.R. NO. 46496; 27 FEB 1940]

Facts:

There was agreement between Ang Tibay and the National Labor Union, Inc (NLU). The NLU alleged that the
supposed lack of leather material claimed by Toribio Teodoro was but a scheme adopted to systematically
discharge all the members of the NLU, from work. And this averment is desired to be proved by the petitioner with
the records of the Bureau of Customs and Books of Accounts of native dealers in leather. That National Worker's
Brotherhood Union of Ang Tibay is a company or employer union dominated by Toribio Teodoro, which was
alleged by the NLU as an illegal one. The CIR, decided the case and elevated it to the Supreme Court, but a
motion for new trial was raised by the NLU. But the Ang Tibay filed a motion for opposing the said motion.

Issue:

Whether or Not, the motion for new trial is meritorious to be granted.

Held:

To begin with the issue before us is to realize the functions of the CIR. The CIR is a special court whose functions
are specifically stated in the law of its creation which is the Commonwealth Act No. 103). It is more an
administrative board than a part of the integrated judicial system of the nation. It is not intended to be a mere
receptive organ of the government. Unlike a court of justice which is essentially passive, acting only when its
jurisdiction is invoked and deciding only cases that are presented to it by the parties litigant, the function of the
CIR, as will appear from perusal of its organic law is more active, affirmative and dynamic. It not only exercises
judicial or quasi-judicial functions in the determination of disputes between employers and employees but its
functions are far more comprehensive and extensive. It has jurisdiction over the entire Philippines, to consider,
investigate, decide, and settle any question, matter controversy or disputes arising between, and/ or affecting
employers and employees or laborers, and landlords and tenants or farm-laborers, and regulates the relations
between them, subject to, and in accordance with, the provisions of CA 103.

As laid down in the case of Goseco v. CIR, the SC had the occasion to point out that the CIR is not narrowly
constrained by technical rules of procedure, and equity and substantial merits of the case, without regard to
technicalities or legal forms and shall not be bound by any technical rules of legal evidence but may inform its
mind in such manner as it may deem just and equitable.

The fact, however, that the CIR may be said to be free from rigidity of certain procedural requirements does not
mean that it can in justiciable cases coming before it, entirely ignore or disregard the fundamental and essential
requirements of due process in trials and investigations of an administrative character. There cardinal primary
rights which must be respected even in proceedings of this character:

Page 50
Section 1-C, SY ’06-‘07
San Beda College of Law – Alabang
Constitutional Law 2 Case Digests

(1) the right to a hearing, which includes the right to present one's cause and submit evidence in
support thereof;
(2) The tribunal must consider the evidence presented;
(3) The decision must have something to support itself;
(4) The evidence must be substantial;
(5) The decision must be based on the evidence presented at the hearing; or at least contained
in the record and disclosed to the parties affected;
(6) The tribunal or body or any of its judges must act on its own independent consideration of the
law and facts of the controversy, and not simply accept the views of a subordinate;
(7) The Board or body should, in all controversial questions, render its decision in such manner
that the parties to the proceeding can know the various Issue involved, and the reason for the
decision rendered.

The failure to grasp the fundamental issue involved is not entirely attributable to the parties adversely affected by
the result. Accordingly, the motion for a new trial should be, and the same is hereby granted, and the entire record
of this case shall be remanded to the CIR, with instruction that it reopen the case receive all such evidence as
may be relevant, and otherwise proceed in accordance with the requirements set forth. So ordered.

ATENEO DE MANILA UNIVERSITY VS. HON. JUDGE IGNACIO CAPULONG


[222 SCRA 644; G.R. 99327; 27 MAY 1993]

Facts:

Leonardo H. Villa, a first year law student of Petitioner University, died of serious physical injuries at Chinese
General Hospital after the initiation rites of Aquila Legis. Bienvenido Marquez was also hospitalized at the Capitol
Medical Center for acute renal failure occasioned by the serious physical injuries inflicted upon him on the same
occasion. Petitioner Dean Cynthia del Castillo created a Joint Administration-Faculty-Student Investigating
Committee which was tasked to investigate and submit a report within 72 hours on the circumstances surrounding
the death of Lennie Villa. Said notice also required respondent students to submit their written statements within
twenty-four (24) hours from receipt. Although respondent students received a copy of the written notice, they
failed to file a reply. In the meantime, they were placed on preventive suspension. The Joint Administration-
Faculty-Student Investigating Committee, after receiving the written statements and hearing the testimonies of
several witness, found a prima facie case against respondent students for violation of Rule 3 of the Law School
Catalogue entitled "Discipline." Respondent students were then required to file their written answers to the formal
charge. Petitioner Dean created a Disciplinary Board to hear the charges against respondent students. The Board
found respondent students guilty of violating Rule No. 3 of the Ateneo Law School Rules on Discipline which
prohibits participation in hazing activities. However, in view of the lack of unanimity among the members of the
Board on the penalty of dismissal, the Board left the imposition of the penalty to the University Administration.
Accordingly, Fr. Bernas imposed the penalty of dismissal on all respondent students. Respondent students filed
with RTC Makati a TRO since they are currently enrolled. This was granted. A TRO was also issued enjoining
petitioners from dismissing the respondents. A day after the expiration of the temporary restraining order, Dean
del Castillo created a Special Board to investigate the charges of hazing against respondent students Abas and
Mendoza. This was requested to be stricken out by the respondents and argued that the creation of the Special
Board was totally unrelated to the original petition which alleged lack of due process. This was granted and
reinstatement of the students was ordered.

Issue:

Was there denial of due process against the respondent students.

Held:

There was no denial of due process, more particularly procedural due process. Dean of the Ateneo Law School,
notified and required respondent students to submit their written statement on the incident. Instead of filing a
reply, respondent students requested through their counsel, copies of the charges. The nature and cause of the
accusation were adequately spelled out in petitioners' notices. Present is the twin elements of notice and hearing.

Respondent students argue that petitioners are not in a position to file the instant petition under Rule 65
considering that they failed to file a motion for reconsideration first before the trial court, thereby by passing the
latter and the Court of Appeals. It is accepted legal doctrine that an exception to the doctrine of exhaustion of
remedies is when the case involves a question of law, as in this case, where the issue is whether or not
respondent students have been afforded procedural due process prior to their dismissal from Petitioner
University.

Minimum standards to be satisfied in the imposition of disciplinary sanctions in academic institutions, such as
petitioner university herein, thus:

Page 51
Section 1-C, SY ’06-‘07
San Beda College of Law – Alabang
Constitutional Law 2 Case Digests

(1) the students must be informed in writing of the nature and cause of any accusation against
them;
(2) that they shall have the right to answer the charges against them with the assistance of
counsel, if desired:
(3) they shall be informed of the evidence against them
(4) they shall have the right to adduce evidence in their own behalf; and
(5) the evidence must be duly considered by the investigating committee or official designated by
the school authorities to hear and decide the case.

US GOVERNMENT VS. JUDGE PURUNGAN


[389 SCRA 623; G.R. NO. 148571, 24 SEPT 2002]

Facts:

The United States of America, pursuant to the existing RP-US extradition treaty, requested the extradition of Mark
B. Jimenez. Upon receipt of the request, the secretary of foreign affairs (SFA) transmitted them to the secretary of
justice (SOJ) for appropriate action. In such event, the RTC held that Jimenez shell be deprived of the right to
notice and hearing during the evaluation stage of the extradition process. Thereafter the US government, through
DOJ, filed Petition for Extradition and Jimenez’s immediate arrest, to avoid flight. Before the RTC could render its
decision, Jimenez filed an "Urgent Manifestation/Ex-Parte Motion," praying that his application for an arrest
warrant be set for hearing, which was granted. During which, the lower court issued its questioned July 3, 2001
Order, directing the issuance of a warrant for his arrest and fixing bail for his temporary liberty at one million
pesos in cash. After Jimenez had surrendered his passport and posted the required cash bond, he was granted
provisional liberty via the challenged Order dated July 4, 2001. Thus, Petition prays for the lifting of the bail Order,
the cancellation of the bond, and the taking of Jimenez into legal custody.

Issue:

Whether or not Jimenez is entitled to notice and hearing before a warrant for his arrest can be issued.

Whether or not he is entitled to bail and to provisional liberty while the extradition proceedings are pending.

Held:

By nature, extradition proceedings are not equivalent to a criminal case in which guilt or innocence is determined.
Consequently, an extradition case is not one in which the constitutional rights of the accused are necessarily
available. Having once escaped the jurisdiction of the requesting state, the reasonable prima facie presumption is
that the person would escape again if given the opportunity. Hence, if the judge is convinced that a prima facie
case exists, he immediately Issue a warrant for the arrest of the potential extraditee and summons him or her to
answer and to appear at scheduled hearings on the petition. Potential extraditees are entitled to the rights to due
process and to fundamental fairness. Due process does not always call for a prior opportunity to be heard. A
subsequent opportunity is sufficient due to the flight risk involved. Indeed, available during the hearings on the
petition and the answer is the full chance to be heard and to enjoy fundamental fairness that is compatible with
the summary nature of extradition.

After being taken into custody, potential extraditees may apply for bail. Since the applicants have a history of
absconding, they have the burden of showing that (a) there is no flight risk and no danger to the community; and
(b) there exist special, humanitarian or compelling circumstances. In extradition cases, bail is not a matter of right;
it is subject to judicial discretion in the context of the peculiar facts of each case.

Page 52
Section 1-C, SY ’06-‘07
San Beda College of Law – Alabang
Constitutional Law 2 Case Digests

EQUAL PROTECTION

Art 3, Sec. 1. “…nor shall any person be denied the equal protection of the laws.”

PEOPLE VS. CAYAT


[68 PHIL 12; G.R. NO. 45987; 5 MAY 1939]

Facts:

“Law prohibits any member of a non-Christian tribe to buy, receive, have in his possession, or drink, any
intoxicating liquors of any kind.” The law, Act No. 1639, exempts only the so-called native wines or liquors which
the members of such tribes have been accustomed to take.

Issue:

Whether or Not the law denies equal protection to one prosecuted and sentenced for violation of said law.

Held:

No. It satisfies the requirements of a valid classification, one of which is that the classification under the law must
rest on real or substantial distinctions.

The distinction is reasonable. The classification between the members of the non- Christian and the members of
the Christian tribes is not based upon accident of birth or parentage but upon the degree of civilization and
culture. The term ‘non-Christian tribes’ refers to a geographical area and more directly to natives of the
Philippines of a low grade civilization usually living in tribal relationship apart from settled communities. The
distinction is reasonable for the Act was intended to meet the peculiar conditions existing in the non- Christian
tribes”

The prohibition is germane to the purposes of the law. It is designed to insure peace and order in and among the
non- Christian tribes has often resulted in lawlessness and crime thereby hampering the efforts of the government
to raise their standards of life and civilization. This law is not limited in its application to conditions existing at the
time of the enactment. It is intended to apply for all times as long as those conditions exists. The Act applies
equally to all members of the class. That it may be unfair in its operation against a certain number of non-
Christians by reason of their degree of culture is not an argument against the equality of its operation nor affect
the reasonableness of the classification thus established.

PASEI VS. DRILON


[163 SCRA 386; L-81958; 30 JUN 1988]

Facts:

Petitioner, Phil association of Service Exporters, Inc., is engaged principally in the recruitment of Filipino workers,
male and female of overseas employment. It challenges the constitutional validity of Dept. Order No. 1 (1998) of
DOLE entitled “Guidelines Governing the Temporary Suspension of Deployment of Filipino Domestic and
Household Workers.” It claims that such order is a discrimination against males and females. The Order does not
apply to all Filipino workers but only to domestic helpers and females with similar skills, and that it is in violation of
the right to travel, it also being an invalid exercise of the lawmaking power. Further, PASEI invokes Sec 3 of Art
13 of the Constitution, providing for worker participation in policy and decision-making processes affecting their
rights and benefits as may be provided by law. Thereafter the Solicitor General on behalf of DOLE submitting to
the validity of the challenged guidelines involving the police power of the State and informed the court that the
respondent have lifted the deployment ban in some states where there exists bilateral agreement with the
Philippines and existing mechanism providing for sufficient safeguards to ensure the welfare and protection of the
Filipino workers.

Issue:

Whether or not there has been a valid classification in the challenged Department Order No. 1.

Held:

Page 53
Section 1-C, SY ’06-‘07
San Beda College of Law – Alabang
Constitutional Law 2 Case Digests

SC in dismissing the petition ruled that there has been valid classification, the Filipino female domestics working
abroad were in a class by themselves, because of the special risk to which their class was exposed. There is no
question that Order No.1 applies only to female contract workers but it does not thereby make an undue
discrimination between sexes. It is well settled hat equality before the law under the constitution does not import a
perfect identity of rights among all men and women. It admits of classification, provided that:

1. Such classification rests on substantial distinctions


2. That they are germane to the purpose of the law
3. They are not confined to existing conditions
4. They apply equally to al members of the same class

In the case at bar, the classifications made, rest on substantial distinctions.

Dept. Order No. 1 does not impair the right to travel. The consequence of the deployment ban has on the right to
travel does not impair the right, as the right to travel is subjects among other things, to the requirements of “public
safety” as may be provided by law. Deployment ban of female domestic helper is a valid exercise of police power.
Police power as been defined as the state authority to enact legislation that may interfere with personal liberty or
property in order to promote general welfare. Neither is there merit in the contention that Department Order No. 1
constitutes an invalid exercise of legislative power as the labor code vest the DOLE with rule making powers.

DUMLAO VS. COMELEC


[95 SCRA 392; L-52245; 22 JAN 1980]

Facts:

Petitioner questions the constitutionality of section 4 of Batas Pambansa Blg. 52 as discriminatory and contrary to
the equal protection and due process guarantees of the Constitution.

Section 4 provided that any retired municipal or provincial city official that already received retirement benefits and
is 65 years of age shall not be qualified to run for the same local elective office from which he has retired.

Issue:

Whether or Not Sec. 4 of BP.52 is unconstitutional being contrary to the equal protection and due process rights.

Held:

No. The guarantee of equal protection is subject to rational classification based on reasonable and real
differentiations. In the present case, employees 65 years of age have been classified differently from younger
employees. The former are subject to compulsory retirement while the latter are not.

Retirement is not a reasonable disqualification for elective local officials because there can be retirees who are
even younger and a 65 year old retiree could be as good as a 65 year old official who is not a retiree. But there is
reason to disqualify a 65 year old elective official who is trying to run for office because there is the “need for new
blood to assume relevance”. When an official has retired he has already declared himself tired and unavailable for
the same government work.

WHEREFORE, the first paragraph of section 4 of Batas pambansa Bilang 52 is hereby declared valid.

TELECOMMUNICATIONS AND BROADCAST ATTORNEYS OF THE PHILS. VS. COMELEC


[289 SCRA 337; G.R. NO. 132922; 21 APR 1998]

Facts:

Petitioner Telecommunications and Broadcast Attorneys of the Philippines, Inc. (TELEBAP) is an organization of
lawyers of radio and television broadcasting companies. It was declared to be without legal standing to sue in this
case as, among other reasons, it was not able to show that it was to suffer from actual or threatened injury as a
result of the subject law. Petitioner GMA Network, on the other hand, had the requisite standing to bring the
constitutional challenge. Petitioner operates radio and television broadcast stations in the Philippines affected by
the enforcement of Section 92, B.P. No. 881.

Petitioners challenge the validity of Section 92, B.P. No. 881 which provides:
“Comelec Time- The Commission shall procure radio and television time to be known as the
“Comelec Time” which shall be allocated equally and impartially among the candidates within the

Page 54
Section 1-C, SY ’06-‘07
San Beda College of Law – Alabang
Constitutional Law 2 Case Digests

area of coverage of all radio and television stations. For this purpose, the franchise of all radio
broadcasting and television stations are hereby amended so as to provide radio or television
time, free of charge, during the period of campaign.”

Petitioner contends that while Section 90 of the same law requires COMELEC to procure print space in
newspapers and magazines with payment, Section 92 provides that air time shall be procured by COMELEC free
of charge. Thus it contends that Section 92 singles out radio and television stations to provide free air time.

Petitioner claims that it suffered losses running to several million pesos in providing COMELEC Time in
connection with the 1992 presidential election and 1995 senatorial election and that it stands to suffer even more
should it be required to do so again this year. Petitioners claim that the primary source of revenue of the radio
and television stations is the sale of air time to advertisers and to require these stations to provide free air time is
to authorize unjust taking of private property. According to petitioners, in 1992 it lost P22,498,560.00 in providing
free air time for one hour each day and, in this year’s elections, it stands to lost P58,980,850.00 in view of
COMELEC’s requirement that it provide at least 30 minutes of prime time daily for such.

Issue:

Whether of not Section 92 of B.P. No. 881 denies radio and television broadcast companies the equal protection
of the laws.

Whether or not Section 92 of B.P. No. 881 constitutes taking of property without due process of law and without
just compensation.

Held:

Petitioner’s argument is without merit. All broadcasting, whether radio or by television stations, is licensed by the
government. Airwave frequencies have to be allocated as there are more individuals who want to broadcast that
there are frequencies to assign. Radio and television broadcasting companies, which are given franchises, do not
own the airwaves and frequencies through which they transmit broadcast signals and images. They are merely
given the temporary privilege to use them. Thus, such exercise of the privilege may reasonably be burdened with
the performance by the grantee of some form of public service. In granting the privilege to operate broadcast
stations and supervising radio and television stations, the state spends considerable public funds in licensing and
supervising them.

The argument that the subject law singles out radio and television stations to provide free air time as against
newspapers and magazines which require payment of just compensation for the print space they may provide is
likewise without merit. Regulation of the broadcast industry requires spending of public funds which it does not do
in the case of print media. To require the broadcast industry to provide free air time for COMELEC is a fair
exchange for what the industry gets.

As radio and television broadcast stations do not own the airwaves, no private property is taken by the
requirement that they provide air time to the COMELEC.

LACSON VS. EXECUTIVE SECRETARY


[301 SCRA 298; G.R. NO. 128096; 20 JAN 1999]

Facts:

Eleven persons believed to be members of the Kuratong Baleleng gang, an organized crime syndicate involved in
bank robberies, were slain by elements of the Anti-Bank Robbery and Intelligence Task Group (ABRITG). Among
those included in the ABRITG were petitioners and petitioner-intervenors.

Acting on a media expose of SPO2 Eduardo delos Reyes, a member of the Criminal Investigation Command, that
what actually transpired was a summary execution and not a shoot-out between the Kuratong Baleleng gang
members and the ABRITG, Ombudsman Aniano Desierto formed a panel of investigators to investigate the said
incident. Said panel found the incident as a legitimate police operation. However, a review board modified the
panel’s finding and recommended the indictment for multiple murder against twenty-six respondents including
herein petitioner, charged as principal, and herein petitioner-intervenors, charged as accessories. After a
reinvestigation, the Ombudsman filed amended informations before the Sandiganbayan, where petitioner was
charged only as an accessory.

The accused filed separate motions questioning the jurisdiction of the Sandiganbayan, asserting that under the
amended informations, the cases fall within the jurisdiction of the Regional Trial Court pursuant to Section 2 of
R.A. 7975. They contend that the said law limited the jurisdiction of the Sandiganbayan to cases where one or

Page 55
Section 1-C, SY ’06-‘07
San Beda College of Law – Alabang
Constitutional Law 2 Case Digests

ore of the “principal accused” are government officals with Salary Grade 27 or higher, or PNP officials with rank of
Chief Superintendent or higher. Thus, they did not qualify under said requisites. However, pending resolution of
their motions, R.A. 8249 was approved amending the jurisdiction of the Sandiganbayan by deleting the word
“principal” from the phrase “principal accused” in Section 2 of R.A. 7975.

Petitioner questions the constitutionality of Section 4 of R.A. 8249, including Section 7 which provides that the
said law shall apply to all cases pending in any court over which trial has not begun as of the approval hereof.

Issue:

Whether or not Sections 4 and 7 of R.A. 8249 violate the petitioners’ right to due process and the equal protection
clause of the Constitution as the provisions seemed to have been introduced for the Sandiganbayan to continue
to acquire jurisdiction over the Kuratong Baleleng case.

Whether or not said statute may be considered as an ex-post facto statute.

Whether or not the multiple murder of the alleged members of the Kuratong Baleleng was committed in relation to
the office of the accused PNP officers which is essential to the determination whether the case falls within the
Sandiganbayan’s or Regional Trial Court’s jurisdiction.

Held:

Petitioner and intervenors’ posture that Sections 4 and 7 of R.A. 8249 violate their right to equal protection of the
law is too shallow to deserve merit. No concrete evidence and convincing argument were presented to warrant
such a declaration. Every classification made by the law is presumed reasonable and the party who challenges
the law must present proof of arbitrariness. The classification is reasonable and not arbitrary when the following
concur: (1) it must rest on substantial distinction; (2) it must be germane to the purpose of the law; (3) must not be
limited to existing conditions only, and (4) must apply equally to all members of the same class; all of which are
present in this case.

Paragraph a of Section 4 provides that it shall apply “to all cases involving” certain public officials and under the
transitory provision in Section 7, to “all cases pending in any court.” Contrary to petitioner and intervenors’
argument, the law is not particularly directed only to the Kuratong Baleleng cases. The transitory provision does
not only cover cases which are in the Sandiganbayan but also in “any court.”

There is nothing ex post facto in R.A. 8249. Ex post facto law, generally, provides retroactive effect of penal laws.
R.A. 8249 is not a penal law. It is a substantive law on jurisdiction which is not penal in character. Penal laws are
those acts of the Legislature which prohibit certain acts and establish penalties for their violations or those that
define crimes and provide for their punishment. R.A. 7975, as regards the Sandiganbayan’s jurisdiction, its mode
of appeal and other procedural matters, has been declared by the Court as not a penal law, but clearly a
procedural statute, one which prescribes rules of procedure by which courts applying laws of all kinds can
properly administer justice. Not being a penal law, the retroactive application of R.A. 8249 cannot be challenged
as unconstitutional.

In People vs. Montejo, it was held that an offense is said to have been committed in relation to the office if it is
intimately connected with the office of the offender and perpetrated while he was in the performance of his official
functions. Such intimate relation must be alleged in the information which is essential in determining the
jurisdiction of the Sandiganbayan. However, upon examination of the amended information, there was no specific
allegation of facts that the shooting of the victim by the said principal accused was intimately related to the
discharge of their official duties as police officers. Likewise, the amended information does not indicate that the
said accused arrested and investigated the victim and then killed the latter while in their custody. The stringent
requirement that the charge set forth with such particularity as will reasonably indicate the exact offense which the
accused is alleged to have committed in relation to his office was not established.

Consequently, for failure to show in the amended informations that the charge of murder was intimately
connected with the discharge of official functions of the accused PNP officers, the offense charged in the subject
criminal cases is plain murder and, therefore, within the exclusive original jurisdiction of the Regional Trial Court
and not the Sandiganbayan.

INT'L. SCHOOL ALLIANCE VS. QUISUMBING


[333 SCRA 13; G.R. NO. 128845; 1 JUN 2000]

Facts:

Page 56
Section 1-C, SY ’06-‘07
San Beda College of Law – Alabang
Constitutional Law 2 Case Digests

Receiving salaries less than their counterparts hired abroad, the local-hires of private respondent School, mostly
Filipinos, cry discrimination. We agree. That the local-hires are paid more than their colleagues in other schools
is, of course, beside the point. The point is that employees should be given equal pay for work of equal value.

Private respondent International School, Inc. (the School, for short), pursuant to Presidential Decree 732, is a
domestic educational institution established primarily for dependents of foreign diplomatic personnel and other
temporary residents. To enable the School to continue carrying out its educational program and improve its
standard of instruction, Section 2(c) of the same decree authorizes the School to
employ its own teaching and management personnel selected by it either locally or abroad, from Philippine or
other nationalities, such personnel being exempt from otherwise applicable laws and regulations attending their
employment, except laws that have been or will be enacted for the protection of employees.
Accordingly, the School hires both foreign and local teachers as members of its faculty, classifying the same into
two: (1) foreign-hires and (2) local-hires.

The School grants foreign-hires certain benefits not accorded local-hires. These include housing, transportation,
shipping costs, taxes, and home leave travel allowance. Foreign-hires are also paid a salary rate twenty-five
percent (25%) more than local-hires. The School justifies the difference on two "significant economic
disadvantages" foreign-hires have to endure, namely: (a) the "dislocation factor" and (b) limited tenure.

Issue:

Whether or Not the grants provided by the school to foreign hires and not to local hires discriminative of their
constitutional right to the equal protection clause.

Held:

The foregoing provisions impregnably institutionalize in this jurisdiction the long honored legal truism of "equal pay
for equal work." Persons who work with substantially equal qualifications, skill, effort and responsibility, under
similar conditions, should be paid similar salaries. This rule applies to the School, its "international character"
notwithstanding.

The School contends that petitioner has not adduced evidence that local-hires perform work equal to that of
foreign-hires. The Court finds this argument a little cavalier. If an employer accords employees the same position
and rank, the presumption is that these employees perform equal work. This presumption is borne by logic and
human experience. If the employer pays one employee less than the rest, it is not for that employee to explain
why he receives less or why the others receive more. That would be adding insult to injury. The employer has
discriminated against that employee; it is for the employer to explain why the employee is treated unfairly.

While we recognize the need of the School to attract foreign-hires, salaries should not be used as an enticement
to the prejudice of local-hires. The local-hires perform the same services as foreign-hires and they ought to be
paid the same salaries as the latter. For the same reason, the "dislocation factor" and the foreign-hires' limited
tenure also cannot serve as valid bases for the distinction in salary rates.

The Constitution enjoins the State to "protect the rights of workers and promote their welfare," "to afford labor full
protection." The State, therefore, has the right and duty to regulate the relations between labor and capital. These
relations are not merely contractual but are so impressed with public interest that labor contracts, collective
bargaining agreements included, must yield to the common good. Should such contracts contain stipulations that
are contrary to public policy, courts will not hesitate to strike down these stipulations.

In this case, we find the point-of-hire classification employed by respondent School to justify the distinction in the
salary rates of foreign-hires and local hires to be an invalid classification. There is no reasonable distinction
between the services rendered by foreign-hires and local-hires.

Wherefore, the petition is given due course. The petition is hereby granted in part. The orders of the secretary of
labor and employment dated June 10, 1996 and march 19, 1997, are hereby reversed and set aside insofar as
they uphold the practice of respondent school of according foreign-hires higher salaries than local-hires.

ORMOC SUGAR COMPANY VS. TREASURER OF ORMOC CITY


[22 SCRA 603; L-23794; 17 FEB 1968]

Facts:

On January 29, 1964, the Municipal Board of Ormoc City passed Ordinance No. 4, Series of 1964, imposing "on
any and all productions of centrifugal sugar milled at the Ormoc Sugar Company, Inc., in Ormoc City a municipal
tax equivalent to one per centum (1%) per export sale to the United States of America and other foreign

Page 57
Section 1-C, SY ’06-‘07
San Beda College of Law – Alabang
Constitutional Law 2 Case Digests

countries." Payments for said tax were made, under protest, by Ormoc Sugar Company, Inc. on March 20, 1964
for P7, 087.50 and on April 20, 1964 for P5, 000, or a total of P12, 087.50.

On June 1, 1964, Ormoc Sugar Company, Inc. filed before the Court of First Instance of Leyte, with service of a
copy upon the Solicitor General, a complaint against the City of Ormoc as well as its Treasurer, Municipal Board
and Mayor, alleging that the afore-stated ordinance is unconstitutional for being violative of the equal protection
clause (Sec. 1[1], Art. III, Constitution) and the rule of uniformity of taxation (Sec. 22[1]), Art. VI, Constitution).

Answering, the defendants asserted that the tax ordinance was within defendant city's power to enact under the
Local Autonomy Act and that the same did not violate the afore-cited constitutional limitations. After pre-trial and
submission of the case on memoranda, the Court of First Instance, on August 6, 1964, rendered a decision that
upheld the constitutionality of the ordinance and declared the taxing power of defendant chartered city broadened
by the Local Autonomy Act to include all other forms of taxes, licenses or fees not excluded in its charter.

Issue:

Whether or Not the ordinance is unconstitutional for being violative of the equal protection clause under Sec. 1[1],
Art. III, Constitution.

Whether or not it was violative of the rule of uniformity of taxation under the Bill of Rights, Sec. 22[1], Art. VI,
Constitution.

Held:

The Constitution in the bill of rights provides: ". . . nor shall any person be denied the equal protection of the
laws." (Sec. 1 [1], Art. III) In Felwa vs. Salas, We ruled that the equal protection clause applies only to persons or
things identically situated and does not bar a reasonable classification of the subject of legislation, and a
classification is reasonable where (1) it is based on substantial distinctions which make real differences; (2) these
are germane to the purpose of the law; (3) the classification applies not only to present conditions but also to
future conditions which are substantially identical to those of the present; (4) the classification applies only to
those who belong to the same class.

A perusal of the requisites instantly shows that the questioned ordinance does not meet them, for it taxes only
centrifugal sugar produced and exported by the Ormoc Sugar Company, Inc. and none other. At the time of the
taxing ordinance's enactment, Ormoc Sugar Company, Inc., it is true, was the only sugar central in the city of
Ormoc. Still, the classification, to be reasonable, should be in terms applicable to future conditions as well. The
taxing ordinance should not be singular and exclusive as to exclude any subsequently established sugar central,
of the same class as plaintiff, for the coverage of the tax. As it is now, even if later a similar company is set up, it
cannot be subject to the tax because the ordinance expressly points only to Ormoc City Sugar Company, Inc. as
the entity to be levied upon.

Appellant, however, is not entitled to interest; on the refund because the taxes were not arbitrarily collected
(Collector of Internal Revenue v. Binalbagan). 6 At the time of collection, the ordinance provided a sufficient basis
to preclude arbitrariness, the same being then presumed constitutional until declared otherwise.

Wherefore, the decision appealed from is hereby reversed, the challenged ordinance is declared unconstitutional
and the defendants-appellees are hereby ordered to refund the P12,087.50 plaintiff-appellant paid under protest.
No costs. So ordered.
PHILIPPINE JUDGES ASSO. VS. PRADO
[227 SCRA 703; G.R. NO. 105371; 11 NOV 1993]

Facts:

The Philippine Postal Corporation issued circular No. 92-28 to implement Section 35 of RA 7354 withdrawing the
franking privilege from the SC, CA, RTCs, MeTCs, MTCs and Land Registration Commission and with certain
other government offices. It is alleged that RA 7354 is discriminatory becasue while withdrawing the franking
privilege from judiciary, it retains the same for the President & Vice-President of the Philippines, Senator &
members of the House of Representatives, COMELEC, National Census & Statistics Office and the general
public. The respondents counter that there is no discrimination because the law is based on a valid classification
in accordance with the equal protection clause.

Issue:

Whether or Not Section 35 of RA 7354 is constitutional.

Held:

Page 58
Section 1-C, SY ’06-‘07
San Beda College of Law – Alabang
Constitutional Law 2 Case Digests

The equal protection of the laws is embraced in the concept of due process, as every unfair discrimination offends
the requirements of justice and fair play. It has nonetheless been embodied in a separate clause in Article III
Section 1 of the Constitution to provide for amore specific guarantee against any form of undue favoritism or
hostility from the government. Arbitrariness in general may be challenged on the basis of the due process clause.
But if the particular act assailed partakes of an unwarranted partiality or prejudice, the sharper weapon to cut it
down is the equal protection clause. Equal protection simply requires that all persons or things similarly situated
should be treated alike, both as to rights conferred and responsibilities imposed. What the clause requires is
equality among equals as determined according to a valid classification. Section 35 of RA 7354 is declared
unconstitutional. Circular No. 92-28 is set aside insofar

Page 59
Section 1-C, SY ’06-‘07
San Beda College of Law – Alabang
Constitutional Law 2 Case Digests

SEARCHES AND SEIZURES

Art 3, Sec. 2. “The right of the people to be secure in their persons, houses, papers, and effects against
unreasonable searches and seizures of whatever nature and for any purpose shall be inviolable, and no search
warrant or warrant of arrest shall issue except upon probable cause to be determined personally by the judge
after examination under oath or affirmation of the complainant and the witnesses he may produce, and particularly
describing the place to be searched and the persons or things to be seized.”

Art 3, Sec. 3. “(1) The privacy of communication and correspondence shall be inviolable except upon lawful
order of the court, or when public safety or order requires otherwise as prescribed by law.
(2) Any evidence obtained in violation of this or the preceding section shall be inadmissible for any purpose
in any proceeding.”

PEOPLE VS. MARTI


[193 SCRA 57; G.R. NO. 81561; 18 JAN 1991]

Facts:

Accused-appellant went to a forwarding agency to send four packages to a friend in Zurich. Initially, the accused
was asked by the proprietress if the packages can be examined. However, he refused. Before delivering said
packages to the Bureau of Customs and the Bureau of Posts, the husband of the proprietress opened said boxes
for final inspection. From that inspection, included in the standard operating procedure and out of curiosity, he
took several grams of its contents.

He brought a letter and the said sample to the National Bureau of Investigation. When the NBI was informed that
the rest of the shipment was still in his office, three agents went back with him. In their presence, the husband
totally opened the packages. Afterwards, the NBI took custody of said packages. The contents , after examination
by forensic chemists, were found to be marijuana flowering tops.

The appellant, while claiming his mail at the Central Post Office, was invited by the agents for questioning. Later
on, the trial court found him guilty of violation of the Dangerous Drugs Act.

Issue:

Whether or Not the items admitted in the searched illegally searched and seized.

Whether or Not custodial investigation properly applied.

Whether or Not the trial court not give credence to the explanation of the appellant on how said packages came to
his possession.

Held:

No. “The case at bar assumes a peculiar character since the evidence sought to be excluded was primarily
discovered and obtained by a private person, acting in a private capacity and without the intervention and
participation of State authorities. Under the circumstances, can accused/appellant validly claim that his
constitutional right against unreasonable searches and seizure has been violated. Stated otherwise, may an act of
a private individual, allegedly in violation of appellant's constitutional rights, be invoked against the State. In the
absence of governmental interference, the liberties guaranteed by the Constitution cannot be invoked against the
State. It was Mr. Job Reyes, the proprietor of the forwarding agency, who made search/inspection of the
packages. Said inspection was reasonable and a standard operating procedure on the part of Mr. Reyes as a
precautionary measure before delivery of packages to the Bureau of Customs or the Bureau of Posts. Second,
the mere presence of the NBI agents did not convert the reasonable search effected by Reyes into a warrantless
search and seizure proscribed by the Constitution. Merely to observe and look at that which is in plain sight is not
a search. Having observed that which is open, where no trespass has been committed in aid thereof, is not
search.”
No. “The law enforcers testified that accused/appellant was informed of his constitutional rights. It is presumed
that they have regularly performed their duties (See. 5(m), Rule 131) and their testimonies should be given full
faith and credence, there being no evidence to the contrary.”
No. “Appellant signed the contract as the owner and shipper thereof giving more weight to the presumption that
things which a person possesses, or exercises acts of ownership over, are owned by him (Sec. 5 [j], Rule 131). At
this point, appellant is therefore estopped to claim otherwise.”

Page 60
Section 1-C, SY ’06-‘07
San Beda College of Law – Alabang
Constitutional Law 2 Case Digests

WATEROUS DRUG VS. NLRC


[280 SCRA 735 ; G.R.NO. 113271; 16 OCT 1997]

Facts:

Catolico was hired as a pharmacist by petitioner Waterous Drug Corporation on 15 August 1988. On 31 July
1989, Catolico received a memorandum from WATEROUS Vice President-General Manager Emma R. Co
warning her not to dispense medicine to employees chargeable to the latter's accounts because the same was a
prohibited practice. On the same date, Co issued another memorandum to Catolico warning her not to negotiate
with suppliers of medicine without consulting the Purchasing Department, as this would impair the company's
control of purchases and, besides she was not authorized to deal directly with the suppliers.

As regards the first memorandum, Catolico did not deny her responsibility but explained that her act was "due to
negligence," since fellow employee Irene Soliven "obtained the medicines in bad faith and through
misrepresentation when she claimed that she was given a charge slip by the Admitting Dept." Catolico then asked
the company to look into the fraudulent activities of Soliven.

In a memorandum dated 21 November 1989, WATEROUS Supervisor Luzviminda E. Bautro warned Catolico
against the "rush delivery of medicines without the proper documents." On 29 January 1990, WATEROUS Control
Clerk Eugenio Valdez informed Co that he noticed an irregularity involving Catolico and Yung Shin
Pharmaceuticals, Inc.
Forthwith, in her memorandum dated 37 January 1990, Co asked Catolico to explain, within twenty-four hours,
her side of the reported irregularity. Catolico asked for additional time to give her explanation, and she was
granted a 48-hour extension from 1 to 3 February 1990. However, on 2 February 1990, she was informed that
effective 6 February 1990 to 7 March 1990, she would be placed on preventive suspension to protect the interests
of the company.

In a letter dated 2 February 1990, Catolico requested access to the file containing Sales Invoice No. 266 for her to
be able to make a satisfactory explanation. In said letter she protested Saldaña's invasion of her privacy when
Saldaña opened an envelope addressed to Catolico.

In a letter to Co dated 10 February 1990, Catolico, through her counsel, explained that the check she received
from YSP was a Christmas gift and not a "refund of overprice." She also averred that the preventive suspension
was ill-motivated, as it sprang from an earlier incident between her and Co's secretary, Irene Soliven.

On 5 March 1990, WATEROUS Supervisor Luzviminda Bautro, issued a memorandum notifying Catolico of her
termination. On 5 May 1990, Catolico filed before the Office of the Labor Arbiter a complaint for unfair labor
practice, illegal dismissal, and illegal suspension. In his decision of 10 May 1993, Labor Arbiter Alex Arcadio
Lopez found no proof of unfair labor practice against petitioners. Nevertheless, he decided in favor of Catolico
because petitioners failed to "prove what alleged as complainant's dishonesty," and to show that any investigation
was conducted. Hence, the dismissal was without just cause and due process. He thus declared the dismissal
and suspension illegal but disallowed reinstatement.

Petitioners seasonably appealed from the decision and urged the NLRC to set it aside because the Labor Arbiter
erred in finding that Catolico was denied due process and that there was no just cause to terminate her services.
In its decision of 30 September 1993, the NLRC affirmed the findings of the Labor Arbiter on the ground that
petitioners were not able to prove a just cause for Catolico's dismissal from her employment. It found that
petitioner's evidence consisted only of the check of P640.00 drawn by YSP in favor of complainant, which her co-
employee saw when the latter opened the envelope. But, it declared that the check was inadmissible in evidence
pursuant to Sections 2 and 3(1 and 2) of Article III of the Constitution. It concluded:

With the smoking gun evidence of respondents being rendered inadmissible, by virtue of the
constitutional right invoked by complainants, respondents' case falls apart as it is bereft of
evidence which cannot be used as a legal basis for complainant's dismissal.

The NLRC then dismissed the appeal for lack of merit, but modified the dispositive portion of the appealed
decision by deleting the award for illegal suspension as the same was already included in the computation of the
aggregate of the awards in the amount of P35,401.86.

Issue:

Whether or Not the dismissal of the private respondent is in violation of the Constitution, under the Bill of Rights.

Held:

Page 61
Section 1-C, SY ’06-‘07
San Beda College of Law – Alabang
Constitutional Law 2 Case Digests

As to the first and second grounds, petitioners insist that Catolico had been receiving "commissions" from YSP, or
probably from other suppliers, and that the check issued to her on 9 November 1989 was not the first or the last.
They also maintained that Catolico occupied a confidential position and that Catolico's receipt of YSP's check,
aggravated by her "propensity to violate company rules," constituted breach of confidence. And contrary to the
findings of NLRC, Catolico was given ample opportunity to explain her side of the controversy.

In her Comment, Catolico asserts that petitioners' evidence is too "flimsy" to justify her dismissal. The check in
issue was given to her, and she had no duty to turn it over to her employer. Company rules do not prohibit an
employee from accepting gifts from clients, and there is no indication in the contentious check that it was meant
as a refund for overpriced medicines. Besides, the check was discovered in violation of the constitutional
provision on the right to privacy and communication; hence, as correctly held by the NLRC, it was inadmissible in
evidence.

Catolico was denied due process. Procedural due process requires that an employee be apprised of the charge
against him, given reasonable time to answer the charge, allowed ample opportunity to be heard and defend
himself, and assisted by a representative if the employee so desires. Ample opportunity connotes every kind of
assistance that management must accord the employee to enable him to prepare adequately for his defense,
including legal representation. In the case at bar, although Catolico was given an opportunity to explain her side,
she was dismissed from the service in the memorandum of 5 March 1990 issued by her Supervisor after receipt
of her letter and that of her counsel. No hearing was ever conducted after the issues were joined through said
letters.

Catolico was also unjustly dismissed. It is settled that the burden is on the employer to prove just and valid cause
for dismissing an employee, and its failure to discharge that burden would result in a finding that the dismissal is
unjustified. It clearly appears then that Catolico's dismissal was based on hearsay information. Catolico's
dismissal then was obviously grounded on mere suspicion, which in no case can justify an employee's dismissal.
Suspicion is not among the valid causes provided by the Labor Code for the termination of employment; and even
the dismissal of an employee for loss of trust and confidence must rest on substantial grounds and not on the
employer's arbitrariness, whims, caprices, or suspicion. Besides, Catolico was not shown to be a managerial
employee, to which class of employees the term "trust and confidence" is restricted.
As regards the constitutional violation upon which the NLRC anchored its decision, that the Bill of Rights does not
protect citizens from unreasonable searches and seizures perpetrated by private individuals. It is not true, as
counsel for Catolico claims, that the citizens have no recourse against such assaults. On the contrary, and as said
counsel admits, such an invasion gives rise to both criminal and civil liabilities.

Finally, since it has been determined by the Labor Arbiter that Catolico's reinstatement would not be to the best
interest of the parties, he correctly awarded separation pay to Catolico. Separation pay in lieu of reinstatement is
computed at one month's salary for every year of service. In this case, however, Labor Arbiter Lopez computed
the separation pay at one-half month's salary for every year of service. Catolico did not oppose or raise an
objection. As such, we will uphold the award of separation pay as fixed by the Labor Arbiter.

WHEREFORE, the instant petition is hereby DISMISSED and the challenged decision and resolution of the
National Labor Relations Commission dated 30 September 1993 and 2 December 1993, respectively, in NLRC-
NCR CA No. 005160-93 are AFFIRMED, except as to its reason for upholding the Labor Arbiter's decision, viz.,
that the evidence against private respondent was inadmissible for having been obtained in violation of her
constitutional rights of privacy of communication and against unreasonable searches and seizures which is
hereby set aside.

STONEHILL VS. DIOKNO


[20 SCRA 383; L-19550; 19 JUN 1967]

Facts:

Upon application of the officers of the government named on the margin1 — hereinafter referred to as
Respondents-Prosecutors — several judges2 — hereinafter referred to as Respondents-Judges — issued, on
different dates,3 a total of 42 search warrants against petitioners herein4 and/or the corporations of which they
were officers,5 directed to the any peace officer, to search the persons above-named and/or the premises of their
offices, warehouses and/or residences, and to seize and take possession of the following personal property to wit:
Books of accounts, financial records, vouchers, correspondence, receipts, ledgers, journals, portfolios,
credit journals, typewriters, and other documents and/or papers showing all business transactions
including disbursements receipts, balance sheets and profit and loss statements and Bobbins (cigarette
wrappers).

as "the subject of the offense; stolen or embezzled and proceeds or fruits of the offense," or "used or intended to
be used as the means of committing the offense," which is described in the applications adverted to above as

Page 62
Section 1-C, SY ’06-‘07
San Beda College of Law – Alabang
Constitutional Law 2 Case Digests

"violation of Central Bank Laws, Tariff and Customs Laws, Internal Revenue (Code) and the Revised Penal
Code."

Petitioners contentions are:


(1) they do not describe with particularity the documents, books and things to be seized;
(2) cash money, not mentioned in the warrants, were actually seized;
(3) the warrants were issued to fish evidence against the aforementioned petitioners in deportation cases filed
against them;
(4) the searches and seizures were made in an illegal manner; and
(5) the documents, papers and cash money seized were not delivered to the courts that issued the warrants, to
be disposed of in accordance with law —

Respondents-prosecutors contentions
(1) that the contested search warrants are valid and have been issued in accordance with law;
(2) that the defects of said warrants, if any, were cured by petitioners' consent; and
(3) that, in any event, the effects seized are admissible in evidence against herein petitioners, regardless of the
alleged illegality of the aforementioned searches and seizures.

The documents, papers, and things seized under the alleged authority of the warrants in question may be split
into two (2) major groups, namely: (a) those found and seized in the offices of the aforementioned corporations,
and (b) those found and seized in the residences of petitioners herein.

Issue:

Whether or not those found and seized in the offices of the aforementioned corporations are obtained legally.

Whether or not those found and seized in the residences of petitioners herein are obtained legally.

Held:

The petitioners have no cause of action to assail the legality of the contested warrants and of the seizures made
in pursuance thereof, for the simple reason that said corporations have their respective personalities, separate
and distinct from the personality of herein petitioners, regardless of the amount of shares of stock or of the
interest of each of them in said corporations, and whatever the offices they hold therein may be. Indeed, it is well
settled that the legality of a seizure can be contested only by the party whose rights have been impaired thereby,
and that the objection to an unlawful search and seizure is purely personal and cannot be availed of by third
parties.

With respect to the documents, papers and things seized in the residences of petitioners herein, the
aforementioned resolution of June 29, 1962, lifted the writ of preliminary injunction previously issued by this Court,
thereby, in effect, restraining herein Respondents-Prosecutors from using them in evidence against petitioners
herein.
Two points must be stressed in connection with this constitutional mandate, namely: (1) that no warrant shall
issue but upon probable cause, to be determined by the judge in the manner set forth in said provision; and (2)
that the warrant shall particularly describe the things to be seized.

None of these requirements has been complied with in the contested warrants. Indeed, the same were issued
upon applications stating that the natural and juridical person therein named had committed a "violation of Central
Ban Laws, Tariff and Customs Laws, Internal Revenue (Code) and Revised Penal Code." In other words, no
specific offense had been alleged in said applications. The averments thereof with respect to the offense
committed were abstract. As a consequence, it was impossible for the judges who issued the warrants to have
found the existence of probable cause, for the same presupposes the introduction of competent proof that the
party against whom it is sought has performed particular acts, or committed specific omissions, violating a given
provision of our criminal laws. As a matter of fact, the applications involved in this case do not allege any specific
acts performed by herein petitioners. It would be the legal heresy, of the highest order, to convict anybody of a
"violation of Central Bank Laws, Tariff and Customs Laws, Internal Revenue (Code) and Revised Penal Code," —
as alleged in the aforementioned applications — without reference to any determinate provision of said laws or
__________________________

1Hon. Jose W. Diokno, in his capacity as Secretary of Justice, Jose Lukban, in his capacity as Acting Director,
National Bureau of Investigation, Special Prosecutors Pedro D. Cenzon, Efren I. Plana and Manuel Villareal, Jr.
and Assistant Fiscal Maneses G. Reyes, City of Manila.
2Hon. Amado Roan, Judge of the Municipal (now City) Court of Manila, Hon. Roman Cansino, Judge of the

Municipal (now City) Court of Manila, Hon. Hermogenes Caluag, Judge of the Court of First Instance of Rizal,
Quezon City Branch, Hon. Eulogio Mencias, Judge of the Court of First Instance of Rizal, Pasig Branch, and Hon.
Damian Jimenez, Judge of the Municipal (now City) Court of Quezon City.
3Covering the period from March 3 to March 9, 1962.

Page 63
Section 1-C, SY ’06-‘07
San Beda College of Law – Alabang
Constitutional Law 2 Case Digests

4HarryS. Stonehill, Robert P. Brooks, John J. Brooks and Karl Beck.


5U.S. Tobacco Corporation, Atlas Cement Corporation, Atlas Development Corporation, Far East Publishing
Corporation (Evening News), Investment Inc., Industrial Business Management Corporation, General Agricultural
Corporation, American Asiatic Oil Corporation, Investment Management Corporation, Holiday Hills, Inc., Republic
Glass Corporation, Industrial and Business Management Corporation, United Housing Corporation, The Philippine
Tobacco-Flue-Curing and Redrying Corporation, Republic Real Estate Corporation and Merconsel Corporation.

BURGOS, SR. V. CHIEF OF STAFF, AFP


[133 SCRA 800; G.R. NO. 64261; 26 DEC 1984]

Facts:

Petitioners assail the validity of 2 search warrants issued on December 7, 1982 by respondent Judge Cruz-Pano
of the then Court of First Instance of Rizal, under which the premises known as No. 19, Road 3, Project 6,
Quezon City, and 784 Units C & D, RMS Building, Quezon Avenue, Quezon City, business addresses of the
"Metropolitan Mail" and "We Forum" newspapers, respectively, were searched, and office and printing machines,
equipment, paraphernalia, motor vehicles and other articles used in the printing, publication and distribution of the
said newspapers, as well as numerous papers, documents, books and other written literature alleged to be in the
possession and control of petitioner Jose Burgos, Jr. publisher-editor of the "We Forum" newspaper, were seized.
As a consequence of the search and seizure, these premises were padlocked and sealed, with the further result
that the printing and publication of said newspapers were discontinued. Respondents contend that petitioners
should have filed a motion to quash said warrants in the court that issued them before impugning the validity of
the same before this Court. Respondents also assail the petition on ground of laches (Failure or negligence for
an unreasonable and unexplained length of time to do that which, by exercising due diligence, could or should
have been done earlier. It is negligence or omission to assert a right within a reasonable time, warranting a
presumption that the party entitled to assert it either has abandoned it or declined to assert it). Respondents
further state that since petitioner had already used as evidence some of the documents seized in a prior criminal
case, he is stopped from challenging the validity of the search warrants.

Petitioners submit the following reasons to nullify the questioned warrants:


1. Respondent Judge failed to conduct an examination under oath or affirmation of the applicant and his
witnesses, as mandated by the above-quoted constitutional provision as well as Sec. 4, Rule 126 of the
Rules of Court.
2. The search warrants pinpointed only one address which would be the former abovementioned address.
3. Articles belonging to his co-petitioners were also seized although the warrants were only directed against
Jose Burgos, Jr.
4. Real properties were seized.
5. The application along with a joint affidavit, upon which the warrants were issued, from the Metrocom
Intelligence and Security Group could not have provided sufficient basis for the finding of a probable
cause upon which a warrant may be validly issued in accordance with Section 3, Article IV of the 1973
Constitution.

Respondents justify the continued sealing of the printing machines on the ground that they have been
sequestered under Section 8 of Presidential Decree No. 885, as amended, which authorizes sequestration of the
property of any person engaged in subversive activities against the government in accordance with implementing
rules and regulations as may be issued by the Secretary of National Defense.

Issue:

Whether or Not the 2 search warrants were validly issued and executed.

Held:

In regard to the quashal of warrants that petitioners should have initially filed to the lower court, this Court takes
cognizance of this petition in view of the seriousness and urgency of the constitutional Issue raised, not to
mention the public interest generated by the search of the "We Forum" offices which was televised in Channel 7
and widely publicized in all metropolitan dailies. The existence of this special circumstance justifies this Court to
exercise its inherent power to suspend its rules. With the contention pertaining to laches, the petitioners gave an
explanation evidencing that they have exhausted other extra-judicial efforts to remedy the situation, negating the
presumption that they have abandoned their right to the possession of the seized property.

On the enumerated reasons:


1. This objection may properly be considered moot and academic, as petitioners themselves conceded
during the hearing on August 9, 1983, that an examination had indeed been conducted by respondent
judge of Col. Abadilla and his witnesses.

Page 64
Section 1-C, SY ’06-‘07
San Beda College of Law – Alabang
Constitutional Law 2 Case Digests

2. The defect pointed out is obviously a typographical error. Precisely, two search warrants were applied for
and issued because the purpose and intent were to search two distinct premises. It would be quite absurd
and illogical for respondent judge to have issued two warrants intended for one and the same place.
3. Section 2, Rule 126, of the Rules of Court, does not require that the property to be seized should be
owned by the person against whom the search warrant is directed. It may or may not be owned by him.
4. Petitioners do not claim to be the owners of the land and/or building on which the machineries were
placed. This being the case, the machineries in question, while in fact bolted to the ground, remain
movable property susceptible to seizure under a search warrant.
5. The broad statements in the application and joint affidavit are mere conclusions of law and does not
satisfy the requirements of probable cause. Deficient of such particulars as would justify a finding of the
existence of probable cause, said allegation cannot serve as basis for the issuance of a search warrant
and it was a grave error for respondent judge to have done so. In Alvarez v. Court of First Instance, this
Court ruled that "the oath required must refer to the truth of the facts within the personal knowledge of the
petitioner or his witnesses, because the purpose thereof is to convince the committing magistrate, not the
individual making the affidavit and seeking the issuance of the warrant, of the existence of probable
cause." Another factor which makes the search warrants under consideration constitutionally
objectionable is that they are in the nature of general warrants. The description of the articles sought to
be seized under the search warrants in question are too general.

With regard to the respondents invoking PD 885, there is an absence of any implementing rules and regulations
promulgated by the Minister of National Defense. Furthermore, President Marcos himself denies the request of
military authorities to sequester the property seized from petitioners. The closure of the premises subjected to
search and seizure is contrary to the freedom of the press as guaranteed in our fundamental law. The search
warrants are declared null and void.

TAMBASEN VS. PEOPLE


[246 SCRA 184; G.R. NO. 89103; 14 JUL 1995]

Facts:

In August 1988, P/Sgt. Natuel applied for issuance of search warrant alleging that he received information that
Petitioner had in his possession at his house “M-16 Armalite rifles, hand grenades, .45 Cal. pistols, dynamite
sticks and subversive documents”, which were “used or intended to be used” for illegal purposes. The application
was granted.

In September, a police team, searched the house of petitioner and seized “2 envelopes containing P14000,
handset with antennae, transceiver with antennae, regulator supply, academy notebook and assorted papers and
handset battery pack”. In October, petitioner moved that the search and seizure be declared illegal and that the
seized articles be returned to him. In December, MTCC, in its order, directed Lt. Col. Torres to return the money
seized to petitioner ruling that any seizure should be limited to the specified items covered thereby. SolGen
petitioned with the RTC for the annulment of the order of MTCC citing that pending the determination of legality of
seizure of the articles, they should remain in custogia legis. RTC granted the petition.

Issue:

Whether or Not the seizure of the articles which were not mentioned in the search warrant was legal.

Held:

Section 2 Article III of the 1987 Constitution requires that a search warrant should particularly describe the things
to be seized. The police acts beyond the parameters of their authority if they seize articles not described in the
search warrants. The evident purpose and intent of the requirement is to limit the things to be seized, to leave the
officers of the law with no discretion; that unreasonable search and seizure may not be made and that abuses
may not be committed.

Petition granted. People of the Philippines is ordered to return the money seized.

PLACER VS. JUDGE VILLANUEVA


[126 SCRA 463; G.R. NOS. L-60349-62; 29 DEC 1983]

Facts:

Petitioners filed informations in the city court and they certified that Preliminary Investigation and Examination had
been conducted and that prima facie cases have been found. Upon receipt of said informations, respondent
judge set the hearing of the criminal cases to determine propriety of issuance of warrants of arrest. After the

Page 65
Section 1-C, SY ’06-‘07
San Beda College of Law – Alabang
Constitutional Law 2 Case Digests

hearing, respondent issued an order requiring petitioners to submit to the court affidavits of prosecution witnesses
and other documentary evidence in support of the informations to aid him in the exercise of his power of judicial
review of the findings of probable cause by petitioners. Petitioners petitioned for certiorari and mandamus to
compel respondent to issue warrants of arrest. They contended that the fiscal’s certification in the informations of
the existence of probable cause constitutes sufficient justification for the judge to issue warrants of arrest.

Issue:

Whether or Not respondent city judge may, for the purpose of issuing warrants of arrest, compel the fiscal to
submit to the court the supporting affidavits and other documentary evidence presented during the preliminary
investigation.

Held:

Judge may rely upon the fiscal’s certification for the existence of probable cause and on the basis thereof, issue a
warrant of arrest. But, such certification does not bind the judge to come out with the warrant. The issuance of a
warrant is not a mere ministerial function; it calls for the exercise of judicial discretion on the part of issuing
magistrate. Under Section 6 Rule 112 of the Rules of Court, the judge must satisfy himself of the existence of
probable cause before issuing a warrant of arrest. If on the face of the information, the judge finds no probable
cause, he may disregard the fiscal’s certification and require submission of the affidavits of witnesses to aid him in
arriving at the conclusion as to existence of probable cause.

Petition dismissed.

SOLIVEN VS. M AKASIAR


[167 SCRA 393; G.R. NO. 82585; 14 NOV 1988]

Facts:

In these consolidated cases, three principal issues were raised: (1) whether or not petitioners were denied due
process when informations for libel were filed against them although the finding of the existence of a prima facie
case was still under review by the Secretary of Justice and, subsequently, by the President; and (2) whether or
not the constitutional rights of Beltran were violated when respondent RTC judge issued a warrant for his arrest
without personally examining the complainant and the witnesses, if any, to determine probable cause.
Subsequent events have rendered the first issue moot and academic. On March 30, 1988, the Secretary of
Justice denied petitioners' motion for reconsideration and upheld the resolution of the Undersecretary of Justice
sustaining the City Fiscal's finding of a prima facie case against petitioners. A second motion for reconsideration
filed by petitioner Beltran was denied by the Secretary of Justice on April 7, 1988. On appeal, the President,
through the Executive Secretary, affirmed the resolution of the Secretary of Justice on May 2, 1988. The motion
for reconsideration was denied by the Executive Secretary on May 16, 1988. With these developments,
petitioners' contention that they have been denied the administrative remedies available under the law has lost
factual support.

Issue:

Whether or Not petitioners were denied due process when informations for libel were filed against them although
the finding of the existence of a prima facie case was still under review by the Secretary of Justice and,
subsequently, by the President.

Whether or Not the constitutional rights of Beltran were violated when respondent RTC judge issued a warrant for
his arrest without personally examining the complainant and the witnesses, if any, to determine probable cause

Held:

With respect to petitioner Beltran, the allegation of denial of due process of law in the preliminary investigation is
negated by the fact that instead of submitting his counter- affidavits, he filed a "Motion to Declare Proceedings
Closed," in effect waiving his right to refute the complaint by filing counter-affidavits. Due process of law does not
require that the respondent in a criminal case actually file his counter-affidavits before the preliminary
investigation is deemed completed. All that is required is that the respondent be given the opportunity to submit
counter-affidavits if he is so minded.

The second issue, raised by petitioner Beltran, calls for an interpretation of the constitutional provision on the
issuance of warrants of arrest. The pertinent provision reads:

Page 66
Section 1-C, SY ’06-‘07
San Beda College of Law – Alabang
Constitutional Law 2 Case Digests

Art. III, Sec. 2. The right of the people to be secure in their persons, houses, papers and effects
against unreasonable searches and seizures of whatever nature and for any purpose shall be
inviolable, and no search warrant or warrant of arrest shall issue except upon probable cause to
be determined personally by the judge after examination nder oath or affirmation of the
complainant and the witnesses he may produce, and particularly describing the place to be
searched and the persons or things to be seized.

The addition of the word "personally" after the word "determined" and the deletion of the grant of authority by the
1973 Constitution to issue warrants to "other responsible officers as may be authorized by law," has apparently
convinced petitioner Beltran that the Constitution now requires the judge to personally examine the complainant
and his witnesses in his determination of probable cause for the issuance of warrants of arrest. This is not an
accurate interpretation.

What the Constitution underscores is the exclusive and personal responsibility of the issuing judge to satisfy
himself of the existence of probable cause. In satisfying himself of the existence of probable cause for the
issuance of a warrant of arrest, the judge is not required to personally examine the complainant and his
witnesses. Following established doctrine and procedure, he shall: (1) personally evaluate the report and the
supporting documents submitted by the fiscal regarding the existence of probable cause and, on the basis
thereof, issue a warrant of arrest; or (2) if on the basis thereof he finds no probable cause, he may disregard the
fiscal's report and require the submission of supporting affidavits of witnesses to aid him in arriving at a conclusion
as to the existence of probable cause.

Sound policy dictates this procedure, otherwise judges would be unduly laden with the preliminary examination
and investigation of criminal complaints instead of concentrating on hearing and deciding cases filed before their
courts. It has not been shown that respondent judge has deviated from the prescribed procedure. Thus, with
regard to the issuance of the warrants of arrest, a finding of grave abuse of discretion amounting to lack or excess
of jurisdiction cannot be sustained. The petitions fail to establish that public respondents, through their separate
acts, gravely abused their discretion as to amount to lack of jurisdiction. Hence, the writs of certiorari and
prohibition prayed for cannot issue.

WHEREFORE, finding no grave abuse of discretion amounting to excess or lack of jurisdiction on the part of the
public respondents, the Court Resolved to DISMISS the petitions in G. R. Nos. 82585, 82827 and 83979. The
Order to maintain the status quo contained in the Resolution of the Court en banc dated April 7, 1988 and
reiterated in the Resolution dated April 26, 1988 is LIFTED.

SALAZAR VS. ACHACOSO


[183 SCRA 145; G.R. NO. 81510; 14 MAR 1990]

Facts:

Rosalie Tesoro of Pasay City in a sworn statement filed with the POEA, charged petitioner with illegal recruitment.
Public respondent Atty. Ferdinand Marquez sent petitioner a telegram directing him to appear to the POEA
regarding the complaint against him. On the same day, after knowing that petitioner had no license to operate a
recruitment agency, public respondent Administrator Tomas Achacoso issued a Closure and Seizure Order No.
1205 to petitioner. It stated that there will a seizure of the documents and paraphernalia being used or intended to
be used as the means of committing illegal recruitment, it having verified that petitioner has— (1) No valid license
or authority from the Department of Labor and Employment to recruit and deploy workers for overseas
employment; (2) Committed/are committing acts prohibited under Article 34 of the New Labor Code in relation to
Article 38 of the same code. A team was then tasked to implement the said Order. The group, accompanied by
mediamen and Mandaluyong policemen, went to petitioner’s residence. They served the order to a certain Mrs.
For a Salazar, who let them in. The team confiscated assorted costumes. Petitioner filed with POEA a letter
requesting for the return of the seized properties, because she was not given prior notice and hearing. The said
Order violated due process. She also alleged that it violated sec 2 of the Bill of Rights, and the properties were
confiscated against her will and were done with unreasonable force and intimidation.

Issue:

Whether or Not the Philippine Overseas Employment Administration (or the Secretary of Labor) can validly issue
warrants of search and seizure (or arrest) under Article 38 of the Labor Code

Held:

Under the new Constitution, “. . . no search warrant or warrant of arrest shall issue except upon probable cause to
be determined personally by the judge after examination under oath or affirmation of the complainant and the

Page 67
Section 1-C, SY ’06-‘07
San Beda College of Law – Alabang
Constitutional Law 2 Case Digests

witnesses he may produce, and particularly describing the place to be searched and the persons or things to be
seized”. Mayors and prosecuting officers cannot issue warrants of seizure or arrest. The Closure and Seizure
Order was based on Article 38 of the Labor Code. The Supreme Court held, “We reiterate that the Secretary of
Labor, not being a judge, may no longer issue search or arrest warrants. Hence, the authorities must go through
the judicial process. To that extent, we declare Article 38, paragraph (c), of the Labor Code, unconstitutional and
of no force and effect… The power of the President to order the arrest of aliens for deportation is, obviously,
exceptional. It (the power to order arrests) cannot be made to extend to other cases, like the one at bar. Under
the Constitution, it is the sole domain of the courts.” Furthermore, the search and seizure order was in the nature
of a general warrant. The court held that the warrant is null and void, because it must identify specifically the
things to be seized.

WHEREFORE, the petition is GRANTED. Article 38, paragraph (c) of the Labor Code is declared
UNCONSTITUTIONAL and null and void. The respondents are ORDERED to return all materials seized as a
result of the implementation of Search and Seizure Order No. 1205.

MORANO VS. VIVO


[20 SCRA 562; G.R. L-22196; 30 JUN 1967]

Facts:

Chan Sau Wah, a Chinese citizen born in Fukien, China arrived in the Philippines on November 1961 to visit her
cousin, Samuel Lee Malaps. She left China and her children by a first marriage: Fu Tse Haw and Fu Yan Kai both
minors, in the care of neighbors in Fukien, China. Chan Sau wah arrived in the Philippines with Fu Yan Fun, her
minor son also by the first marriage. Chan Sau Wah and her minor son Fu Yan Fun were permitted only into the
Philippines under a temporary visitor's visa for two months and after they posted a cash bond of 4,000 pesos. On
January 1962, Chan Sau Wah married Esteban Morano, a native-born Filipino citizen. Born to this union on
September 1962 was Esteban Morano, Jr. To prolong their stay in the Philippines, Chan Sau Wah and Fu Yan
Fun obtained several extensions. The last extension expired on September 10, 1962. In a letter dated August 31,
1962, the Commissioner of Immigration ordered Chan Sau Wah and her son, Fu Yan Fun, to leave the country on
or before September 10, 1962 with a warning that upon failure so to do, he will issue a warrant for their arrest and
will cause the confiscation of their bond.

Issue:

Whether or Not the issuance of the warrant of arrest is unconstitutional.

Held:

Chan Sau Wah entered the Philippines on a tourist-temporary visitor's visa. She is a non-immigrant. Under
Section 13 just quoted, she may therefore be admitted if she were a qualified and desirable alien and subject to
the provisions of the last paragraph of Section 9. Therefore, first, she must depart voluntarily to some foreign
country; second, she must procure from the appropriate consul the proper visa; and third, she must thereafter
undergo examination by the officials of the Bureau of Immigration at the port of entry for determination of her
admissibility in accordance with the requirements of the immigration Act. This Court in a number of cases has
ruled, and consistently too, that an alien admitted as a temporary visitor cannot change his or her status without
first departing from the country and complying with the requirements of Section 9 of the Immigration Act. The
gravamen of petitioners' argument is that Chan Sau Wah has, since her entry, married in Manila a native-born
Filipino, Esteban Morano. It will not particularly help analysis for petitioners to appeal to family solidarity in an
effort to thwart her deportation. Chan Sau Wah, seemingly is not one who has a high regard for such solidarity.
Proof: She left two of her children by the first marriage, both minors, in the care of neighbors in Fukien,
China.Then, the wording of the statute heretofore adverted to is a forbidding obstacle which will prevent this Court
from writing into the law an additional provision that marriage of a temporary alien visitor to a Filipino would ipso
facto make her a permanent resident in his country. This is a field closed to judicial action. No breadth of
discretion is allowed. We cannot insulate her from the State's power of deportation. it would be an easy matter for
an alien woman to enter the Philippines as a temporary visitor, go through a mock marriage, but actually live with
another man as husband and wife, and thereby skirt the provisions of our immigration law. Also, a woman of
undesirable character may enter this country, ply a pernicious trade, marry a Filipino, and again throw overboard
Sections 9 and 13 of the Act. Such a flanking movement, we are confident, is impermissible.Recently we
confirmed the rule that an alien wife of a Filipino may not stay permanently without first departing from the
Philippines. Reason: Discourage entry under false pretenses.

HARVEY V. DEFENSOR-SANTIAGO
[162 SCRA 840; G.R. NO. 82544; 28 JUN 1988]

Page 68
Section 1-C, SY ’06-‘07
San Beda College of Law – Alabang
Constitutional Law 2 Case Digests

Facts:

This is a petition for Habeas Corpus. Petitioners are the following: American nationals Andrew Harvey, 52 and
Jonh Sherman 72. Dutch Citizen Adriaan Van Den Elshout, 58. All reside at Pagsanjan Laguna respondent
Commissioner Miriam Defensor Santiago issued Mission Orders to the Commission of Immigration and
Deportation (CID) to apprehended petitioners at their residences. The “Operation Report” read that Andrew
Harvey was found together with two young boys. Richard Sherman was found with two naked boys inside his
room. While Van Den Elshout in the “after Mission Report” read that two children of ages 14 and 16 has been
under his care and subjects confirmed being live-in for sometime now.

Seized during the petitioner’s apprehension were rolls of photo negatives and photos of suspected child
prostitutes shown in scandalous poses as well as boys and girls engaged in sex. Posters and other literature
advertising the child prostitutes were also found.

Petitioners were among the 22 suspected alien pedophiles. They were apprehended 17 February1988 after close
surveillance for 3 month of the CID in Pagsanjan, Laguna. 17 of the arrested aliens opted for self-deportation.
One released for lack of evidence, another charged not for pedophile but working with NO VISA, the 3 petitioners
chose to face deportation proceedings. On 4 March1988, deportation proceedings were instituted against aliens
for being undesirable aliens under Sec.69 of Revised Administrative Code.

Warrants of Arrest were issued 7March1988 against petitioners for violation of Sec37, 45 and 46 of Immigration
Act and sec69 of Revised Administrative Code. Trial by the Board of Special Inquiry III commenced the same
date. Petition for bail was filed 11March 1988 but was not granted by the Commissioner of Immigration. 4
April1988 Petitioners filed a petition for Writ of Habeas Corpus. The court heard the case on oral argument on 20
April 1988.

Issue:

Whether or Not the Commissioner has the power to arrest and detain petitioners pending determination of
existence of probable cause.

Whether or Not there was unreasonable searches and seizures by CID agents.

Whether or Not the writ of Habeas Corpus may be granted to petitioners.

Held:

While pedophilia is not a crime under the Revised Penal Code, it violates the declared policy of the state to
promote and protect the physical, moral, spiritual and social well being of the youth. The arrest of petitioners was
based on the probable cause determined after close surveillance of 3 months. The existence of probable cause
justified the arrest and seizure of articles linked to the offense. The articles were seized as an incident to a lawful
arrest; therefore the articles are admissible evidences (Rule 126, Section12 of Rules on Criminal Procedure).

The rule that search and seizures must be supported by a valid warrant of arrest is not an absolute rule. There
are at least three exceptions to this rule. 1.) Search is incidental to the arrest. 2.) Search in a moving vehicle. 3.)
Seizure of evidence in plain view. In view of the foregoing, the search done was incidental to the arrest.

The filing of the petitioners for bail is considered as a waiver of any irregularity attending their arrest and estops
them from questioning its validity. Furthermore, the deportation charges and the hearing presently conducted by
the Board of Special Inquiry made their detention legal. It is a fundamental rule that habeas corpus will not be
granted when confinement is or has become legal, although such confinement was illegal at the beginning.

The deportation charges instituted by the Commissioner of Immigration are in accordance with Sec37 (a) of the
Philippine Immigration Act of 1940 in relation to sec69 of the Revised Administrative code. Section 37 (a)
provides that aliens shall be arrested and deported upon warrant of the Commissioner of Immigration and
Deportation after a determination by the Board of Commissioners of the existence of a ground for deportation
against them. Deportation proceedings are administrative in character and never construed as a punishment but
a preventive measure. Therefore, it need not be conducted strictly in accordance with ordinary Court
proceedings. What is essential is that there should be a specific charge against the alien intended to be arrested
and deported. A fair hearing must also be conducted with assistance of a counsel if desired.

Page 69
Section 1-C, SY ’06-‘07
San Beda College of Law – Alabang
Constitutional Law 2 Case Digests

Lastly, the power to deport aliens is an act of the State and done under the authority of the sovereign power. It a
police measure against the undesirable aliens whose continued presence in the country is found to be injurious to
the public good and tranquility of the people.

SALES VS. SANDIGANBAYAN


[369 SCRA 293 G.R. NO. 143802; 16 NOV 2001]

Facts:

The petitioner, the incumbent mayor of Pagudpud Ilocos Norte, shot the former mayor and his political rival Atty.
Benemerito. After the shooting, he surrendered himself and hence the police inspector and wife of the victim filed
a criminal complaint for murder against him. The judge after conducting the preliminary examination (p.e. for
brevity) found probable cause and issued a warrant of arrest. Also after conducting the preliminary investigation
(p.i. for brevity), he issued a resolution forwarding the case to the prosecutor for appropriate action. Petitioner
received a subpoena directing him to file his counter affidavit, affidavit of witnesses and other supporting
documents. He did it the following day. While proceedings are ongoing, he filed a petition for habeas corpus with
the C.A alleging that: the warrant was null and void because the judge who issued it was a relative by affinity of
the private respondent and the p.e. and the p.i. were illegal and irregular as the judge doesn’t have jurisdiction on
the case. The C.A. granted the petition holding that the judge was a relative by affinity by 3 rd degree to the private
respondent and the p.i. he conducted has 2 stages, the p.e. and the p.i. proper. The proceeding now consists only
of one stage. He conducted the requisite investigation prior to the issuance of warrant of arrest. Moreover he did
not complete it. He only examined the witness of the complainant. But the prosecution instead of conducting p.i.
of his own forwarded the records to the Ombudsman (OMB for brevity) for the latter to conduct the same. The
OMB directed the petitioner to submit his counter affidavit, but he did not comply with it finding the same
superfluous. The graft investigator recommended the filing of information for murder which the OMB approved.
Petitioner received a copy of the resolution but prevented seeking reconsideration thereof he filed a motion to
defer issuance of warrant of arrest pending the determination of probable cause. The Sandiganbayan denied the
motion. This is now a petition for review on the decision of the Sandiganbayan,

Issue:

Whether or Not the OMB followed the procedure in conducting preliminary investigation.

Whether or Not petitioner was afforded an opportunity to be heard and to submit controverting evidence.

Held:

The proper procedure in the conduct of preliminary investigation was not followed because of the following
reasons. Firstly, the preliminary investigation was conducted by 3 different investigators, none of whom completed
the preliminary investigation There was not one continuous proceeding but rather, cases of passing the buck, the
last one being the OMB throwing the buck to the Sandiganbayan. Secondly, the charge of murder is a non
bailable offense. The gravity of the offense alone should have merited a deeper and more thorough preliminary
investigation. The OMB did nothing of the sort but wallowed the resolution of the graft investigator. He did a worse
job than the judge, by actually adopting the resolution of the graft investigator without doing anything and threw
everything to the Sandiganbayan for evaluation. Thirdly, a person under preliminary investigation by the OMB is
entitled to a motion for reconsideration, as maintained by the Rules of Procedure by the OMB. The filing of the
motion for reconsideration is an integral part of the preliminary investigation proper. The denial thereof is
tantamount to the denial of the right itself to a preliminary investigation. This fact alone renders preliminary
investigation conducted in this case incomplete. And lastly, it was patent error for the Sandiganbayan to have
relied purely on the OMB’s certification of probable cause given the prevailing facts of the case much more so in
the face of the latter’s flawed report and one side factual findings.

The court cannot accept the Sandiganbayan’s assertion of having found probable cause on its own, considering
the OMB’s defective report and findings, which merely rekied on the testimonies of the witnesses for the
prosecution and disregarded the evidence for the defense.

Judgment is rendered setting aside the resolution of the Sandiganbayan, ordering the Sandiganbayan to quash
the warrant of arrest and remanding the OMB for completion of the preliminary investigation.

SILVA VS. PRESIDING JUDGE


[203 SCRA 140; G.R. No. 81756; 21 Oct 1991]

Facts:

Page 70
Section 1-C, SY ’06-‘07
San Beda College of Law – Alabang
Constitutional Law 2 Case Digests

Sgt. Villamor, chief of the PC Narcom Detachment in Dumaguete City filed an "application for search warrant" and
"Deposition of witness" against petitioner Nicomedes Silva and Martin Silva. Judge Nickarter Ontal, then the
presiding judge of RTC of Dumaguete issued Search Warrant No.1 pursuant to the said applications for violation
of RA 6425 Dangerous Drugs ACT of 1972. Such warrant states that there is a probable cause to believe that Mr.
Tama Silva has the possession and control of marijuana dried leaves, cigarette and joint. The warrant authorizes
Sgt. Villamor to make an immediate search at any time of the room of Mr. Tama Silva at the residence of his
father Comedes Silva and to open aparadors, lockers, cabinets, cartons and containers to look for said illegal
drugs. In the course of the search, the officers seized money belonging to Antonieta Silva in the amount of
P1,231.40. Petitioner filed a motion to quash Search Warrant No.1 on the ground that 1) it was issued on the sole
basis of mimeographed 2) the judge failed to personally examine the complainant and witness by searching
questions and answers.

Issue:

Whether or Not Search Warrant No.1 is invalid. WON the officers abused their authority in seizing the money of
Antonieta Silva.

Held:

Search Warrant No. 1 is invalid due to the failure of the judge to examine the witness in the form of searching
questions and answers. The questions asked were leading as they are answerable by mere yes or no. Such
questions are not sufficiently searching to establish probable cause. The questions were already mimeographed
and all the witness had to do was fill in their answers on the blanks provided. Judge Ontal is guilty of grave abuse
of discretion when he rejected the motion of Antonieta Silva seeking the return of her money.

The officers who implemented the search warrant clearly abused their authority when they seized the money of
Antonieta Silva. The warrant did not indicate the seizure of money but only for marijuana leaves, cigarettes..etc.
Search Warrant No. 1 is declared null and void.

*** Sec 4 Rule 126 Rules of Court


Examination of the complainant, record -the judge before issuing the warrant, personally examine in the form of
searching questions and answers, in writing and under oath the complainant and any witness he may produce the
facts personally known to them and attach to the record their sworn statements together with their affidavits.

VEROY VS. LAYAGUE


[210 SCRA 97; G.R. No. 95630; 18 Jun 1992]

Facts:

Petitioners are husband and wife who owned and formerly resided at No. 13 Isidro St., Skyline Village. Catalunan
Grande, Davao City. When petitioner Leopoldo Veroy was promoted to the position of Assistant Administrator of
the Social Security System sometime in June, 1988, he and his family transferred to 130 K-8th St., East Kamias,
Quezon City, where they are presently residing. The care and upkeep of their residence in Davao City was left to
two (2) houseboys, Jimmy Favia and Eric Burgos, who had their assigned quarters at a portion of the premises.
The Veroys would occasionally send money to Edna Soguilon for the salary of the said houseboys and other
expenses for the upkeep of their house. While the Veroys had the keys to the interior of the house, only the key to
the kitchen, where the circuit breakers were located, was entrusted to Edna Soguilon to give her access in case of
an emergency. Hence, since 1988, the key to the master's bedroom as well as the keys to the children's rooms
were retained by herein Petitioners so that neither Edna Soguilon nor the caretakers could enter the house.

Police Officers had an information that the petitioner’s residence was being used as a safehouse of rebel soldiers.
They were able to enter the yard with the help of the caretakers but did not enter the house since the owner was
not present and they did not have a search warrant. Petitioner Ma. Luisa was contacted by telephone in her
Quezon City residence by Capt. Obrero to ask permission to search the house in Davao City as it was reportedly
being used as a hideout and recruitment center of rebel soldiers. Petitioner Ma. Luisa Veroy responded that she is
flying to Davao City to witness the search but relented if the search would not be conducted in the presence of
Major Ernesto Macasaet, an officer of the PC/INP, Davao City and a long time family friend of the Veroys.

The following day, Capt. Obrero and Major Macasaet met at the house of herein petitioners in Skyline Village to
conduct the search pursuant to the authority granted by petitioner Ma. Luisa Veroy. The caretakers facilitated their
entry into the yard, and using the key entrusted to Edna Soguilon, they were able to gain entrance into the
kitchen. However, a locksmith by the name of George Badiang had to be employed to open the padlock of the
door leading to the children's room. Capt. Obrero and Major Macasaet then entered the children's room and
conducted the search. Capt. Obrero recovered a .45 cal. handgun with a magazine containing seven (7) live
bullets in a black clutch bag inside an unlocked drawer. Three (3) half-full jute sacks containing printed materials
of RAM-SFP were also found in the children's room. A search of the children's recreation and study area revealed

Page 71
Section 1-C, SY ’06-‘07
San Beda College of Law – Alabang
Constitutional Law 2 Case Digests

a big travelling bag containing assorted polo shirts, men's brief, two (2) pieces polo barong and short sleeve
striped gray polo. sweat shirt, two (2) pairs men's socks, a towel made in U.S.A., one blanket, a small black bag,
Gandhi brand, containing a book entitled "Islamic Revolution Future Path of the Nation", a road map of the
Philippines, a telescope, a plastic bag containing assorted medicines and religious pamphlets was found in the
master's bedroom. Sgt. Leo Justalero was instructed by Capt. Obrero to make an inventory and receipt of the
articles seized, in the house.

The case was referred for preliminary investigation to Quezon City Assistant Prosecutor Rodolfo Ponferrada who
was designated Acting Provincial Prosecutor for Davao City. In a resolution dated August 6, 1990, Fiscal
Ponferrada recommended the filing of an information against herein petitioners for Violation of Presidential
Decree No. 1866 (Illegal Possession of Firearms and Ammunitions in Furtherance of Rebellion). No bail was
recommended.

Issue:

Whether or Not Presidential Decree No. 1866, or at least the third paragraph of Section 1 thereof, is
unconstitutional for being violative of the due process and equal protection clauses of the Constitution.

Held:

The issue of constitutionality of Presidential Decree No. 1866 has been laid to rest in the case of Misolas v.
Panga, G.R. No. 83341, January 30, 1990 (181 SCRA 648), where this Court held that the declaration of
unconstitutionality of the third paragraph of Section 1 of Presidential Decree No. 1866 is wanting in legal basis
since it is neither a bill of attainder nor does it provide a possibility of a double jeopardy.

Petitioners' contention that Republic Act 6968 has repealed Presidential Decree No. 1866 is bereft of merit. It is a
cardinal rule of statutory construction that where the words and phrases of a statute are not obscure or
ambiguous. its meaning and the intention of the legislature must be determined from the language employed, and
where there is no ambiguity in the words, there is no room for construction. Petitioners contend that Section 1 of
Presidential Decree No. 1866 is couched in general or vague terms. The terms "deal in", "acquire", "dispose" or
"possess" are capable of various interpretations such that there is no definiteness as to whether or not the
definition includes "constructive possession" or how the concept of constructive possession should be applied.
Petitioners were not found in actual possession of the firearm and ammunitions. They were in Quezon City while
the prohibited articles were found in Davao City. Yet they were being charged under Presidential Decree No.
1866 upon the sole circumstance that the house wherein the items were found belongs to them.

Petitioners question the admissibility in evidence of the articles seized in violation of their constitutional right
against unreasonable search and seizure. Petitioners aver that while they concede that Capt. Obrero had
permission from Ma. Luisa Veroy to break open the door of their residence, it was merely for the purpose of
ascertaining thereat the presence of the alleged "rebel" soldiers. The permission did not include any authority to
conduct a room to room search once inside the house. The items taken were, therefore, products of an illegal
search, violative of their constitutional rights As such, they are inadmissible in evidence against them.

The Constitution guarantees the right of the people to be secure in their persons, houses, papers and effects
against unreasonable searches and seizures (Article III, Section 2 of the 1987 Constitution). However, the rule
that searches and seizures must be supported by a valid warrant is not an absolute one. Among the recognized
exceptions thereto are: (1) a search incidental to an arrest; (2) a search of a moving vehicle; and (3) seizure of
evidence in plain view (People v. Lo Ho Wing, G.R. No. 88017, January 21, 1991 [193 SCRA 122]).

None of these exceptions pertains to the case at bar. The reason for searching the house of herein petitioners is
that it was reportedly being used as a hideout and recruitment center for rebel soldiers. While Capt. Obrero was
able to enter the compound, he did not enter the house because he did not have a search warrant and the owners
were not present. This shows that he himself recognized the need for a search warrant, hence, he did not persist
in entering the house but rather contacted the Veroys to seek permission to enter the same. Permission was
indeed granted by Ma. Luisa Veroy to enter the house but only to ascertain the presence of rebel soldiers. Under
the circumstances it is undeniable that the police officers had ample time to procure a search warrant but did not.

Undeniably, the offense of illegal possession of firearms is malum prohibitum but it does not follow that the
subject thereof is necessarily illegal per se. Motive is immaterial in mala prohibita but the subjects of this kind of
offense may not be summarily seized simply because they are prohibited. A search warrant is still necessary.
Hence, the rule having been violated and no exception being applicable, the articles seized were confiscated
illegally and are therefore protected by the exclusionary principle. They cannot be used as evidence against the
petitioners in the criminal action against them for illegal possession of firearms. (Roan v. Gonzales, 145 SCRA
689-690 [1986]). Besides, assuming that there was indeed a search warrant, still in mala prohibita, while there is

Page 72
Section 1-C, SY ’06-‘07
San Beda College of Law – Alabang
Constitutional Law 2 Case Digests

no need of criminal intent, there must be knowledge that the same existed. Without the knowledge or
voluntariness there is no crime.

PREMISES CONSIDERED, the petition as granted and the criminal case against the petitioners for illegal
possession of firearms is DISMISSED.

PEOPLE VS. DEL ROSARIO


[234 SCRA 246; G.R. NO. 109633; 20 JUL 1994]

Facts:

Accused was charged and convicted by the trial court of illegal possession of firearms and illegal possession and
sale of drugs, particularly methamphetamine or shabu. After the issuance of the search warrant, which authorized
the search and seizure of an undetermined quantity of methamphetamine and its paraphernalia’s, an entrapment
was planned that led to the arrest of del Rosario and to the seizure of the shabu, its paraphernalia’s and of a .22
caliber pistol with 3 live ammunition.

Issue:

Whether or Not the seizure of the firearms was proper.

Held:

No. Sec 2 art. III of the constitution specifically provides that a search warrant must particularly describe the
things to be seized. In herein case, the only objects to be seized that the warrant determined was the
methamphetamine and the paraphernalia’s therein. The seizure of the firearms was unconstitutional.

Wherefore the decision is reversed and the accused is acquitted.

PEOPLE VS. GESMUNDO


[219 SCRA 743; G.R. NO. 89373; 19 MAR 1993]

Facts:

According to the prosecution, in the morning of Nov. 17, 1986, PO Jose Luciano gave money and instructed his
civilian informer to buy marijuana from the accused at the Cocoland Hotel. He actually saw the accused selling
marijuana to his civilian informer and that same day Luciano applied for a search warrant.

About 2pm that day, a police raiding team armed with a search warrant went to the Brgy captain for them to be
accompanied in serving the said warrant at the residence of the accused. The police was allowed to enter the
house upon the strength of the warrant shown to the accused. The accused begged the police not to search and
to leave the house. The police still searched the house and was led to the kitchen. She pointed a metal basin on
top of a table as the hiding place of died marijuana flowering tops contained in a plastic bag marked ISETANN.
The police also recovered from a native “uway” cabinet dried marijuana flowering tops wrapped in 3 pieces of
komiks paper.

According to the accused, when the police arrived at her house, she saw Sgt. Yte and PFC Jose Luciano. She
invited Sgt. Yte to enter her house while Luciano was left in the jeep that was parked near the house. While inside
the house Yte showed the accused something he claimed as a search warrant, when someone coming from the
kitchen uttered “eto na” They proceeded to the kitchen and saw Luciano holding a plastic bag with four other
companions. They confronted the accused and insisted that the bags belonged to her. Accused denied the
accusation and told them that she doesn’t know anything about it. She was made to sign a prepared document.
She was brought to the police station and was detained.

The court renders judgment finding the accused guilty.

Issue:

Whether or Not the evidence was properly obtained by the police.

Held:

Page 73
Section 1-C, SY ’06-‘07
San Beda College of Law – Alabang
Constitutional Law 2 Case Digests

In the investigation report prepared by Luciano stated that during the search they discovered a hole at the
backyard of the house of the suspect, there was a big biscuit can inside the hole and on top of the cover a flower
pot was placed wherein the marijuana was kept. However, there was no mention of any marijuana obtained from
a flower pot in any of their testimonies. There were inconsistencies insofar the prosecution is concerned, as to
what was recovered and where, the trial court concluded that these inconsistencies are trivial. There must
sufficient evidence that the marijuana was actually surrendered by the accused. As held in PP vs. Remorosa,
Irreconcilable and unexplained contradictions in the testimonies of the prosecution witnesses cast doubt on the
guilt of appellant and his culpability to the crime charged.

The claim that the marijuana was planted was strengthen as the police violated sec 7, rule 126 rules of the court
provides no search of a house, room or any other premise shall be made except in the presence of the lawful
occupant thereof or any member of his family or in the absence of the latter, in the presence of two (2) witnesses
of sufficient age and discretion residing in the same locality. This requirement is mandatory to ensure regularity in
the execution of the search warrant. Violation of said rule is in fact punishable under Article 130 of the Revised
Penal Code.

The document (PAGPAPATUNAY) was inadmissible to the court as the accused was not informed of her right not
to sign the document neither was she informed that she has the right to the assistance of a counsel and the fact
that it may be used as evidence against her. It was not proved that the marijuana belonged to her. Not only does
the law require the presence of witnesses when the search is conducted, but it also imposes upon the person
making the search the duty to issue a detailed receipt for the property seized. He is likewise required to deliver
the property seized to the judge who issued the warrant, together with a true and accurate inventory thereof duly
verified under oath. Again, these duties are mandatory and are required to preclude substitution of the items
seized by interested parties.

The guilt of the accused was has not been established. Judgment is reversed.

UMIL VS. RAMOS


[187 SCRA 311; G.R. NO. 81567; 3 OCT 1991]

Facts:

On 1 February 1988, military agents were dispatched to the St. Agnes Hospital, Roosevelt Avenue, Quezon City,
to verify a confidential information which was received by their office, about a "sparrow man" (NPA member) who
had been admitted to the said hospital with a gunshot wound. That the wounded man in the said hospital was
among the five (5) male "sparrows" who murdered two (2) Capcom mobile patrols the day before, or on 31
January 1988 at about 12:00 o'clock noon, before a road hump along Macanining St., Bagong Barrio, Caloocan
City. The wounded man's name was listed by the hospital management as "Ronnie Javellon," twenty-two (22)
years old of Block 10, Lot 4, South City Homes, Biñan, Laguna however it was disclosed later that the true name
of the wounded man was Rolando Dural. In view of this verification, Rolando Dural was transferred to the
Regional Medical Servicesof the CAPCOM, for security reasons. While confined thereat, he was positively
identified by the eyewitnesses as the one who murdered the 2 CAPCOM mobile patrols.

Issue:

Whether or Not Rolando was lawfully arrested.

Held:

Rolando Dural was arrested for being a member of the NPA, an outlawed subversive organization. Subversion
being a continuing offense, the arrest without warrant is justified as it can be said that he was committing as
offense when arrested. The crimes rebellion, subversion, conspiracy or proposal to commit such crimes, and
crimes or offenses committed in furtherance therefore in connection therewith constitute direct assaults against
the state and are in the nature of continuing crimes.

PEOPLE VS. SUCRO


[195 SCRA 388; G.R. No. 93239; 18 Mar 1991]

Facts:

Page 74
Section 1-C, SY ’06-‘07
San Beda College of Law – Alabang
Constitutional Law 2 Case Digests

Pat. Fulgencio went to Arlie Regalado’s house at C. Quimpo to monitor activities of Edison SUCRO (accused).
Sucro was reported to be selling marijuana at a chapel 2 meters away from Regalado’s house. Sucro was
monitored to have talked and exchanged things three times. These activities are reported through radio to P/Lt.
Seraspi. A third buyer was transacting with appellant and was reported and later identified as Ronnie Macabante.
From that moment, P/Lt.Seraspi proceeded to the area. While the police officers were at the Youth Hostel in
Maagama St. Fulgencio told Lt. Seraspi to intercept. Macabante was intercepted at Mabini and Maagama
crossing in front of Aklan Medical center. Macabante saw the police and threw a tea bag of marijuana on the
ground. Macabante admitted buying the marijuana from Sucro in front of the chapel.

The police team intercepted and arrested SUCRO at the corner of C. Quimpo and Veterans. Recovered were 19
sticks and 4 teabags of marijuana from a cart inside the chapel and another teabag from Macabante.

Issue:

Whether or Not arrest without warrant is lawful.

Whether or Not evidence from such arrest is admissible.

Held:

Search and seizures supported by a valid warrant of arrest is not an absolute rule. Rule 126, Sec 12 of Rules of
Criminal Procedure provides that a person lawfully arrested may be searched for dangerous weapons or
anything, which may be used as proff of the commission of an offense, without a search warrant.(People v.
Castiller) The failure of the police officers to secure a warrant stems from the fact that their knowledge required
from the surveillance was insufficient to fulfill requirements for its issuance. However, warantless search and
seizures are legal as long as PROBABLE CAUSE existed. The police officers have personal knowledge of the
actual commission of the crime from the surveillance of the activities of the accused. As police officers were the
ones conducting the surveillance, it is presumed that they are regularly in performance of their duties.

PEOPLE V. RODRIGUEZA
[205 SCRA 791; G.R. No. 95902; 4 Feb 1992]

Facts:

NARCOM agents staged a buy-bust operation, after gaining information that there was an ongoing illegal traffic of
prohibited drugs in Tagas, Albay. The participating agents were given money treated with ultraviolet powder.
One of the agents went to said location, asked for a certain Don. Thereafter, the Don, herein accused, met with
him and “a certain object wrapped in a plastic” later identified as marijuana was given in exchange for P200. The
agent went back to headquarters and made a report, based on which, a team was subsequently organized and a
raid was conducted in the house of the father of the accused. During the raid, the NARCOM agents were able to
confiscate dried marijuana leaves and a plastic syringe among others. There was no authorization by any search
warrant. The accused was found positive of ultraviolet powder. The lower court, considering the evidences
obtained and testimonies from the prosecution, found him guilty of violating the Dangerous Drugs Act of 1972 and
sentenced him to reclusion perpetua.

Issue:

Whether or Not the lower court was correct in its judgment.

Held:

The NARCOM agents’ procedure in the entrapment of the accused failed to meet the qualification that the
suspected drug dealer must be caught red-handed in the act of selling marijuana to a person posing as a buyer,
since the operation was conducted after the actual exchange. Said raid also violated accused’ right against
unreasonable search and seizure, as the situation did not fall in the circumstances wherein a search may be
validly made even without a search warrant, i.e. when the search is incidental to a lawful arrest; when it involves
prohibited articles in plain view. The NARCOM agents could not have justified their act by invoking the urgency
and necessity of the situation because the testimonies of the prosecution witnesses reveal that the place had
already been put under surveillance for quite some time. Had it been their intention to conduct the raid, then they
should, because they easily could, have first secured a search warrant during that time. The Court further notes
the confusion and ambiguity in the identification of the confiscated marijuana leaves and other prohibited drug
paraphernalia presented as evidence against appellant:

Page 75
Section 1-C, SY ’06-‘07
San Beda College of Law – Alabang
Constitutional Law 2 Case Digests

CIC Taduran, who acted as the poseur buyer, testified that appellant sold him 100 grams of dried marijuana
leaves wrapped in a plastic bag. Surprisingly, and no plausible explanation has been advanced therefor, what
were submitted to and examined by the PCCL and thereafter utilized as evidence against the appellant were the
following items:

One (1) red and white colored plastic bag containing the following:

Exh. "A"—Thirty (30) grams of suspected dried marijuana fruiting tops contained inside a
transparent plastic bag.
Exh. "B"— Fifty (50) grams of suspected dried marijuana leaves and seeds contained inside a
white colored plastic labelled "Robertson".
Exh. "C"— Four (4) aluminum foils each containing suspected dried marijuana fruiting tops having
a total weight of seven grams then further wrapped
with a piece of aluminum foil.
Exh. "D"— Five (5) small transparent plastic bags each containing suspected dried marijuana
fruiting tops having a total weight of seventeen grams.
Exh. "E"— One plastic syringe.

Evidently, these prohibited articles were among those confiscated during the so-called follow-up raid in the house
of Rodrigueza’s father. The unanswered question then arises as to the identity of the marijuana leaves that
became the basis of appellant's conviction. In People vs. Rubio, this Court had the occasion to rule that the
plastic bag and the dried marijuana leaves contained therein constitute the corpus delicti of the crime. As such,
the existence thereof must be proved with certainty and conclusiveness. Failure to do so would be fatal to the
cause of the prosecution. Conviction is reversed and set aside and accused is acquitted.

PEOPLE VS. SY CHUA


[396 SCRA 657; G.R. No.136066-67; 4 Feb 2003]

Facts:

Accused-appellant Binad Sy Chua was charged with violation of Section 16, Article III of R.A. 6425, as amended
by R.A. 7659, and for Illegal Possession of Ammunitions and Illegal Possession of Drugs in two separate
Informations.

SPO2 Nulud and PO2 Nunag received a report from their confidential informant that accused-appellant was about
to deliver drugs that night at the Thunder Inn Hotel in Balibago, Angeles City. So, the PNP Chief formed a team of
operatives. The group positioned themselves across McArthur Highway near Bali Hai Restaurant, fronting the
hotel. The other group acted as their back up.

Afterwards, their informer pointed to a car driven by accused-appellant which just arrived and parked near the
entrance of the hotel. After accused-appellant alighted from the car carrying a sealed Zest-O juice box, SPO2
Nulud and PO2 Nunag hurriedly accosted him and introduced themselves as police officers. As accused-
appellant pulled out his wallet, a small transparent plastic bag with a crystalline substance protruded from his right
back pocket. Forthwith, SPO2 Nulud subjected him to a body search which yielded twenty (20) pieces of live .22
caliber firearm bullets from his left back pocket. When SPO2 Nunag peeked into the contents of the Zest-O box,
he saw that it contained a crystalline substance. SPO2 Nulud instantly confiscated the small transparent plastic
bag, the Zest-O juice box, the twenty (20) pieces of .22 caliber firearm bullets and the car used by accused-
appellant. SPO2 Nulud and the other police operatives who arrived at the scene brought the confiscated items to
the office of Col. Guttierez at the PNP Headquarters in Camp Pepito, Angeles City.

Accused-appellant vehemently denied the accusation against him and narrated a different version of the incident.

Accused-appellant alleged that he was driving the car of his wife to follow her and his son to Manila. He felt
sleepy, so he decided to take the old route along McArthur Highway. He stopped in front of a small store near
Thunder Inn Hotel to buy cigarettes and candies. While at the store, he noticed a man approaches and examines
the inside of his car. When he called the attention of the onlooker, the man immediately pulled out a .45 caliber
gun and made him face his car with raised hands. The man later on identified himself as a policeman. During the
course of the arrest, the policeman took out his wallet and instructed him to open his car. He refused, so the
policeman took his car keys and proceeded to search his car. At this time, the police officer’s companions arrived
at the scene in two cars. PO2 Nulud, who just arrived at the scene, pulled him away from his car in a nearby
bank, while the others searched his car.

Page 76
Section 1-C, SY ’06-‘07
San Beda College of Law – Alabang
Constitutional Law 2 Case Digests

Thereafter, he was brought to a police station and was held inside a bathroom for about fifteen minutes until Col.
Guttierez arrived, who ordered his men to call the media. In the presence of reporters, Col. Guttierez opened the
box and accused-appellant was made to hold the box while pictures were being taken.

The lower court acquitted Sy Chua for the Illegal Possession of Ammunitions, yet convicted him for Illegal
Possession of 1,955.815 grams of shabu. Hence, this appeal to the Court.

Issue:

Whether or Not the arrest of accused-appellant was lawful; and (2) WON the search of his person and the
subsequent confiscation of shabu allegedly found on him were conducted in a lawful and valid manner.

Held:

The lower court believed that since the police received information that the accused will distribute illegal drugs
that evening at the Thunder Inn Hotel and its vicinities. The police officer had to act quickly and there was no
more time to secure a search warrant. The search is valid being akin to a “stop and frisk”.

The trial court confused the concepts of a “stop-and-frisk” and of a search incidental to a lawful arrest. These two
types of warrantless searches differ in terms of the requisite quantum of proof before they may be validly effected
and in their allowable scope.

In a search incidental to a lawful arrest, as the precedent arrest determines the validity of the incidental search,
the legality of the arrest is questioned, e.g., whether an arrest was merely used as a pretext for conducting a
search. In this instance, the law requires that there first be arrest before a search can be made—the process
cannot be reversed. Accordingly, for this exception to apply, two elements must concur: (1) the person to be
arrested must execute an overt act indicating that he has just committed, is actually committing, or is attempting
to commit a crime; and (2) such overt act is done in the presence or within the view of the arresting officer.

We find the two aforementioned elements lacking in the case at bar. Accused-appellant did not act in a suspicious
manner. For all intents and purposes, there was no overt manifestation that accused-appellant has just
committed, is actually committing, or is attempting to commit a crime. “Reliable information” alone, absent any
overt act indicative of a felonious enterprise in the presence and within the view of the arresting officers, is not
sufficient to constitute probable cause that would justify an in flagrante delicto arrest.

With regard to the concept of “stop-and frisk”: mere suspicion or a hunch will not validate a “stop-and-frisk”. A
genuine reason must exist, in light of the police officer’s experience and surrounding conditions, to warrant the
belief that the person detained has weapons concealed about him. Finally, a “stop-and-frisk” serves a two-fold
interest: (1) the general interest of effective crime prevention and detection for purposes of investigating possible
criminal behavior even without probable cause; and (2) the interest of safety and self-preservation which permit
the police officer to take steps to assure himself that the person with whom he deals is not armed with a deadly
weapon that could unexpectedly and fatally be used against the police officer.

A stop-and-frisk was defined as the act of a police officer to stop a citizen on the street, interrogate him, and pat
him for weapon(s) or contraband. It should also be emphasized that a search and seizure should precede the
arrest for this principle to apply. The foregoing circumstances do not obtain in the case at bar. To reiterate,
accused-appellant was first arrested before the search and seizure of the alleged illegal items found in his
possession. The apprehending police operative failed to make any initial inquiry into accused-appellant’s business
in the vicinity or the contents of the Zest-O juice box he was carrying. The apprehending police officers only
introduced themselves when they already had custody of accused-appellant.

In the case at bar, neither the in flagrante delicto nor the “stop and frisk” principles is applicable to justify the
warrantless arrest and consequent search and seizure made by the police operatives on accused-appellant.

Wherefore, accused-appellant Binad Sy Chua is hereby Acquitted.

GO VS. COURT OF APPEALS


[206 SCRA 138; G.R. NO. 101837; 11 FEB 1992]

Facts:

Petitioner, while traveling in the wrong direction on a one-way street, almost had a collision with another vehicle.
Petitioner thereafter got out of his car, shot the driver of the other vehicle, and drove off. An eyewitness of the
incident was able to take down petitioner’s plate number and reported the same to the police, who subsequently
ordered a manhunt for petitioner. 6 days after the shooting, petitioner presented himself in the police station,
accompanied by 2 lawyers, the police detained him. Subsequently a criminal charge was brought against him.

Page 77
Section 1-C, SY ’06-‘07
San Beda College of Law – Alabang
Constitutional Law 2 Case Digests

Petitioner posted bail, the prosecutor filed the case to the lower court, setting and commencing trial without
preliminary investigation. Prosecutor reasons that the petitioner has waived his right to preliminary investigation
as bail has been posted and that such situation, that petitioner has been arrested without a warrant lawfully, falls
under Section 5, Rule 113 and Section 7, Rule 112 of The 1985 Rules of Criminal Procedure which provides for
the rules and procedure pertaining to situations of lawful warrantless arrests. Petitioner in his petition for certiorari
assails such procedure and actions undertaken and files for a preliminary investigation.

Issue:

Whether or Not warrantless arrest of petitioner was lawful.

Whether or Not petitioner effectively waived his right to preliminary investigation.

Held:

Petitioner and prosecutor err in relying on Umil v. Ramos, wherein the Court upheld the warrantless arrest as
valid effected 1 to 14 days from actual commission of the offenses, which however constituted “continuing
crimes,” i.e. subversion, membership in an outlawed organization, etc. There was no lawful warrantless arrest
under Section 5, Rule 113. This is because the arresting officers were not actually there during the incident, thus
they had no personal knowledge and their information regarding petitioner were derived from other sources.
Further, Section 7, Rule 112, does not apply.

Petitioner was not arrested at all, as when he walked in the police station, he neither expressed surrender nor any
statement that he was or was not guilty of any crime. When a complaint was filed to the prosecutor, preliminary
investigation should have been scheduled to determine probable cause. Prosecutor made a substantive error,
petitioner is entitled to preliminary investigation, necessarily in a criminal charge, where the same is required
appear thereat. Petition granted, prosecutor is ordered to conduct preliminary investigation, trial for the criminal
case is suspended pending result from preliminary investigation, petitioner is ordered released upon posting a bail
bond.

CALLANTA VS. VILLANUEVA


[77 SCRA 377; G.R. NOS. 24646 & L-24674; 20 JUN 1977]

Facts:

Two complaints for grave oral defamation were filed against Faustina Callanta. The City Judge of Dagupan City,
Felipe Villanueva, denied the motions to quash the complaints. Thus, petitioner Callanta brought the suits for
certiorari in the Supreme Court. Petitioner questions the validity of the issuance of warrant of arrest by
respondent, arguing that the City Fiscal should have conducted the preliminary investigation. According to
petitioner’s counsel, there was jurisdictional infirmity. After the issuance of the warrants of arrest and the bail fixed
at P600, petitioner posted the bail bond, thus obtaining her provisional liberty. The City Fiscal in this case did not
disagree with the judge’s investigation, and agreed with the complaints filed.

Issue:

Whether or Not petitioner’s contentions are to be given merit.

Held:

Based on many precedent cases of the Supreme Court, “where the accused has filed bail and waived the
preliminary investigation proper, he has waived whatever defect, if any, in the preliminary examination conducted
prior to the issuance of the warrant of arrest”. In the case at bar, it is futile for the petitioner to question the validity
of the issuance of the warrant of arrest, because she posted the bail bond. Petitioner also erred in arguing that
only the City Fiscal can conduct a preliminary investigation. According to the Charter of the City of Dagupan, “the
City Court of Dagupan City may also conduct preliminary investigation for any offense, without regard to the limits
of punishment, and may release, or commit and bind over any person charged with such offense to secure his
appearance before the proper court”. Petition for certiorari is denied. Restraining order issued by the Court is lifted
and set aside.

POSADAS VS. COURT OF APPEALS

Page 78
Section 1-C, SY ’06-‘07
San Beda College of Law – Alabang
Constitutional Law 2 Case Digests

[188 SCRA 288; G.R. NO. 89139; 2 AUG 1990]

Facts:

Members of the Integrated National Police (INP) of the Davao Metrodiscom assigned with the Intelligence Task
Force, Pat. Ursicio Ungab and Pat. Umbra Umpar conducted surveillance along Magallanes Street, Davao City.
While in the vicinity of Rizal Memorial Colleges they spotted petitioner carrying a "buri" bag and they noticed him
to be acting suspiciously. They approached the petitioner and identified themselves as members of the INP.
Petitioner attempted to flee but his attempt to get away was unsuccessful. They then checked the "buri" bag of the
petitioner where they found one (1) caliber .38 Smith & Wesson revolver with Serial No. 770196, two (2) rounds of
live ammunition for a .38 caliber gun, a smoke (tear gas) grenade, and two (2) live ammunitions for a .22 caliber
gun. They brought the petitioner to the police station for further investigation. In the course of the same, the
petitioner was asked to show the necessary license or authority to possess firearms and ammunitions found in his
possession but he failed to do so. He was then taken to the Davao Metrodiscom office and the prohibited articles
recovered from him were indorsed to M/Sgt. Didoy the officer then on duty. He was prosecuted for illegal
possession of firearms and ammunitions in the Regional Trial Court of Davao City.

Issue:

Whether or Not the warantless search is valid.

Held:

In justifying the warrantless search of the buri bag then carried by the petitioner, argues that under Section 12,
Rule 136 of the Rules of Court a person lawfully arrested may be searched for dangerous weapons or anything
used as proof of a commission of an offense without a search warrant. It is further alleged that the arrest without a
warrant of the petitioner was lawful under the circumstances.

in the case at bar, there is no question that, indeed, it is reasonable considering that it was effected on the basis
of a probable cause. The probable cause is that when the petitioner acted suspiciously and attempted to flee with
the buri bag there was a probable cause that he was concealing something illegal in the bag and it was the right
and duty of the police officers to inspect the same.

It is too much indeed to require the police officers to search the bag in the possession of the petitioner only after
they shall have obtained a search warrant for the purpose. Such an exercise may prove to be useless, futile and
much too late.

Clearly, the search in the case at bar can be sustained under the exceptions heretofore discussed, and hence,
the constitutional guarantee against unreasonable searches and seizures has not been violated.

PEOPLE V. MENGOTE
[210 SCRA 174; G.R. NO. 87059; 22 JUN 1992]

Facts:

The Western Police District received a telephone call from an informer that there were three suspicious looking
persons at the corner of Juan Luna and North Bay Boulevard in Tondo, Manila. A surveillance team of
plainclothesmen was forthwith dispatched to the place. The patrolmen saw two men looking from side to side, one
of whom holding his abdomen. They approached the persons and identified themselves as policemen, whereupon
the two tried to run but unable to escape because the other lawmen surrounded them. The suspects were then
searched. One of them the accused-appellant was found with a .38 caliber with live ammunitions in it, while his
companion had a fan knife. The weapons were taken from them and they were turned over to the police
headquarters for investigation. An information was filed before the RTC convicting the accused of illegal
possession of firearm arm. A witness testified that the weapon was among the articles stolen at his shop, which
he reported to the police including the revolver. For his part, Mengote made no effort to prove that he owned the
fire arm or that he was licensed to possess it but instead, he claimed that the weapon was planted on him at the
time of his arrest. He was convicted for violation of P.D.1866 and was sentenced to reclusion perpetua. In his
appeal he pleads that the weapon was not admissible as evidence against him because it had been illegally
seized and therefore the fruit of a poisonous tree.

Issue:

Whether or not the warrantless search and arrest was illegal.

Page 79
Section 1-C, SY ’06-‘07
San Beda College of Law – Alabang
Constitutional Law 2 Case Digests

Held:

An evidence obtained as a result of an illegal search and seizure inadmissible in any proceeding for any purpose
as provided by Art. III sec 32 of the Constitution. Rule 113 sec.5 of the Rules of Court, provides arrest without
warrant lawful when: (a) the person to be arrested has committed, is actually committing, or is attempting to
commit an offense, (b) when the offense in fact has just been committed, and he has personal knowledge of the
facts indicating the person arrested has committed it and (c) the person to be arrested has escaped from a penal
establishment or a place where he is serving final judgment or temporarily confined while his case is pending, or
has escaped while being transferred from one confinement to another.

These requirements have not been established in the case at bar. At the time of the arrest in question, the
accused appellant was merely looking from side to side and holding his abdomen, according to the arresting
officers themselves. There was apparently no offense that has just been committed or was being actually
committed or at least being attempt by Mengote in their presence. Moreover a person may not be stopped and
frisked in a broad daylight or on a busy street on unexplained suspicion.

Judgment is reversed and set aside. Accused-appellant is acquitted.

PEOPLE VS. TANGLIBEN


[184 SCRA 220; G.R. No.L-63630; 6 Apr 1990]

Facts:

Patrolmen Silverio and Romeo Punzalan were conducting surveillance at the San Fernando Victory Liner
Terminal. At around 9:30pm they noticed a person, Medel Tangliben, carrying a traveling bag who acted
suspiciously. They confronted him, inspected his bag, and there they found marijuana leaves. The accused was
then taken to the Police Headquarters for further investigations. The TC found Tangliben guilty of violating sec.4
art. 2 of the RA 6425 or the Dangerous Drugs Act of 1972.

Issue:

Whether or Not there was an unlawful search due to lack of search warrant.

Held;

No. Rule 113 sec. 5 provides the a peace officer or a private person may w/o a warrant arrest a person when in
his presence the person to be arrested has committed, is committing, or is attempting to commit an offense.

In the present case, the accused was found to have been committing possession of marijuana and can be
therefore searched lawfully even without a search warrant. Another reason is that this case poses urgency on the
part of the arresting police officers. It was found out that an informer pointed to the accused telling the policemen
that the accused was carrying marijuana. The police officers had to act quickly and there was not enough time to
secure a search warrant.

PEOPLE VS. MALMSTEDT


[198 SCRA 401; G.R. No. 91107; 19 Jun 1991]

Facts:

In an information filed against the accused- appellant Mikael Malmstead was charged before the RTC of La
Trinidad, Benguet, for violation of Section 4, Art. II of Republic Act 6425, as amended, otherwise known as the
Dangerous Drugs Act of 1972, as amended.
Accused Mikael Malmstedt, a Swedish national, entered the Philippines for the third time in December 1988 as a
tourist. He had visited the country sometime in 1982 and 1985.

In the evening of 7 May 1989, accused left for Baguio City. Upon his arrival thereat in the morning of the following
day, he took a bus to Sagada and stayed in that place for two (2) days. Then in the 7 in the morning of May 11,
1989, the accused went to Nangonogan bus stop in Sagada.

At about 8: 00 o'clock in the morning of that same day (11 May 1989), Captain Alen Vasco, the Commanding
Officer of the First Regional Command (NARCOM) stationed at Camp Dangwa, ordered his men to set up a
temporary checkpoint at Kilometer 14, Acop, Tublay, Mountain Province, for the purpose of checking all vehicles
coming from the Cordillera Region. The order to establish a checkpoint in the said area was prompted by
persistent reports that vehicles coming from Sagada were transporting marijuana and other prohibited drugs.
Moreover, information was received by the Commanding Officer of NARCOM, that same morning that a

Page 80
Section 1-C, SY ’06-‘07
San Beda College of Law – Alabang
Constitutional Law 2 Case Digests

Caucasian coming from Sagada had in his possession prohibited drugs. The group composed of seven (7)
NARCOM officers, in coordination with Tublay Police Station, set up a checkpoint at the designated area at about
10:00 o'clock in the morning and inspected all vehicles coming from the Cordillera Region.

The two (2) NARCOM officers started their inspection from the front going towards the rear of the bus. Accused
who was the sole foreigner riding the bus was seated at the rear thereof.

During the inspection, CIC Galutan noticed a bulge on accused's waist. Suspecting the bulge on accused's waist
to be a gun, the officer asked for accused's passport and other identification papers. When accused failed to
comply, the officer required him to bring out whatever it was that was bulging on his waist. The bulging object
turned out to be a pouch bag and when accused opened the same bag, as ordered, the officer noticed four (4)
suspicious-looking objects wrapped in brown packing tape, prompting the officer to open one of the wrapped
objects. The wrapped objects turned out to contain hashish, a derivative of marijuana.

Thereafter, accused was invited outside the bus for questioning. But before he alighted from the bus, accused
stopped to get two (2) travelling bags from the luggage carrier. Upon stepping out of the bus, the officers got the
bags and opened them. A teddy bear was found in each bag. Feeling the teddy bears, the officer noticed that
there were bulges inside the same which did not feel like foam stuffing. It was only after the officers had opened
the bags that accused finally presented his passport.

Accused was then brought to the headquarters of the NARCOM at Camp Dangwa, La Trinidad, Benguet for
further investigation. At the investigation room, the officers opened the teddy bears and they were found to also
contain hashish. Representative samples were taken from the hashish found among the personal effects of
accused and the same were brought to the PC Crime Laboratory for chemical analysis.
In the chemistry report, it was established that the objects examined were hashish. a prohibited drug which is a
derivative of marijuana. Thus, an information was filed against accused for violation of the Dangerous Drugs Act.
ACCUSED’S DEFENSE
During the arraignment, accused entered a plea of "not guilty." For his defense, he raised the issue of illegal
search of his personal effects. He also claimed that the hashish was planted by the NARCOM officers in his
pouch bag and that the two (2) travelling bags were not owned by him, but were merely entrusted to him by an
Australian couple whom he met in Sagada. He further claimed that the Australian couple intended to take the
same bus with him but because there were no more seats available in said bus, they decided to take the next ride
and asked accused to take charge of the bags, and that they would meet each other at the Dangwa Station.

The trial court found the guilt of the accused Mikael Malmstedt established beyond reasonable doubt.

Seeking the reversal of the decision of the trial court finding him guilty of the crime charged, accused argues that
the search of his personal effects was illegal because it was made without a search warrant and, therefore, the
prohibited drugs which were discovered during the illegal search are not admissible as evidence against him.

Issue:

Whether or Not the contention of the accused is valid, and therefore the RTC ruling be reversed.

Held:

The Constitution guarantees the right of the people to be secure in their persons, houses, papers and effects
against unreasonable searches and seizures. However, where the search is made pursuant to a lawful arrest,
there is no need to obtain a search warrant. A lawful arrest without a warrant may be made by a peace officer or a
private person under the following circumstances.

Sec. 5 Arrest without warrant; when lawful. –– A peace officer or a private person may, without a
warrant, arrest a person:

(a) When, in his presence, the person to be arrested has committed is actually committing,
or is attempting to commit an offense;
(b) When an offense has in fact just been committed, and he has personal knowledge of
facts indicating that the person to be arrested has committed it; and
(c) When the person to be arrested is a prisoner who has escaped from a penal
establishment or place where he is serving final judgment or temporarily confined while
his case is pending, or has escaped while being transferred from one confinement to
another.

Accused was searched and arrested while transporting prohibited drugs (hashish). A crime was actually being
committed by the accused and he was caught in flagrante delicto. Thus, the search made upon his personal
effects falls squarely under paragraph (1) of the foregoing provisions of law, which allow a warrantless search

Page 81
Section 1-C, SY ’06-‘07
San Beda College of Law – Alabang
Constitutional Law 2 Case Digests

incident to a lawful arrest. While it is true that the NARCOM officers were not armed with a search warrant when
the search was made over the personal effects of accused, however, under the circumstances of the case, there
was sufficient probable cause for said officers to believe that accused was then and there committing a crime.

Probable cause has been defined as such facts and circumstances which could lead a reasonable, discreet and
prudent man to believe that an offense has been committed, and that the objects sought in connection with the
offense are in the place sought to be searched. Warrantless search of the personal effects of an accused has
been declared by this Court as valid, because of existence of probable cause, where the smell of marijuana
emanated from a plastic bag owned by the accused, 10 or where the accused was acting suspiciously, 11 and
attempted to flee.

The appealed judgment of conviction by the trial court is hereby affirmed. Costs against the accused-appellant.

PEOPLE VS. AMMINUDIN


[163 SCRA 402; G.R. L-74869; 6 Jul 1988]

Facts:

Idel Aminnudin, accused-appellant was arrested on June 25, 1984, shortly after disembarking from the M/V
Wilcon 9 at about 8:30 in the evening, in Iloilo City. The PC officers who were in fact waiting for him because of a
tip from one their informers simply accosted him, inspected his bag and finding what looked liked marijuana
leaves took him to their headquarters for investigation. The two bundles of suspect articles were confiscated from
him and later taken to the NBI laboratory for examination. It was found to contain three kilos of what were later
analyzed as marijuana leaves by an NBI forensic examiner. An information for violation of the Dangerous Drugs
Act was filed against him. Later, the information was amended to include Farida Ali y Hassen, who had also been
arrested with him that same evening and likewise investigated. Both were arraigned and pleaded not guilty.
Subsequently, the fiscal filed a motion to dismiss the charge against Ali on the basis of a sworn statement of the
arresting officers absolving her after a 'thorough investigation." The motion was granted, and trial proceeded only
against the accused-appellant, who was eventually convicted . In his defense, Aminnudin disclaimed the
marijuana, averring that all he had in his bag was his clothing consisting of a jacket, two shirts and two pairs of
pants. He alleged that he was arbitrarily arrested and immediately handcuffed. His bag was confiscated without a
search warrant. At the PC headquarters, he was manhandled to force him to admit he was carrying the marijuana,
the investigator hitting him with a piece of wood in the chest and arms even as he parried the blows while he was
still handcuffed. He insisted he did not even know what marijuana looked like and that his business was selling
watches and sometimes cigarettes. However the RTC rejected his allegations. Saying that he only has two
watches during that time and that he did not sufficiently proved the injuries allegedly sustained.

Issue:

Whether or not search of defendant’s bag is legal.

Held:

The search was illegal. Defendant was not caught in flagrante delicto, which could allow warrantless arrest or
search. At the moment of his arrest, he was not committing a crime. Nor was he about to do so or had just done
so. To all appearances, he was like any of the other passengers innocently disembarking from the vessel. The
said marijuana therefore could not be appreciated as evidence against the defendant, and furthermore he is
acquitted of the crime as charged.

PEOPLE VS. SAYCON


[236 SCRA 325; G.R. NO. 110995; 5 SEPT 1994]

Facts:

On or about 8 July 1992, at about 6:00 in the morning, the Coastguard personnel received information from
NARCOM agent Ruben Laddaran that a suspected "shabu" courier by the name of Alvaro Saycon was on board
the MV Doña Virginia, which was arriving at that moment in Dumaguete City. Upon receipt of the information, the
Coastguard chief officer CPO Tolin, instructed them to intercept the suspect. A combined team of NARCOM
agents and Philippine Coastguard personnel consisting of CPO Tolin, a certain Miagme, and Senior Police
Officers Ruben Laddaran and Winifredo Noble of NARCOM posted themselves at the gate of Pier 1. The MV
Doña Virginia docked at 6:00 a.m. that same morning at Pier 1 in Dumaguete City. Alvaro Saycon alighted from

Page 82
Section 1-C, SY ’06-‘07
San Beda College of Law – Alabang
Constitutional Law 2 Case Digests

the boat carrying a black bag and went through the checkpoint manned by the Philippine Coastguard where he
was identified by police officer Winifredo Noble of NARCOM. Saycon was then invited to the Coastguard
Headquarters at the Pier area. He willingly went with them. At the headquarters, the coastguard asked Saycon to
open his bag, and the latter willingly obliged. In it were personal belongings and a maong wallet. Inside that
maong wallet, there was a Marlboro pack containing the suspected "shabu". When police officer Winifredo Noble
asked Saycon whether the Marlboro pack containing the suspected "shabu" was his, Saycon merely bowed his
head. Then Saycon, his bag and the suspected "shabu" were brought to the NARCOM office for booking. When
Alvaro Saycon was arrested, the NARCOM agents did not have a warrant of arrest. The PNP's Forensic Analyst
declared in court that she had conducted an examination of the specimens and found out that the specimens
weighed 4.2 grams in total, consisted of methamphetamine hydrochloride, more widely known as "shabu."

Issue:

Whether or Not the warrantless search was valid.

Held:

The warrantless search was valid, as the accused was a passenger of a motor vehicle. There was probable
cause to believe that the accused was carrying prohibited drugs. Three weeks earlier, agents of the Narcotics
Command bought methamine hydrochloride from him. An agent of the Narcotics Command reported that the
accused would be arriving on board the vessel and carrying methamphetamine hydrochloride with him. Drug
couriers do not go about their trade with some external sign indicating that they are transporting prohibited drugs.
This must be taken into account in determining probable cause.

PEOPLE VS. MUSA


[217 SCRA 597; G.,R. NO. 96177; 27 JAN 1993]

Facts:

A civilian informer gave the information that Mari Musa was engaged in selling marijuana in Suterville,
Zamboanga City. Sgt. Ani was ordered by NARCOM leader T/Sgt. Belarga, to conduct a surveillance and test
buy on Musa. The civilian informer guided Ani to Musa’s house and gave the description of Musa. Ani was able to
buy one newspaper-wrapped dried marijuana for P10.00.

The next day, a buy-bust was planned. Ani was to raise his right hand if he successfully buys marijuana from
Musa. As Ani proceeded to the house, the NARCOM team positioned themselves about 90 to 100 meters away.
From his position, Belarga could see what was going on. Musa came out of the house and asked Ani what he
wanted. Ani said he wanted more marijuana and gave Musa the P20.00 marked money. Musa went into the
house and came back, giving Ani two newspaper wrappers containing dried marijuana. Ani opened and
inspected it. He raised his right hand as a signal to the other NARCOM agents, and the latter moved in and
arrested Musa inside the house. Belarga frisked Musa in the living room but did not find the marked money (gave
it to his wife who slipped away). T/Sgt. Belarga and Sgt. Lego went to the kitchen and found a ‘cellophane
colored white and stripe hanging at the corner of the kitchen.’ They asked Musa about its contents but failed to
get a response. So they opened it and found dried marijuana leaves inside. Musa was then placed under arrest.

Issue:

Whether or Not the seizure of the plastic bag and the marijuana inside it is unreasonable, hence, inadmissible as
evidence.

Held:

Yes. It constituted unreasonable search and seizure thus it may not be admitted as evidence. The warrantless
search and seizure, as an incident to a suspect’s lawful arrest, may extend beyond the person of the one arrested
to include the premises or surroundings under his immediate control. Objects in the ‘plain view’ of an officer who
has the right to be in the position to have that view are subject to seizure and may be presented as evidence. The
‘plain view’ doctrine is usually applied where a police officer is not searching for evidence against the accused,
but nonetheless inadvertently comes across an incriminating object. It will not justify the seizure of the object
where the incriminating nature of the object is not apparent from the ‘plain view’ of the object.

Page 83
Section 1-C, SY ’06-‘07
San Beda College of Law – Alabang
Constitutional Law 2 Case Digests

In the case at bar, the plastic bag was not in the ‘plain view’ of the police. They arrested the accused in the living
room and moved into the kitchen in search for other evidences where they found the plastic bag. Furthermore,
the marijuana inside the plastic bag was not immediately apparent from the ‘plain view’ of said object.

Therefore, the ‘plain view’ does not apply. The plastic bag was seized illegally and cannot be presented in
evidence pursuant to Article III Section 3 (2) of the Constitution.

PITA VS. COURT OF APPEALS


[178 SCRA 362; G.R. NO. 80806; 5 OCT 1989]

Facts:

On December 1 and 3, 1983, pursuing an Anti-Smut Campaign initiated by the Mayor of the City of Manila,
Ramon D. Bagatsing, elements of the Special Anti-Narcotics Group, Auxilliary Services Bureau, Western Police
District, INP of the Metropolitan Police Force of Manila, seized and confiscated from dealers, distributors,
newsstand owners and peddlers along Manila sidewalks, magazines, publications and other reading materials
believed to be obscene, pornographic and indecent and later burned the seized materials in public at the
University belt along C.M. Recto Avenue, Manila, in the presence of Mayor Bagatsing and several officers and
members of various student organizations.

Among the publications seized, and later burned, was "Pinoy Playboy" magazines published and co-edited by
plaintiff Leo Pita.

Plaintiff filed a case for injunction with prayer for issuance of the writ of preliminary injunction against Mayor
Bagatsing and Narcisco Cabrera, as superintendent of Western Police District of the City of Manila, seeking to
enjoin said defendants and their agents from confiscating plaintiff’s magazines or from preventing the sale or
circulation thereof claiming that the magazine is a decent, artistic and educational magazine which is not per se
obscene, and that the publication is protected by the Constitutional guarantees of freedom of speech and of the
press. Plaintiff also filed an Urgent Motion for issuance of a temporary restraining order against indiscriminate
seizure, confiscation and burning of plaintiff's "Pinoy Playboy" Magazines, pending hearing on the petition for
preliminary injunction. The Court granted the temporary restraining order. The case was set for trial upon the
lapse of the TRO. RTC ruled that the seizure was valid. This was affirmed by the CA.

Issue:

Whether or Not the seizure was illegal.

Held:

The Court ruled that the government authorities have not shown the required proof to justify a ban and to warrant
confiscation of the literature. First of all, they were not possessed of a lawful court order: (1) finding the said
materials to be pornography, and (2) authorizing them to carry out a search and seizure, by way of a search
warrant. The court provides the following guidelines to be observed:

1. The authorities must apply for the issuance of a search warrant from a judge, if in their opinion an
obscenity seizure is in order;
2. The authorities must convince the court that the materials sought to be seized are obscene and pose a
clear and present danger of an evil substantive enough to warrant State interference and action;
3. The judge must determine whether or not the same are indeed obscene. The question is to be resolved
on a case-to-case basis and on the judge’s sound discretion;
4. If in the opinion of the court, probable cause exists, it shall issue the search warrant prayed for;
5. The proper suit is then brought in the court under Article 201 of the RPC (Obscene publications).
6. Any conviction is subject to appeal. The appellate court may assess whether or not the properties seized
are indeed obscene.

GUANZON VS. DE VILLA


[181 SCRA 623; G.R. 80508; 30 JAN 1990]

Facts:

The 41 petitioners alleged that the "saturation drive" or "aerial target zoning" that were conducted in their place
(Tondo Manila) were unconstitutional. They alleged that there is no specific target house to be search and that
there is no search warrant or warrant of arrest served. Most of the policemen are in their civilian clothes and
without nameplates or identification cards. The residents were rudely rouse from their sleep by banging on the
walls and windows of their houses. The residents were at the point of high-powered guns and herded like cows.
Men were ordered to strip down to their briefs for the police to examine their tattoo marks. The residents

Page 84
Section 1-C, SY ’06-‘07
San Beda College of Law – Alabang
Constitutional Law 2 Case Digests

complained that they're homes were ransacked, tossing their belongings and destroying their valuables. Some of
their money and valuables had disappeared after the operation. The residents also reported incidents of
maulings, spot-beatings and maltreatment. Those who were detained also suffered mental and physical torture to
extract confessions and tactical informations. The respondents said that such accusations were all lies.
Respondents contends that the Constitution grants to government the power to seek and cripple subversive
movements for the maintenance of peace in the state. The aerial target zoning were intended to flush out
subversives and criminal elements coddled by the communities were the said drives were conducted. They said
that they have intelligently and carefully planned months ahead for the actual operation and that local and foreign
media joined the operation to witness and record such event.

Issue:

Whether or Not the saturation drive committed consisted of violation of human rights.

Held:

It is not the police action per se which should be prohibited rather it is the procedure used or the methods which
"offend even hardened sensibilities" .Based on the facts stated by the parties, it appears to have been no
impediment to securing search warrants or warrants of arrest before any houses were searched or individuals
roused from sleep were arrested. There is no showing that the objectives sought to be attained by the "aerial
zoning" could not be achieved even as th rights of the squatters and low income families are fully protected.
However, the remedy should not be brought by a tazpaer suit where not one victim complaints and not one
violator is properly charged. In the circumstances of this taxpayers' suit, there is no erring soldier or policeman
whom the court can order prosecuted. In the absence of clear facts no permanent relief can be given.

In the meantime where there is showing that some abuses were committed, the court temporary restraint the
alleged violations which are shocking to the senses. Petition is remanded to the RTC of Manila.

PEOPLE VS. ARUTA


[288 SCRA 626; G.R. NO. 120515; 13 APR 1998]

Facts:

On Dec. 13, 1988, P/Lt. Abello was tipped off by his informant that a certain “Aling Rosa” will be arriving from
Baguio City with a large volume of marijuana and assembled a team. The next day, at the Victory Liner Bus
terminal they waited for the bus coming from Baguio, when the informer pointed out who “Aling Rosa” was, the
team approached her and introduced themselves as NARCOM agents. When Abello asked “aling Rosa” about the
contents of her bag, the latter handed it out to the police. They found dried marijuana leaves packed in a plastic
bag marked “cash katutak”.

Instead of presenting its evidence, the defense filed a demurrer to evidence alleging the illegality of the search
and seizure of the items. In her testimony, the accused claimed that she had just come from Choice theatre where
she watched a movie “Balweg”. While about to cross the road an old woman asked her for help in carrying a
shoulder bag, when she was later on arrested by the police. She has no knowledge of the identity of the old
woman and the woman was nowhere to be found. Also, no search warrant was presented.

The trial court convicted the accused in violation of the dangerous drugs of 1972

Issue:

Whether or Not the police correctly searched and seized the drugs from the accused.

Held:

The following cases are specifically provided or allowed by law:

1. Warrantless search incidental to a lawful arrest recognized under Section 12, Rule 126 of the Rules
of Court 8 and by prevailing jurisprudence
2. Seizure of evidence in "plain view," the elements of which are: (a) a prior valid intrusion based on the
valid warrantless arrest in which the police are legally present in the pursuit of their official duties; (b)
the evidence was inadvertently discovered by the police who had the right to be where they are; (c)
the evidence must be immediately apparent, and (d) "plain view" justified mere seizure of evidence
without further search;
3. Search of a moving vehicle. Highly regulated by the government, the vehicle's inherent mobility
reduces expectation of privacy especially when its transit in public thoroughfares furnishes a highly
reasonable suspicion amounting to probable cause that the occupant committed a criminal activity;

Page 85
Section 1-C, SY ’06-‘07
San Beda College of Law – Alabang
Constitutional Law 2 Case Digests

4. Consented warrantless search;


5. Customs search;
6. Stop and Frisk;
7. Exigent and Emergency Circumstances.

The essential requisite of probable cause must still be satisfied before a warrantless search and seizure can be
lawfully conducted.

The accused cannot be said to be committing a crime, she was merely crossing the street and was not acting
suspiciously for the Narcom agents to conclude that she was committing a crime. There was no legal basis to
effect a warrantless arrest of the accused’s bag, there was no probable cause and the accused was not lawfully
arrested.

The police had more than 24 hours to procure a search warrant and they did not do so. The seized marijuana was
illegal and inadmissible evidence.

RULE 113, RULES OF COURT

Section 5. Arrest without warrant; when lawful. — A peace officer or a private person may, without a warrant,
arrest a person:

(a) When, in his presence, the person to be arrested has committed, is actually committing, or is
attempting to commit an offense;
(b) When an offense has just been committed, and he has probable cause to believe based on personal
knowledge of facts or circumstances that the person to be arrested has committed it; and
(c) When the person to be arrested is a prisoner who has escaped from a penal establishment or place
where he is serving final judgment or is temporarily confined while his case is pending, or has
escaped while being transferred from one confinement to another.

In cases falling under paragraph (a) and (b) above, the person arrested without a warrant shall be forthwith
delivered to the nearest police station or jail and shall be proceeded against in accordance with section 7 of Rule
112.

RULE 126, RULES OF COURT

Section 2. Court where application for search warrant shall be filed. — An application for search warrant shall be
filed with the following:

a) Any court within whose territorial jurisdiction a crime was committed.


b) For compelling reasons stated in the application, any court within the judicial region where the crime was
committed if the place of the commission of the crime is known, or any court within the judicial region where
the warrant shall be enforced.

However, if the criminal action has already been filed, the application shall only be made in the court where the
criminal action is pending.

Section 7. Right to break door or window to effect search. — The officer, if refused admittance to the place of
directed search after giving notice of his purpose and authority, may break open any outer or inner door or
window of a house or any part of a house or anything therein to execute the warrant or liberate himself or any
person lawfully aiding him when unlawfully detained therein.

Section 12. Delivery of property and inventory thereof to court; return and proceedings thereon.

(a) The officer must forthwith deliver the property seized to the judge who issued the warrant, together with a
true inventory thereof duly verified under oath.
(b) Ten (10) days after issuance of the search warrant, the issuing judge shall ascertain if the return has
been made, and if none, shall summon the person to whom the warrant was issued and require him to
explain why no return was made. If the return has been made, the judge shall ascertain whether section
11 of this Rule has been complained with and shall require that the property seized be delivered to him.
The judge shall see to it that subsection (a) hereof has been complied with.
(c) The return on the search warrant shall be filed and kept by the custodian of the log book on search
warrants who shall enter therein the date of the return, the result, and other actions of the judge.

A violation of this section shall constitute contempt of court.

Page 86
Section 1-C, SY ’06-‘07
San Beda College of Law – Alabang
Constitutional Law 2 Case Digests

Page 87
Section 1-C, SY ’06-‘07
San Beda College of Law – Alabang
Constitutional Law 2 Case Digests

LIBERTY OF ABODE AND OF TRAVEL

Art 3, Sec. 6. “The liberty of abode and of changing the same within the limits prescribed by law shall not be
impaired except upon lawful order of the court. Neither shall the right to travel be impaired except in the interest of
national security, public safety, or public health, as may be provided by law.”

CAUNCA VS. SALAZAR


[82 PHIL 851; NO.L-2690; 1 JAN 1949]

Facts:

This is an action for habeas corpus brought by Bartolome Caunca in behalf of his cousin Estelita Flores who was
employed by the Far Eastern Employment Bureau, owned by Julia Salazar, respondent herein. An advanced
payment has already been given to Estelita by the employment agency, for her to work as a maid. However,
Estelita wanted to transfer to another residence, which was disallowed by the employment agency. Further she
was detained and her liberty was restrained. The employment agency wanted that the advance payment, which
was applied to her transportation expense from the province should be paid by Estelita before she could be
allowed to leave.

Issue:

Whether or Not an employment agency has the right to restrain and detain a maid without returning the advance
payment it gave?

Held:

An employment agency, regardless of the amount it may advance to a prospective employee or maid, has
absolutely no power to curtail her freedom of movement. The fact that no physical force has been exerted to keep
her in the house of the respondent does not make less real the deprivation of her personal freedom of movement,
freedom to transfer from one place to another, freedom to choose one’s residence. Freedom may be lost due to
external moral compulsion, to founded or groundless fear, to erroneous belief in the existence of an imaginary
power of an impostor to cause harm if not blindly obeyed, to any other psychological element that may curtail the
mental faculty of choice or the unhampered exercise of the will. If the actual effect of such psychological spell is to
place a person at the mercy of another, the victim is entitled to the protection of courts of justice as much as the
individual who is illegally deprived of liberty by duress or physical coercion.

M ANOTOC VS. COURT OF APPEALS


[142 SCRA 149; G.R. NO. L-62100; 30 MAY 1986]

Facts:

Petitioner was charged with estafa. He posted bail. Petitioner filed before each of the trial courts a motion entitled,
"motion for permission to leave the country," stating as ground therefor his desire to go to the United States,
"relative to his business transactions and opportunities." The prosecution opposed said motion and after due
hearing, both trial judges denied the same. Petitioner thus filed a petition for certiorari and mandamus before the
then Court of Appeals seeking to annul the orders dated March 9 and 26, 1982, of Judges Camilon and Pronove,
respectively, as well as the communication-request of the Securities and Exchange Commission, denying his
leave to travel abroad. He likewise prayed for the issuance of the appropriate writ commanding the Immigration
Commissioner and the Chief of the Aviation Security Command (AVSECOM) to clear him for departure. The
Court of Appeals denied the petition.

Petitioner contends that having been admitted to bail as a matter of right, neither the courts which granted him
bail nor the Securities and Exchange Commission which has no jurisdiction over his liberty could prevent him from
exercising his constitutional right to travel.

Issue:

Page 88
Section 1-C, SY ’06-‘07
San Beda College of Law – Alabang
Constitutional Law 2 Case Digests

Whether or Not his constitutional right to travel has been violated.

Held:

A court has the power to prohibit a person admitted to bail from leaving the Philippines. This is a necessary
consequence of the nature and function of a bail bond. The condition imposed upon petitioner to make himself
available at all times whenever the court requires his presence operates as a valid restriction on his right to travel.
Indeed, if the accused were allowed to leave the Philippines without sufficient reason, he may be placed beyond
the reach of the courts. Petitioner has not shown the necessity for his travel abroad. There is no indication that the
business transactions cannot be undertaken by any other person in his behalf.

M ARCOS VS. M ANGLAPUS


[177 SCRA 668; G.R. NO. 88211; 15 SEPT 1989]

Facts:

This case involves a petition of mandamus and prohibition asking the court to order the respondents Secretary of
Foreign Affairs, etc. To issue a travel documents to former Pres. Marcos and the immediate members of his
family and to enjoin the implementation of the President's decision to bar their return to the Philippines. Petitioners
assert that the right of the Marcoses to return in the Philippines is guaranteed by the Bill of Rights, specifically
Sections 1 and 6. They contended that Pres. Aquino is without power to impair the liberty of abode of the
Marcoses because only a court may do so within the limits prescribed by law. Nor the President impair their right
to travel because no law has authorized her to do so.

They further assert that under international law, their right to return to the Philippines is guaranteed particularly by
the Universal Declaration of Human Rights and the International Covenant on Civil and Political Rights, which has
been ratified by the Philippines.

Issue:

Whether or not, in the exercise of the powers granted by the constitution, the President (Aquino) may prohibit the
Marcoses from returning to the Philippines.

Held:

"It must be emphasized that the individual right involved is not the right to travel from the Philippines to other
countries or within the Philippines. These are what the right to travel would normally connote. Essentially, the right
involved in this case at bar is the right to return to one's country, a distinct right under international law,
independent from although related to the right to travel. Thus, the Universal Declaration of Human Rights and the
International Covenant on Civil and Political Rights treat the right to freedom of movement and abode within the
territory of a state, the right to leave the country, and the right to enter one's country as separate and distinct
rights. What the Declaration speaks of is the "right to freedom of movement and residence within the borders of
each state". On the other hand, the Covenant guarantees the right to liberty of movement and freedom to choose
his residence and the right to be free to leave any country, including his own. Such rights may only be restricted
by laws protecting the national security, public order, public health or morals or the separate rights of others.
However, right to enter one's country cannot be arbitrarily deprived. It would be therefore inappropriate to
construe the limitations to the right to return to ones country in the same context as those pertaining to the liberty
of abode and the right to travel.

The Bill of rights treats only the liberty of abode and the right to travel, but it is a well considered view that the right
to return may be considered, as a generally accepted principle of International Law and under our Constitution as
part of the law of the land.

The court held that President did not act arbitrarily or with grave abuse of discretion in determining that the return
of the Former Pres. Marcos and his family poses a serious threat to national interest and welfare. President
Aquino has determined that the destabilization caused by the return of the Marcoses would wipe away the gains
achieved during the past few years after the Marcos regime.

The return of the Marcoses poses a serious threat and therefore prohibiting their return to the Philippines, the
instant petition is hereby DISMISSED.

SILVERIO VS. COURT OF APPEALS


[195 SCRA 760 ; G.R. 94284; 8 APR 1991]

Page 89
Section 1-C, SY ’06-‘07
San Beda College of Law – Alabang
Constitutional Law 2 Case Digests

Facts:

Petitioner was charged with violation of Section 2 (4) of the revised securities act. Respondent filed to cancel the
passport of the petitioner and to issue a hold departure order. The RTC ordered the DFA to cancel petitioner’s
passport, based on the finding that the petitioner has not been arraigned and there was evidence to show that the
accused has left the country with out the knowledge and the permission of the court.

Issue:

Whether or Not the right to travel may be impaired by order of the court.

Held:

The bail bond posted by petitioner has been cancelled and warrant of arrest has been issued by reason that he
failed to appear at his arraignments. There is a valid restriction on the right to travel, it is imposed that the
accused must make himself available whenever the court requires his presence. A person facing criminal charges
may be restrained by the Court from leaving the country or, if abroad, compelled to return (Constitutional Law,
Cruz, Isagani A., 1987 Edition, p. 138). So it is also that "An accused released on bail may be re-arrested without
the necessity of a warrant if he attempts to depart from the Philippines without prior permission of the Court where
the case is pending (ibid., Sec. 20 [2nd
par. ]).

Article III, Section 6 of the 1987 Constitution should be interpreted to mean that while the liberty of travel may be
impaired even without Court Order, the appropriate executive officers or administrative authorities are not armed
with arbitrary discretion to impose limitations. They can impose limits only on the basis of "national security, public
safety, or public health" and "as may be provided by law," a limitive phrase which did not appear in the 1973 text
(The Constitution, Bernas, Joaquin G.,S.J., Vol. I, First Edition, 1987, p. 263). Apparently, the phraseology in the
1987 Constitution was a reaction to the ban on international travel imposed under the previous regime when there
was a Travel Processing Center, which issued certificates of eligibility to travel upon application of an interested
party (See Salonga vs. Hermoso & Travel Processing Center, No. 53622, 25 April 1980, 97 SCRA 121).

Holding an accused in a criminal case within the reach of the Courts by preventing his departure from the
Philippines must be considered as a valid restriction on his right to travel so that he may be dealt with in
accordance with law. The offended party in any criminal proceeding is the People of the Philippines. It is to their
best interest that criminal prosecutions should run their course and proceed to finality without undue delay, with
an accused holding himself amenable at all times to Court Orders and processes

DEFENSOR-SANTIAGO VS. VASQUEZ


[217 SCRA 633; G.R. NOS. 99289-90; 27 JAN 1993]

Facts:

An information was filed against petitioner with the Sandiganbayan for violation of the Anti Graft and Corrupt
Practices Act. The order of arrest was issued with bail for release fixed at Php. 15,000 so she filed a motion for
acceptance of cash bail bond. On the same day the Sandiganbayan issued a resolution authorizing the petitioner
to post cash bond which the later filed in the amount of Php.15, 000. Her arraignment was set, but petitioner
asked for the cancellation of her bail bond and that she be allowed provisional release on recognizance. The
Sandiganbayan deferred it. The Sandiganbayan issued a hold departure order against petitioner, by reason of the
announcement she made that she would be leaving for the U.S. to accept a fellowship a Harvard. In the instant
motion she submitted before the S.C. she argues that her right to travel is impaired.

Issue:

Whether or Not the petitioner’s right to travel is impaired.

Held:

The petitioner does not deny and as a matter of fact even made a public statement, that she he every intension of
leaving the country to pursue higher studies abroad. The court upholds the course of action of the Sandiganbayan
in taking judicial notice of such fact of petitioners pal to go abroad and in thereafter issuing a sua sponte the hold
departure order is but an exercise of respondent court’s inherent power to preserve and to maintain effectiveness
of its jurisdiction over the case and the person of the accused.

Page 90
Section 1-C, SY ’06-‘07
San Beda College of Law – Alabang
Constitutional Law 2 Case Digests

Also, the petitioner assumed obligations, when she posted bail bond. She holds herself amenable at all times to
the orders and process of eth court. She may legally be prohibited from leaving the country during the pendency
of the case. (Manotoc v. C.A.)

M ARCOS VS. SANDIGANBAYAN


[247 SCRA 127; G.R. NO. 115132-34; 9 AUG 1995]

Facts:

This is a petition for certiorari to set aside as arbitrary and in grave abuse of discretion resolutions of the
Sandiganbayan's First Division denying petitioner's motion for leave to travel abroad for medical treatment.

The former first lady Imelda Marcos was found guilty by the First Division of the Sandiganbayan of violating 3 of
the Anti Graft and Corrupt Practices Act. After conviction she filed a "Motion for Leave to Travel Abroad" to seek
diagnostic tests and treatment by practitioners of oriental medicine in China allegedly because of "a serious and
life threatening medical condition" requiring facilities not available in the Philippines that was denied. Then she
again filed an "Urgent Ex-Parte Motion for Permission to Travel Abroad" to undergo diagnosis and treatment in
China. This was supported by several medical reports that were prepared by her doctor Roberto Anastacio.

Again another Motion to leave was filed by Mrs. Marcos to US and Europe for treatment of several Heart diseases
alleging that the tests were not available here.

The presiding justice, Garchitorena, contacted Dr. Gregorio B. Patacsil, Officer-in-Charge of the Philippine Heart
Center, and later wrote him a letter, asking for "expert opinion on coronary medicine". The court still found no
merit to allow the petitioners motion to leave and denied all of the motions.

Petitioner filed a motion for reconsideration and a "Motion to Admit Clinical Summary and to Resolve Motion for
Reconsideration." Attached was a recent medical report and letters of Vice President Joseph E. Estrada offering
to be guarantor for the return of petitioner and those of twenty four members of the House of Representatives
requesting the court to allow petitioner to travel abroad. This was also denied by the Court also stating their
express disapproval of the involvement of the VP and the Cabinet members so as to influence the resolutions,
decisions or orders or any judicial action of respondent court.

Issue:

Whether or Not the Sandiganbayan erred in disallowing the Motion for Leave to Travel Abroad because it (1)
disregarded the medical findings (2) it motu propio contacted a third party asking the latter to give an opinion on
petitioner's motion and medical findings (3) said that there was no necessity to get medical treatment abroad.

Held:

No. The contention of the petitioner that was invalid to contact a third party asking the latter to give an opinion on
petitioner's motion and medical findings was erroneous. Respondent court had to seek expert opinion because
petitioner's motion was based on the advice of her physician. The court could not be expected to just accept the
opinion of petitioner's physician in resolving her request for permission to travel. What would be objectionable
would be if respondent court obtained information without disclosing its source to the parties and used it in
deciding a case against them.

In disregarding the medical reports, the petitioner failed to prove the necessity for a trip abroad. It should be
emphasized that considering the fact that she is facing charges before the courts in several cases, in two of which
she was convicted although the decision is still pending reconsideration, petitioner did not have an absolute right
to leave the country and the burden was on her to prove that because of danger to health if not to her life there
was necessity to seek medical treatment in foreign countries.

On the third issue, the Court ordered petitioner to undergo several tests which summarily states that the required
medical treatment was available here in the Philippines and that the expertise and facilities here were more than
adequate to cater to her medical treatment. The heart ailments of the petitioner were not as severe as that was
reported by Dr. Anastacio.

Wherefore, the petitioner is Dismissed without prejudice to the filling of another motion for leave to travel abroad,
should petitioner still desire, based on her heart condition. In such an event the determination of her medical
condition should be made by joint panel of medical specialists recommended by both the accused and the
prosecution.

RUBI VS. PROVINCIAL BOARD OF MINDORO

Page 91
Section 1-C, SY ’06-‘07
San Beda College of Law – Alabang
Constitutional Law 2 Case Digests

[39 PHIL 660; NO. 14078; 7 MAR 1919]

Facts:

The provincial board of Mindoro adopted resolution No. 25 wherein non-Christian inhabitants (uncivilized tribes)
will be directed to take up their habitation on sites on unoccupied public lands. It is resolved that under section
2077 of the Administrative Code, 800 hectares of public land in the sitio of Tigbao on Naujan Lake be selected as
a site for the permanent settlement of Mangyanes in Mindoro. Further, Mangyans may only solicit homesteads on
this reservation providing that said homestead applications are previously recommended by the provincial
governor.

In that case, pursuant to Section 2145 of the Revised Administrative Code, all the Mangyans in the townships of
Naujan and Pola and the Mangyans east of the Baco River including those in the districts of Dulangan and Rubi's
place in Calapan, were ordered to take up their habitation on the site of Tigbao, Naujan Lake. Also, that any
Mangyan who shall refuse to comply with this order shall upon conviction be imprisoned not exceed in sixty days,
in accordance with section 2759 of the revised Administrative Code.

Said resolution of the provincial board of Mindoro were claimed as necessary measures for the protection of the
Mangyanes of Mindoro as well as the protection of public forests in which they roam, and to introduce civilized
customs among them.

It appeared that Rubi and those living in his rancheria have not fixed their dwelling within the reservation of
Tigbao and are liable to be punished.

It is alleged that the Manguianes are being illegally deprived of their liberty by the provincial officials of that
province. Rubi and his companions are said to be held on the reservation established at Tigbao, Mindoro, against
their will, and one Dabalos is said to be held under the custody of the provincial sheriff in the prison at Calapan for
having run away form the reservation.

Issue:

Whether or Not Section 2145 of the Administrative Code deprive a person of his liberty pf abode. Thus, WON
Section 2145 of the Administrative Code of 1917 is constitutional.

Held:

The Court held that section 2145 of the Administrative Code does not deprive a person of his liberty of abode and
does not deny to him the equal protection of the laws, and that confinement in reservations in accordance with
said section does not constitute slavery and involuntary servitude. The Court is further of the opinion that section
2145 of the Administrative Code is a legitimate exertion of the police power. Section 2145 of the Administrative
Code of 1917 is constitutional.
Assigned as reasons for the action: (1) attempts for the advancement of the non-Christian people of the province;
and (2) the only successfully method for educating the Manguianes was to oblige them to live in a permanent
settlement. The Solicitor-General adds the following; (3) The protection of the Manguianes; (4) the protection of
the public forests in which they roam; (5) the necessity of introducing civilized customs among the Manguianes.

One cannot hold that the liberty of the citizen is unduly interfered without when the degree of civilization of the
Manguianes is considered. They are restrained for their own good and the general good of the Philippines.

“Liberty regulated by law": Implied in the term is restraint by law for the good of the individual and for the greater
good of the peace and order of society and the general well-being. No man can do exactly as he pleases.

None of the rights of the citizen can be taken away except by due process of law.

Therefore, petitioners are not unlawfully imprisoned or restrained of their liberty. Habeas corpus can, therefore,
not issue.

Page 92
Section 1-C, SY ’06-‘07
San Beda College of Law – Alabang
Constitutional Law 2 Case Digests

FREEDOM OF RELIGION

Art 3, Sec. 5. “No law shall be made respecting an establishment of religion, or prohibiting the free exercise
thereof. The free exercise and enjoyment of religious profession and worship, without discrimination or
preference, shall forever be allowed. No religious test shall be required for the exercise of civil or political rights.”

AGLIPAY VS. RUIZ


[64 PHIL 201; G.R. NO. 45459; 13 MAR 1937]

Facts:

Petitioner seeks the issuance of a writ of prohibition against respondent Director of Posts from issuing and selling
postage stamps commemorative of the 33rd International Eucharistic Congress. Petitioner contends that such act
is a violation of the Constitutional provision stating that no public funds shall be appropriated or used in the benefit
of any church, system of religion, etc. This provision is a result of the principle of the separation of church and
state, for the purpose of avoiding the occasion wherein the state will use the church, or vice versa, as a weapon
to further their ends and aims. Respondent contends that such issuance is in accordance to Act No. 4052,
providing for the appropriation funds to respondent for the production and issuance of postage stamps as would
be advantageous to the government.

Issue:

Whether or Not there was a violation of the freedom to religion.

Held:

What is guaranteed by our Constitution is religious freedom and not mere religious toleration. It is however not an
inhibition of profound reverence for religion and is not a denial of its influence in human affairs. Religion as a
profession of faith to an active power that binds and elevates man to his Creator is recognized. And in so far as it
instills into the minds the purest principles of morality, its influence is deeply felt and highly appreciated. The
phrase in Act No. 4052 “advantageous to the government” does not authorize violation of the Constitution. The
issuance of the stamps was not inspired by any feeling to favor a particular church or religious denomination.
They were not sold for the benefit of the Roman Catholic Church. The postage stamps, instead of showing a
Catholic chalice as originally planned, contains a map of the Philippines and the location of Manila, with the words
“Seat XXXIII International Eucharistic Congress.” The focus of the stamps was not the Eucharistic Congress but
the city of Manila, being the seat of that congress. This was to “to advertise the Philippines and attract more
tourists,” the officials merely took advantage of an event considered of international importance. Although such
issuance and sale may be inseparably linked with the Roman Catholic Church, any benefit and propaganda
incidentally resulting from it was no the aim or purpose of the Government.

GARCES VS. ESTENZO


[104 SCRA 510; G.R. L-53487; 25 MAY 1981]

Facts:

Two resolutions of the Barangay Council of Valencia, Ormoc City were passed:

a. Resolution No. 5- Reviving the traditional socio-religious celebration every fifth of April. This provided
for the acquisition of the image of San Vicente Ferrer and the construction of a waiting shed. Funds
for the said projects will be obtained through the selling of tickets and cash donations.
b. Resolution No. 6- The chairman or hermano mayor of the fiesta would be the caretaker of the image
of San Vicente Ferrer and that the image would remain in his residence for one year and until the
election of his successor. The image would be made available to the Catholic Church during the
celebration of the saint’s feast day.

These resolutions have been ratified by 272 voters, and said projects were implemented. The image was
temporarily placed in the altar of the Catholic Church of the barangay. However, after a mass, Father Sergio
Marilao Osmeña refused to return the image to the barangay council, as it was the church’s property since church
funds were used in its acquisition.

Resolution No. 10 was passed for the authorization of hiring a lawyer for the replevin case against the priest for
the recovery of the image. Resolution No. 12 appointed Brgy. Captain Veloso as a representative to the case. The
priest, in his answer assailed the constitutionality of the said resolutions. The priest with Andres Garces, a
member of the Aglipayan Church, contends that Sec. 8 Article IV1 and Sec 18(2) Article VIII) 2 of the constitution
was violated.

Page 93
Section 1-C, SY ’06-‘07
San Beda College of Law – Alabang
Constitutional Law 2 Case Digests

Issue:

Whether or Not any freedom of religion clause in the Constitution violated.

Held:

No. As said by the Court this case is a petty quarrel over the custody of the image. The image was purchased in
connection with the celebration of the barrio fiesta and not for the purpose of favoring any religion nor interfering
with religious matters or beliefs of the barrio residents. Any activity intended to facilitate the worship of the patron
saint(such as the acquisition) is not illegal. Practically, the image was placed in a layman’s custody so that it could
easily be made available to any family desiring to borrow the image in connection with prayers and novena. It was
the council’s funds that were used to buy the image, therefore it is their property. Right of the determination of
custody is their right, and even if they decided to give it to the Church, there is no violation of the Constitution,
since private funds were used. Not every government activity which involves the expenditure of public funds and
which has some religious tint is violative of the constitutional provisions regarding separation of church and state,
freedom of worship and banning the use of public money or property.

AMERICAN BIBLE SOCIETY VS. CITY OF MANILA


[101PHIL 386; G.R. NO. 9637; 30 APR 1957]

Facts:

New York’s Education Law requires local public school authorities to lend textbooks free of charge to all students
in grade 7 to 12, including those in private schools. The Board of Education contended that said statute was
invalid and violative of the State and Federal Constitutions. An order barring the Commissioner of Education
(Allen) from removing appellant’s members from office for failure to comply with the requirement and an order
preventing the use of state funds for the purchase of textbooks to be lent to parochial schools were sought for.
The trial court held the statute unconstitutional. The Appellate Division reversed the decision and dismissed the
complaint since the appellant have no standing. The New York Court of Appeals, ruled that the appellants have
standing but the law is not unconstitutional.

Issue:

Whether or Not the said ordinances are constitutional and valid (contention: it restrains the free exercise and
enjoyment of the religious profession and worship of appellant).

Held:

Section 1, subsection (7) of Article III of the Constitution, provides that:

(7) No law shall be made respecting an establishment of religion, or prohibiting the free exercise thereof,
and the free exercise and enjoyment of religious profession and worship, without discrimination or
preference, shall forever be allowed. No religion test shall be required for the exercise of civil or political
rights.

The provision aforequoted is a constitutional guaranty of the free exercise and enjoyment of religious profession
and worship, which carries with it the right to disseminate religious information.

It may be true that in the case at bar the price asked for the bibles and other religious pamphlets was in some
instances a little bit higher than the actual cost of the same but this cannot mean that appellant was engaged in
the business or occupation of selling said "merchandise" for profit. For this reason. The Court believe that the
provisions of City of Manila Ordinance No. 2529, as amended, cannot be applied to appellant, for in doing so it
would impair its free exercise and enjoyment of its religious profession and worship as well as its rights of
dissemination of religious beliefs.

With respect to Ordinance No. 3000, as amended, the Court do not find that it imposes any charge upon the
enjoyment of a right granted by the Constitution, nor tax the exercise of religious practices.

It seems clear, therefore, that Ordinance No. 3000 cannot be considered unconstitutional, however inapplicable to
said business, trade or occupation of the plaintiff. As to Ordinance No. 2529 of the City of Manila, as amended, is
also not applicable, so defendant is powerless to license or tax the business of plaintiff Society.
WHEREFORE, defendant shall return to plaintiff the sum of P5,891.45 unduly collected from it.

GERMAN VS. BARANGAN


[135 SCRA 514; G.R. NO. 68828; 27 MAR 1985]

Page 94
Section 1-C, SY ’06-‘07
San Beda College of Law – Alabang
Constitutional Law 2 Case Digests

Facts:

Petitioners converged at J.P. Laurel Street to hear Mass at the St. Jude Chapel, which adjoined Malacañang.
Respondent barred them for security reasons. Petitioners filed a petition for mandamus.

Issue:

Whether or Not there was a violation of the constitutional freedom.

Held:

Petitioners' intention was not really to perform an act of religious worship but to conduct an anti- government
demonstration since they wore yellow T-shirts, raised their clenched fists and shouted anti- government slogans.
While every citizen has the right to religious freedom, the exercise must be done in good faith. Besides, the
restriction was reasonable as it was designed to protect the lives of the President and his family, government
officials and diplomatic and foreign guests transacting business with Malacanang. The restriction was also
intended to secure the executive offices within the Malacanang grounds from possible external attacks and
disturbances. (Minority opinion) The sole justification for a prior restraint or limitation on the exercise of the
freedom of religion is the existence of a grave and imminent, of a serious evil to public safety, public morals,
public health or any other legitimate public interest that the State has a right to prevent. The burden to show the
existence of grave and imminent danger lies on the officials who would restrain petitioners. Respondents were in
full control and had the capability to stop any untoward move. There was no clear and present danger of any
serious evil to public safety or the security of Malacanang.

EBRALINAG VS. DIVISION SUPERINTENDENT OF CEBU


[219 SCRA 256 ; G.R. NO. 95770; 1 MAR 1993]

Facts:

Two special civil actions for certiorari, Mandamus and Prohibition were filed and consolidated for raising same
issue. Petitioners allege that the public respondents acted without or in excess of their jurisdiction and with grave
abuse of discretion. Respondents ordered expulsion of 68 HS and GS students of Bantayan, Pinamungajan,
Caracar, Taburan and Asturias in Cebu. Public school authorities expelled these students for refusing to salute
the flag, sing the national anthem and recite the “Panatang Makabayan” required by RA1265. They are
Jehovah’s Witnesses believing that by doing these is religious worship/devotion akin to idolatry against their
teachings. They contend that to compel transcends constitutional limits and invades protection against official
control and religious freedom. The respondents relied on the precedence of Gerona et al v. Secretary of
Education. Gerona doctrine provides that we are a system of separation of the church and state and the flag is
devoid of religious significance and it doesn’t involve any religious ceremony. The freedom of religious belief
guaranteed by the Constitution does not mean exception from non-discriminatory laws like the saluting of flag and
singing national anthem. This exemption disrupts school discipline and demoralizes the teachings of civic
consciousness and duties of citizenship.

Issue:

Whether or Not religious freedom has been violated.

Held:

Religious freedom is a fundamental right of highest priority. The 2 fold aspect of right to religious worship is: 1.)
Freedom to believe which is an absolute act within the realm of thought. 2.) Freedom to act on one’s belief
regulated and translated to external acts. The only limitation to religious freedom is the existence of grave and
present danger to public safety, morals, health and interests where State has right to prevent. The expulsion of
the petitioners from the school is not justified.

The 30 yr old previous GERONA decision of expelling and dismissing students and teachers who refuse to obey
RA1265 is violates exercise of freedom of speech and religious profession and worship. Jehovah’s Witnesses
may be exempted from observing the flag ceremony but this right does not give them the right to disrupt such
ceremonies. In the case at bar, the Students expelled were only standing quietly during ceremonies. By
observing the ceremonies quietly, it doesn’t present any danger so evil and imminent to justify their expulsion.
What the petitioner’s request is exemption from flag ceremonies and not exclusion from public schools. The
expulsion of the students by reason of their religious beliefs is also a violation of a citizen’s right to free education.
The non-observance of the flag ceremony does not totally constitute ignorance of patriotism and civic
consciousness. Love for country and admiration for national heroes, civic consciousness and form of government
are part of the school curricula. Therefore, expulsion due to religious beliefs is unjustified.

Page 95
Section 1-C, SY ’06-‘07
San Beda College of Law – Alabang
Constitutional Law 2 Case Digests

Petition for Certiorari and Prohibition is GRANTED. Expulsion is ANNULLED.

FONACIER VS. COURT OF APPEALS


[96 PHIL 417; G.R. L-5917; 28 JAN 1955]

Facts:

Case was filed by Iglesia Filipina Independiente (IFI), represented by its supreme bishop Gerardo Bayaca,
against Bishop Fonacier seeking to render an accounting of his administration of all the temporal properties and
to recover the same on the ground that he ceased to be the supreme bishop of IFI. Isabelo De los Reyes Jr. had
been elected as the Supreme Bishop.

Petitioner claims that he was not properly removed as Supreme Bishop and his legal successor was Juan Jamias.
He claims that the there was an accounting of his administration and was turned over to bishop Jamias. Also, that
Isabelo De los Reyes and Bayaca have abandoned their faith and formally joined the Prostestant Episcopal
Church of America.

CFI rendered judgment declaring Isabelo De Los Reyes, Jr. as the sole and legitimate Supreme Bishop of IFI and
ordered Fonacier to render an accounting of his admistration
CA affirmed the decision of the CFI

Issue:

Whether or not the petitioner should still be regarded as the legitimate supreme bishop of IFI.

Held:

Supreme Court affirmed CA’s decision. The legitimate Supreme Bishop of IFI is Isabelo De los Reyes, Jr. The
Supreme Court affirms the validity of the election of Bishop Delos Reyes as the Supreme Bishop based on their
internal laws

To finally dispose of the property issue, the Court, citing Watson v. Jones,368 declared that the rule in property
controversies within religious congregations strictly independent of any other superior ecclesiastical association
(such as the Philippine Independent Church) is that the rules for resolving such controversies should be those of
any voluntary association. If the congregation adopts the majority rule then the majority should prevail; if it adopts
adherence to duly constituted authorities within the congregation, then that should be followed.

PAMIL VS. TELECOM


[86 SCRA 413; G.R. 34854; 20 NOV 1978]

Facts:

Fr. Margarito Gonzaga was elected as Municipal Mayor in Alburquerque, Bohol. Petitioner, also an aspirant for
said office, then filed a suit for quo warranto for Gonzaga’s disqualification based on the Administrative Code
provision: “In no case shall there be elected or appointed to a municipal office ecclesiastics, soldiers in active
service, persons receiving salaries or compensation from provincial or national funds, or contractors for public
works of the municipality." The respondent Judge, in sustaiing Fr. Gonzaga’s right to the office, ruled that the
provision had already been impliedly repealed by the Election Code of 1971. Petitioner on the other hand argues
that there was no implied repeal.

Issue:

Whether or Not Fr. Gonzaga is eligible for the position of municipal mayor, according to law.

Whether or Not the prohibition regarding elected or appointed ecclesiastics is constitutional.

Held:

The court was divided. Five voted that the prohibition was not unconstitutional. Seven others voted that the
provision was impliedly repealed. However, the minority vote overruled the seven. According to the dissenting
seven, there are three reasons for the said provision to be inoperative. First, the 1935 Constitution stated, “No
religious test shall be required for the exercise of civil or political rights.” Second, said section 2175 is superseded
by the Constitution. Third, section 2175 has been repealed by Sec. 23 of the Election Code (1971): “Appointive

Page 96
Section 1-C, SY ’06-‘07
San Beda College of Law – Alabang
Constitutional Law 2 Case Digests

public office holders and active members of the Armed Forces are no longer disqualified from running for an
elective office”. Ecclesiastics were no longer included in the enumeration of persons ineligible under the said
Election Code. On the other hand, the controlling five argued: Section 2175 of the Administrative Code deals with
a matter different from that of section 23 of the Election Code. Also, section 2175 of the Administrative Code did
not violate the right to freedom of religion because it did not give any requirement for a religious test.

The view of the dissenting seven failed to obtain a vote of eight members, so it was not controlling. The provision
of the Administrative Code remained operative.

ESTRADA VS. ESCRITOR


[492 SCRA 1 ; AM NO P-02-1651; 22 JUN 2006]

Facts:

Escritor is a court interpreter since 1999 in the RTC of Las Pinas City. She has been living with Quilapio, a man
who is not her husband, for more than twenty five years and had a son with him as well. Respondent’s husband
died a year before she entered into the judiciary while Quilapio is still legally married to another woman.

Complainant Estrada requested the Judge of said RTC to investigate respondent. According to complainant,
respondent should not be allowed to remain employed therein for it will appear as if the court allows such act.

Respondent claims that their conjugal arrangement is permitted by her religion—the Jehovah’s Witnesses and the
Watch Tower and the Bible Trace Society. They allegedly have a ‘Declaration of Pledging Faithfulness’ under the
approval of their congregation. Such a declaration is effective when legal impediments render it impossible for a
couple to legalize their union.

Issue:

Whether or Not the State could penalize respondent for such conjugal arrangement.

Held:

No. The State could not penalize respondent for she is exercising her right to freedom of religion. The free
exercise of religion is specifically articulated as one of the fundamental rights in our Constitution. As Jefferson put
it, it is the most inalienable and sacred of human rights. The State’s interest in enforcing its prohibition cannot be
merely abstract or symbolic in order to be sufficiently compelling to outweigh a free exercise claim. In the case at
bar, the State has not evinced any concrete interest in enforcing the concubinage or bigamy charges against
respondent or her partner. Thus the State’s interest only amounts to the symbolic preservation of an unenforced
prohibition.

Furthermore, a distinction between public and secular morality and religious morality should be kept in mind. The
jurisdiction of the Court extends only to public and secular morality.

The Court further states that our Constitution adheres the benevolent neutrality approach that gives room for
accommodation of religious exercises as required by the Free Exercise Clause. This benevolent neutrality could
allow for accommodation of morality based on religion, provided it does not offend compelling state interests.
Assuming arguendo that the OSG has proved a compelling state interest, it has to further demonstrate that the
state has used the least intrusive means possible so that the free exercise is not infringed any more than
necessary to achieve the legitimate goal of the state. Thus the conjugal arrangement cannot be penalized for it
constitutes an exemption to the law based on her right to freedom of religion.

ISLAMIC DA'WAH COUNCIL OF THE PHILIPPINES VS. EXECUTIVE SECRETARY


[405 SCRA 497;GR 153888; 9 JUL 2003]

Facts:

Petitioner Islamic Da'wah Council of the Philippines, Inc. (IDCP) is a corporation that operates under Department
of Social Welfare and Development, a non-governmental organization that extends voluntary services to the
Filipino people, especially to Muslim communities. It claims to be a federation of national Islamic organizations
and an active member of international organizations such as the Regional Islamic Da'wah Council of Southeast
Asia and the Pacific (RISEAP) and The World Assembly of Muslim Youth. The RISEAP accredited petitioner to
issue halal certifications in the Philippines. Thus, among the functions petitioner carries out is to conduct

Page 97
Section 1-C, SY ’06-‘07
San Beda College of Law – Alabang
Constitutional Law 2 Case Digests

seminars, orient manufacturers on halal food and issue halal certifications to qualified products and
manufacturers.

Petitioner alleges that, the actual need to certify food products as halal and also due to halal food producers'
request, petitioner formulated in 1995 internal rules and procedures based on the Qur'an and the Sunnah for the
analysis of food, inspection thereof and issuance of halal certifications. In that same year, petitioner began to
issue, for a fee, certifications to qualified products and food manufacturers. Petitioner even adopted for use on its
halal certificates a distinct sign or logo registered in the Philippine Patent Office.

On 2001, respondent Office of the Executive Secretary issued EO 465 creating the Philippine Halal Certification
Scheme and designating respondent OMA to oversee its implementation. Under the EO, respondent OMA has
the exclusive authority to issue halal certificates and perform other related regulatory activities.

Issue:

Whether or Not EO violates the constitutional provision on the separation of Church and State.

Held:

It is unconstitutional for the government to formulate policies and guidelines on the halal certification scheme
because said scheme is a function only religious organizations, entity or scholars can lawfully and validly perform
for the Muslims. According to petitioner, a food product becomes halal only after the performance of Islamic
religious ritual and prayer. Thus, only practicing Muslims are qualified to slaughter animals for food. A government
agency like herein respondent OMA cannot therefore perform a religious function like certifying qualified food
products as halal. Without doubt, classifying a food product as halal is a religious function because the standards
used are drawn from the Qur'an and Islamic beliefs. By giving OMA the exclusive power to classify food products
as halal, EO 46 encroached on the religious freedom of Muslim organizations like herein petitioner to interpret for
Filipino Muslims what food products are fit for Muslim consumption. Also, by arrogating to itself the task of issuing
halal certifications, the State has in effect forced Muslims to accept its own interpretation of the Qur'an and
Sunnah on halal food.

In the case at bar, we find no compelling justification for the government to deprive Muslim organizations, like
herein petitioner, of their religious right to classify a product as halal, even on the premise that the health of
Muslim Filipinos can be effectively protected by assigning to OMA the exclusive power to issue halal certifications.
The protection and promotion of the muslim Filipinos' right to health are already provided for in existing laws and
ministered to by government agencies charged with ensuring that food products released in the market are fit for
human consumption, properly labeled and safe. Unlike EO 46, these laws do not encroach on the religious
freedom of Muslims.

Page 98
Section 1-C, SY ’06-‘07
San Beda College of Law – Alabang
Constitutional Law 2 Case Digests

FREEDOM OF EXPRESSION

Art 3, Sec. 4. “No law shall be passed abridging the freedom of speech, of expression, or of the press, or the
right of the people peaceably to assemble and petition the government for redress of grievances.”

Art 3, Sec. 7. “The right of the people to information on matters of public concern shall be recognized. Access
to official records, and to documents, and papers pertaining to official acts, transactions, or decisions, as well as
to government research data used as basis for policy development, shall be afforded the citizen, subject to such
limitations as may be provided by law.”

Art 3, Sec. 8. “The right of the people, including those employed in the public and private sectors, to form
unions, associations, or societies for purposes not contrary to law shall not be abridged.”

Art 3, Sec. 18. “(1) No person shall be detained solely by reason of his political beliefs and aspirations.”

NEAR VS. MINNESOTA


[283 US 697]

Facts:

A complaint alleged that the defendants, on September 24, 1927, and on eight subsequent dates in October and
November, 1927, published and circulated editions of “The Saturday Press”(published in Minneapolis) which were
'largely devoted to malicious, scandalous and defamatory articles'(based on Session Laws of Minnesota). The
articles charged, in substance, provides that a Jewish gangster was in control of gambling, bootlegging, and
racketeering in Minneapolis, and that law enforcing officers and agencies were not energetically performing their
duties. Most of the charges were directed against the chief of police; he was charged with gross neglect of duty,
illicit relations with gangsters, and with participation in graft. The county attorney was charged with knowing the
existing conditions and with failure to take adequate measures to remedy them. The mayor was accused of
inefficiency and dereliction. One member of the grand jury was stated to be in sympathy with the gangsters. A
special grand jury and a special prosecutor were demanded to deal with the situation in general, and, in particular,
to investigate an attempt to assassinate one Guilford, one of the original defendants, who, it appears from the
articles, was shot by gangsters after the first issue of the periodical had been published. Now defendants
challenged the Minnesota statute which provides for the abatement, as a public nuisance, of a malicious,
scandalous and defamatory news paper, magazine or other periodical. The District Court ruled against
defendants. Hence the appeal.

Issue:

Whether or Not the proceeding authorized by the statute herein constitutes an infringement of the freedom of the
press.

Held:

Yes. The insistence that the statute is designed to prevent the circulation of scandal which tends to disturb the
public peace and to provoke assaults and the commission of crime is unavailing.

The reason for the enactment, as the state court has said, is that prosecutions to enforce penal statutes for libel
do not result in 'efficient repression or suppression of the evils of scandal.' In the present instance, the proof was
that nine editions of the newspaper or periodical in question were published on successive dates, and that they
were chiefly devoted to charges against public officers and in relation to the prevalence and protection of crime. In
such a case, these officers are not left to their ordinary remedy in a suit for libel, or the authorities to a prosecution
for criminal libel. The statute not only operates to suppress the offending newspaper or periodical, but to put the
publisher under an effective censorship.

Every freeman has an undoubted right to lay what sentiments he pleases before the public; to forbid this, is to
destroy the freedom of the press; but if he publishes what is improper, mischievous or illegal, he must take the
consequence of his own temerity.

The liberty of the press was to be unrestrained, but he who used it was to be responsible in case of its abuse.'
Public officers, whose character and conduct remain open to debate and free discussion in the press, find their
remedies for false accusations in actions under libel laws providing for redress and punishment, and not in
proceedings to restrain the publication of newspapers and periodicals.

Page 99
Section 1-C, SY ’06-‘07
San Beda College of Law – Alabang
Constitutional Law 2 Case Digests

Characterizing the publication as a business, and the business as a nuisance, does not permit an invasion of the
constitutional immunity against restraint. Nor can it be said that the constitutional freedom from previous restraint
is lost because charges are made of derelictions which constitute crimes.

The preliminary freedom, by virtue of the very reason for its existence, does not depend, as this court has said, on
proof of truth.

GROSJEAN VS. AMERICAN PRESS CO.


[297 US 233]

Facts:

The nine publishers(corporations) who brought the suit publish thirteen newspapers and these thirteen
publications are the only ones within the state of Louisiana having each a circulation of more than 20,000 copies
per week. The suit assailed Act No. 231 of the Louisiana Legislature, as their freedom of the press was abridged
in contravention to the due process clause.

Issue:

Whether or Not Act 23 unconstitutional.

Held:

Yes. Freedom of speech and of the press are rights of the same fundamental character, safeguarded by the due
process of law clause. The word 'liberty' contained in that amendment embraces not only the right of a person to
be free from physical restraint, but the right to be free in the enjoyment of all his faculties as well.
The Act operates as a restraint in a double sense. First, its effect is to curtail the amount of revenue realized from
advertising; and, second, its direct tendency is to restrict circulation. This is plain enough when we consider that,
if it were increased to a high degree, as it could be if valid it well might result in destroying both advertising and
circulation.

Judge Cooley has laid down the test to be applied: The evils to be prevented were not the censorship of the press
merely, but any action of the government by means of which it might prevent such free and general discussion of
public matters as seems absolutely essential to prepare the people for an intelligent exercise of their rights as
citizens.

The tax here involved is bad not because it takes money from the pockets of the appellees. It is bad because, it is
seen to be a deliberate and calculated device in the guise of a tax to limit the circulation of information to which
the public is entitled in virtue of the constitutional guaranties. A free press stands as one of the great interpreters
between the government and the people.

The form in which the tax is imposed is in itself suspicious. It is not measured or limited by the volume of
advertisements. It is measured alone by the extent of the circulation of the publication in which the advertisements
are carried, with the plain purpose of penalizing the publishers and curtailing the circulation of a selected group of
newspapers.

NEW YORK TIMES VS. UNITED STATES


[403 US 713]

Facts:

The court granted certiorari in the cases in which the United States seeks to enjoin the New York Times and the
Washington Post from publishing the contents of a classified study entitled "History of U.S. Decision-Making
Process on Viet Nam Policy." Said articles reveal the workings of government that led to the Vietnam war. The
Government argues that "the authority of the Executive Department to protect the nation against publication of
information whose disclosure would endanger the national security stems from two interrelated sources: the
constitutional power of the President over the conduct of foreign affairs and his authority as Commander-in-Chief.
In such case the Executive Branch seeks judicial aid in preventing publication. The court ruled in favor of the
newspaper companies hence the appeal.

Issue:

Whether or not the freedom of the press was abridged.

Page 100
Section 1-C, SY ’06-‘07
San Beda College of Law – Alabang
Constitutional Law 2 Case Digests

Held:

Yes. To find that the President has "inherent power" to halt the publication of news by resort to the courts would
wipe out the First Amendment (Bill of Rights) and destroy the fundamental liberty and security of the very people
the Government hopes to make "secure."

No branch of government could abridge the people's rights granted by the Constitution including the freedom of
the press. The language of the First Amendment support the view that the press must be left free to publish news,
whatever the source, without censorship, injunctions, or prior restraints. The press was protected so that it could
bare the secrets of government and inform the people. Only a free and unrestrained press can effectively expose
deception in government. And paramount among the responsibilities of a free press is the duty to prevent any part
of the government from deceiving the people and sending them off to distant lands to die of foreign fevers and
foreign shot and shell.

GONZALES VS. COMELEC


[27 SCRA 835; G.R. L-27833; 18 APR 1969]

Facts:

RA 4880 which took effect on June 17, 1967, prohibiting the too early nomination of candidates and limiting the
period of election campaign or partisan political activity was challenged on constitutional grounds. More precisely,
the basic liberties of free speech and free press, freedom of assembly and freedom of association are invoked to
nullify the act. Petitioner Cabigao was, at the time of the filing the petition, an incumbent councilor in the 4th
District of Manila and the Nacionalista Party official candidate for Vice-Mayor of Manila to which he was
subsequently elected on November 11, 1967; petitioner Gonzales, on the other hand, is a private individual, a
registered voter in the City of Manila and a political leader of his co-petitioner. There was the further allegation
that the nomination of a candidate and the fixing of period of election campaign are matters of political expediency
and convenience which only political parties can regulate or curtail by and among themselves through self-
restraint or mutual understanding or agreement and that the regulation and limitation of these political matters
invoking the police power, in the absence of clear and present danger to the state, would render the constitutional
rights of petitioners meaningless and without effect. Senator Lorenzo M. Tañada was asked to appear as amicus
curiae, and elucidated that Act No. 4880 could indeed be looked upon as a limitation on the preferred rights of
speech and press, of assembly and of association. He did justify its enactment however under the clear and
present danger doctrine, there being the substantive evil of elections, whether for national or local officials, being
debased and degraded by unrestricted campaigning, excess of partisanship and undue concentration in politics
with the loss not only of efficiency in government but of lives as well. The Philippine Bar Association, the Civil
Liberties Union, the U.P. Law Center and the U.P. Women Lawyers' Circle were requested to give their opinions.
Respondents contend that the act was based on the police power of the state.

Issue:

Whether or Not RA 4880 unconstitutional.

Held:

Yes. As held in Cabansag v. Fernandez there are two tests that may supply an acceptable criterion for
permissible restriction on freedom of speech. These are the “clear and present danger” rule and the 'dangerous
tendency' rule. The first, means that the evil consequence of the comment or utterance must be extremely serious
and the degree of imminence extremely high before the utterance can be punished. The danger to be guarded
against is the 'substantive evil' sought to be prevented. It has the advantage of establishing according to the
above decision a definite rule in constitutional law. It provides the criterion as to what words may be publicly
established. The "dangerous tendency rule" is such that “If the words uttered create a dangerous tendency which
the state has a right to prevent, then such words are punishable.” It is not necessary that some definite or
immediate acts of force, violence, or unlawfulness be advocated. It is sufficient that such acts be advocated in
general terms. Nor is it necessary that the language used be reasonably calculated to incite persons to acts of
force, violence, or unlawfulness. It is sufficient if the natural tendency and probable effect of the utterance be to
bring about the substantive evil which the legislative body seeks to prevent.

The challenged statute could have been more narrowly drawn and the practices prohibited more precisely
delineated to satisfy the constitutional requirements as to a valid limitation under the clear and present danger
doctrine. As the author Tañada clearly explained, such provisions were deemed by the legislative body to be part
and parcel of the necessary and appropriate response not merely to a clear and present danger but to the actual
existence of a grave and substantive evil of excessive partisanship, dishonesty and corruption as well as violence
that of late has invariably marred election campaigns and partisan political activities in this country.

Page 101
Section 1-C, SY ’06-‘07
San Beda College of Law – Alabang
Constitutional Law 2 Case Digests

The very idea of a government, republican in form, implies a right on the part of its citizens to meet peaceably for
consultation in respect to public affairs and to petition for redress of grievances. As in the case of freedom of
expression, this right is not to be limited, much less denied, except on a showing of a clear and present danger of
a substantive evil that Congress has a right to prevent.

The prohibition of any speeches, announcements or commentaries, or the holding of interviews for or against the
election of any party or candidate for public office and the prohibition of the publication or distribution of campaign
literature or materials, against the solicitation of votes whether directly or indirectly, or the undertaking of any
campaign literature or propaganda for or against any candidate or party is repugnant to a constitutional command.

IGLESIA NI CRISTO VS. COURT OF APPEALS


[259 SCRA 529; G.R. NO. 119673; 26 JUL 1996]

Facts:

Petitioner has a television program entitled "Ang Iglesia ni Cristo" aired on Channel 2 every Saturday and on
Channel 13 every Sunday. The program presents and propagates petitioner's religious beliefs, doctrines and
practices often times in comparative studies with other religions. Petitioner submitted to the respondent Board of
Review for Moving Pictures and Television the VTR tapes of its TV program Series Nos. 116, 119, 121 and 128.
The Board classified the series as "X" or not for public viewing on the ground that they "offend and constitute an
attack against other religions which is expressly prohibited by law." On November 28, 1992, it appealed to the
Office of the President the classification of its TV Series No. 128 which allowed it through a letter of former
Executive Secretary Edelmiro A. Amante, Sr., addressed for Henrietta S. Mendez reversing the decision of the
respondent Board. According to the letter the episode in is protected by the constitutional guarantee of free
speech and expression and no indication that the episode poses any clear and present danger. Petitioner also
filed Civil Case. Petitioner alleged that the respondent Board acted without jurisdiction or with grave abuse of
discretion in requiring petitioner to submit the VTR tapes of its TV program and in x-rating them. It cited its TV
Program Series Nos. 115, 119, 121 and 128. In their Answer, respondent Board invoked its power under PD No.
19861 in relation to Article 201 of the Revised Penal Code. The Iglesia ni Cristo insists on the literal translation of
the bible and says that our (Catholic) veneration of the Virgin Mary is not to be condoned because nowhere it is
found in the bible. The board contended that it outrages Catholic and Protestant's beliefs. RTC ruled in favor of
petitioners. CA however reversed it hence this petition.

Issue:

Whether or Not the "ang iglesia ni cristo" program is not constitutionally protected as a form of religious exercise
and expression.

Held:

Yes. Any act that restrains speech is accompanied with presumption of invalidity. It is the burden of the
respondent Board to overthrow this presumption. If it fails to discharge this burden, its act of censorship will be
struck down. This is true in this case. So-called "attacks" are mere criticisms of some of the deeply held dogmas
and tenets of other religions. RTC’s ruling clearly suppresses petitioner's freedom of speech and interferes with its
right to free exercise of religion. “attack” is different from “offend” any race or religion. The respondent Board may
disagree with the criticisms of other religions by petitioner but that gives it no excuse to interdict such criticisms,
however, unclean they may be. Under our constitutional scheme, it is not the task of the State to favor any religion
by protecting it against an attack by another religion. Religious dogmas and beliefs are often at war and to
preserve peace among their followers, especially the fanatics, the establishment clause of freedom of religion
prohibits the State from leaning towards any religion. Respondent board cannot censor the speech of petitioner
Iglesia ni Cristo simply because it attacks other religions, even if said religion happens to be the most numerous
church in our country. The basis of freedom of religion is freedom of thought and it is best served by encouraging
the marketplace of dueling ideas. It is only where it is unavoidably necessary to prevent an immediate and grave
danger to the security and welfare of the community that infringement of religious freedom may be justified, and
only to the smallest extent necessary to avoid the danger. There is no showing whatsoever of the type of harm
the tapes will bring about especially the gravity and imminence of the threatened harm. Prior restraint on speech,
including religious speech, cannot be justified by hypothetical fears but only by the showing of a substantive and
imminent evil. It is inappropriate to apply the clear and present danger test to the case at bar because the issue
involves the content of speech and not the time, place or manner of speech. Allegedly, unless the speech is first
allowed, its impact cannot be measured, and the causal connection between the speech and the evil
apprehended cannot be established. The determination of the question as to whether or not such vilification,
exaggeration or fabrication falls within or lies outside the boundaries of protected speech or expression is a
judicial function which cannot be arrogated by an administrative body such as a Board of Censors." A system of
prior restraint may only be validly administered by judges and not left to administrative agencies.

Page 102
Section 1-C, SY ’06-‘07
San Beda College of Law – Alabang
Constitutional Law 2 Case Digests

ADIONG VS. COMELEC


[207 SCRA 712; G.R. NO. 103956; 31 MAR 1992]

Facts:

COMELEC promulgated Resolution No. 2347 which provides that decals and stickers may be posted only in any
of the authorized posting areas, prohibiting posting in "mobile" places, public or private. Petitioner Blo Umpar
Adiong, a senatorial candidate in the May 11, 1992 elections now assails the Resolution. In addition, the petitioner
believes that with the ban on radio, television and print political advertisements, he, being a neophyte in the field
of politics stands to suffer grave and irreparable injury with this prohibition.

Issue:

Whether or Not the COMELEC’s prohibition unconstitutional.

Held:

The prohibition unduly infringes on the citizen's fundamental right of free speech. The preferred freedom of
expression calls all the more for the utmost respect when what may be curtailed is the dissemination of
information to make more meaningful the equally vital right of suffrage. The so-called balancing of interests —
individual freedom on one hand and substantial public interests on the other — is made even more difficult in
election campaign cases because the Constitution also gives specific authority to the Commission on Elections to
supervise the conduct of free, honest, and orderly elections. When faced with border line situations where
freedom to speak by a candidate or party and freedom to know on the part of the electorate are invoked against
actions intended for maintaining clean and free elections, the police, local officials and COMELEC, should lean in
favor of freedom. The regulation of election campaign activity may not pass the test of validity if it is too general in
its terms or not limited in time and scope in its application, if it restricts one's expression of belief in a candidate or
one's opinion of his or her qualifications, if it cuts off the flow of media reporting, and if the regulatory measure
bears no clear and reasonable nexus with the constitutionally sanctioned objective.

The posting of decals and stickers in mobile places like cars and other moving vehicles does not endanger any
substantial government interest. There is no clear public interest threatened by such activity so as to justify the
curtailment of the cherished citizen's right of free speech and expression. Under the clear and present danger rule
not only must the danger be patently clear and pressingly present but the evil sought to be avoided must be so
substantive as to justify a clamp over one's mouth or a writing instrument to be stilled. The regulation strikes at the
freedom of an individual to express his preference and, by displaying it on his car, to convince others to agree
with him. A sticker may be furnished by a candidate but once the car owner agrees to have it placed on his private
vehicle, the expression becomes a statement by the owner, primarily his own and not of anybody else. The
restriction as to where the decals and stickers should be posted is so broad that it encompasses even the citizen's
private property, which in this case is a privately-owned vehicle. In consequence of this prohibition, another
cardinal rule prescribed by the Constitution would be violated. Section 1, Article III of the Bill of Rights provides
that no person shall be deprived of his property without due process of law.

The prohibition on posting of decals and stickers on "mobile" places whether public or private except in the
authorized areas designated by the COMELEC becomes censorship.

NATIONAL PRESS CLUB VS. COMELEC


[201 SCRA 1; G.R. NO. 1026653; 5 MAR 1992]

Facts:

Petitioners in these cases consist of representatives of the mass media which are prevented from selling or
donating space and time for political advertisements; two (2) individuals who are candidates for office (one for
national and the other for provincial office) in the coming May 1992 elections; and taxpayers and voters who claim
that their right to be informed of election Issue and of credentials of the candidates is being curtailed. It is
principally argued by petitioners that Section 11 (b) of Republic Act No. 6646 1 invades and violates the
constitutional guarantees comprising freedom of expression. Petitioners maintain that the prohibition imposed by
Section 11 (b) amounts to censorship, because it selects and singles out for suppression and repression with
criminal sanctions, only publications of a particular content, namely, media-based election or political propaganda
during the election period of 1992. It is asserted that the prohibition is in derogation of media's role, function and
duty to provide adequate channels of public information and public opinion relevant to election Issue. Further,
petitioners contend that Section 11 (b) abridges the freedom of speech of candidates, and that the suppression of
media-based campaign or political propaganda except those appearing in the Comelec space of the newspapers
and on Comelec time of radio and television broadcasts, would bring about a substantial reduction in the quantity

Page 103
Section 1-C, SY ’06-‘07
San Beda College of Law – Alabang
Constitutional Law 2 Case Digests

or volume of information concerning candidates and Issue in the election thereby curtailing and limiting the right of
voters to information and opinion.

Issue:

Whether or Not Section 11 (b) of Republic Act No. 6646 constitutional.

Held:

Yes. It seems a modest proposition that the provision of the Bill of Rights which enshrines freedom of speech,
freedom of expression and freedom of the press has to be taken in conjunction with Article IX (C) (4) which may
be seen to be a special provision applicable during a specific limited period — i.e., "during the election period." In
our own society, equality of opportunity to proffer oneself for public office, without regard to the level of financial
resources that one may have at one's disposal, is clearly an important value. One of the basic state policies given
constitutional rank by Article II, Section 26 of the Constitution is the egalitarian demand that "the State shall
guarantee equal access to opportunities for public service and prohibit political dynasties as may be defined by
law." The essential question is whether or not the assailed legislative or administrative provisions constitute a
permissible exercise of the power of supervision or regulation of the operations of communication and information
enterprises during an election period, or whether such act has gone beyond permissible supervision or regulation
of media operations so as to constitute unconstitutional repression of freedom of speech and freedom of the
press. The Court considers that Section 11 (b) has not gone outside the permissible bounds of supervision or
regulation of media operations during election periods.

Section 11 (b) is limited in the duration of its applicability and enforceability. By virtue of the operation of Article IX
(C) (4) of the Constitution, Section 11 (b) is limited in its applicability in time to election periods. Section 11 (b)
does not purport in any way to restrict the reporting by newspapers or radio or television stations of news or
news-worthy events relating to candidates, their qualifications, political parties and programs of government.
Moreover, Section 11 (b) does not reach commentaries and expressions of belief or opinion by reporters or
broadcasters or editors or commentators or columnists in respect of candidates, their qualifications, and programs
and so forth, so long at least as such comments, opinions and beliefs are not in fact advertisements for particular
candidates covertly paid for. In sum, Section 11 (b) is not to be read as reaching any report or commentary other
coverage that, in responsible media, is not paid for by candidates for political office. Section 11 (b) as designed to
cover only paid political advertisements of particular candidates.

The limiting impact of Section 11 (b) upon the right to free speech of the candidates themselves is not unduly
repressive or unreasonable.

US VS. BUSTOS
[37 PHIL. 731; G.R. L-12592; 8 MAR 1918]

Facts:

In the latter part of 1915, numerous citizens of the Province of Pampanga assembled, and prepared and signed a
petition to the Executive Secretary(privileged communication) through the law office of Crossfield and O'Brien,
and five individuals signed affidavits, charging Roman Punsalan, justice of the peace of Macabebe and Masantol,
Pampanga, with malfeasance in office and asking for his removal. The specific charges against the justice of the
peace include the solicitation of money from persons who have pending cases before the judge. Now, Punsalan
alleged that accused published a writing which was false, scandalous, malicious, defamatory, and libelous against
him.

Issue:

Whether or Not accused is entitled to constitutional protection by virtue of his right to free speech and free press.

Held:

Yes. The guaranties of a free speech and a free press include the right to criticize judicial conduct. The
administration of the law is a matter of vital public concern. Whether the law is wisely or badly enforced is,
therefore, a fit subject for proper comment. If the people cannot criticize a justice of the peace or a judge the
same as any other public officer, public opinion will be effectively suppressed. It is a duty which every one owes to
society or to the State to assist in the investigation of any alleged misconduct. It is further the duty of all who know
of any official dereliction on the part of a magistrate or the wrongful act of any public officer to bring the facts to
the notice of those whose duty it is to inquire into and punish them.

The right to assemble and petition is the necessary consequence of republican institutions and the complement of
the part of free speech. Assembly means a right on the part of citizens to meet peaceably for consultation in

Page 104
Section 1-C, SY ’06-‘07
San Beda College of Law – Alabang
Constitutional Law 2 Case Digests

respect to public affairs. Petition means that any person or group of persons can apply, without fear of penalty, to
the appropriate branch or office of the government for a redress of grievances. The persons assembling and
petitioning must, of course, assume responsibility for the charges made. All persons have an interest in the pure
and efficient administration of justice and of public affairs.

Public policy, the welfare of society, and the orderly administration of government have demanded protection for
public opinion. The inevitable and incontestable result has been the development and adoption of the doctrine of
privilege. All persons have an interest in the pure and efficient administration of justice and of public affairs. The
duty under which a party is privileged is sufficient if it is social or moral in its nature and this person in good faith
believes he is acting in pursuance thereof although in fact he is mistaken. Although the charges are probably not
true as to the justice of the peace, they were believed to be true by the petitioners. Good faith surrounded their
action. Probable cause for them to think that malfeasance or misfeasance in office existed is apparent. The ends
and the motives of these citizens— to secure the removal from office of a person thought to be venal — were
justifiable. In no way did they abuse the privilege.

In the usual case malice can be presumed from defamatory words. Privilege destroys that presumption. A
privileged communication should not be subjected to microscopic examination to discover grounds of malice or
falsity.

PITA VS. COURT OF APPEALS


[178 SCRA 362; G.R. NO.80806; 5 OCT 1989]

Facts:

On December 1 and 3, 1983, pursuing an Anti-Smut Campaign initiated by the Mayor of the City of Manila,
Ramon D. Bagatsing, elements of the Special Anti-Narcotics Group, Auxilliary Services Bureau, Western Police
District, INP of the Metropolitan Police Force of Manila, seized and confiscated from dealers, distributors,
newsstand owners and peddlers along Manila sidewalks, magazines, publications and other reading materials
believed to be obscene, pornographic and indecent and later burned the seized materials in public at the
University belt along C.M. Recto Avenue, Manila, in the presence of Mayor Bagatsing and several officers and
members of various student organizations.

Among the publications seized, and later burned, was "Pinoy Playboy" magazines published and co-edited by
plaintiff Leo Pita.

Plaintiff filed a case for injunction with prayer for issuance of the writ of preliminary injunction against Mayor
Bagatsing and Narcisco Cabrera, as superintendent of Western Police District of the City of Manila, seeking to
enjoin said defendants and their agents from confiscating plaintiff’s magazines or from preventing the sale or
circulation thereof claiming that the magazine is a decent, artistic and educational magazine which is not per se
obscene, and that the publication is protected by the Constitutional guarantees of freedom of speech and of the
press. Plaintiff also filed an Urgent Motion for issuance of a temporary restraining order against indiscriminate
seizure, confiscation and burning of plaintiff's "Pinoy Playboy" Magazines, pending hearing on the petition for
preliminary injunction. The Court granted the temporary restraining order. The case was set for trial upon the
lapse of the TRO. RTC ruled that the seizure was valid. This was affirmed by the CA.

Issue:

Whether or Not the seizure violative of the freedom of expression of the petitioner.

Held:

Freedom of the press is not without restraint as the state has the right to protect society from pornographic
literature that is offensive to public morals, as indeed we have laws punishing the author, publishers and sellers of
obscene publications. However, It is easier said than done to say, that if the pictures here in question were used
not exactly for art's sake but rather for commercial purposes, the pictures are not entitled to any constitutional
protection. Using the Kottinger rule: the test of obscenity is "whether the tendency of the matter charged as
obscene, is to deprave or corrupt those whose minds are open to such immoral influences and into whose hands
a publication or other article charged as being obscene may fall." Another is whether it shocks the ordinary and
common sense of men as an indecency. Ultimately "whether a picture is obscene or indecent must depend upon
the circumstances of the case and that the question is to be decided by the "judgment of the aggregate sense of
the community reached by it." The government authorities in the instant case have not shown the required proof
to justify a ban and to warrant confiscation of the literature First of all, they were not possessed of a lawful court
order: (1) finding the said materials to be pornography, and (2) authorizing them to carry out a search and seizure,
by way of a search warrant. The court provides that the authorities must apply for the issuance of a search
warrant from a judge, if in their opinion an obscenity seizure is in order and that;

Page 105
Section 1-C, SY ’06-‘07
San Beda College of Law – Alabang
Constitutional Law 2 Case Digests

1. The authorities must convince the court that the materials sought to be seized are obscene and pose a
clear and present danger of an evil substantive enough to warrant State interference and action;
2. The judge must determine whether or not the same are indeed obscene. The question is to be resolved
on a case-to-case basis and on the judge’s sound discretion;

AYER PRODUCTIONS VS. CAPULONG


[160 SCRA 861; G.R. NO. L-82380; 29 APR 1988]

Facts:

Petitioner McElroy an Australian film maker, and his movie production company, Ayer Productions, envisioned,
sometime in 1987, for commercial viewing and for Philippine and international release, the historic peaceful
struggle of the Filipinos at EDSA. The proposed motion picture entitled "The Four Day Revolution" was endorsed
by the MTRCB as and other government agencies consulted. Ramos also signified his approval of the intended
film production.

It is designed to be viewed in a six-hour mini-series television play, presented in a "docu-drama" style, creating
four fictional characters interwoven with real events, and utilizing actual documentary footage as background.
David Williamson is Australia's leading playwright and Professor McCoy (University of New South Wales) is an
American historian have developed a script.

Enrile declared that he will not approve the use, appropriation, reproduction and/or exhibition of his name, or
picture, or that of any member of his family in any cinema or television production, film or other medium for
advertising or commercial exploitation. petitioners acceded to this demand and the name of Enrile was deleted
from the movie script, and petitioners proceeded to film the projected motion picture. However, a complaint was
filed by Enrile invoking his right to privacy. RTC ordered for the desistance of the movie production and making of
any reference to plaintiff or his family and from creating any fictitious character in lieu of plaintiff which
nevertheless is based on, or bears substantial or marked resemblance to Enrile. Hence the appeal.

Issue:

Whether or Not freedom of expression was violated.

Held:

Yes. Freedom of speech and of expression includes the freedom to film and produce motion pictures and exhibit
such motion pictures in theaters or to diffuse them through television. Furthermore the circumstance that the
production of motion picture films is a commercial activity expected to yield monetary profit, is not a
disqualification for availing of freedom of speech and of expression.

The projected motion picture was as yet uncompleted and hence not exhibited to any audience. Neither private
respondent nor the respondent trial Judge knew what the completed film would precisely look like. There was, in
other words, no "clear and present danger" of any violation of any right to privacy. Subject matter is one of public
interest and concern. The subject thus relates to a highly critical stage in the history of the country.

At all relevant times, during which the momentous events, clearly of public concern, that petitioners propose to
film were taking place, Enrile was a "public figure:" Such public figures were held to have lost, to some extent at
least, their right to privacy.

The line of equilibrium in the specific context of the instant case between the constitutional freedom of speech and
of expression and the right of privacy, may be marked out in terms of a requirement that the proposed motion
picture must be fairly truthful and historical in its presentation of events.

LOPEZ VS. SANDIGANBAYAN


[34 SCRA 116; L-26549; 31 JUL 1970]

Facts:

In the early part of January, 1956, there appeared on the front page of The Manila Chronicle, of which petitioner
Lopez was the publisher, as well as on other dailies, a news story of a sanitary inspector assigned to the Babuyan
Islands, Fidel Cruz, sending a distress signal to a passing United States Airforce plane which in turn relayed the

Page 106
Section 1-C, SY ’06-‘07
San Beda College of Law – Alabang
Constitutional Law 2 Case Digests

message to Manila. An American Army plane dropping on the beach of an island an emergency-sustenance kit
containing, among other things, a two-way radio set. He utilized it to inform authorities in Manila that the people in
the place were living in terror, due to a series of killings committed since Christmas of 1955. Losing no time, the
Philippines defense establishment rushed to the island a platoon of scout rangers. Upon arriving Major
Encarnacion and his men found, instead of the alleged killers, a man named Fidel Cruz who merely wanted
transportation home to Manila. In view of this finding, Major Encarnacion branded as a "hoax," the report of
respondent.

This Week Magazine of the Manila Chronicle, then edited by Gatbonton, devoted a pictorial article to it in its issue
of January 15, 1956. Mention was made that while Fidel Cruz’ story turned out to be false it brought attention to
the government that people in that most people in the area are sick sick, only two individuals able to read and
write, food and clothing being scarce.

The magazine carried photographs of the person purporting to be Fidel Cruz. Unfortunately, the pictures that were
published were that of private respondent Fidel G. Cruz, a businessman contractor from Santa Maria, Bulacan. It
turned out that the photographs of respondent Cruz and that of Fidel Cruz, sanitary inspector, were on file in the
library of the Manila Chronicle but when the news quiz format was prepared, the two photographs were in
advertently switched. However a correction was published immediately.

Respondent sued petitioners in the Court of First Instance of Manila for the recovery of damages alleging the
defamatory character of the above publication of his picture. Defense interposed that they are beating the
deadline. The court ruled in his favor. Hence the appeal.

Issue:

Whether or Not petitioners abused the freedom of the press.

Held:

No. The SC, quoting Quisumbing v. Lopez, found for plaintiff, but with reduced damages, since the error in this
case could have been checked considering that this was a weekly magazine and not a daily. The ruling: "there is
no evidence in the record to prove that the publication of the news item under consideration was prompted by
personal ill will or spite, or that there was intention to do harm,' and that on the other hand there was 'an honest
and high sense of duty to serve the best interests of the public, without self-seeking motive and with malice
towards none.' Every citizen of course has the right to enjoy a good name and reputation, but we do not consider
that the respondents, under the circumstances of this case, had violated said right or abused the freedom of the
press. The newspapers should be given such leeway and tolerance as to enable them to courageously and
effectively perform their important role in our democracy. In the preparation of stories, press reporters and editors
usually have to race with their deadlines; and consistently with good faith and reasonable care, they should not be
held to account, to a point of suppression, for honest mistakes or imperfection in the choice of words.

“No inroads on press freedom should be allowed in the guise of punitive action visited on what otherwise could be
characterized as libel whether in the form of printed words or a defamatory imputation resulting from the
publication of respondent's picture with the offensive caption as in the case here complained of. This is merely to
underscore the primacy that freedom of the press enjoys.”

PRIMICIAS VS. FUGOSO


[80 PHIL 71; L-1800; 27 JAN 1948]

Facts:

An action was instituted by the petitioner for the refusal of the respondent to issue a permit to them to hold a
public meeting in Plaza Miranda for redress of grievances to the government. The reason alleged by the
respondent in his defense for refusing the permit is, "that there is a reasonable ground to believe, basing upon
previous utterances and upon the fact that passions, specially on the part of the losing groups, remains bitter and
high, that similar speeches will be delivered tending to undermine the faith and confidence of the people in their
government, and in the duly constituted authorities, which might threaten breaches of the peace and a disruption
of public order." Giving emphasis as well to the delegated police power to local government. Stating as well
Revised Ordinances of 1927 prohibiting as an offense against public peace, and penalizes as a misdemeanor,
"any act, in any public place, meeting, or procession, tending to disturb the peace or excite a riot; or collect with
other persons in a body or crowd for any unlawful purpose; or disturb or disquiet any congregation engaged in
any lawful assembly." Included herein is Sec. 1119, Free use of Public Place. 1

Issue:

Whether or Not the freedom of speech was violated.

Page 107
Section 1-C, SY ’06-‘07
San Beda College of Law – Alabang
Constitutional Law 2 Case Digests

Held:

Yes. Dealing with the ordinance, specifically, Sec. 1119, said section provides for two constructions: (1) the Mayor
of the City of Manila is vested with unregulated discretion to grant or refuse, to grant permit for the holding of a
lawful assembly or meeting, parade, or procession in the streets and other public places of the City of Manila; (2)
The right of the Mayor is subject to reasonable discretion to determine or specify the streets or public places to be
used with the view to prevent confusion by overlapping, to secure convenient use of the streets and public places
by others, and to provide adequate and proper policing to minimize the risk of disorder. The court favored the
second construction. First construction tantamount to authorizing the Mayor to prohibit the use of the streets.
Under our democratic system of government no such unlimited power may be validly granted to any officer of the
government, except perhaps in cases of national emergency.

The Mayor’s first defense is untenable. Fear of serious injury cannot alone justify suppression of free speech and
assembly. It is the function of speech to free men from the bondage of irrational fears. To justify suppression of
free speech there must be reasonable ground to fear that serious evil will result if free speech is practiced. There
must be reasonable ground to believe that the danger apprehended is imminent. There must be reasonable
ground to believe that the evil to be prevented is a serious one . The fact that speech is likely to result in some
violence or in destruction of property is not enough to justify its suppression. There must be the probability of
serious injury to the state.

ZALDIVAR VS. SANDIGANBAYAN


[170 SCRA 1; G.R. NO. 79690-707; 1 FEB 1989]

Facts:

The case stemmed from the resolution of the Supreme Court stopping the respondent from investigating graft
cases involving Antique Gov. Enrique Zaldivar. The Court ruled that since the adoption of the 1987 Constitution,
respondent’s powers as Tanodbayan have been superseded by the creation of the Office of the Ombudsman, he
however becomes the Special Prosecutor of the State, and can only conduct an investigation and file cases only
when so authorized by the Ombudsman. A motion for reconsideration was filed by the respondent wherein he
included statements which were unrelated in the Issue raised in the Court. This include: (a)That he had been
approached twice by a leading member of the court and he was asked to 'go slow on Zaldivar and 'not to be too
hard on him; (b) That he "was approached and asked to refrain from investigating the COA report on illegal
disbursements in the Supreme Court because 'it will embarass the Court;" and (c) that in several instances, the
undersigned respondent was called over the phone by a leading member of the Court and was asked to dismiss
the cases against two Members of the Court." Statements of the respondent saying that the SC’s order
'"heightens the people's apprehension over the justice system in this country, especially because the people have
been thinking that only the small fly can get it while big fishes go scot-free” was publicized in leading newspapers.

Now, the Court Resolved to require respondent to explain in writing why he should not be punished for contempt
of court for making such public statements reported in the media. Respondent then sought to get some members
of the Court to inhibit themselves in the resolution of the Zaldivar case for alleged bias and prejudice against him.
A little later, he in effect asked the whole Court to inhibit itself from passing upon the Issue involved in proceeding
and to pass on responsibility for this matter to the Integrated Bar of the Philippines, upon the ground that
respondent cannot expect due process from this Court, that the Court has become incapable of judging him
impartially and fairly. The Court found respondent guilty of contempt of court and indefinitely suspended from the
practice of law. Now, he assails said conviction, invoking his freedom of speech. Counsel for respondent urges
that it is error "for this Court to apply the "visible tendency" rule rather than the "clear and present danger" rule in
disciplinary and contempt charges."

Issue:

Whether or Not there was a violation of the freedom of speech/expression.

Held:

There was no violation. The Court did not purport to announce a new doctrine of "visible tendency," it was simply
paraphrasing Section 3 (d) of Rule 71 of the Revised Rules of Court which penalizes a variety of contumacious
conduct including: "any improper conduct tending, directly or indirectly, to impede, obstruct or degrade the
administration of justice."

Under either the "clear and present danger" test or the "balancing-of-interest test," the Court held that the
statements made by respondent Gonzalez are of such a nature and were made in such a manner and under such
circumstances, as to transcend the permissible limits of free speech. What is here at stake is the authority of the
Supreme Court to confront and prevent a "substantive evil" consisting not only of the obstruction of a free and fair

Page 108
Section 1-C, SY ’06-‘07
San Beda College of Law – Alabang
Constitutional Law 2 Case Digests

hearing of a particular case but also the avoidance of the broader evil of the degradation of the judicial system of
a country and the destruction of the standards of professional conduct required from members of the bar and
officers of the courts, which has some implications to the society.
REYES VS. BAGATSING
[125 SCRA 553; L-65366; 9 NOV 1983]

Facts:

Petitioner sought a permit from the City of Manila to hold a peaceful march and rally on October 26, 1983 from
2:00 to 5:00 in the afternoon, starting from the Luneta to the gates of the United States Embassy. Once there, and
in an open space of public property, a short program would be held. The march would be attended by the local
and foreign participants of such conference. That would be followed by the handing over of a petition based on
the resolution adopted at the closing session of the Anti-Bases Coalition. There was likewise an assurance in the
petition that in the exercise of the constitutional rights to free speech and assembly, all the necessary steps would
be taken by it "to ensure a peaceful march and rally. However the request was denied. Reference was made to
persistent intelligence reports affirming the plans of subversive/criminal elements to infiltrate or disrupt any
assembly or congregations where a large number of people is expected to attend. Respondent suggested that a
permit may be issued if it is to be held at the Rizal Coliseum or any other enclosed area where the safety of the
participants themselves and the general public may be ensured. An oral argument was heard and the mandatory
injunction was granted on the ground that there was no showing of the existence of a clear and present danger of
a substantive evil that could justify the denial of a permit. However Justice Aquino dissented that the rally is
violative of Ordinance No. 7295 of the City of Manila prohibiting the holding of rallies within a radius of five
hundred (500) feet from any foreign mission or chancery and for other purposes. Hence the Court resolves.

Issue:

Whether or Not the freedom of expression and the right to peaceably assemble violated.

Held:

Yes. The invocation of the right to freedom of peaceable assembly carries with it the implication that the right to
free speech has likewise been disregarded. It is settled law that as to public places, especially so as to parks and
streets, there is freedom of access. Nor is their use dependent on who is the applicant for the permit, whether an
individual or a group. There can be no legal objection, absent the existence of a clear and present danger of a
substantive evil, on the choice of Luneta as the place where the peace rally would start. Time immemorial Luneta
has been used for purposes of assembly, communicating thoughts between citizens, and discussing public
questions.
Such use of the public places has from ancient times, been a part of the privileges, immunities, rights, and
liberties of citizens.

With regard to the ordinance, there was no showing that there was violation and even if it could be shown that
such a condition is satisfied it does not follow that respondent could legally act the way he did. The validity of his
denial of the permit sought could still be challenged.

A summary of the application for permit for rally: The applicants for a permit to hold an assembly should inform
the licensing authority of the date, the public place where and the time when it will take place. If it were a private
place, only the consent of the owner or the one entitled to its legal possession is required. Such application
should be filed well ahead in time to enable the public official concerned to appraise whether there may be valid
objections to the grant of the permit or to its grant but at another public place. It is an indispensable condition to
such refusal or modification that the clear and present danger test be the standard for the decision reached.
Notice is given to applicants for the denial.

BAYAN VS. EXECUTIVE SECRETARY ERMITA


[488 SCRA 226; G.R. NO. 169838; 25 APR 2006]

Facts:

Rallies of September 20, October 4, 5 and 6, 2005 is at issue. BAYAN’s rally was violently dispersed. 26
petitioners were injured, arrested and detained when a peaceful mass action they was preempted and violently
dispersed by the police. KMU asserts that the right to peaceful assembly, are affected by Batas Pambansa No.
880 and the policy of “Calibrated Preemptive Response” (CPR) being followed to implement it. KMU, et al., claim
that on October 4, 2005, a rally KMU co-sponsored was to be conducted at the Mendiola bridge but police
blocked them along C.M. Recto and Lepanto Streets and forcibly dispersed them, causing injuries to several of
their members. They further allege that on October 6, 2005, a multi-sectoral rally which KMU also co-sponsored
was scheduled to proceed along España Avenue in front of the UST and going towards Mendiola bridge. Police
officers blocked them along Morayta Street and prevented them from proceeding further. They were then forcibly
dispersed, causing injuries on one of them. Three other rallyists were arrested.

Page 109
Section 1-C, SY ’06-‘07
San Beda College of Law – Alabang
Constitutional Law 2 Case Digests

All petitioners assail Batas Pambansa No. 880 The Public Assembly Act of 1985, some of them in toto and others
only Sections 4, 5, 6, 12, 13(a), and 14(a), as well as the policy of CPR. They seek to stop violent dispersals of
rallies under the “no permit, no rally” policy and the CPR policy announced on Sept. 21, 2005.

Petitioners Bayan, et al., contend that BP 880 is clearly a violation of the Constitution and the International
Covenant on Civil and Political Rights and other human rights treaties of which the Philippines is a signatory.

They argue that B.P. No. 880 requires a permit before one can stage a public assembly regardless of the
presence or absence of a clear and present danger. It also curtails the choice of venue and is thus repugnant to
the freedom of expression clause as the time and place of a public assembly form part of the message for which
the expression is sought.

Petitioners Jess del Prado, et al., in turn, argue that B.P. No. 880 is unconstitutional as it is a curtailment of the
right to peacefully assemble and petition for redress of grievances because it puts a condition for the valid
exercise of that right. It also characterizes public assemblies without a permit as illegal and penalizes them and
allows their dispersal. Thus, its provisions are not mere regulations but are actually prohibitions. Regarding the
CPR policy, it is void for being an ultra vires act that alters the standard of maximum tolerance set forth in B.P.
No. 880, aside from being void for being vague and for lack of publication.

KMU, et al., argue that the Constitution sets no limits on the right to assembly and therefore B.P. No. 880 cannot
put the prior requirement of securing a permit. And even assuming that the legislature can set limits to this right,
the limits provided are unreasonable: First, allowing the Mayor to deny the permit on clear and convincing
evidence of a clear and present danger is too comprehensive. Second, the five-day requirement to apply for a
permit is too long as certain events require instant public assembly, otherwise interest on the issue would possibly
wane.As to the CPR policy, they argue that it is preemptive, that the government takes action even before the
rallyists can perform their act, and that no law, ordinance or executive order supports the policy. Furthermore, it
contravenes the maximum tolerance policy of B.P. No. 880 and violates the Constitution as it causes a chilling
effect on the exercise by the people of the right to peaceably assemble.

Respondents argued that petitioners have no standing. BP 880 entails traffic re-routing to prevent grave public
inconvenience and serious or undue interference in the free flow of commerce and trade. It is content-neutral
regulation of the time, place and manner of holding public assemblies. According to Atienza RA. 7160 gives the
Mayor power to deny a permit independently of B.P. No. 880. and that the permit is for the use of a public place
and not for the exercise of rights; and that B.P. No. 880 is not a content-based regulation because it covers all
rallies.

Issue:

Whether or Not BP 880 and the CPR Policy unconstitutional.

Held:

No question as to standing. Their right as citizens to engage in peaceful assembly and exercise the right of
petition, as guaranteed by the Constitution, is directly affected by B.P. No. 880. B.P. 880 is not an absolute ban of
public assemblies but a restriction that simply regulates the time, place and manner of the assemblies. It refers to
all kinds of public assemblies that would use public places. The reference to “lawful cause” does not make it
content-based because assemblies really have to be for lawful causes, otherwise they would not be “peaceable”
and entitled to protection. Maximum tolerance1 is for the protection and benefit of all rallyists and is independent
of the content of the expressions in the rally. There is, likewise, no prior restraint, since the content of the speech
is not relevant to the regulation.

The so-called calibrated preemptive response policy has no place in our legal firmament and must be struck down
as a darkness that shrouds freedom. It merely confuses our people and is used by some police agents to justify
abuses. Insofar as it would purport to differ from or be in lieu of maximum tolerance, this was declared null and
void.

The Secretary of the Interior and Local Governments, are DIRECTED to take all necessary steps for the
immediate compliance with Section 15 of Batas Pambansa No. 880 through the establishment or designation of at
least one suitable freedom park or plaza in every city and municipality of the country. After thirty (30) days from
the finality of this Decision, subject to the giving of advance notices, no prior permit shall be required to exercise
the right to peaceably assemble and petition in the public parks or plazas of a city or municipality that has not yet
complied with Section 15 of the law.

FERNANDO VS. ESTORNINOS


[G.R. NO 159751; 6 DEC 2006]

Page 110
Section 1-C, SY ’06-‘07
San Beda College of Law – Alabang
Constitutional Law 2 Case Digests

Facts:

Acting on reports of sale and distribution of pornographic materials, officers of the PNP Criminal Investigation and
Detection Group in the National Capital Region conducted police surveillance on the store Gaudencio E.
Fernando Music Fair (Music Fair) in Quiapo. A Search Warrant for violation of Article 201 of RPC against
petitioner and a certain Warren Tingchuy and the seizure of the following items:

a. Copies of New Rave Magazines with nude obscene pictures;


b. Copies of IOU Penthouse Magazine with nude obscene pictures;
c. Copies of Hustler International Magazine with nude obscene pictures; and
d. Copies of VHS tapes containing pornographic shows.

The police searched the premises and confiscated twenty-five VHS tapes(among of which is “Kahit sa Pangarap
Lang” with Myra Manibog as actress who is naked) and ten different magazines(Dalaga, Penthouse, Swank,
Erotic, Rave, Playhouse, Gallery, QUI), which they deemed pornographic. Petitioners were charged and
convicted. CA affirmed the decision hence this appeal.

Issue:

Whether or Not the CA erred in affirming RTC’s decision.

Held:

No. As obscenity is an unprotected speech which the State has the right to regulate, the State in pursuing its
mandate to protect the public from obscene, immoral and indecent materials must justify the regulation or
limitation. (Kottinger Rule Applied).

M ALABANAN VS. RAMENTO


[129 SCRA 359; G.R. NO.62270; 21 MAY 1984]

Facts:

Petitioners were officers of the Supreme Student Council of respondent University. They sought and were granted
by the school authorities a permit to hold a meeting from 8:00 A.M. to 12:00 P.M, on August 27, 1982. Pursuant to
such permit, along with other students, they held a general assembly at the Veterinary Medicine and Animal
Science basketball court (VMAS), the place indicated in such permit, not in the basketball court as therein stated
but at the second floor lobby. At such gathering they manifested in vehement and vigorous language their
opposition to the proposed merger of the Institute of Animal Science with the Institute of Agriculture. The same
day, they marched toward the Life Science Building and continued their rally. It was outside the area covered by
their permit. Even they rallied beyond the period allowed. They were asked to explain on the same day why they
should not be held liable for holding an illegal assembly. Then on September 9, 1982, they were informed that
they were under preventive suspension for their failure to explain the holding of an illegal assembly. The validity
thereof was challenged by petitioners both before the Court of First Instance of Rizal against private respondents
and before the Ministry of Education, Culture, and Sports. Respondent Ramento found petitioners guilty of the
charge of illegal assembly which was characterized by the violation of the permit granted resulting in the
disturbance of classes and oral defamation. The penalty was suspension for one academic year. Hence this
petition.

Issue:

Whether on the facts as disclosed resulting in the disciplinary action and the penalty imposed, there was an
infringement of the right to peaceable assembly and its cognate right of free speech.

Held:

Yes. Student leaders are likely to be assertive and dogmatic. They would be ineffective if during a rally they speak
in the guarded and judicious language of the academe. But with the activity taking place in the school premises
and during the daytime, no clear and present danger of public disorder is discernible. This is without prejudice to
the taking of disciplinary action for conduct, "materially disrupts classwork or involves substantial disorder or
invasion of the rights of others."

The rights to peaceable assembly and free speech are guaranteed students of educational institutions.
Necessarily, their exercise to discuss matters affecting their welfare or involving public interest is not to be
subjected to previous restraint or subsequent punishment unless there be a showing of a clear and present
danger to a substantive evil that the state, has a right to present. As a corollary, the utmost leeway and scope is

Page 111
Section 1-C, SY ’06-‘07
San Beda College of Law – Alabang
Constitutional Law 2 Case Digests

accorded the content of the placards displayed or utterances made. The peaceable character of an assembly
could be lost, however, by an advocacy of disorder under the name of dissent, whatever grievances that may be
aired being susceptible to correction through the ways of the law. If the assembly is to be held in school premises,
permit must be sought from its school authorities, who are devoid of the power to deny such request arbitrarily or
unreasonably. In granting such permit, there may be conditions as to the time and place of the assembly to avoid
disruption of classes or stoppage of work of the non-academic personnel. Even if, however, there be violations of
its terms, the penalty incurred should not be disproportionate to the offense.

NON VS. DAMES


[185 SCRA 523; G.R. NO. 89317; 20 MAY 1990]

Facts:

Petitioners, students in private respondent Mabini Colleges, Inc. in Daet, Camarines Norte, were not allowed to
re-enroll by the school for the academic year 1988-1989 for leading or participating in student mass actions
against the school in the preceding semester. The subject of the protests is not, however, made clear in the
pleadings.

Petitioners filed a petition in the court seeking their readmission or re-enrollment to the school, but the trial court
dismissed the petition. They now petition the court to reverse its ruling in Alcuaz vs. PSBA 1, which was also
applied in the case. The court said that petitioners waived their privilege to be admitted for re-enrollment with
respondent college when they adopted, signed, and used its enrollment form for the first semester of school year
1988-89, which states that: The Mabini College reserves the right to deny admission of students whose
scholarship and attendance are unsatisfactory and to require withdrawal of students whose conduct discredits the
institution and/or whose activities unduly disrupts or interfere with the efficient operation of the college. Students,
therefore, are required to behave in accord with the Mabini College code of conduct and discipline.

Issue:

Whether or Not the students’ right to freedom of speech and assembly infringed.

Held:

Yes. The protection to the cognate rights of speech and assembly guaranteed by the Constitution is similarly
available to students is well-settled in our jurisdiction. However there are limitations. The permissible limitation on
Student Exercise of Constitutional Rights within the school presupposes that conduct by the student, in class or
out of it, which for any reason whether it stems from time, place, or type of behavior should not materially disrupt
classwork or must not involve substantial disorder or invasion of the rights of others.

IN RE: TULFO
[A.M. NO. 90-4-1545-0; 17 APR 1990]

Facts:

In Oct. 13, 1989, Tulfo wrote an article in his column in PDI 'On Target' stating that the Supreme Court rendered
an idiotic decision in legalizing checkpoints, and again on Oct. 16, 1989, where he called the Supreme Court
stupid and "sangkatutak na mga bobo justices of the Philippine Supreme Court". Tulfo was required to show
cause why he should not be punished for contempt. Tulfo said that he was just reacting emotionally because he
had been a victim of harassment in the checkpoints, and "idiotic" meant illogical and unwise, and "bobo" was just
quoted from other attorneys, and since the case had been decided and terminated, there was not contempts.
Lastly, the article does not pose any clear and present danger to the Supreme court.

Issue:

Whether or Not Tulfo is in contempt.

Held:

Yes. At the time Tulfo wrote the article, the checkpoints case had not yet been decided upon, and the Supreme
Court was still acting on an MR filed from the CA. The power to punish is inherent as it is essential for self-
preservation. Contempt of court is defiance of the authority, justice and dignity of the courts. It brings disrepute to
the court. There are two kinds of publications which can be punished for contempt:

a. those whose object is to affect the decision in a pending case.

Page 112
Section 1-C, SY ’06-‘07
San Beda College of Law – Alabang
Constitutional Law 2 Case Digests

b. those whose object is to bring courts to discredit. Tulfo's article constituted both.

It should have been okay to criticize if respectful language was used, but if its object is only to degrade and
ridicule, then it is clearly an obstruction of justice. Nothing constructive can be gained from them. Being emotional
is no excuse for being insulting. Quoting is not an excuse also, because at the end of his article, Tulfo said, "So
you bobo justices, watch out!" Also, he said he was not sorry for having written the articles.

Tulfo is found in contempt of court and is gravely censured.

PBM EMPLOYEES VS. PBM


[51 SCRA 189; G.R. NO. L-31195; 5 JUN 1993]

Facts:

The petitioner Philippine Blooming Mills Employees Organization (PBMEO) is a legitimate labor union composed
of the employees of the respondent Philippine Blooming Mills Co., Inc., and petitioners. Benjamin Pagcu and
Rodulfo Munsod are officers and members of the petitioner Union. Petitioners claim that on March 1, 1969, they
decided to stage a mass demonstration at Malacañang on March 4, 1969, in protest against alleged abuses of the
Pasig police. PBMEO thru Pagcu confirmed the planned demonstration and stated that the demonstration or rally
cannot be cancelled because it has already been agreed upon in the meeting. Pagcu explained further that the
demonstration has nothing to do with the Company because the union has no quarrel or dispute with
Management. The Management, thru Atty. C.S. de Leon, Company personnel manager, informed PBMEO that
the demonstration is an inalienable right of the union guaranteed by the Constitution but emphasized that any
demonstration for that matter should not unduly prejudice the normal operation of the Company. Workers who
without previous leave of absence approved by the Company, particularly , the officers present who are the
organizers of the demonstration, who shall fail to report for work the following morning shall be dismissed,
because such failure is a violation of the existing CBA and, therefore, would be amounting to an illegal strike.
Because the petitioners and their members numbering about 400 proceeded with the demonstration despite the
pleas of the respondent Company that the first shift workers should not be required to participate in the
demonstration and that the workers in the second and third shifts should be utilized for the demonstration from 6
A.M. to 2 P.M. on March 4, 1969, filed a charge against petitioners and other employees who composed the first
shift, for a violation of Republic Act No. 875(Industrial Peace Act), and of the CBA providing for 'No Strike and No
Lockout.' Petitioners were held guilty in by CIR for bargaining in bad faith, hence this appeal.

Issue:

Whether or Not the petitioners right to freedom of speech and to peaceable assemble violated.

Held:

Yes. A constitutional or valid infringement of human rights requires a more stringent criterion, namely existence of
a grave and immediate danger of a substantive evil which the State has the right to prevent. This is not present in
the case. It was to the interest herein private respondent firm to rally to the defense of, and take up the cudgels
for, its employees, so that they can report to work free from harassment, vexation or peril and as consequence
perform more efficiently their respective tasks enhance its productivity as well as profits. Herein respondent
employer did not even offer to intercede for its employees with the local police. In seeking sanctuary behind their
freedom of expression well as their right of assembly and of petition against alleged persecution of local
officialdom, the employees and laborers of herein private respondent firm were fighting for their very survival,
utilizing only the weapons afforded them by the Constitution — the untrammelled enjoyment of their basic human
rights. The pretension of their employer that it would suffer loss or damage by reason of the absence of its
employees from 6 o'clock in the morning to 2 o'clock in the afternoon, is a plea for the preservation merely of their
property rights. The employees' pathetic situation was a stark reality — abused, harassment and persecuted as
they believed they were by the peace officers of the municipality. As above intimated, the condition in which the
employees found themselves vis-a-vis the local police of Pasig, was a matter that vitally affected their right to
individual existence as well as that of their families. Material loss can be repaired or adequately compensated.
The debasement of the human being broken in morale and brutalized in spirit-can never be fully evaluated in
monetary terms. As heretofore stated, the primacy of human rights — freedom of expression, of peaceful
assembly and of petition for redress of grievances — over property rights has been sustained. To regard the
demonstration against police officers, not against the employer, as evidence of bad faith in collective bargaining
and hence a violation of the collective bargaining agreement and a cause for the dismissal from employment of
the demonstrating employees, stretches unduly the compass of the collective bargaining agreement, is "a potent
means of inhibiting speech" and therefore inflicts a moral as well as mortal wound on the constitutional
guarantees of free expression, of peaceful assembly and of petition. Circulation is one of the aspects of freedom
of expression. If demonstrators are reduced by one-third, then by that much the circulation of the Issue raised by
the demonstration is diminished. The more the participants, the more persons can be apprised of the purpose of

Page 113
Section 1-C, SY ’06-‘07
San Beda College of Law – Alabang
Constitutional Law 2 Case Digests

the rally. Moreover, the absence of one-third of their members will be regarded as a substantial indication of
disunity in their ranks which will enervate their position and abet continued alleged police persecution.

Page 114
Section 1-C, SY ’06-‘07
San Beda College of Law – Alabang
Constitutional Law 2 Case Digests

THE IMPAIRMENT CLAUSE

Art 3, Sec. 10. “No law impairing the obligation of contracts shall be passed.”

RUTTER VS. ESTEBAN


[93 PHIL 68; NO.L-3708; 18 MAY 1953]

Facts:

On August 20,1941 Rutter sold to Esteban two parcels of land situated in the Manila for P9,600 of which P4,800
were paid outright, and the balance was made payable as follows: P2,400 on or before August 7, 1942, and
P2,400 on or before August 27, 1943, with interest at the rate of 7 percent per annum. To secure the payment of
said balance of P4,800, a first mortgage has been constituted in favor of the plaintiff. Esteban failed to pay the two
installments as agreed upon, as well as the interest that had accrued and so Rutter instituted an action to recover
the balance due, the interest due and the attorney's fees. The complaint also contains a prayer for sale of the
properties mortgaged in accordance with law. Esteban claims that this is a prewar obligation contracted and that
he is a war sufferer, having filed his claim with the Philippine War Damage Commission for the losses he had
suffered as a consequence of the last war; and that under section 2 of RA 342(moratorium law), payment of his
obligation cannot be enforced until after the lapse of eight years. The complaint was dismissed. A motion for
recon was made which assails the constitutionality of RA 342.

Issue:

Whether or Not RA 342 unconstitutional on non-impairment clause grounds.

Held:

Yes. The moratorium is postponement of fulfillment of obligations decreed by the state through the medium of the
courts or the legislature. Its essence is the application of police power. The economic interests of the State may
justify the exercise of its continuing and dominant protective power notwithstanding interference with contracts.
The question is not whether the legislative action affects contracts incidentally, or directly or indirectly, but
whether the legislation is addressed to a legitimate end and the measures taken are reasonable and appropriate
to that end.

However based on the President’s general SONA and consistent with what the Court believes to be as the only
course dictated by justice, fairness and righteousness, declared that the continued operation and enforcement of
RA 342 at the present time is unreasonable and oppressive, and should not be prolonged should be declared null
and void and without effect. This holds true as regards Executive Orders Nos. 25 and 32, with greater force and
reason considering that said Orders contain no limitation whatsoever in point of time as regards the suspension of
the enforcement and effectivity of monetary obligations.

ORTIGAS VS. FEATI BANK


[94 SCRA 533; NO.L-24670; 14 DEC 1979]

Facts:

Plaintiff is engaged in real estate business, developing and selling lots to the public, particularly the Highway Hills
Subdivision along EDSA. On March 4, 1952, plaintiff, as vendor, and Augusto Padilla and Natividad Angeles, as
vendees, entered into separate agreements of sale on installments over two parcels of land of the Subdivision.
On July 19, 1962, the said vendees transferred their rights and interests over the aforesaid lots in favor of one
Emma Chavez. Upon completion of payment of the purchase price, the plaintiff executed the corresponding
deeds of sale in favor of Emma Chavez. Both the agreements (of sale on installment) and the deeds of sale
contained the stipulations or restrictions that:

1. The parcel of land shall be used exclusively for residential purposes, and she shall not be entitled to take
or remove soil, stones or gravel from it or any other lots belonging to the Seller.
2. All buildings and other improvements (except the fence) which may be constructed at any time in said lot
must be, (a) of strong materials and properly painted, (b) provided with modern sanitary installations
connected either to the public sewer or to an approved septic tank, and (c) shall not be at a distance of
less than two (2) meters from its boundary lines.

Page 115
Section 1-C, SY ’06-‘07
San Beda College of Law – Alabang
Constitutional Law 2 Case Digests

Eventually said lots were bought by defendant. Lot 5 directly from Chavez and Lot 6 from Republic Flour Mills by
deed of exchange, with same restrictions. Plaintiff claims that restriction is for the beautification of the subdivision.
Defendant claimed of the commercialization of western part of EDSA. Defendant began constructing a
commercial bank building. Plaintiff demand to stop it, which forced him to file a case, which was later dismissed,
upholding police power. Motion for recon was denied, hence the appeal.

Issue:

Whether or Not non-impairment clause violated.

Held:

No. Resolution is a valid exercise of police power. EDSA, a main traffic artery which runs through several cities
and municipalities in the Metro Manila area, supports an endless stream of traffic and the resulting activity, noise
and pollution are hardly conducive to the health, safety or welfare of the residents in its route. Health, safety,
peace, good order and general welfare of the people in the locality are justifications for this. It should be stressed,
that while non-impairment of contracts is constitutionally guaranteed, the rule is not absolute, since it has to be
reconciled with the legitimate exercise of police power.

LOZANO VS. M ARTINEZ


[146 SCRA 323; NO.L-63419; 18 DEC 1986]

Facts:

A motion to quash the charge against the petitioners for violation of the BP 22 was made, contending that no
offense was committed, as the statute is unconstitutional. Such motion was denied by the RTC. The petitioners
thus elevate the case to the Supreme Court for relief. The Solicitor General, commented that it was premature for
the accused to elevate to the Supreme Court the orders denying their motions to quash. However, the Supreme
Court finds it justifiable to intervene for the review of lower court's denial of a motion to quash.

Issue:

Whether or Not BP 22 impairs freedom of contract. Whether or not BP 22 transgresses the constitutional inhibition
against imprisonment for debt.

Held:

The freedom of contract which is constitutionally protected is freedom to enter into "lawful" contracts. Contracts
which contravene public policy are not lawful. Checks can not be categorized as mere contracts. It is a
commercial instrument which, in this modem day and age, has become a convenient substitute for money; it
forms part of the banking system and therefore not entirely free from the regulatory power of the state.

The offense punished by BP 22 is the act of making and issuing a worthless check or a check that is dishonored
upon its presentation for payment. It is not the non-payment of an obligation which the law punishes. The law is
not intended or designed to coerce a debtor to pay his debt. The thrust of the law is to prohibit, under pain of
penal sanctions, the making of worthless checks and putting them in circulation.

Page 116
Section 1-C, SY ’06-‘07
San Beda College of Law – Alabang
Constitutional Law 2 Case Digests

EX POST FACTO LAWS

Art 3, Sec. 22. “No ex post facto law or bill of attainder shall be enacted.”

PEOPLE VS. FERRER


[48 SCRA 382; NOS.L-32613-14; 27 DEC 1972]

Facts:

Hon. Judge Simeon Ferrer is the Tarlac trial court judge that declared RA1700 or the Anti-Subversive Act of 1957
as a bill of attainder. Thus, dismissing the information of subversion against the following: 1.) Feliciano Co for
being an officer/leader of the Communist Party of the Philippines (CPP) aggravated by circumstances of contempt
and insult to public officers, subversion by a band and aid of armed men to afford impunity. 2.) Nilo Tayag and 5
others, for being members/leaders of the NPA, inciting, instigating people to unite and overthrow the Philippine
Government. Attended by Aggravating Circumstances of Aid or Armed Men, Craft, and Fraud. The trial court is
of opinion that 1.) The Congress usurped the powers of the judge 2.) Assumed judicial magistracy by pronouncing
the guilt of the CPP without any forms of safeguard of a judicial trial. 3.) It created a presumption of organizational
guilt by being members of the CPP regardless of voluntariness.

The Anti Subversive Act of 1957 was approved 20June1957. It is an act to outlaw the CPP and similar
associations penalizing membership therein, and for other purposes. It defined the Communist Party being
although a political party is in fact an organized conspiracy to overthrow the Government, not only by force and
violence but also by deceit, subversion and other illegal means. It declares that the CPP is a clear and present
danger to the security of the Philippines. Section 4 provided that affiliation with full knowledge of the illegal acts
of the CPP is punishable. Section 5 states that due investigation by a designated prosecutor by the Secretary of
Justice be made prior to filing of information in court. Section 6 provides for penalty for furnishing false evidence.
Section 7 provides for 2 witnesses in open court for acts penalized by prision mayor to death. Section 8 allows
the renunciation of membership to the CCP through writing under oath. Section 9 declares the constitutionality of
the statute and its valid exercise under freedom if thought, assembly and association.

Issue:

Whether or not RA1700 is a bill of attainder/ ex post facto law.

Whether or Not RA1700 violates freedom of expression.

Held:

The court holds the VALIDITY Of the Anti-Subversion Act of 1957.

A bill of attainder is solely a legislative act. It punishes without the benefit of the trial. It is the substitution of
judicial determination to a legislative determination of guilt. In order for a statute be measured as a bill of
attainder, the following requisites must be present: 1.) The statute specifies persons, groups. 2.) the statute is
applied retroactively and reach past conduct. (A bill of attainder relatively is also an ex post facto law.)

In the case at bar, the statute simply declares the CPP as an organized conspiracy for the overthrow of the
Government for purposes of example of SECTION 4 of the Act. The Act applies not only to the CPP but also to
other organizations having the same purpose and their successors. The Act’s focus is on the conduct not person.
Membership to this organizations, to be UNLAWFUL, it must be shown that membership was acquired with the
intent to further the goals of the organization by overt acts. This is the element of MEMBERSHIP with
KNOWLEDGE that is punishable. This is the required proof of a member’s direct participation. Why is
membership punished. Membership renders aid and encouragement to the organization. Membership makes
himself party to its unlawful acts.

Furthermore, the statute is PROSPECTIVE in nature. Section 4 prohibits acts committed after approval of the
act. The members of the subversive organizations before the passing of this Act is given an opportunity to
escape liability by renouncing membership in accordance with Section 8. The statute applies the principle of
mutatis mutandis or that the necessary changes having been made.

The declaration of that the CPP is an organized conspiracy to overthrow the Philippine Government should not be
the basis of guilt. This declaration is only a basis of Section 4 of the Act. The EXISTENCE OF SUBSTANTIVE
EVIL justifies the limitation to the exercise of “Freedom of Expression and Association” in this matter. Before the
enactment of the statute and statements in the preamble, careful investigations by the Congress were done. The

Page 117
Section 1-C, SY ’06-‘07
San Beda College of Law – Alabang
Constitutional Law 2 Case Digests

court further stresses that whatever interest in freedom of speech and association is excluded in the prohibition of
membership in the CPP are weak considering NATIONAL SECURITY and PRESERVATION of DEMOCRACY.

The court set basic guidelines to be observed in the prosecution under RA1700. In addition to proving
circumstances/ evidences of subversion, the following elements must also be established:

1. Subversive Organizations besides the CPP, it must be proven that the organization purpose is to
overthrow the present Government of the Philippines and establish a domination of a FOREIGN
POWER. Membership is willfully and knowingly done by overt acts.
2. In case of CPP, the continued pursuance of its subversive purpose. Membership is willfully and
knowingly done by overt acts.

The court did not make any judgment on the crimes of the accused under the Act. The Supreme Court set aside
the resolution of the TRIAL COURT.

BAYOT VS. SANDIGANBAYAN


[128 SCRA 383; NO.L-61776 TO NO.L-61861; 23 MAR 1984]

Facts:

Bayot is one of the several persons who was accused in more than 100 counts of estafa thru falsification of Public
documents before the Sandiganbayan. The said charges started from his alleged involvement as a government
auditor of the commission on audit assigned to the Ministry of education and culture, with some other employees
from the said ministry. The bureau of treasury and the teacher’s camp in Baguio City for the preparation and
encashment of fictitious TCAA checks for the nom-existent obligations of the teacher’s camp resulting in damage
to the government of several millions. The 1st 32 cases were filed on july 25, 1987, while Bayot ran for municipal
mayor of Amadeo Cavite and was elected on January 1980. but on May 1980 Sandiganbayan promulgated a
decision convicting the accused together with his other co-accused in all but one of the thirty two cases filed
against them.

On Mach 16, 1982 Batas Pambansa Blg 195 was passed amending RA 3019.

Issue:

Whether or Not it would be violative of the constitutional guarantee against an ex post facto law.

Held:

The court finds no merit in the petitioner’s contention that RA 3019 as amended by Batas Pambansa Blg 195,
which includes the crime of estafa through falsification of Public Documents as among crimes subjecting the
public officer charged therewith with suspension from public office pending action in court, is a penal provision
which violates the constitutional prohibition against the enactment of ex post facto law. Accdg to the RPC
suspension from employment and public office during trial shall not be considered as a penalty. It is not a penalty
because it is not a result of a judicial proceeding. In fact, if acquitted the official who is suspended shall be entitled
to reinstatement and the salaries and benefits which he failed to receive during suspension. And does not violate
the constitutional provision against ex post facto law.

The claim of the petitioner that he cannot be suspended because he is currently occupying a position diffren tfrom
that under which he is charged is untenable. The amendatory provision clearly states that any incumbent public
officer against whom any criminal prosecution under a valid information under RA 3019 for any offense involving
fraud upon the government or public funds or property or whatever stage of execution and mode of participation
shall be suspended from office. The use of the word “office” applies to any office which the officer charged may be
holding and not only the particular office under which he was charged.

PEOPLE VS. SANDIGANBAYAN


[211 SCRA 241; G.R. NO. 101724; 3 JUL 1992]

Facts:

Page 118
Section 1-C, SY ’06-‘07
San Beda College of Law – Alabang
Constitutional Law 2 Case Digests

Two letter complaints were filed with the Tanodbayan by Teofilo Gelacio on October 28,1986 and December 9,
1986, a political leader of Governor Valentina Plaza, wife of Congressman Democrito Plaza of Agusan del Sur,
shortly after private respondent had replaced Mrs. Plaza as OIC/provincial Governor of Agusan del Sur on March
1986 The complaint questioned the issuance to Governor Paredes, when he was still the provincial attorney in
1976 of a free patent title for a lot in the Rosario public land subdivision in San Francisco, Agusan del Sur. He
misrepresented to a Lands Inspector of the Bureau of Lands that the lands subject herein are disposable lands,
thereby inducing said inspector to recommend approval of his application for free patent. On August 10, 1989 an
information for violation of RA 3019 Anti-Graft and Corrupt Practices Act was then filed in the Sandiganbayan
after an ex parte preliminary investigation. A motion to quash the information was filed by the private respondent
contending among others that he is charged for an offence which has prescribed. Said motion was granted. The
crime was committed on January 21, 1976, period of prescription was 10 years, therefore it has prescribed in
1986. Now the motion to quash was being assailed.

Issue:

Whether or Not the motion to quash validly granted.

Held:

Yes. RA 3019, being a special law the computation of the period for the prescription of the crime is governed by
Sec. 29 of Act No. 3326, which begins to run from the day of the commission of the crime and not the discovery of
it. Additionally, BP 195 which was approved on March 16, 1982, amending Sec. 11 of RA 3019 by increasing ten
to fifteen years of the period for the prescription or extinguishment of a violation of RA 3019 may not be given
retroactive application to the crime which was committed by Paredes, as it is prejudicial to the accused. To apply
BP 195 to Paredes would make it an ex post facto law1 for it would alter his situation to his disadvantage by
making him criminally liable for a crime that had already been extinguished under the law existing when it was
committed.

Page 119
Section 1-C, SY ’06-‘07
San Beda College of Law – Alabang
Constitutional Law 2 Case Digests

NON-IMPRISONMENT FOR DEBT

Art 3, Sec. 20. “No person shall be imprisoned for debt or non-payment of a poll tax.”

SERAFIN VS. LINDAYAG


[67 SCRA 166; ADM. MATTER. NO. 297-MJ; 30 SEPT 1975]

Facts:

Plaintiff failed to pay a simple indebtedness for P1500 Carmelito Mendoza, then municipal secretary and his wife
Corazon Mendoza and therefore an estafa case was filed against her. Complainant admitted complaint. Now
complainant filed a case against respondent Judge for not dismissing the case and issuing a warrant of arrest as
it falls on the category of a simple indebtedness, since elements of estafa are not present. Further she contended
that no person should be imprisoned for non-payment of a loan of a sum of money. Two months after respondent
dismissed plaintiff’s case. (Judge here committed gross ignorance of law. Even if complainant desisted case was
pursued.)

Issue:

Whether or Not there was a violation committed by the judge when it ordered the imprisonment of plaintiff for non-
payment of debt?

Held:

Yes. Since plaintiff did not commit any offense as, his debt is considered a simple loan granted by her friends to
her. There is no collateral or security because complainant was an old friend of the spouses who lent the money
and that when they wrote her a letter of demand she promised to pay them and said that if she failed to keep her
promise, they could get her valuable things at her home. Under the Constitution she is protected. Judge therefore
in admitting such a "criminal complaint" that was plainly civil in aspects from the very face of the complaint and the
"evidence" presented, and issuing on the same day the warrant of arrest upon his utterly baseless finding "that
the accused is probably guilty of the crime charged," respondent grossly failed to perform his duties properly.

LOZANO VS. M ARTINEZ


[146 SCRA 323; NO.L-63419; 18 DEC 1986]

Facts:

A motion to quash the charge against the petitioners for violation of the BP 22 was made, contending that no
offense was committed, as the statute is unconstitutional. Such motion was denied by the RTC. The petitioners
thus elevate the case to the Supreme Court for relief. The Solicitor General, commented that it was premature for
the accused to elevate to the Supreme Court the orders denying their motions to quash. However, the Supreme
Court finds it justifiable to intervene for the review of lower court's denial of a motion to quash.

Issue:

Whether or not BP 22 is constitutional as it is a proper exercise of police power of the State.

Held:

The enactment of BP 22 a valid exercise of the police power and is not repugnant to the constitutional inhibition
against imprisonment for debt.

The offense punished by BP 22 is the act of making and issuing a worthless check or a check that is dishonored
upon its presentation for payment. It is not the non-payment of an obligation which the law punishes. The law is
not intended or designed to coerce a debtor to pay his debt.

The law punishes the act not as an offense against property, but an offense against public order. The thrust of the
law is to prohibit, under pain of penal sanctions, the making of worthless checks and putting them in circulation.
An act may not be considered by society as inherently wrong, hence, not malum in se but because of the harm
that it inflicts on the community, it can be outlawed and criminally punished as malum prohibitum. The state can
do this in the exercise of its police power.

Page 120
Section 1-C, SY ’06-‘07
San Beda College of Law – Alabang
Constitutional Law 2 Case Digests

INVOLUNTARY SERVITUDE

Art 3, Sec. 18. “(2) No involuntary servitude in any form shall exist except as a punishment for a crime
whereof the party shall have been duly convicted.”

CAUNCA VS. SALAZAR


[82 PHIL 851; NO.L-2690; 1 JAN 1949]

Facts:

This is an action for habeas corpus brought by Bartolome Caunca in behalf of his cousin Estelita Flores who was
employed by the Far Eastern Employment Bureau, owned by Julia Salazar, respondent herein. An advanced
payment has already been given to Estelita by the employment agency, for her to work as a maid. However,
Estelita wanted to transfer to another residence, which was disallowed by the employment agency. Further she
was detained and her liberty was restrained. The employment agency wanted that the advance payment, which
was applied to her transportation expense from the province should be paid by Estelita before she could be
allowed to leave.

Issue:

Whether or Not an employment agency has the right to restrain and detain a maid without returning the advance
payment it gave?

Held:

An employment agency, regardless of the amount it may advance to a prospective employee or maid, has
absolutely no power to curtail her freedom of movement. The fact that no physical force has been exerted to keep
her in the house of the respondent does not make less real the deprivation of her personal freedom of movement,
freedom to transfer from one place to another, freedom to choose one’s residence. Freedom may be lost due to
external moral compulsion, to founded or groundless fear, to erroneous belief in the existence of an imaginary
power of an impostor to cause harm if not blindly obeyed, to any other psychological element that may curtail the
mental faculty of choice or the unhampered exercise of the will. If the actual effect of such psychological spell is to
place a person at the mercy of another, the victim is entitled to the protection of courts of justice as much as the
individual who is illegally deprived of liberty by duress or physical coercion.

Page 121
Section 1-C, SY ’06-‘07
San Beda College of Law – Alabang
Constitutional Law 2 Case Digests

THE WRIT OF HABEAS CORPUS

Art 3, Sec. 15. “The privilege of the writ of habeas corpus shall not be suspended except in cases of invasion or
rebellion when the public safety requires it.”

LANSANG VS. GARCIA


[42 SCRA 448; L-33964; 11 Dec 1971]

Facts:

In the evening of August 21, 1971, at about 9 p.m., while the Liberal Party of the Philippines was holding a public
meeting at Plaza Miranda, Manila, for the presentation of its candidates in the general elections scheduled for
November 8, 1971, two hand grenades were thrown at the platform where said candidates and other persons
were. Eight persons were killed and many more injured. Proclamation 889 was issued by the President
suspending privilege of writ of habeas corpus stating that there is a conspiracy of rebellion and insurrection in
order to forcibly seize political power. Petitions for writ of habeas corpus were filed by persons (13) who have
been arrested without a warrant.

It was stated that one of the safeguards of the proclamation was that it is to be applied to persons caught in
flagrante delicto. Incidentally, Proc. 889-A was issued as an amendment, inserting the word “actually staging”.
Proc. 889-B was also issued lifting the suspension of privilege in 27 provinces, 3 sub-provinces and 26 cities.
Proc. 889-C was issued restoring the suspension in 13 provinces and cities(mostly in Mindanao). Proc. 889-D
further lifted the suspension in 7 provinces and 4 cities. Only 18 provinces and sub-provinces and 2 cities whose
privilege was suspended. Petitioners maintained that Proclamation No. 889 did not declare the existence of actual
"invasion insurrection or rebellion or imminent danger thereof, however it became moot and academic since it was
amended. Petitioners further contend that public safety did not require the issuance of proclamations stating: (a)
that there is no rebellion; (b) that, prior to and at the time of the suspension of the privilege, the Government was
functioning normally, as were the courts; (c) that no untoward incident, confirmatory of an alleged July-August
Plan, has actually taken place after August 21, 1971; (d) that the President's alleged apprehension, because of
said plan, is non-existent and unjustified; and (e) that the Communist forces in the Philippines are too small and
weak to jeopardize public safety to such extent as to require the suspension of the privilege of the writ of habeas
corpus.

A resolution was issued by majority of the Court having tentatively arrived at a consensus that it may inquire in
order to satisfy itself of the existence of the factual bases for the proclamations. Now the Court resolves after
conclusive decision reached by majority.

Issue:

Whether or Not the authority to decide whether the exigency has arisen requiring suspension (of the privilege of
the writ of habeas corpus) belongs to the President and his decision is final and conclusive upon the courts and
upon all other persons.

Whether or Not public safety require the suspension of the privilege of the writ of habeas corpus decreed in
Proclamation No. 889-A.

Held:

The President has authority however it is subject to judicial review. Two conditions must concur for the valid
exercise of the authority to suspend the privilege to the writ (a) there must be "invasion, insurrection, or rebellion"
or "imminent danger thereof," and (b) "public safety" must require the suspension of the privilege. President has
three (3) courses of action: (a) to call out the armed forces; (b) to suspend the privilege of the writ of habeas
corpus; and (c) to place the Philippines or any part thereof under martial law. He had, already, called out the
armed forces, proved inadequate. Of the two other alternatives, the suspension of the privilege is the least harsh.

Petitioners contention that CPP-NPA has no ability, is negatived by the killing of 5 mayors, 20 barrio captains and
3 chiefs of police; that there were fourteen (14) meaningful bombing incidents in the Greater Manila Area in 1970.
CPP has managed to infiltrate or establish and control nine major labor organizations; has exploited the (11)
major student or youth organizations; about thirty (30) mass organizations actively advancing the CPP.

Page 122
Section 1-C, SY ’06-‘07
San Beda College of Law – Alabang
Constitutional Law 2 Case Digests

RIGHTS OF THE ACCUSED

Art 3, Sec. 12. “(1) Any person under investigation for the commission of an offense shall have the right to be
informed of his right to remain silent and to have competent and independent counsel preferably of his own
choice. If the person cannot afford the services of counsel, he must be provided with one. These rights cannot be
waived except in writing and in the presence of counsel.
(2) No torture, force, violence, threat, intimidation, or any other means which vitiate the free will shall be used
against him. Secret detention places, solitary, incommunicado, or other similar forms of detention are prohibited.
(3) Any confession or admission obtained in violation of this or Section 17 hereof shall be inadmissible in
evidence against him.
(4) The law shall provide for penal and civil sanctions for violations of this section as well as compensation to
and rehabilitation of victims of torture or similar practices, and their families.”

Art 3, Sec. 14. “(1) No person shall be held to answer for a criminal offense without due process of law.
(2) In all criminal prosecutions, the accused shall be presumed innocent until the contrary is proved, and shall
enjoy the right to be heard by himself and counsel, to be informed of the nature and cause of the accusation
against him, to have a speedy, impartial, and public trial, to meet the witnesses face to face, and to have
compulsory process to secure the attendance of witnesses and the production of evidence in his behalf. However,
after arraignment, trial may proceed notwithstanding the absence of the accused provided that he has been duly
notified and his failure to appear is unjustifiable.”

Art 3, Sec. 11. “Free access to the courts and quasi-judicial bodies and adequate legal assistance shall not be
denied to any person by reason of poverty.”

Art 3, Sec. 16. “All persons shall have the right to a speedy disposition of their cases before all judicial, quasi-
judicial, or administrative bodies.”

Art 3, Sec. 17. “No person shall be compelled to be a witness against himself.”

Art 3, Sec. 19. “(1) Excessive fines shall not be imposed, nor cruel, degrading or inhuman punishment inflicted.
Neither shall death penalty be imposed, unless, for compelling reasons involving heinous crimes, the Congress
hereafter provides for it. Any death penalty already imposed shall be reduced to reclusion perpetua.
(2) The employment of physical, psychological, or degrading punishment against any prisoner or detainee or
the use of substandard or inadequate penal facilities under subhuman conditions shall be dealt with by law.”

Art 3, Sec. 21. “No person shall be twice put in jeopardy of punishment for the same offense. If an act is punished
by a law and an ordinance, conviction or acquittal under either shall constitute a bar to another prosecution for the
same act.”

GAMBOA VS. CRUZ


[162 SCRA 642;L-56291; 27 JUN 1988]

Facts:

Petitioner was arrested for vagrancy without a warrant. During a line-up of 5 detainees including petitioner, he
was identified by a complainant to be a companion in a robbery, thereafter he was charged. Petitioner filed a
Motion to Acquit on the ground that the conduct of the line-up, without notice and in the absence of his counsel
violated his constitutional rights to counsel and to due process. The court denied said motion. Hearing was set,
hence the petition.

Issue:

Whether or Not petitioner’s right to counsel and due process violated.

Held:

No. The police line-up was not part of the custodial inquest, hence, petitioner was not yet entitled, at such stage,
to counsel. He had not been held yet to answer for a criminal offense. The moment there is a move or even an
urge of said investigators to elicit admissions or confessions or even plain information which may appear innocent
or innocuous at the time, from said suspect, he should then and there be assisted by counsel, unless he waives
the right, but the waiver shall be made in writing and in the presence of counsel.

On the right to due process, petitioner was not, in any way, deprived of this substantive and constitutional right, as
he was duly represented by a counsel. He was accorded all the opportunities to be heard and to present evidence

Page 123
Section 1-C, SY ’06-‘07
San Beda College of Law – Alabang
Constitutional Law 2 Case Digests

to substantiate his defense; only that he chose not to, and instead opted to file a Motion to Acquit after the
prosecution had rested its case. What due process abhors is the absolute lack of opportunity to be heard.

PEOPLE VS. JUDGE AYSON


[175 SCRA 216; G.R. NO. 85215; 7 JUL 1989]

Facts:

Felipe Ramos was a ticket freight clerk of the Philippine Airlines, assigned at its Baguio City station. It was alleged
that he was involved in irregularities in the sales of plane tickets, the PAL management notified him of an
investigation to be conducted. That investigation was scheduled in accordance with PAL's Code of Conduct and
Discipline, and the Collective Bargaining Agreement signed by it with the Philippine Airlines Employees'
Association (PALEA) to which Ramos pertained. A letter was sent by Ramos stating his willingness to settle the
amount of P76,000. The findings of the Audit team were given to him, and he refuted that he misused proceeds of
tickets also stating that he was prevented from settling said amounts. He proffered a compromise however this
did not ensue. Two months after a crime of estafa was charged against Ramos. Ramos pleaded not guilty.
Evidence by the prosecution contained Ramos’ written admission and statement, to which defendants argued that
the confession was taken without the accused being represented by a lawyer. Respondent Judge did not admit
those stating that accused was not reminded of his constitutional rights to remain silent and to have counsel. A
motion for reconsideration filed by the prosecutors was denied. Hence this appeal.

Issue:

Whether or Not the respondent Judge correct in making inadmissible as evidence the admission and statement of
accused.

Held:

No. Section 20 of the 1987 constitution provides that the right against self-incrimination (only to witnesses other
than accused, unless what is asked is relating to a different crime charged- not present in case at bar).

This is accorded to every person who gives evidence, whether voluntarily or under compulsion of subpoena, in
any civil, criminal, or administrative proceeding. The right is not to "be compelled to be a witness against himself.”
It prescribes an "option of refusal to answer incriminating questions and not a prohibition of inquiry." the right can
be claimed only when the specific question, incriminatory in character, is actually put to the witness. It cannot be
claimed at any other time. It does not give a witness the right to disregard a subpoena, to decline to appear before
the court at the time appointed, or to refuse to testify altogether. It is a right that a witness knows or should know.
He must claim it and could be waived.

Rights in custodial interrogation as laid down in miranda v. Arizona: the rights of the accused include:

1) he shall have the right to remain silent and to counsel, and to be informed of such right.
2) nor force, violence, threat, intimidation, or any other means which vitiates the free will shall be used
against him.
3) any confession obtained in violation of these rights shall be inadmissible in evidence.

The individual may knowingly and intelligently waive these rights and agree to answer or make a statement. But
unless and until such rights and waivers are demonstrated by the prosecution at the trial, no evidence obtained as
a result of interrogation can be used against him.

PEOPLE VS. M AQUEDA


[242 SCRA 565; G.R. NO.112983; 22 MAR 1994]

Facts:

British Horace William Barker (consultant of WB) was slain inside his house in Tuba, Benguet while his Filipino
wife, Teresita Mendoza was badly battered with lead pipes on the occasion of a robbery. Two household helpers
of the victims identified Salvamante (a former houseboy of the victims) and Maqueda as the robbers. Mike
Tabayan and his friend also saw the two accused a kilometer away from the house of the victims that same
morning, when the two accused asked them for directions.

Maqueda was then arrested in Guinyangan, Quezon. He was taken to Calauag, Quezon where he signed a
Sinumpaang Salaysay wherein he narrated his participation in the crime. According to SPO3 Molleno, he
informed Maqueda of his constitutional rights before he signed such document. Afterwards he was brought to the
Benguet Provincial Jail. While he was under detention, Maqueda filed a Motion to Grant Bail. He stated therein

Page 124
Section 1-C, SY ’06-‘07
San Beda College of Law – Alabang
Constitutional Law 2 Case Digests

that "he is willing and volunteering to be a State witness in the above entitled case, it appearing that he is the
least guilty among the accused in this case."

Maqueda also admitted his involvement in the commission of the robbery to Prosecutor Zarate and to Salvosa.

Issue:

Whether or Not the trial court was correct in holding that the Sinumpaan Salaysay is admissible as evidence.

Held:

No. The Sinumpaang Salaysay is inadmissible because it was in clear violation of the constitutional rights of the
accused. First, he was not informed of his right to remain silent and his right to counsel. Second, he cannot be
compelled to be a witness against himself. At the time of the confession, the accused was already facing charges
in court. He no longer had the right to remain silent and to counsel but he had the right to refuse to be a witness
and not to have any prejudice whatsoever result to him by such refusal. And yet, despite his knowing fully well
that a case had already been filed in court, he still confessed when he did not have to do so.

The contention of the trial court that the accused is not entitled to such rights anymore because the information
has been filed and a warrant of arrest has been issued already, is untenable. The exercise of the rights to remain
silent and to counsel and to be informed thereof under Section 12(1) of the Bill of Rights are not confined to that
period prior to the filing of a criminal complaint or information but are available at that stage when a person is
"under investigation for the commission of an offense."

Pursuant to Section 12(3) of the Bill of Rights therefore, such extra-judicial admission is inadmissible as evidence.

As to the admissions made by Maqueda to Prosecutor Zarate and Ray Dean Salvosa, the trial court admitted their
testimony thereon only to prove the tenor of their conversation but not to prove the truth of the admission because
such testimony was objected to as hearsay. Maqueda voluntarily and freely made them to Prosecutor Zarate not
in the course of an investigation, but in connection with Maqueda's plea to be utilized as a state witness; and as to
the other admission (Salvosa), it was given to a private person therefore admissible.

Note: a distinction between a confession and admission has been made by the SC:
Admission of a party. — The act, declaration or omission of party as to a relevant fact may be given in evidence
against him.

Confession. — The declaration of an accused acknowledging his guilt of the offense charged, or of any offense
necessarily included therein, may be given in evidence against him.

PEOPLE VS. BANDULA


[232 SCRA 566; G.R. NO. 89223; 27 MAY 1994]

Facts:

Six armed men barged into the compound of Polo Coconut Plantation in Tanjay, Negros Oriental. The armed men
were identified by Security Guard, including accused. Salva and Pastrano, security guards were hogtied and
accused proceeded to the Atty. Garay, counsel of plantation. They ransacked the place and took with them
money and other valuables. Atty. Garay was killed. Accused-appellant is charged with robbery with homicide
along with 3 others who were acquitted for insufficiency of evidence. Appellant was convicted.

Now, appellant argues that the extrajudicial confessions he and accused Dionanao executed suffer from
constitutional infirmities, hence, inadmissible in evidence considering that they were extracted under duress and
intimidation, and were merely countersigned later by the municipal attorney who, by the nature of his position,
was not entirely an independent counsel nor counsel of their choice. Consequently, without the extrajudicial
confessions, the prosecution is left without sufficient evidence to convict him of the crime charged.

Issue:

Whether or Not extrajudicial confessions of appellant is admissible as evidence against him.

Held:

No. When accused-appellant Bandula and accused Dionanao were investigated immediately after their arrest,
they had no counsel present. If at all, counsel came in only a day after the custodial investigation with respect to
accused Dionanao, and two weeks later with respect to appellant Bandula. And, counsel who supposedly
assisted both accused was Atty. Ruben Zerna, the Municipal Attorney of Tanjay. On top of this, there are telltale

Page 125
Section 1-C, SY ’06-‘07
San Beda College of Law – Alabang
Constitutional Law 2 Case Digests

signs that violence was used against the accused. Certainly, these are blatant violations of the Constitution which
mandates in
Sec. 12, Art. III. Irregularities present include:

1. The investigators did not inform the accused of their right to remain silent and to have competent and
independent counsel, preferably of their own choice, even before attempting to elicit statements that
would incriminate them.
2. Investigators continuously disregard the repeated requests of the accused for medical assistance.
Reason for Accused Sedigo’s "black eye" which even
Pat. Baldejera admitted is not established, as well as Bandula’s fractured rib.
3. Counsel must be independent. He cannot be a special counsel, public or private prosecutor, counsel of
the police, or a municipal attorney whose interest is admittedly adverse to the accused.

PEOPLE VS. LUCERO


[244 SCRA 425; G.R. NO.97936; 29 MAY 1995]

Facts:

Alejandro Lucero, Bienvenido Echavez, Balbino Echavez, Peter Doe, Richard Doe and John Doe were charged
with the crime of robbery with homicide.

The prosecution:

Accused-appellant (alighted from a gray-reddish car), armed with handgun, blocked the way of the said
complainant who was on board a Mercedes Benz passing along Road 14, Mindanao Avenue, Pag-asa, QC, rob
and carry away cash money; one gold necklace with cross pendant, 7 karat; one gold Rolex watch; one 3 karat
gold ring; one 2 karat gold ring, domino style; one solid gold bracelet; all worth P363,600.00, belonging to DR.
DEMETRIO Z. MADRID. Accused shot LORENZO BERNALES y ALERIA, a driver of the said offended party,
thus inflicting upon him mortal wounds, which resulted to the instantaneous death of ALERIA.

Only the accused Echavez brothers and Alejandro Lucero were apprehended.

When Lucero told him that he had no lawyer, in due time, Atty. Diosdado Peralta conferred with Lucero. He
apprised Lucero of his constitutional rights. He observed no reaction from Lucero. Nonetheless, Atty. Peralta
gathered the impression that Lucero understood his advice.

When the investigator started asking the preliminary questions, Atty. Peralta left to attend the wake of his friend.
The next morning, Lucero was accompanied by CIS agents to Atty. Peralta's house. The extrajudicial statement of
Lucero was presented to Atty. Peralta. It was already signed by Lucero.

The three accused denied complicity in the crime charged.

Appellant Lucero's defense is alibi. He testified that he was at his house in Caloocan City.

He said he was surprised when several unidentified men accosted him while he was walking towards his house.
They chased him, handcuffed and blindfolded him and pushed him into a jeep. He was blindfolded the whole night
and did not know where he was taken. The men turned out to be police officers.
The next day, he learned he was in Camp Crame. He claimed that he was tortured. He was not informed of the
offense for which he was being investigated. Neither did they reveal the identity of the complainant.

Lucero denied knowing Dr. Madrid, the Echavez brothers and the other accused in this case. He said he only met
Dr. Madrid at the CIS Office during the police line-up. He was made to line-up four (4) times before Dr. Madrid
finally identified him on the fourth time.

Lucero also claimed he signed the extrajudicial confession under duress. He denied engaging the services of
Atty, Peralta. He likewise confirmed that Atty. Peralta was not present during his actual custodial interrogation.

After trial, the court a quo acquitted the Echavez brothers for insufficient evidence. The trial court, however,
convicted accused Lucero GUILTY as principal by direct participation of Robbery with Homicide and sentenced to
suffer an imprisonment term of RECLUSION PERPETUA.

Issue:

Whether or Not the lower court erred in convicting accused-appellant.

Held:

Page 126
Section 1-C, SY ’06-‘07
San Beda College of Law – Alabang
Constitutional Law 2 Case Digests

Appellant's conviction cannot be based on his extrajudicial confession.

Constitution requires that a person under investigation for the commission of a crime should be provided with
counsel. The Court have constitutionalized the right to counsel because of hostility against the use of duress and
other undue influence in extracting confessions from a suspect. Force and fraud tarnish confessions can render
them inadmissible.

The records show that Atty. Peralta, who was not the counsel of choice of appellant. Atty. Peralta himself
admitted he received no reaction from appellant although his impression was that appellant understood him. More
so, it was during his absence that appellant gave an uncounselled confession.

Constitution requires the right to counsel, it did not mean any kind of counsel but effective and vigilant counsel.
The circumstances clearly demonstrate that appellant received no effective counseling from Atty. Peralta.

Whereof, Decision convicting appellant Alejandro Lucero y Cortel is hereby reversed.

PEOPLE VS. AGUSTIN


[240 SCRA 541; G.R. NO. 110290; 25 JAN 1995]

Facts:

Dr. Bayquen, a dentist, together with his son, Anthony; Anthony's girlfriend, Anna Theresa; his daughter, Dominic;
and Danny, a family friend, were on their way aboard their Brasilia to the doctor's residence at Malvar Street,
Baguio City. While they were cruising along Malvar Street and nearing the Baptist church, a man came out from
the right side of a car parked about two meters to the church. The man approached the Brasilia, aimed his
armalite rifle through its window, and fired at the passengers. The Brasilia swerved and hit a fence. The gunman
immediately returned to the parked car which then sped away. All those in the car were hit and Dr. Bayquen and
Anna Theresa died on the spot. Dominic was bale to get out of the Brasilia to run to the Alabanza store where she
telephoned her mother. Later, she and her mother brought her father and Anthony to the hospital. Danny went
home and was then brought to the Hospital for treatment.

Accused Quiaño, an alleged former military agent who had been picked up by the police authorities, confessed
during the investigation conducted by Baguio City Fiscal Erdolfo Balajadia in his office that he was the triggerman.
He implicated Abenoja, Jr., who engaged him to kill Dr. Bayquen for a fee, Cartel, who provided the armalite, and
a certain "Jimmy." During the investigation, Wilfredo Quiaño was assisted by Atty. Reynaldo Cajucom.
Stenographic notes of the proceedings during the investigation as transcribed with the sworn statement of Quiaño
was signed, with the assistance of Atty. Cajucom, and swore to before City Fiscal Balajadia. The following day,
Agustin was apprehended, and was investigated and was afforded the privileges like that of Quijano. Agustin’s
defense interpose that he was forced to admit involvement at gunpoint in the Kennon Road. He further declared
that although he was given a lawyer, Cajucom (a law partner of the private prosecutor), he nevertheless, asked
for his uncle Atty. Oliver Tabin, and that Atty. Cajucom interviewed him from only two minutes in English and
Tagalog but not in Ilocano, the dialect he understands. The promise that he would be discharged as a witness did
not push through since Quijano escaped. However the RTC convicted him, since conspiracy was established.
Hence the appeal.

Issue:

Whether or Not accused-appellant’s extrajudicial statements admissible as evidence.

Held:

No. Extrajudicial statement is not extrajudicial confession. In a confession, there is an acknowledgment of guilt of
the accused, while an admission is a statement direct or implied of facts pertinent to the issue. The rule on
inadmissibility, however expressly includes admissions, not just confessions.The extrajudicial admission of the
appellant, contained in twenty-two pages appear to be signed by him and Atty. Cajucom but for reasons not
explained in the records, the transcript of the notes which consists of twelve pages was not signed by the
appellant. Since the court cannot even read or decipher the stenographic notes it cannot be expected that
appellant, who is a farmer and who reached only the fourth grade, to read or decipher its contents. The appellant,
therefore was deprived of his rights under Section 12(1), Article III of the Constitution. Firstly, he was not fully and
properly informed of his rights. The appellant was not explicitly told of his right to have a competent and
independent counsel of his choice, specifically asked if he had in mind any such counsel and, if so, whether he
could afford to hire his services, and, if he could not, whether he would agree to be assisted by one to be provided
for him. He was not categorically informed that he could waive his rights to remain silent and to counsel and that
this waiver must be in writing and in the presence of his counsel. He had, in fact, waived his right to remain silent
by agreeing to be investigated. Yet, no written waiver of such right appears in the transcript and no other

Page 127
Section 1-C, SY ’06-‘07
San Beda College of Law – Alabang
Constitutional Law 2 Case Digests

independent evidence was offered to prove its existence. In short, after the appellant said that he wanted to be
assisted by counsel, the City fiscal, through suggestive language, immediately informed him that Atty. Cajucom
was ready to assist him. Moreso said counsel is not independent since he is an associate of the private
prosecutor.

PEOPLE VS. BOLANOS


[211 SCRA 262; G.R. NO. 101808; 3 JUL 1992]

Facts:

Oscar Pagdalian was murdered in Marble Supply, Balagtas Bulacan. According to Pat. Rolando Alcantara and
Francisco Dayao, deceased was with two companions on the previous night, one of whom the accused who had
a drinking spree with the deceased. When they apprehended the accused they found the firearm of the deceased
on the chair where the accused was allegedly seated. They boarded accused along with Magtibay, other accused
on the police vehicle and brought them to the police station. While in the vehicle Bolanos admitted that he killed
the deceased. RTC convicted him hence the appeal.

Issue:

Whether or Not accused-appellant deprived of his constitutional right to counsel.

Held:

Yes. Being already under custodial investigation while on board the police patrol jeep on the way to the Police
Station where formal investigation may have been conducted, appellant should have been informed of his
Constitutional rights under Article III, Section 12 of the 1987 Constitution, more particularly par. 1 and par. 3.

PEOPLE VS. MACAM


[238 SCRA 306; G.R. NOS. 91011-12; 24 NOV 1994]

Facts:

Prosecution’s version:

On Aug 18,1987, Eduardo Macam, Antonio Cedro, Eugenio Cawilan Jr., Danilo Roque and Ernesto Roque went
to the house of Benito Macam (uncle of Eduardo Macam) located at 43 Ferma Road QC. Upon the arrival of the
accused, Benito invited the former to have lunch. Benito asked his maid Salvacion Enrera to call the companions
of Eduardo who were waiting in a tricycle outside the house. A. Cedro, E. Cawilan and D. Roque entered the
house while E. Roque remained in the tricycle. After all the accused had taken their lunch, Eduardo Macam
grabbed the clutch bag of Benito Macam and pulled out his uncle’s gun then declared a hold-up. They tied up the
wife (Leticia Macam), children, maid (Salvacion) and Nilo Alcantara and brought them to the room upstairs. After
a while Leticia was brought to the bathroom and after she screamed she was stabbed and killed by A. Cedro.
Benito, Nilo and Salvacion was also stabbed but survived. The total value of the items taken was P536, 700.00.

Defense’s version:

Danilo Roque stated that he being a tricycle driver drove the 4 accused to Benito’s house for a fee of P50.00.
Instead of paying him, he was given a calling card by Eduardo Macam so that he can be paid the following day.
Upon arriving, he went with the accused inside the house to have lunch. Thereafter he washed the dishes and
swept the floor. When Eugenio Cawilan pulled a gun and announced the hold-up, he was asked to gather some
things and which he abided out of fear. While putting the said thins inside the car of Benito (victim) he heard the
accused saying “kailangan patayin ang mga taong yan dahil kilala ako ng mga yan”. Upon hearing such phrase
he escaped and went home using his tricycle. He also testified that his brother Ernesto Roque has just arrived
from the province and in no way can be involved in the case at bar. On the following day, together with his
brother, they went to the factory of the Zesto Juice (owned by the father of Eduardo Macam) for him to get his
payment (50.00) . He and his brother was suddenly apprehended by the security guards and brought to the police
headquarters in Q.C. They were also forced to admit certain things.

After which, he together with all the accused, in handcuffs and bore contusions on their faces caused by blows
inflicted in their faces during investigation, was brought to the QC General Hospital before each surviving victims
and made to line-up for identification. Eugenio Cawilan was also charged with Anti-fencing Law but was acquitted
in the said case.

Issue:

Page 128
Section 1-C, SY ’06-‘07
San Beda College of Law – Alabang
Constitutional Law 2 Case Digests

Whether or Not their right to counsel has been violated. WON the arrest was valid. WON the evidence from the
line-up is admissible.

Held:

It is appropriate to extend the counsel guarantee to critical stages of prosecution even before trial. A police line-up
is considered a “critical” stage of the proceedings. Any identification of an uncounseled accused made in a police
line-up is inadmissible. HOWEVER, the prosecution did not present evidence regarding appellant’s identification
at the line-up. The witnesses identified the accused again in open court. Also, accused did not object to the in-
court identification as being tainted by illegal line-up.

The arrest of the appellants was without a warrant. HOWEVER, they are estopped from questioning the legality of
such arrest because they have not moved to quash the said information and therefore voluntarily submitted
themselves to the jurisdiction of the trial court by entering a plea of not guilty and participating in trial.

The court believed the version of the prosecution. Ernesto Roque, while remaining outside the house served as a
looked out.

Wherefore, decision of lower court is Affirmed. Danilo Roque and Ernesto Roque is guilty of the crime of robbery
with homicide as co-conspirators of the other accused to suffer reclusion perpetua.

Things taken: 2 toygun, airgun riffle, CO2 refiller, TV, betamax tapes, betamax rewinder, Samsonite attache case,
typewriter, chessboard, TOYOTA Crown Car Plate No. CAS-997, assorted jewelry. .22 gun and money.

PEOPLE VS. DY
[158 SCRA 111; G.R. 74517; 23 FEB 1988]

Facts:

Pat. Padilla reported along with Benny Dy, with caliber .38 as suspect to the shooting incident at "Benny's Bar," at
Sitio Angol, Manoc-Manoc Malay, Aklan (Boracay) situated on the Island which caused the death of Christian
Langel Philippe, tourist, 24 years old and a Swiss nationale. He was charged with the Murder With the Use of
Unlicensed firearms. Appellant alleges that he carried the victim to the shore to be brought to the hospital to save
the latter, and who facilitated the surrender to Pat. Padilla a gun which his helper found the following morning
while cleaning the bar. Accused posted bail which was granted. The accused denied having made any oral
confession alleging that he went to Pat. Padilla not to report the incident but to state that a boy helper in the bar
had found a gun on the sand floor while cleaning and that Pat. Padilla picked up the gun from the bar at his
request. The Accused argues that even if he did make such a confession, the same would be inadmissible in
evidence. He was found guilty in the RTC. Hence the appeal.

Issue:

Whether or Not the lower court correct in saying that the constitutional procedure on custodial interrogation is not
applicable in the instant case.

Held:

YES. Appellant's assertion that the gun he had surrendered was merely found by a boy helper while cleaning the
bar deserves no credence for, if it were so, it would have been absurd for him to have placed himself under police
custody in the early morning after the incident. Sworn Complaint for "Murder with Use of Unlicensed Firearm"
signed by the Chief of Police also attests to Appellant's oral confession. That Complaint forms part of the record of
the proceedings before the Municipal Circuit Trial Court of Buruanga, Aklan, and is prima facie evidence of the
facts therein stated. Appellant's voluntary surrender implies no violation as "no warrant of arrest is issued for the
apprehension of the accused for the reason that he is already under police custody before the filing of the
complaint." What was told by the Accused to Pat, Padilla was a spontaneous statement not elicited through
questioning, but given in ordinary manner. No written confession was sought to be presented in evidence as a
result of formal custodial investigation.

NAVALLO VS. SANDIGANBAYAN


[234 SCRA 177; G.R. NO. 97214; 18 JUL 1994]

Facts:

Page 129
Section 1-C, SY ’06-‘07
San Beda College of Law – Alabang
Constitutional Law 2 Case Digests

Accused was the Collecting and Disbursing Officer of the Numancia National Vocational School, which school is
also located at del Carmen, Surigao del Norte. His duties included the collection of tuition fees, preparation of
vouchers for salaries of teachers and employees, and remittance of collections exceeding P500.00 to the National
Treasury. An information for malversation of public funds was filed. A warrant of arrest was issued, but accused-
petitioner could not be found. on 10 December 1978, Presidential Decree No. 1606 took effect creating the
Sandiganbayan and conferring on it original and exclusive jurisdiction over crimes committed by public officers
embraced in Title VII of the Revised Penal Code. On 15 November 1984, Navallo was finally arrested. He was
released on provisional liberty upon the approval of his property bail bond. When arraigned by the RTC on 18 July
1985, he pleaded not guilty. Upon motion of the prosecution, the RTC transferred the case and transmitted its
records to the Sandiganbayan. Special Prosecutor Luz L. Quiñones-Marcos opined that since Navallo had
already been arraigned before the case was transferred to the Sandiganbayan, the RTC should continue taking
cognizance of the case. The matter was referred to the Office of the Ombudsman which held otherwise. The
information was then docketed with the Sandiganbayan. A new order for Navallo's arrest was issued by the
Sandiganbayan. The warrant was returned with a certification by the RTC Clerk of Court that the accused had
posted a bail bond. Navallo filed a motion to quash, contending (1) that the Sandiganbayan had no jurisdiction
over the offense and the person of the accused and (2) that since the accused had already been arraigned by the
RTC, the attempt to prosecute him before the Sandiganbayan would constitute double jeopardy. However this
was denied and trial ensued and he was found guilty.

Issue:

Whether or Not the constitutional right against double jeopardy and in custodial investigations in favor of the
accused violated.

Held:

No. Double jeopardy requires the existence of the following requisites:

(1) The previous complaint or information or other formal charge is sufficient in form and substance to sustain
a conviction;
(2) The court has jurisdiction to try the case;
(3) The accused has been arraigned and has pleaded to the charge; and
(4) The accused is convicted or acquitted or the case is dismissed without his express consent.

The RTC was devoid of jurisdiction when it conducted an arraignment of the accused which by then had already
been conferred on the Sandiganbayan. Moreover, neither did the case there terminate with conviction or acquittal
nor was it dismissed.

No. Appellant is not in custodial investigation. A person under a normal audit examination is not under custodial
investigation. An audit examiner himself can hardly be deemed to be the law enforcement officer contemplated in
the above rule. In any case, the allegation of his having been "pressured" to sign the Examination Report
prepared by Dulguime (examined cash, as ordered by Espino, the provincial auditor) appears to be belied by his
own testimony.

PEOPLE VS. ALICANDO


[251 SCRA 293; G.R. NO. 117487; 2 DEC 1995]

Facts:

Appellant was charged with the crime of rape with homicide of Khazie Mae Penecilla, a minor, four years of age,
choking her with his right hand. The incident happened after appellant drank liquor. A neighbor, Leopoldo
Santiago found the victim’s body and the parents and police were informed. Appellant was living in his uncle's
house some five arm's length from Penecilla's house. Appellant was arrested and interrogated by PO3 Danilo
Tan. He verbally confessed his guilt without the assistance of counsel. On the basis of his uncounselled verbal
confession and follow up interrogations, the police came to know and recovered from appellant's house, Khazie
Mae's green slippers, a pair of gold earrings, a buri mat, a stained pillow and a stained T-shirt all of which were
presented as evidence for the prosecution. He was arraigned with the assistance of Atty. Rogelio Antiquiera of the
PAO. Appellant pleaded guilty. The RTC convicted him. Hence an automatic review for the imposition of death
penalty.

Issue:

Whether or Not the death penalty proper.

Held:

Page 130
Section 1-C, SY ’06-‘07
San Beda College of Law – Alabang
Constitutional Law 2 Case Digests

No. The records do not reveal that the Information against the appellant was read in the language or dialect
known to him. The Information against the appellant is written in the English language. It is unknown whether the
appellant knows the English language. Neither is it known what dialect is understood by the appellant. Nor is
there any showing that the Information couched in English was translated to the appellant in his own dialect
before his plea of guilt. The RTC violated section 1(a) of Rule 116, the rule implementing the constitutional right of
the appellant to be informed of the nature and cause of the accusation against him. It also denied appellant his
constitutional right to due process of law. It is urged that we must presume that the arraignment of the appellant
was regularly conducted. When life is at stake, we cannot lean on this rebuttable presumption. There could be no
presumption. The court must be sure.

The trial court violated section 3 of Rule 116 when it accepted the plea of guilt of the appellant. Said section
requires that the court shall conduct a searching inquiry the voluntariness and full comprehension of the
consequences of his plea and require the prosecution to prove his guilt and the precise degree of culpability. The
accused may also present evidence in his behalf. The trial court simply inquired if appellant had physical marks of
maltreatment. It did not ask the appellant when he was arrested, who arrested him, how and where he was
interrogated, whether he was medically examined before and after his interrogation, etc. It limited its efforts trying
to discover late body marks of maltreatment as if involuntariness is caused by physical abuse alone.

Further, there are physical evidence to prove Khazie was raped. These consists of a pillow with bloodstains in its
center 14 and the T-shirt 15 of the accused colored white with bloodstains on its bottom. These physical evidence
are evidence of the highest order. They strongly corroborate the testimony of Luisa Rebada that the victim was
raped.These are inadmissible evidence for they were gathered by PO3 Danilo Tan of the Iloilo City PNP as a
result of custodial interrogation where appellant verbally confessed to the crime without the benefit of counsel.

PEOPLE VS. DE GUZMAN


[224 SCRA 93; G.R. NOS. 98321-24; 30 JUN 1993]

Facts:

All the accused were charged before the Regional Trial Court of Cebu with three counts of murder and one count
of frustrated murder in four Informations. The victim Jose Bantug was found with gunshots in the head, body, and
skull. The other three informations charged them with the murder of Francisco Carteciano y Sorilla and Antonio S.
Carteciano, and the frustrated murder of Lorna V. Carteciano. The other 8 accused were acquitted on the ground
of reasonable doubt, while Victor Nuñez was found guilty. The facts shown by evidence are: One morning, Major
Antonio Carteciano was driving his private jeep Camp General Arcadio Maxilom in Lahug, Cebu City where he
was stationed as medical officer of the PC/INP Provincial Command. In the front seat with him is his wife Lorna,
and at the backseat are his mother in law, son, brother Francisco, neighbor Bantug, and Bantug’s son. Near the
intersection, gunshots were heard from the left side of the street. Major Carteciano took his .45 cal pistol and
fired. However, gunshots were fired in succession, and Major Carteciano, his brother Francisco, Jose Bantug, and
his wife Lorna were hit. When the jeep stopped, several gunmen approached them. Nuñez demanded Lorna to
give Nuñez her husband’s pistol. Lorna asked to take her valuables instead. Then, Nuñez shot Major Carteciano’s
head point blank. Then the gunmen hijacked another jeep and took off. Lorna, her mother Juanita Ricaplaza, and
her son Reiser Carteciano positively identified the accused. Lorna identified Nuñez as the one who shot her
husband. Nuñez claimed that his arrest was illegal and that he was deprived of his right to counsel when he was
subjected to a paraffin test without the assistance of counsel.

Issue:

Whether or not the accused Nuñez’s constitutional right was violated

Held:

No. Nuñez pleaded not guilty at the arraignment. Therefore, he is estopped from questioning the validity of his
arrest. Furthermore, the illegal arrest of an accused is not sufficient cause for setting aside a valid judgment
rendered upon a sufficient complaint after trial free from error. The witnesses also positively identified the
accused, so he cannot question the credibility of the witnesses. Regarding his right to counsel, the Supreme
Court held that-- the right to counsel attaches only upon the start of an investigation, that is, when the
investigating officer starts to ask questions to elicit information and/or confessions or admissions from the
accused. At such point or stage, the person being interrogated must be assisted by counsel to avoid the
pernicious practice of extorting false or coerced admissions or confessions from the lips of the person undergoing
interrogation. In the case at bar, when accused was subjected to a paraffin test, he was not then under custodial
investigation. Accused-appellant also argued that since his co-accused were acquitted, then their acquittal
negates conspiracy among them, and he should not be convicted with the charges filed. However, the Court held
that conspiracy was still proven by the evidence, and the other co-accused were acquitted only because there
was reasonable doubt. Therefore, accused-appellant is still convicted of the four charges against him.

Page 131
Section 1-C, SY ’06-‘07
San Beda College of Law – Alabang
Constitutional Law 2 Case Digests

We, therefore, find that the conviction of accused-appellant for the crimes charged has been established beyond
reasonable doubt and the penalty imposed is in accordance with law. However, the civil indemnity imposed by the
trial court should be increased to P50,000 in conformity with our recent rulings on the matter.

WHEREFORE, except for the modification that the civil indemnity to be paid by accused-appellant Victor Nuñez,
Jr. to the heirs of each victim who died is hereby increased to P50,000, the appealed decision is hereby affirmed
in all other respects, with costs against accused-appellant

PEOPLE VS. JUDGE DONATO


[198 SCRA 130; G.R. NO.79269; 5 JUN 1991]

Facts:

Private respondent and his co-accused were charged of rebellion on October 2, 1986 for acts committed before
and after February 1986. Private respondent filed with a Motion to Quash alleging that: (a) the facts alleged do not
constitute an offense; (b) the Court has no jurisdiction over the offense charged; (c) the Court has no jurisdiction
over the persons of the defendants; and (d) the criminal action or liability has been extinguished. This was denied.
May 9, 1987 Respondent filed a petition for bail, which was opposed that the respondent is not entitled to bail
anymore since rebellion became a capital offense under PD 1996, 942 and 1834 amending ART. 135 of RPC. On
5 June 1987 the President issued Executive Order No. 187 repealing, among others, P.D. Nos. 1996, 942 and
1834 and restoring to full force and effect Article 135 of the Revised Penal Code as it existed before the
amendatory decrees. Judge Donato now granted the bail, which was fixed at P30,000.00 and imposed a condition
that he shall report to the court once every two months within the first ten days of every period thereof. Petitioner
filed a supplemental motion for reconsideration indirectly asking the court to deny bail to and to allow it to present
evidence in support thereof considering the "inevitable probability that the accused will not comply with this main
condition of his bail. It was contended that:

1. The accused has evaded the authorities for thirteen years and was an escapee from detention when
arrested; (Chairman of CPP-NPA)
2. He was not arrested at his residence as he had no known address;
3. He was using the false name "Manuel Mercado Castro" at the time of his arrest and presented a Driver's
License to substantiate his false identity;
4. The address he gave "Panamitan, Kawit, Cavite," turned out to be also a false address;
5. He and his companions were on board a private vehicle with a declared owner whose identity and
address were also found to be false;
6. Pursuant to Ministry Order No. 1-A dated 11 January 1982 , a reward of P250,000.00 was offered and
paid for his arrest.

This however was denied. Hence the appeal.

Issue:

Whether or Not the private respondent has the right to bail.

Held:

Yes. Bail in the instant case is a matter of right. It is absolute since the crime is not a capital offense, therefore
prosecution has no right to present evidence. It is only when it is a capital offense that the right becomes
discretionary. However it was wrong for the Judge to change the amount of bail from 30K to 50K without hearing
the prosecution.

Republic Act No. 6968 approved on 24 October 1990, providing a penalty of reclusion perpetua to the crime of
rebellion, is not applicable to the accused as it is not favorable to him.

Accused validly waived his right to bail in another case(petition for habeas corpus). Agreements were made
therein: accused to remain under custody, whereas his co-detainees Josefina Cruz and Jose Milo Concepcion will
be released immediately, with a condition that they will submit themselves in the jurisdiction of the court. Said
petition for HC was dismissed. Bail is the security given for the release of a person in custody of the law. Ergo,
there was a waiver. We hereby rule that the right to bail is another of the constitutional rights which can be
waived. It is a right which is personal to the accused and whose waiver would not be contrary to law, public order,
public policy, morals, or good customs, or prejudicial to a third person with a right recognized by law.

CARPIO VS. M AGLALANG


[196 SCRA 41; G.R. NO. 78162; 19 APR 1991]

Page 132
Section 1-C, SY ’06-‘07
San Beda College of Law – Alabang
Constitutional Law 2 Case Digests

Facts:

On January 8, 1987, information for the murder of Mayor Jose Payumo of Dinalupihan Bataan was filed against
Escaño and ten other unindentified persons by the provincial fiscal in the RTC of Bataan at Balanga. Four days
later, the Acting Executive Judge of said court issued an order of arrest against Escaño recommending no bail for
his provisional liberty. Pat. Cesar Diego who acted on the warrant returned to the court with a certification issued
by NBI agent Gonzales, stating therein that accused was still under investigation.

Through counsel Rolando T. Cainoy, Escaño filed in court an urgent ex-parte motion for his commitment at the
provincial jail of Bataan on the ground that he wanted to be where his family and counsel could have easy access
to him. He alleged therein that his detention at the NBI headquarters in Manila was irregular and in defiance of the
warrant of arrest issued by the court. This was granted.

A motion for reconsideration was filed by Director Carpio stating that the NBI needed physical custody of Escaño
for the identification of the other accused in the case who were still the objects of a manhunt by NBI agents; that
in view of the finding of NBI agents that the other accused and suspects in the case were subversive elements or
members of the New People's Army, it was for the best interest of Escaño that he be detained at the NBI lock-up
cell where security measures were adequate; and that the NBI would produce the person of Escaño before the
court whenever required and every time that there would be a hearing on the case. However another motion was
executed by Escaño stating that he now wants to be detained in the NBI, alleging that he did not authorize his
counsel to execute the first motion. Also, Escaño's counsel Rolando T. Cainoy filed an application for bail stating
that Escaño was arrested by NBI agents on December 7, 1986 without a warrant having been presented to him
and that since then he had been detained in the lock-up cell of the NBI; that said agents, also without a warrant,
searched his house when he was arrested; that he was subjected to inhuman torture and forced to admit
participation in the killing of Mayor Payumo and to implicate other persons, and that during the custodial
investigation, he was not represented by counsel. In opposing said application, the public prosecutor averred that
the accused was charged with a capital offense for which no bail may be availed of, that the reasons advanced in
said application would be overcome by strong and sufficient evidence; and that during the custodial investigation,
he was represented by counsel. The court granted the application for bail fixing the same at P30,000, having
found no sufficient evidence against accused. Director Carpio was ordered to justify his actions and so as not to
be considered in contempt.

Issue:

Whether or Not the order granting right to bail was proper.

Held:

No. The order granting bail had been rendered moot not only by the fact that he had been released from NBI
custody, but also because Escaño jumped bail and did not appear on the date set for his arraignment.
Notwithstanding, the Court resolved the issue of the legality of the order granting bail to Escaño. Although the
right to bail is principally for the benefit of the accused, in the judicial determination of the availability of said right,
the prosecution should be afforded procedural due process. Thus, in the summary proceeding on a motion
praying for admission to bail, the prosecution should be given the opportunity to present evidence and, thereafter,
the court should spell out at least a resume of the evidence on which its order granting or denying bail is based.
Otherwise, the order is defective and voidable. In the case at bar the RTC erred in not summarizing the factual
basis of its order granting bail, the court merely stated the number of prosecution witnesses but not their
respective testimonies, and concluded that the evidence presented by the prosecution was not "sufficiently
strong" to deny bail to Escaño.

The facts, however, that Mayor Payumo was killed on August 20, 1986 when the 1973 Constitution allowing the
death penalty was still in force and that the application for bail was made on March 5, 1987 during the effectivity
of the 1987 Constitution which abolished the death penalty, should not have gotten in the way of resolving the
application for bail in accordance with the Constitution and procedural rules. Section 13, Article III of the
Constitution explicitly provides that "(a)ll persons, except those charged with offenses punishable by reclusion
perpetua when evidence of guilt is strong, shall before conviction, be bailable by sufficient sureties, or be released
on recognizance as may be provided by law." As the phrase "capital offenses" has been replaced by the phrase
"offenses punishable by reclusion perpetua, 25 crimes punishable by reclusion perpetua instead of those
punishable by the death penalty, when evidence of guilt is strong, are the exceptions to the rule that the right to
bail should be made available to all accused. As the court itself acknowledged in its order of April 2, 1987 that
"capital punishment" in Section 4, Rule 114 has been amended to reclusion perpetua, the court should have
proceeded accordingly: i.e., resolved the application for bail pursuant to Section 13, Article III of the Constitution.
It did not have to invoke the abolition of the death penalty and the lack of legislative enactment restoring it in
justifying the grant of bail. All it had to do was to determine whether evidence of guilt is strong in the light of the
provision of Section 13, Article III.

Page 133
Section 1-C, SY ’06-‘07
San Beda College of Law – Alabang
Constitutional Law 2 Case Digests

The RTC has the discretion in the consideration of the strength of the evidence at hand. However, in the exercise
of said discretion, the court is controlled by the following: first, the applicable provisions of the Constitution and the
statutes; second, by the rules which this Court may promulgate; and third, by those principles of equity and justice
that are deemed to be part of the laws of the land. 27 The lower court not only failed to properly apply the
pertinent provisions of the Constitution and the Rules but it also disregarded equity and justice by its failure to
take into account the factual milieu surrounding the detention of Escaño

PEOPLE VS. FORTES


[223 SCRA 619; G.R. NO. 90643; 25 JUN 1993]

Facts:

Agripino Gine of Barangay Naburacan, Municipality of Matnog, Province of Sorsogon, accompanied his 13-year
old daughter, Merelyn, to the police station of the said municipality to report a rape committed against the latter by
the accused. Following this, the accused was apprehended and charged. A bond of P25000 was granted for
accused’s provisional release. The MCTC found him guilty. An appeal to RTC was filed, the request for the fixing
of bond was denied. Now accused assails denial of bail on the ground that the same amounted to an undue
denial of his constitutional right to bail.

Issue:

Whether or Not the accused’s right to bail violated.

Held:

No. It is clear from Section 13, Article III of the 1987 Constitution and Section 3, Rule 114 of the Revised Rules of
Court, as amended, that before conviction bail is either a matter of right or of discretion. It is a matter of right when
the offense charged is punishable by any penalty lower than reclusion perpetua. To that extent the right is
absolute. If the offense charged is punishable by reclusion perpetua bail becomes a matter of discretion. It shall
be denied if the evidence of guilt is strong. The court's discretion is limited to determining whether or not evidence
of guilt is strong. But once it is determined that the evidence of guilt is not strong, bail also becomes a matter of
right. If an accused who is charged with a crime punishable by reclusion perpetua is convicted by the trial court
and sentenced to suffer such a penalty, bail is neither a matter of right on the part of the accused nor of discretion
on the part of the court.

COMMENDADOR VS. DE VILLA


[200 SCRA 80; G.R. NO. 93177; 2 AUG 1991]

Facts:

The petitioners in G.R. Nos. 93177 and 96948 who are officers of the AFP were directed to appear in person
before the Pre-Trial Investigating Officers for the alleged participation the failed coup on December 1 to 9, 1989.
Petitioners now claim that there was no pre-trial investigation of the charges as mandated by Article of War 71. A
motion for dismissal was denied. Now, their motion for reconsideration. Alleging denial of due process.

In G.R. No. 95020, Ltc Jacinto Ligot applied for bail on June 5, 1990, but the application was denied by GCM
No.14. He filed with the RTC a petition for certiorari and mandamus with prayer for provisional liberty and a writ of
preliminary injunction. Judge of GCM then granted the provisional liberty. However he was not released
immediately. The RTC now declared that even military men facing court martial proceedings can avail the right to
bail.

The private respondents in G.R. No. 97454 filed with SC a petition for habeas corpus on the ground that they
were being detained in Camp Crame without charges. The petition was referred to RTC. Finding after hearing that
no formal charges had been filed against the petitioners after more than a year after their arrest, the trial court
ordered their release.

Issue:

Whether or Not there was a denial of due process.

Whether or not there was a violation of the accused right to bail.

Held:

Page 134
Section 1-C, SY ’06-‘07
San Beda College of Law – Alabang
Constitutional Law 2 Case Digests

NO denial of due process. Petitioners were given several opportunities to present their side at the pre-trial
investigation, first at the scheduled hearing of February 12, 1990, and then again after the denial of their motion of
February 21, 1990, when they were given until March 7, 1990, to submit their counter-affidavits. On that date,
they filed instead a verbal motion for reconsideration which they were again asked to submit in writing. They had
been expressly warned in the subpoena that "failure to submit counter-affidavits on the date specified shall be
deemed a waiver of their right to submit controverting evidence." Petitioners have a right to pre-emptory
challenge. (Right to challenge validity of members of G/SCM)

It is argued that since the private respondents are officers of the Armed Forces accused of violations of the
Articles of War, the respondent courts have no authority to order their release and otherwise interfere with the
court-martial proceedings. This is without merit. * The Regional Trial Court has concurrent jurisdiction with the
Court of Appeals and the Supreme Court over petitions for certiorari, prohibition or mandamus against inferior
courts and other bodies and on petitions for habeas corpus and quo warranto.

The right to bail invoked by the private respondents has traditionally not been recognized and is not available in
the military, as an exception to the general rule embodied in the Bill of Rights. The right to a speedy trial is given
more emphasis in the military where the right to bail does not exist.

On the contention that they had not been charged after more than one year from their arrest, there was
substantial compliance with the requirements of due process and the right to a speedy trial. The AFP Special
Investigating Committee was able to complete the pre-charge investigation only after one year because hundreds
of officers and thousands of enlisted men were involved in the failed coup.

Accordingly, in G.R. No. 93177, the petition is dismissed for lack of merit. In G.R. No. 96948, the petition is
granted, and the respondents are directed to allow the petitioners to exercise the right of peremptory challenge
under article 18 of the articles of war. In G.R. Nos. 95020 and 97454, the petitions are also granted, and the
orders of the respondent courts for the release of the private respondents are hereby reversed and set aside. No
costs.

M ANOTOC VS. COURT OF APPEALS


[142 SCRA 149; G.R. NO. L-62100; 30 MAY 1986]

Facts:

Petitioner was charged with estafa. He posted bail. Petitioner filed before each of the trial courts a motion entitled,
"motion for permission to leave the country," stating as ground therefor his desire to go to the United States,
"relative to his business transactions and opportunities." The prosecution opposed said motion and after due
hearing, both trial judges denied the same. Petitioner thus filed a petition for certiorari and mandamus before the
then Court of Appeals seeking to annul the orders dated March 9 and 26, 1982, of Judges Camilon and Pronove,
respectively, as well as the communication-request of the Securities and Exchange Commission, denying his
leave to travel abroad. He likewise prayed for the issuance of the appropriate writ commanding the Immigration
Commissioner and the Chief of the Aviation Security Command (AVSECOM) to clear him for departure. The
Court of Appeals denied the petition.
Petitioner contends that having been admitted to bail as a matter of right, neither the courts which granted him
bail nor the Securities and Exchange Commission which has no jurisdiction over his liberty could prevent him from
exercising his constitutional right to travel.

Issue:

Whether or Not the right to bail a matter of right.

Held:

The right to bail is a matter of right if the charge is not a capital offense or punishable by reclusion perpetua to
death.

A court has the power to prohibit a person admitted to bail from leaving the Philippines. This is a necessary
consequence of the nature and function of a bail bond.

The condition imposed upon petitioner to make himself available at all times whenever the court requires his
presence operates as a valid restriction on his right to travel.

Indeed, if the accused were allowed to leave the Philippines without sufficient reason, he may be placed beyond
the reach of the courts.

Page 135
Section 1-C, SY ’06-‘07
San Beda College of Law – Alabang
Constitutional Law 2 Case Digests

Petitioner has not shown the necessity for his travel abroad. There is no indication that the business transactions
cannot be undertaken by any other person in his behalf.

CALLANTA VS. VILLANUEVA


[77 SCRA 377; G.R. NOS. 24646 & L-24674; 20 JUN 1977]

Facts:

Two complaints for grave oral defamation were filed against Faustina Callanta. The City Judge of Dagupan City,
Felipe Villanueva, denied the motions to quash the complaints. Thus, petitioner Callanta brought the suits for
certiorari in the Supreme Court. Petitioner questions the validity of the issuance of warrant of arrest by
respondent, arguing that the City Fiscal should have conducted the preliminary investigation. According to
petitioner’s counsel, there was jurisdictional infirmity. After the issuance of the warrants of arrest and the bail fixed
at P600, petitioner posted the bail bond, thus obtaining her provisional liberty. The City Fiscal in this case did not
disagree with the judge’s investigation, and agreed with the complaints filed.

Issue:

Whether or Not petitioner’s contentions are to be given merit.

Held:

Based on many precedent cases of the Supreme Court, “where the accused has filed bail and waived the
preliminary investigation proper, he has waived whatever defect, if any, in the preliminary examination conducted
prior to the issuance of the warrant of arrest”. In the case at bar, it is futile for the petitioner to question the validity
of the issuance of the warrant of arrest, because she posted the bail bond. Petitioner also erred in arguing that
only the City Fiscal can conduct a preliminary investigation. According to the Charter of the City of Dagupan, “the
City Court of Dagupan City may also conduct preliminary investigation for any offense, without regard to the limits
of punishment, and may release, or commit and bind over any person charged with such offense to secure his
appearance before the proper court”. Petition for certiorari is denied. Restraining order issued by the Court is lifted
and set aside.

TATAD VS. SANDIGANBAYAN


[159 SCRA 70; G.R. NOS. L-72335-39; 21 MAR 1988]

Facts:

The complainant, Antonio de los Reyes, originally filed what he termed "a report" with the Legal Panel of the
Presidential Security Command (PSC) on October 1974, containing charges of alleged violations of Rep. Act No.
3019 against then Secretary of Public Information Francisco S. Tatad. The "report" was made to "sleep" in the
office of the PSC until the end of 1979 when it became widely known that Secretary (then Minister) Tatad had a
falling out with President Marcos and had resigned from the Cabinet. On December 12, 1979, the 1974 complaint
was resurrected in the form of a formal complaint filed with the Tanodbayan. The Tanodbayan acted on the
complaint on April 1, 1980 which was around two months after petitioner Tatad's resignation was accepted by
Pres. Marcos by referring the complaint to the CIS, Presidential Security Command, for investigation and report.
On June 16, 1980, the CIS report was submitted to the Tanodbayan, recommending the filing of charges for graft
and corrupt practices against former Minister Tatad and Antonio L. Cantero. By October 25, 1982, all affidavits
and counter-affidavits were in the case was already for disposition by the Tanodbayan. However, it was only on
June 5, 1985 that a resolution was approved by the Tanodbayan. Five criminal informations were filed with the
Sandiganbayan on June 12, 1985, all against petitioner Tatad alone. (1) Section 3, paragraph (e) of RA. 3019 for
giving D' Group, a private corporation controlled by his brother-in-law, unwarranted benefits, advantage or
preference in the discharge of his official functions; (2) Violation of Section 3, paragraph (b) for receiving a check
of P125,000.00 from Roberto Vallar, President/General Manager of Amity Trading Corporation as consideration
for the release of a check of P588,000.00 to said corporation for printing services rendered for the Constitutional
Convention Referendum in 1973; (3) Violation of Section 7 on three (3) counts for his failure to file his Statement
of Assets and Liabilities for the calendar years 1973, 1976 and 1978. A motion to quash the information was
made alleging that the prosecution deprived accused of due process of law and of the right to a speedy
disposition of the cases filed against him. It was denied hence the appeal.

Issue:

Whether or not petitioner was deprived of his rights as an accused.

Page 136
Section 1-C, SY ’06-‘07
San Beda College of Law – Alabang
Constitutional Law 2 Case Digests

Held:

YES. Due process (Procedural) and right to speedy disposition of trial were violated. Firstly, the complaint came
to life, as it were, only after petitioner Tatad had a falling out with President Marcos. Secondly, departing from
established procedures prescribed by law for preliminary investigation, which require the submission of affidavits
and counter-affidavits by the complainant and the respondent and their witnesses, the Tanodbayan referred the
complaint to the Presidential Security Command for finding investigation and report. The law (P.D. No. 911)
prescribes a ten-day period for the prosecutor to resolve a case under preliminary investigation by him from its
termination. While we agree with the respondent court that this period fixed by law is merely "directory," yet, on
the other hand, it can not be disregarded or ignored completely, with absolute impunity. A delay of close to three
(3) years can not be deemed reasonable or justifiable in the light of the circumstance obtaining in the case at bar.

GALMAN VS. SANDIGANBAYAN


[144 SCRA 43; G.R. NO.72670; 12 SEP 1986]

Facts:

Assassination of former Senator Benigno "Ninoy" Aquino, Jr. He was killed from his plane that had just landed at
the Manila International Airport. His brain was smashed by a bullet fired point-blank into the back of his head by
an assassin. The military investigators reported within a span of three hours that the man who shot Aquino
(whose identity was then supposed to be unknown and was revealed only days later as Rolando Galman) was a
communist-hired gunman, and that the military escorts gunned him down in turn.

President was constrained to create a Fact Finding Board to investigate due to large masses of people who joined
in the ten-day period of national mourning yearning for the truth, justice and freedom.

The fact is that both majority and minority reports were one in rejecting the military version stating that "the
evidence shows to the contrary that Rolando Galman had no subversive affiliations. Only the soldiers in the
staircase with Sen. Aquino could have shot him; that Ninoy's assassination was the product of a military
conspiracy, not a communist plot. Only difference between the two reports is that the majority report found all the
twenty-six private respondents above-named in the title of the case involved in the military conspiracy; " while the
chairman's minority report would exclude nineteen of them.

Then Pres. Marcos stated that evidence shows that Galman was the killer.

Petitioners pray for issuance of a TRO enjoining respondent court from rendering a decision in the two criminal
cases before it, the Court resolved by nine-to-two votes 11 to issue the restraining order prayed for. The Court
also granted petitioners a five-day period to file a reply to respondents' separate comments and respondent
Tanodbayan a three-day period to submit a copy of his 84-page memorandum for the prosecution.

But ten days later, the Court by the same nine-to-two-vote ratio in reverse, resolved to dismiss the petition and to
lift the TRO issued ten days earlier enjoining the Sandiganbayan from rendering its decision. The same Court
majority denied petitioners' motion for a new 5-day period counted from receipt of respondent Tanodbayan's
memorandum for the prosecution (which apparently was not served on them).

Thus, petitioners filed a motion for reconsideration, alleging that the dismissal did not indicate the legal ground for
such action and urging that the case be set for a full hearing on the merits that the people are entitled to due
process.

However, respondent Sandiganbayan issued its decision acquitting all the accused of the crime charged,
declaring them innocent and totally absolving them of any civil liability. Respondents submitted that with the
Sandiganbayan's verdict of acquittal, the instant case had become moot and academic. Thereafter, same Court
majority denied petitioners' motion for reconsideration for lack of merit.

Hence, petitioners filed their motion to admit their second motion for reconsideration alleging that respondents
committed serious irregularities constituting mistrial and resulting in miscarriage of justice and gross violation of
the constitutional rights of the petitioners and the sovereign people of the Philippines to due process of law.

Issue:

Whether or not petitioner was deprived of his rights as an accused.

Whether or not there was a violation of the double jeopardy clause.

Held:

Page 137
Section 1-C, SY ’06-‘07
San Beda College of Law – Alabang
Constitutional Law 2 Case Digests

Petitioners' second motion for reconsideration is granted and ordering a re-trial of the said cases which should be
conducted with deliberate dispatch and with careful regard for the requirements of due process.

Deputy Tanodbayan Manuel Herrera (made his expose 15 months later when former Pres. was no longer around)
affirmed the allegations in the second motion for reconsideration that he revealed that the Sandiganbayan
Justices and Tanodbayan prosecutors were ordered by Marcos to whitewash the Aquino-Galman murder case.
Malacañang wanted dismissal to the extent that a prepared resolution was sent to the Investigating Panel.
Malacañang Conference planned a scenario of trial where the former President ordered then that the resolution
be revised by categorizing the participation of each respondent; decided that the presiding justice, Justice
Pamaran, (First Division) would personally handle the trial. A conference was held in an inner room of the Palace.
Only the First Lady and Presidential Legal Assistant Justice Lazaro were with the President. The conferees were
told to take the back door in going to the room where the meeting was held, presumably to escape notice by the
visitors in the reception hall waiting to see the President. During the conference, and after an agreement was
reached, Pres. Marcos told them 'Okay, mag moro-moro na lamang kayo;' and that on their way out of the room
Pres. Marcos expressed his thanks to the group and uttered 'I know how to reciprocate'.

The Court then said that the then President (code-named Olympus) had stage-managed in and from Malacañang
Palace "a scripted and predetermined manner of handling and disposing of the Aquino-Galman murder case;"
and that "the prosecution in the Aquino-Galman case and the Justices who tried and decided the same acted
under the compulsion of some pressure which proved to be beyond their capacity to resist. Also predetermined
the final outcome of the case" of total absolution of the twenty-six respondents-accused of all criminal and civil
liability. Pres. Marcos came up with a public statement aired over television that Senator Aquino was killed not by
his military escorts, but by a communist hired gun. It was, therefore, not a source of wonder that President Marcos
would want the case disposed of in a manner consistent with his announced theory thereof which, at the same
time, would clear his name and his administration of any suspected guilty participation in the assassination. such
a procedure would be a better arrangement because, if the accused are charged in court and subsequently
acquitted, they may claim the benefit of the doctrine of double jeopardy and thereby avoid another prosecution if
some other witnesses shall appear when President Marcos is no longer in office.

More so was there suppression of vital evidence and harassment of witnesses. The disappearance of witnesses
two weeks after Ninoy's assassination. According to J. Herrera, "nobody was looking for these persons because
they said Marcos was in power. The assignment of the case to Presiding Justice Pamaran; no evidence at all that
the assignment was indeed by virtue of a regular raffle, except the uncorroborated testimony of Justice Pamaran
himself. The custody of the accused and their confinement in a military camp, instead of in a civilian jail. The
monitoring of proceedings and developments from Malacañang and by Malacañang personnel. The partiality of
Sandiganbayan betrayed by its decision: That President Marcos had wanted all of the twenty-six accused to be
acquitted may not be denied. In rendering its decision, the Sandiganbayan overdid itself in favoring the
presidential directive. Its bias and partiality in favor of the accused was clearly obvious. The evidence presented
by the prosecution was totally ignored and disregarded.

The record shows that the then President misused the overwhelming resources of the government and his
authoritarian powers to corrupt and make a mockery of the judicial process in the Aquino-Galman murder cases.
"This is the evil of one-man rule at its very worst." Our Penal Code penalizes "any executive officer who shall
address any order or suggestion to any judicial authority with respect to any case or business coming within the
exclusive jurisdiction of the courts of justice."

Impartial court is the very essence of due process of law. This criminal collusion as to the handling and treatment
of the cases by public respondents at the secret Malacañang conference (and revealed only after fifteen months
by Justice Manuel Herrera) completely disqualified respondent Sandiganbayan and voided ab initio its verdict.
The courts would have no reason to exist if they were allowed to be used as mere tools of injustice, deception and
duplicity to subvert and suppress the truth. More so, in the case at bar where the people and the world are entitled
to know the truth, and the integrity of our judicial system is at stake.

There was no double jeopardy. Courts' Resolution of acquittal was a void judgment for having been issued
without jurisdiction. No double jeopardy attaches, therefore. A void judgment is, in legal effect, no judgment at all.
By it no rights are divested. It neither binds nor bars anyone. All acts and all claims flowing out of it are void.

Motion to Disqualify/Inhibit should have been resolved ahead. In this case, petitioners' motion for reconsideration
of the abrupt dismissal of their petition and lifting of the TRO enjoining the Sandiganbayan from rendering its
decision had been taken cognizance of by the Court which had required the respondents', including the
Sandiganbayan's, comments. Although no restraining order was issued anew, respondent Sandiganbayan should
not have precipitately issued its decision of total absolution of all the accused pending the final action of this
Court. All of the acts of the respondent judge manifest grave abuse of discretion on his part amounting to lack of
jurisdiction which substantively prejudiced the petitioner.

Page 138
Section 1-C, SY ’06-‘07
San Beda College of Law – Alabang
Constitutional Law 2 Case Digests

With the declaration of nullity of the proceedings, the cases must now be tried before an impartial court with an
unbiased prosecutor. Respondents accused must now face trial for the crimes charged against them before an
impartial court with an unbiased prosecutor with all due process.

The function of the appointing authority with the mandate of the people, under our system of government, is to fill
the public posts. Justices and judges must ever realize that they have no constituency, serve no majority nor
minority but serve only the public interest as they see it in accordance with their oath of office, guided only the
Constitution and their own conscience and honor.

PEOPLE VS. DRAMAYO


[42 SCRA 60; G.R. L-21325; 29 OCT 1971]

Facts:

Dramayo brought up the idea of killing Estelito Nogaliza so that he could not testify in the robbery case where he
is an accused. The idea was for Dramayo and Ecubin to ambush Estelito, who was returning from Sapao. The
others were to station themselves nearby. Only Dramayo and Ecubin were convicted in the RTC for murder.
Hence the appeal

Issue:

Whether or not the accused’s criminal liability proved beyond reasonable doubt.

Held:
Yes. It is to be admitted that the starting point is the Presumption of innocence. So it must be, according to the
Constitution. That is a right safeguarded both appellants. Accusation is not, according to the fundamental law,
synonymous with guilt. It is incumbent on the prosecution demonstrate that culpability lies. Appellants were not
even called upon then to offer evidence on their behalf. Their freedom is forfeit only if the requisite quantum of
proof necessary for conviction be in existence. Their guilt be shown beyond reasonable doubt. What is required
then is moral certainty. "By reasonable doubt is meant that which of possibility may arise, but it is doubt
engendered by an investigation of the whole proof and an inability, after such investigation, to let the mind rest
easy upon the certainty of guilt. Absolute certain of guilt is not demanded by the law to convict of any carnal
charge but moral certainty is required, and this certainty is required as to every proposition of proof regular to
constitute the offense."

The judgment of conviction should not have occasioned any surprise on the part of the two appellants, as from
the evidence deserving of the fullest credence, their guilt had been more than amply demonstrated. The
presumption of innocence could not come to their rescue as it was more than sufficiently overcome by the proof
that was offered by the prosecution. The principal contention raised is thus clearly untenable. It must be stated
likewise that while squarely advanced for the first time, there had been cases where this Court, notwithstanding a
majority of the defendants being acquitted, the element of conspiracy likewise being allegedly present, did hold
the party or parties, responsible for the offense guilty of the crime charged, a moral certainty having arisen as to
their capability.

DUMLAO VS. COMELEC


[95 SCRA 392; L-52245; 22 JAN 1980]

Facts:

Petitioner Dumlao questions the constitutionality of Sec. 4 of Batas Pambansa Blg 52 as discriminatory and
contrary to equal protection and due process guarantees of the Constitution. Sec. 4 provides that any retired
elective provicial or municipal official who has received payments of retirement benefits and shall have been 65
years of age at the commencement of the term of office to which he seeks to be elected, shall not be qualified to
run for the same elective local office from which he has retired. According to Dumlao, the provision amounts to
class legislation. Petitioners Igot and Salapantan Jr. also assail the validity of Sec. 4 of Batas Pambansa Blg 52,
which states that any person who has committed any act of disloyalty to the State, including those amounting to
subversion, insurrection, rebellion, or other similar crimes, shall not be qualified for any of the offices covered by
the act, or to participate in any partisan activity therein: provided that a judgment of conviction of those crimes
shall be conclusive evidence of such fact and the filing of charges for the commission of such crimes before a civil
court or military tribunal after preliminary investigation shall be prima facie evidence of such fact.

Issue:

Whether or not the aforementioned statutory provisions violate the Constitution and thus, should be declared null
and void

Page 139
Section 1-C, SY ’06-‘07
San Beda College of Law – Alabang
Constitutional Law 2 Case Digests

Held:

In regards to the unconstitutionality of the provisions, Sec. 4 of BP Blg 52 remains constitutional and valid. The
constitutional guarantee of equal protection of the laws is subject to rational classification. One class can be
treated differently from another class. In this case, employees 65 years of age are classified differently from
younger employees. The purpose of the provision is to satisfy the “need for new blood” in the workplace. In
regards to the second paragraph of Sec. 4, it should be declared null and void for being violative of the
constitutional presumption of innocence guaranteed to an accused. “Explicit is the constitutional provision that, in
all criminal prosecutions, the accused shall be presumed innocent until the contrary is proved, and shall enjoy the
right to be heard by himself and counsel (Article IV, section 19, 1973 Constitution). An accusation, according to
the fundamental law, is not synonymous with guilt. The challenged proviso contravenes the constitutional
presumption of innocence, as a candidate is disqualified from running for public office on the ground alone that
charges have been filed against him before a civil or military tribunal. It condemns before one is fully heard. In
ultimate effect, except as to the degree of proof, no distinction is made between a person convicted of acts of
dislotalty and one against whom charges have been filed for such acts, as both of them would be ineligible to run
for public office. A person disqualified to run for public office on the ground that charges have been filed against
him is virtually placed in the same category as a person already convicted of a crime with the penalty of arresto,
which carries with it the accessory penalty of suspension of the right to hold office during the term of the sentence
(Art. 44, Revised Penal Code).”

And although the filing of charges is considered as but prima facie evidence, and therefore, may be rebutted, yet.
there is "clear and present danger" that because of the proximity of the elections, time constraints will prevent one
charged with acts of disloyalty from offering contrary proof to overcome the prima facie evidence against him.
Additionally, it is best that evidence pro and con of acts of disloyalty be aired before the Courts rather than before
an administrative body such as the COMELEC. A highly possible conflict of findings between two government
bodies, to the extreme detriment of a person charged, will thereby be avoided. Furthermore, a
legislative/administrative determination of guilt should not be allowed to be substituted for a judicial determination.
Being infected with constitutional infirmity, a partial declaration of nullity of only that objectionable portion is
mandated. It is separable from the first portion of the second paragraph of section 4 of Batas Pambansa Big. 52
which can stand by itself.

Wherefore, the first paragraph of section 4 of Batas pambansa Bilang 52 is hereby declared valid and that portion
of the second paragraph of section 4 of Batas Pambansa Bilang 52 is hereby declared null and void, for being
violative of the constitutional presumption of innocence guaranteed to an accused.

PEOPLE VS. ALCANTARA


[240 SCRA 122; G.R. NO. 91283; 17 JAN 1995]

Facts:

On July 19, 1988, Venancio Patricio, accompanied by Larry Salvador, drove a ten-wheeler truck a Coca-Cola
plant in Antipolo to load cases of softdrinks. They were about to leave the plant at 10:00pm when several men
approached them to hitch for a ride. Ascertaining that Salvador knew appellant, Venancio accommodated
appellant's request. Appellant had four companions. At Ortigas Ave., one of them poked a gun at Venancio and
grabbed the steering wheel. At the North Diversion Road, Venancio and Salvador(helper) were brought down
from the vehicle and tied to the fence of the expressway, thereafter they were stabbed and left bleeding to death.
Venancio survived but Salvador did not.Appellant was arrested in the vicinity of Otis Street in Pandacan, Manila.
A few days later, he was turned over to the Constabulary Highway Patrol Group. Sgt. Alberto Awanan brought the
appellant to the MCU hospital and was presented to Venancio for identification. Appellant was brought to the
Headquarters at Camp Crame where he confessed.

Appelant’s Defense: Denial and alibi. He said that he was just applying to be a driver and stayed there even if he
was told that no work was available, to confirm with the truck drivers. While he was applying for CONCEPCION
TRUCKING located across Otis street from the Coca-cola plant. He was arrested. He denied any knowledge of
the "hit" on the Coca-cola delivery truck. He remained in the custody of the police for two days and two nights. On
the third day of his detention, he was turned over to the Constabulary Highway Patrol Group. Appellant was the
brought to the MCU hospital. He was made to confront Venancio whom he saw for the first time. CHPG Sgt.
Awanan asked Venancio twice if appellant was among those who hijacked the truck he was driving. On both
times, Venancio did not respond. Undaunted, Sgt. Awanan, called to a photographer present, forced appellant to
stand about a foot from Venancio, and told the latter to just point at the suspect. "Basta ituro mo lang," Sgt.
Awanan directed. Venancio obeyed, and pictures of him pointing to the suspect were taken. From the hospital,
appellant was brought to the Constabulary Highway Patrol Group headquarters at Camp Crame. Without being
apprised of his rights nor provided with counsel, he was interrogated and urged to confess his guilt. He balked. At
ten o'clock that night, hours after questioning began, appellant's interrogators started boxing him and kicking him.
He was also hit on the back with a chair, and electrocuted. Still, he refused to admit to the crime. In the midst of

Page 140
Section 1-C, SY ’06-‘07
San Beda College of Law – Alabang
Constitutional Law 2 Case Digests

his ordeal, appellant heard someone say, "Tubigan na iyan." He was then blindfolded and brought to another
room where he was made to lie down. Water was slowly and continuously poured on his face, over his mouth.
Appellant could no longer bear the pain caused by the water treatment. Finally, he confessed to being one of the
hijackers. He was led to another room, where he was handcuffed and left until the following day. Later, he was
made to sign prepared statements containing his full confession.

Alcantara was arraigned under an information charging him and four others (at large) with the crime of robbery
with Homicide and Frustrated Homicide.

The trial court convicted the accused despite the following inconsistency between Venancio’s affidavit and
testimony:

Affidavit Testimony
1. mentioned 5 assailants - only Alcantara was identified
2. stabbing was preceded by a - only 3 assailants had a
3. conference by all assailants - conference
4. claimed to have allowed assailants - failed to identify Alcantara
5. to hitch a ride because Alcantara - at the hospital and in open
6. was familiar to them - court (pointed to another person)

Issue:

Whether or not the rights of the accused was violated.

Held:

YES. The people’s evidence failed to meet the quantum required to overcome the presumption. The second
identification which correctly pointed to accused by Venancio should not be credited. There is no reason for him to
err as they know each other for 3 years. It was also incorrect to give too much weight to Police Sgt. Awanan’s
testimony as to the “previous identification” at the hospital. The testimony of Sgt. Awanan was not corroborated by
Venancio.

The identification procedure was irregular. Due process demands that the identification procedure of criminal
suspects must be free from impermissible suggestions as the influence of improper suggestion probably accounts
for more miscarriages of justice than any other single factor. Conviction must be based on the strength of the
prosecution and not the weakness of the defense. There was blatant violation of the constitutional rights of
appellant as an accused. Appellant belongs to the economically deprived in our society. He is nearly illiterate(third
grade education). Our Constitution and our laws strictly ordain their protection following the Magsaysay
desideratum that those who have less in life should have more in law.

CORPUZ VS. REPUBLIC


[194 SCRA 73; G.R. NO. 74259; 14 FEB 1991]

Facts:

Generoso Corpuz is the Supervising Accounting Clerk in the Office of the Provincial Treasurer of Nueva Viscaya.
He was designated Acting Supervising Cashier in the said office. In this capacity, he received collections,
disbursed funds and made bank deposits and withdrawals pertaining to government accounts. On April 13, 1981
his designation as Acting Supervising Cashier was terminated and a transfer of accountabilities was effected
between him and his successor. The Certificate of turnover revealed a shortage of P72,823.00. He was able to
pay only P10,159.50. After a final demand letter for the total of P50,596.07 which was not met, a case of
malversation was filed against him. Corpuz did not deny such facts but he insists that the shortage was
malversed by other persons. He alleged that Paymaster Diosdado Pineda through 1 of 4 separate checks (PNB)
issued and encashed such checks while he was of leave. Also, Acting Deputy Provincial Treasurer Bernardo
Aluning made to post the amount on his cashbook although he had not received the said amount. He was
convicted in Sandiganbayan.

Issue:

Whether or Not Corpuz is guilty of malversation.

Held:

Page 141
Section 1-C, SY ’06-‘07
San Beda College of Law – Alabang
Constitutional Law 2 Case Digests

It is a subtle way of camouflaging the embezzlement of the money equivalent when 1 of the 4 checks issued and
encashed in the same day was entered in the accused’s cash book 3 months after such encashments. Also,
Corpuz claim that he was absent when Paymaster Diosdado Pineda through 1 of 4 separate checks (PNB) issued
and encashed such checks, was not proven.

Post-Audit is not a preliminary requirement to filing a malversation case. The failure of the public officer to have
duly forthcoming any public funds with which he is chargeable, upon demand by an authorized officer shall be a
prima facie evidence that he has put such missing funds to personal use.

The equipoise rule(balancing test) which is the presumption of innocence is applicable only where the evidence of
the parties is evenly balance, in which case the scale of justice should be tilt in favor of the accused. There is no
such balance in the case at bar. The evidence of the prosecution is overwhelming and has not been overcome by
the petitioner with his claims. The presumed innocence must yield to the positive finding that he is guilty of
malversation.

Wherefore his petition is denied. He is guilty as principal of Malversation of Public Funds.

PEOPLE VS. HOLGADO


[85 PHIL 752; G.R.L-2809; 22 MAR 1950]

Facts:

Appellant Frisco Holgado was charged in the court of First Instance of Romblon with slight illegal detention
because according to the information, being a private person, he did "feloniously and without justifiable motive,
kidnap and detain one Artemia Fabreag in the house of Antero Holgado for about eight hours thereby depriving
said Artemia Fabreag of her personal liberty. He pleaded guilty (without a counsel) and said that he was just
instructed by Mr. Ocampo, which no evidence was presented to indict the latter.

Issue:

Whether or Not there was any irregularity in the proceedings in the trial court.

Held:

Yes. Rule 112, section 3 of ROC that : “If the defendant appears without attorney, he must be informed by the
court that it is his right to have attorney being arraigned., and must be asked if he desires the aid of attorney, the
Court must assign attorney de oficio to defend him. A reasonable time must be allowed for procuring attorney.”
This was violated. Moreso the guarantees of our Constitution that "no person shall be held to answer for a
criminal offense without due process of law", and that all accused "shall enjoy the right to be heard by himself and
counsel." In criminal cases there can be no fair hearing unless the accused be given the opportunity to be heard
by counsel.

The trial court failed to inquire as to the true import of the qualified plea of accused. The record does not show
whether the supposed instructions of Mr. Ocampo was real and whether it had reference to the commission of the
offense or to the making of the plea guilty. No investigation was opened by the court on this matter in the
presence of the accused and there is now no way of determining whether the supposed instruction is a good
defense or may vitiate the voluntariness of the confession. Apparently the court became satisfied with the fiscal's
information that he had investigated Mr. Ocampo and found that the same had nothing to do with this case. Such
attitude of the court was wrong for the simple reason that a mere statement of the fiscal was not sufficient to
overcome a qualified plea of the accused. But above all, the court should have seen to it that the accused be
assisted by counsel especially because of the qualified plea given by him and the seriousness of the offense
found to be capital by the court.

PEOPLE VS. MAGSI


[124 SCRA 64; G.R. NO.L-32888; 12 AUG 1983]

Facts:

Soon after appellant was apprehended on August 20, 1970, his arraignment was scheduled before the Criminal
Circuit Court of San Fernando, La Union. The case was actually set and rescheduled for six (6) times, first of
which was on August 1, 1970. On that date, despite appointment by the court of Atty. Mario Rivera as de officio
counsel for the accused, hearing was re-set to September 8, 1970 on motion of Atty. Rivera, who was prompted
to ask for it because of accused desire to be represented by a de parte counsel. Prior to the next hearing, Atty.
Rivera moved to withdraw as de officio counsel and it was favorably acted on by the court on September 7, 1970.
At the second hearing on September 8, 1970, for failure of the de officio and de parte counsels to appear, despite

Page 142
Section 1-C, SY ’06-‘07
San Beda College of Law – Alabang
Constitutional Law 2 Case Digests

a second call of the case, the hearing was re-set for the next day and the court appointed Atty. Dominador
Cariaso de officio counsel for the accused. On the third hearing date, neither the de parte nor the de officio
counsel was in Court, so Atty. Rivera was reappointed that day as de officio counsel for arraignment purposes
only. The accused del Rosario entered a plea of guilty but qualified it with the allegation that he committed the
crime out of fear of his co-accused Eloy Magsi and the other coaccused. Appellant was found guilty of murder and
made to suffer the death penalty.

Issue:

Whether or not there was a violation of the rights of the accused.

Held:

YES. The desire to speed up the disposition of cases should not be effected at the sacrifice of the basic rights of
the accused. Citing People vs. Domingo (55 SCRA 243-244): the trial courts should exercise solicitous care
before sentencing the accused on a plea of guilty especially in capital offenses by first insuring that the accused
fully understands the gravity of the offense, the severity of the consequences attached thereto as well as the
meaning and significance of his plea of guilty; and that the prudent and proper thing to do in capital cases is to
take testimony, to assure the court that the accused has not misunderstood the nature and effect of his plea of
guilty. Mere pro-forma appointment of de officio counsel, who fails to genuinely protect the interests of the
accused, resetting of hearing by the court for alleged reception of evidence when in fact none was conducted,
perfunctory queries addressed to the accused whether he understands the charges and the gravity of the penalty,
are not sufficient compliance.

SORIANO VS. SANDIGANBAYAN


[131 SCRA 184; G.R. NO.L-65952; 31 JUL 1984]

Facts:

Tan was accused of qualified theft. The petitioner, who was an Asst. Fiscal, was assigned to investigate. In the
course of the investigation, petitioner demanded Php.4000 from Tan as price for dismissing the case. Tan
reported it to the NBI which set up an entrapment. Tan was given a Php.2000, marked bill, and he had supplied
the other half. The entrapment succeeded and an information was filed with the Sandiganbayan. After trial, the
Sandiganbayan rendered a decision finding the petitioner guilty as a principal in violating the Anti Graft and
Corrupt Practices Act (R.A.3019). A motion for reconsideration was denied by the Sandiganbayan, hence this
instant petition.

Issue:

Whether or Not the investigation conducted by the petitioner can be regarded as contract or transaction within the
purview of .RA.3019.

Held:

R.A. 3019 Sec.3. Corrupt practices of public officers - In addition to acts or omissions of public officers already
penalized by existing laws, the following shall constitute corrupt practices of any public officer and are hereby
declared to be unlawful: xxx b. Directly or indirectly requesting or receiving any gift, present, share percentage or
benefit, for himself or for other person, in connection with any contract or transaction between the Govt. and any
other party wherein the public officer in his official capacity has to intervene under the law.

The petitioner stated that the facts make out a case of direct bribery under Art.210 of the RPC and not a violation
of R.A. 3019 sec.3 (b). The offense of direct bribery is not the offense charged and is not included in the offense
charged which is violation of R.A.3019 sec.3 (b).

The respondent claimed that, transaction as used hereof, is not limited to commercial or business transaction, but
includes all kinds of transaction whether commercial, civil, or administrative in nature.

The court agrees with the petitioner. It is obvious that the investigation conducted by the petitioner was neither a
contract nor transaction. A transaction like a contract is one which involves some consideration as in credit
transactions. And this element is absent in the investigation conducted by the petitioner.

Judgment modified. Petitioner is guilty of direct bribery under Art.210 of the RPC.

Page 143
Section 1-C, SY ’06-‘07
San Beda College of Law – Alabang
Constitutional Law 2 Case Digests

BORJA VS. MENDOZA


[77 SCRA 422; G.R. NO.L-45667; 20 JUN 1977]

Facts:

Borja was accused of slight physical injuries in the City of Cebu. However, he was not arraigned. That not
withstanding, respondent Judge Senining proceeded with the trial in absentia and rendered a decision finding
petitioner guilty of the crime charged. The case was appealed to the Court o First Instance in Cebu presided by
respondent Judge Mendoza. It was alleged that the failure to arraign him is a violation of his constitutional rights.
It was also alleged that without any notice to petitioner and without requiring him to submit his memorandum, a
decision on the appealed case was rendered The Solicitor General commented that the decision should be
annulled because there was no arraignment.

Issue:

Whether or Not petitioner’s constitutional right was violated when he was not arraigned.

Held:

Yes. Procedural due process requires that the accused be arraigned so that he may be informed as to why he
was indicted and what penal offense he has to face, to be convicted only on a showing that his guilt is shown
beyond reasonable doubt with full opportunity to disprove the evidence against him. It is also not just due process
that requires an arraignment. It is required in the Rules that an accused, for the first time, is granted the
opportunity to know the precise charge that confronts him. It is imperative that he is thus made fully aware of
possible loss of freedom, even of his life, depending on the nature of the crime imputed to him. At the very least
then, he must be fully informed of why the prosecuting arm of the state is mobilized against him. Being arraigned
is thus a vital aspect of the constitutional rights guaranteed him. Also, respondent Judge Senining convicted
petitioner notwithstanding the absence of an arraignment. With the violation of the constitutional right to be heard
by himself and counsel being thus manifest, it is correct that the Solicitor General agreed with petitioner that the
sentence imposed on him should be set aside for being null. The absence of an arraignment can be invoked at
anytime in view of the requirements of due process to ensure a fair and impartial trial.

Wherefore, the petition for certiorari is granted. The decision of respondent Judge Romulo R. Senining dated
December 28, 1973, finding the accused guilty of the crime of slight physical injuries, is nullified and set aside.
Likewise, the decision of respondent Judge Rafael T. Mendoza dated November 16, 1976, affirming the aforesaid
decision of Judge Senining, is nullified and set aside. The case is remanded to the City Court of Cebu for the
prosecution of the offense of slight physical injuries, with due respect and observance of the provisions of the
Rules of Court, starting with the arraignment of petitioner.

CONDE VS. RIVERA


[45 PHIL 650; G.R. NO. 21741; 25 JAN 1924]

Facts:

Aurelia Conde, formerly a municipal midwife in Lucena, Tayabas, has been forced to respond to no less the five
information for various crimes and misdemeanors, has appeared with her witnesses and counsel at hearings no
less than on eight different occasions only to see the cause postponed, has twice been required to come to the
Supreme Court for protection, and now, after the passage of more than one year from the time when the first
information was filed, seems as far away from a definite resolution of her troubles as she was when originally
charged.

Issue:

Whether or Not petitioner has been denied her right to a speedy and impartial trial.

Held:

Philippine organic and statutory law expressly guarantee that in all criminal prosecutions the accused shall enjoy
the right to have a speedy trial. Aurelia Conde, like all other accused persons, has a right to a speedy trial in order
that if innocent she may go free, and she has been deprived of that right in defiance of law. We lay down the legal
proposition that, where a prosecuting officer, without good cause, secures postponements of the trial of a
defendant against his protest beyond a reasonable period of time, as in this instance for more than a year, the

Page 144
Section 1-C, SY ’06-‘07
San Beda College of Law – Alabang
Constitutional Law 2 Case Digests

accused is entitled to relief by a proceeding in mandamus to compel a dismissal of the information, or if he be


restrained of his liberty, by habeas corpus to obtain his freedom.

PEOPLE VS. TAMPAL


[244 SCRA 202; G.R. NO. 102485; 22 MAY 1995]

Facts:

Luis Tampal, Domingo Padumon, Arsenio Padumon, Samuel Padumon, Pablito Suco, Dario Suco and Galvino
Cadling were charged of robbery with homicide and multiple serious physical injuries in the Regional Trial Court of
Zamboanga with Hon. Wilfredo Ochotorena as presiding judge. However, only private respondents, Luis Tampal,
Domingo Padumon, Arsenio Padumon, and Samuel Padumon were arrested, while the others remained at large.

The case was set for hearing on July 26, 1991, but Assistant Provincial Prosecutor Wilfredo Guantero moved for
postponement due to his failure to contact the material witnesses. The case was reset without any objection from
the defense counsel. The case was called on September 20, 1991 but the prosecutor was not present. The
respondent judge considered the absence of the prosecutor as unjustified, and dismissed the criminal case for
failure to prosecute. The prosecution filed a motion for reconsidereation, claiming that his absence was because
such date was a Muslim holiday and the office of the Provincial prosecutor was closed on that day. The motion
was denied by respondent judge.

Issue:

Whether or Not the postponement is a violation of the right of the accused to a speedy disposition of their cases.

Whether or Not the dismissal serves as a bar to reinstatement of the case.

Held:

In determining the right of an accused to speedy disposition of their case, courts should do more than a
mathematical computation of the number of postponements of the scheduled hearings of the case. What are
violative of the right of the accused to speedy trial are unjustified postponements which prolong trial for an
unreasonable length of time. In the facts above, there was no showing that there was an unjust delay caused by
the prosecution, hence, the respondent judge should have given the prosecution a fair opportunity to prosecute its
case.

The private respondents cannot invoke their right against double jeopardy. In several cases it was held that
dismissal on the grounds of failure to prosecute is equivalent to an acquittal that would bar another prosecution
for the same offense, but in this case, this does not apply, considering that the rights of the accused to a speedy
trial was not violated by the State. Therefore, the order of dismissal is annulled and the case is remanded to the
court of origin for further proceedings.

REPUBLIC ACT NO. 8493 “THE SPEEDY TRIAL ACT”

The arraignment of an accused shall be held within 30 days from filing of the information, or from the date the
accused has appeared before the justice, judge or court in which the charge is pending, whichever date last
occurs. Thereafter, where a plea of not guilty is entered, the accused shall have at least 15 days to prepare for
trial. Trial shall commence within 30 days from arraignment as fixed by the court. In no case shall the entire trial
period exceed 180 days from the 1st day of trial, except as otherwise authorized by the Chief Justice of the
Supreme Court.

RE: REQUEST FOR LIVE TV OF TRIAL OF JOSEPH ESTRADA


[360 SCRA 248; A.M. NO 01-4-03-SC; 29 JUN 2001]

Facts:

The Kapisanan ng mga Brodkaster ng Pilipinas (KBP) sent a letter requesting the Court to allow live media
coverage of the anticipated trial of the plunder and other criminal cases filed against former President Joseph E.
Estrada before the Sandiganbayan in order "to assure the public of full transparency in the proceedings of an
unprecedented case in our history." The request was seconded by Mr. Cesar N. Sarino and, still later, by Senator
Renato Cayetano and Attorney Ricardo Romulo.

Issue:

Page 145
Section 1-C, SY ’06-‘07
San Beda College of Law – Alabang
Constitutional Law 2 Case Digests

Whether or Not live media coverage of the trial of the plunder and other criminal cases filed against former
President Joseph E. Estrada should be permitted by the court.

Held:

The propriety of granting or denying the instant petition involve the weighing out of the constitutional guarantees
of freedom of the press and the right to public information, on the other hand, along with the constitutional power
of a court to control its proceedings in ensuring a fair and impartial trial. When these rights race against one
another, jurisprudence tells us that the right of the accused must be preferred to win.

Due process guarantees the accused a presumption of innocence until the contrary is proved in a trial that is not
lifted about its individual settings nor made an object of public’s attention and where the conclusions reached are
induced not by any outside force or influence but only be evidence and argument given in open court, where
fitting dignity and calm ambiance is demanded.

An accused has a right to a public trial but it is a right that belongs to him, more than anyone else, where his life
or liberty can be held critically in balance. A public trial aims to ensure that he is fairly dealt with and would not be
unjustly condemned and that his rights are not compromised in secret conclaves of long ago. A public trial is not
synonymous with publicized trial, it only implies that the court doors must be open to those who wish to come, sit
in the available seats, conduct themselves with decorum and observe the trial process.

The courts recognize the constitutionally embodied freedom of the press and the right to public information. It also
approves of media's exalted power to provide the most accurate and comprehensive means of conveying the
proceedings to the public. Nevertheless, within the courthouse, the overriding consideration is still the paramount
right of the accused to due process which must never be allowed to suffer diminution in its constitutional
proportions.

PEOPLE VS. SALAS


[143 SCRA 163; G.R. NO. L-66469; 29 JUL 1986]

Facts:

At about 6:00 o'clock in the morning of March 6, 1992, a 60 year old woman, identified as Virginia Talens was
found lying dead in a canal at Bo. San Nicolas, Mexico, Pampanga; she was last seen alive at about 3:00 o'clock
early morning of March 6, 1992 by Orlando Pangan and Richard Pangan who were with her going home coming
from the wake of one Leonardo Flores; both Orlando and Richard Pangan testified that accused was with them in
going home at about 3:00 o'clock in the morning of March 6, 1992; Orlando and Richard Pangan reached first
their house and left the two on the way and that was the last time Virginia was seen alive; just a few minutes after
reaching his house and while inside his house, Orlando Pangan heard a shout; another woman, one Serafia
Gutierrez, testified that she likewise was awakened by a shout at about 3:00 in the morning; Dr. Aguda who
autopsied the victim found hematoma on the head and chest, an abrasion on the left chin and stabwound on the
neck which stabwound, the doctor claims, was the cause of death of the victim; Police Investigator Gonzales who
immediately responded upon report, recovered at the scene a pin, the victim's wristwatch, earring, a ring and
P135.00 money; he likewise found on March 9, 1992 when he continued his investigation bloodstain on the front
door of the house of the accused which bloodstain when submitted for examination was found to be of human
blood; one Resultay was with Virginia Talens at about 5:00 afternoon of March 5, 1992 in going to the wake, who
claims that Virginia had money on a purse as while they were on the way Virginia bet on a jueteng she saw
Virginia got money from her purse a P500.00 bill but as she had no change she instead took P8.00 from her other
pocket; one Ramil Talens, a son of the victim corroborated the claim of Resultay that Virginia had with her at that
time money worth P2,000.00 as in the morning of March 5, 1992 he gave her mother for safekeeping the sum of
P1,500.00 which he claims his mother placed in her purse and claims further that at the wake, he asked and was
given P50.00 by his mother as he also participated in the gambling thereat, however, the purse of Virginia
containing about P2,000.00 was no longer to be found when she was found dead; Orlando Pangan saw the
accused gambled in the wake; Virginia likewise gambled at the wake; accused had been working for three days
before March 6 at Sta. Ana, Pampanga and up to March 5, 1992, but the following day, he did not anymore report
for work at Sta. Ana, Pampanga, was no longer to be found and was last seen at about 3:00 morning together
with Virginia Talens on their way home coming from the wake; the parents of [the] accused were informed by
Investigator Gonzales that their son was the suspect and adviced them to surrender him, but since March 6, 1992
when accused left Mexico, Pampanga, he returned only on September 19, 1992 at Arayat, Pampanga, not at
Mexico, Pampanga where he was ultimately apprehended by the Mexico Police on September 22, 1992 after
chancing on a radio message by the police of Arayat to their Provincial commander that a vehicular incident
occurred at Arayat, Pampanga where one Elmer Salas was the victim and was hospitalized at the district hospital
at Arayat, Pampanga where he used the name of Rommel Salas and not Elmer Salas. The trial court rendered
convicting Salas for Robbery with Homicide

Issue:

Page 146
Section 1-C, SY ’06-‘07
San Beda College of Law – Alabang
Constitutional Law 2 Case Digests

Whether or Not there is evidence sufficient to sustain a conviction of the appellant of the crime of Robbery with
Homicide.

Whether or Not the appellant’s crime homicide or robbery with homicide.

Held:

There was no eyewitness or direct evidence; either to the robbery or to the homicide and none of the things
allegedly stolen were ever recovered. However, direct evidence is not the only matrix from which the trial court
may draw its findings and conclusion of culpability. Resort to circumstantial evidence is essential when to insist on
direct testimony would result in setting felons free.

For circumstantial evidence to be sufficient to support a conviction, all the circumstances must be consistent with
each other, consistent with the theory that the accused is guilty of the offense charged, and at the same time
inconsistent with the hypothesis that he is innocent and with every other possible, rational hypothesis excepting
that of guilt. All the circumstances established must constitute an unbroken chain which leads to one and fair and
reasonable conclusion pointing solely to the accused, to the exclusion of all other persons, as the author of the
crime. The facts and circumstances consistent with the guilt of the accused and inconsistent with his innocence
can constitute evidence which, in weight and probative value, may be deemed to surpass even direct evidence in
its effect on the court.

The fatal stabbing of Virginia Talens occurred at around 3:00 a.m. of March 6, 1992. Appellant hastily abandoned
his house in Barrio San Nicolas, Mexico, Pampanga, his residence since childhood, on that very date. Appellant
was nowhere when his co-worker and barrio mate, Eduardo Bagtas, came to appellant's house to fetch him for
work at around 6:30 to 7:00 a.m. of March 6, 1992. Appellant also abandoned his job as a painter in Sta. Ana,
Pampanga, on March 6, 1992, the date of the crime, leaving behind an unfinished painting project. He was not
seen again from said date. Police investigators found human bloodstains on the front door of appellant's house,
on his clothing, and on his yellow slippers after the victim was killed. Despite efforts of the police to find appellant
as the principal suspect, a fact known to appellant's family and neighbors, appellant did not present himself to the
authorities. Appellant was apprehended only a full six months after the date of the crime, following his
confinement in a hospital in Arayat, Pampanga because he was sideswiped by a Victory Liner bus in Arayat.
When hospitalized, appellant used the alias Rommel Salas, instead of his true name Elmer Salas. These
circumstances denote flight, which when unexplained, has always been considered by the courts as indicative of
guilt.

Both appellant and victim gambled at the wake they attended. The victim was, in fact, enjoying a winning streak
when her son, Ramil Talens, came to fetch her but which he failed to do because his mother was winning, and
she refused to leave. The purse of Talens containing cash was gone when her corpse was found in the canal with
a stab wound and bruises. What was left was a safety pin which victim used to fasten the missing purse to her
clothes.

Denial is an inherently weak defense which must be buttressed by strong evidence of non-culpability to merit
credibility. Denial is negative and self-serving and cannot be given greater evidentiary weight over the testimonies
of credible witnesses who positively testified that appellant was at the locus criminis and was the last person seen
with the victim alive.

The absence of evidence showing any improper motive on the part of the principal witness for the prosecution to
falsely testify against the appellant strongly tends to buttress the conclusion that no such improper motive exists
and that the testimony of said witnesses deserve full faith and credit.

The essence of voluntary surrender is spontaneity and the intent of the accused to give himself up and submit
himself unconditionally to the authorities either because he acknowledges his guilt or he wants to save the State
the trouble of having to effect his arrest. Spontaneity and intent to give one's self up are absent where the
accused went into hiding for six months after the incident and had to resort to an alias when he was involved in an
accident being investigated by the police authorities.

Robbery with Homicide is a special complex crime against property. Homicide is incidental to the robbery which is
the main purpose of the criminal. In charging Robbery with Homicide, the onus probandi is to establish: "(a) the
taking of personal property with the use of violence or intimidation against a person; (b) the property belongs to
another; (c) the taking is characterized with animus lucrandi; and (d) on the occasion of the robbery or by reason
thereof, the crime of homicide, which is used in the generic sense, was committed." Although there was no
witness as to the actual robbing of the victim, there is testimony that the victim had more or less P2,000.00; and
wore gold earrings valued at P750.00. These were never recovered.

While there is indeed no direct proof that Virginia Talens was robbed at the time she was killed, we may conclude
from four circumstances that the robbery occasioned her killing: (1) Both appellant and victim gambled at the

Page 147
Section 1-C, SY ’06-‘07
San Beda College of Law – Alabang
Constitutional Law 2 Case Digests

wake. (2) The appellant knew that victim was winning. (3) The victim was last seen alive with appellant. (4) The
victim's purse containing her money and earrings were missing from her body when found.

The decision of the regional trial court is affirmed. Costs against appellant. So ordered.

PEOPLE VS. MAGPALAO


[197 SCRA 79; G.R. NO. 92415; 14 MAY 1991]

Facts:

Eleven (11) people rode in a Ford Fiera going to Baguio. Namely they are: Felizardo Galvez, Jimmy Jetwani,
Simeon Calama, Rene Salonga, Eduardo Lopez, Adolfo Quiambao, Aliman Bara-akal, Anwar Hadji Edris,
Gumanak Ompa and defendant-appelants in this case, Omar Magpalao and Rex Magumnang.

After an hour of driving, the car stopped so that one of the passengers could urinate. While the car was stopped
the Bara-akal, Edris, Ompa, Magpalao and Magumnang pointed guns and knives at the other passengers and
divested them of their properties.

On of the robbers then ordered Galvez to drive the car towards the precipice (bangin). When the car was near the
precipice, Galvez then stepped to the brakes. The other passengers jumped out of the car and went to different
directions to escape. Galvez however, was left in side the car and was stabbed by one of the robbers. The
robbers then escaped. Quiambao, who owned the car helped Galvez to get to a hospital. Galvez died in the
hospital. The robbers were then apprehended with the exception of Edris who remain at large. Mangumnang
however escaped while being in detention and Bara-akal died inside the jail. Since Mangumnang was not
arrested, the trial in absentia continued as to him. Ompa, Magpalao, and Magumnang were all held guilty as
principal by direct participation of the crime of Robbery with Homicide.

Issue:

Whether or Not the lower court erred in failing to apply the Constitutional mandate on the presumption of
innocence and proof beyond reasonable doubt when it allowed the trial in absentia to push through on the part of
defendant-appellant Magumnang.

Held:

The Court affirmed the decision of the lower court. The reason is that the lower court has jurisdiction over
Magumnang the moment the latter was in custody. Jurisdiction once acquired is not lost upon the instance of
parties but until the case is terminated. Since all the requisites of trial in absentia are complete, the court has
jurisdiction over Magumnang.

In addition, Magumnang was presumed innocent during his trial in absentia. The prosecution had strong evidence
against him as proof beyond reasonable doubt that he is a principal by direct participation in the crime of Robbery
with Homicide. Thus, the Constitutional mandate was not violated.

PEOPLE VS. ACABAL


[226 SCRA 694 ; G.R. NO. 103604, 23 SEP 1993]

Facts:

The accusatory portion in the information for murder. Facts are as follows:

"That sometime in the evening of the 28th of January, 1980, at Nagbinlod, Municipality of Sta. Catalina, Province
of Negros Oriental, Philippines, and within the jurisdiction of this Honorable Court, the accused, including several
'John Does', conspiring and confederating with one another, with intent to kill, and with treachery and evident
premeditation and being then armed with bolos and 'pinuti', did then and there willfully, unlawfully and feloniously
attack, assault and use personal violence on the person of one Rizalina Apatan Silvano while the latter was about
to leave her house and inflicting upon her injuries, to wit: 'right leg amputated below the knee; left leg hacked
behind the knee; abdomen hacked with viscerae evacerated,' and did then and there set the house on fire while
the aforementioned Rizalina Apatan Silvano was inside said house trying to escape therefrom, and allowing her to
be burned inside said house which was burned to the ground, thereby causing upon said Rizalina Apatan Silvano
her death and burning her beyond recognition.

But on 16 May 1987, a fire gutted the building where Branch 37 was located and the records of these two cases
were burned. The records were subsequently reconstituted upon petition of the prosecuting fiscal. The
testimonies of the witnesses were retaken, however, before it could commence, accused Engracio Valeriano

Page 148
Section 1-C, SY ’06-‘07
San Beda College of Law – Alabang
Constitutional Law 2 Case Digests

jumped bail and the warrant for his arrest issued on 16 November 1987 was returned unserved because he could
not be found. An alias warrant for his arrest was issued on 26 June 1989, but he remains at large up to the
present.

After the completion of the re-taking of the testimonies of the witnesses in Branch 37, Criminal Cases Nos. 4584
and 4585 were re-raffled to Branch 33 of the trial court, then presided over by Judge Pacifico S. Bulado.

The decision of the trial court, per Judge Pacifico S. Bulado, dated 31 October 1991 but promulgated on 20
December 1991, contained no specific dispositive portion. Its rulings are found in the last two paragraphs which
read as follows:

"The elements of murder in this case, Criminal Case No. 4585 for the killing of Rizalina Apatan-Silvano having
been proved by the prosecution beyond doubt, the accused JUANITO RISMUNDO, MACARIO ACABAL and
ABUNDIO NAHID, considering the attendant qualifying aggravating circumstances of nighttime, use of fire by
burning the house of victim Rizalina Apatan-Silvano in order to forcibly drive her out of her house and hack her to
death, the abuse of superior strength, the penalty impossable [sic] here will be in its maximum degree, that is
reclusion perpetua taking into account Article 248 of the Revised Penal Code, the penalty now for murder is
Reclusion Temporal to Reclusion Perpetua, and for all the accused to indemnify the heirs of the victim the sum of
Thirty Thousand (P30,000.00) Pesos since this case occurred [sic] in 1980. For the wounding of the victim Wilson
A. Silvano, this Court believes that simple frustrated homicide only is committed by the accused Engracio
Valeriano only.

But since the person who actually inflicted the injuries of victim Wilson Silvano, accused Engracio Valeriano only
is nowhere to be found, hence, not brought to the bar of justice, he being a fugitive or at large, no penalty could
be imposed on him since he is beyond the jurisdiction of this court to reach. All the other two (2) accused,
JUANITO RISMUNDO and ABUNDIO NAHID are hereby ordered and declared absolved from any criminal
responsibility from frustrated homicide.

The bail bond put up by the three accused, namely: Juanito Rismundo, Macario Acabal and Abundio Nahid are
hereby ordered cancelled and let a warrant of arrest be issued for their immediate confinement."

Issue:

Whether or not the judgment complied with the Rules of Court.

Whether or not the cancellation of the bail bonds of the accused is valid.

Whether or not the accused may be tried in absentia.

Whether or not the accused is guilty of the crime of frustrated murder.

Held:

We find that the decision substantially complies with the Rules of Court on judgments as it did sentence the
accused-appellants to reclusion perpetua. A judgment of conviction shall state (a) the legal qualification of the
offense constituted by the acts committed by the accused, and the aggravating or mitigating circumstances
attending the commission, if there are any; (b) the participation of the accused in the commission of the offense,
whether as principal, accomplice or accessory after the fact; (c) the penalty imposed upon the accused; and (d)
the civil liability or damages caused by the wrongful act to be recovered from the accused by the offended party, if
there is any, unless the enforcement of the civil liability by a separate action has been reserved or waived.

It is obvious that they clearly understood that they were found guilty beyond reasonable doubt of the crime of
murder and were sentenced to suffer the penalty of reclusion perpetua in Criminal Case No. 4585. Were it
otherwise, they would not have declared in open court their intention to appeal immediately after the promulgation
of the decision and would not have subsequently filed their written notice of appeal.

Accused-appellants contend that the trial court did not impose any sentence and so cannot cancel anymore their
bail bonds and direct their arrest and immediate commitment because it already lost jurisdiction over their persons
when they perfected their appeal.

The decision did impose the penalty of reclusion perpetua. Since the order cancelling their bail bonds and
directing their arrest is contained in the decision itself, it is apparent that their abovementioned contention is highly
illogical. At the time the order in question was made, the trial court still had jurisdiction over the persons of the
accused-appellants.

The trial court further erred in holding that no penalty could be imposed on accused Engracio Valeriano in
Criminal Case No. 4584 because he "is nowhere to be found, hence, not brought to the bar of justice, he being a

Page 149
Section 1-C, SY ’06-‘07
San Beda College of Law – Alabang
Constitutional Law 2 Case Digests

fugitive or at large." The court ignored the fact that Engracio jumped bail after he had been arraigned, just before
the retaking of evidence commenced. Paragraph (2), Section 14, Article III of the Constitution permits trial in
absentia after the accused has been arraigned provided he has been duly notified of the trial and his failure to
appear thereat is unjustified. One who jumps bail can never offer a justifiable reason for his non-appearance
during the trial.

Accordingly, after the trial in absentia, the court can render judgment in the case and promulgation may be made
by simply recording the judgment in the criminal docket with a copy thereof served upon his counsel, provided
that the notice requiring him to be present at the promulgation is served through his bondsmen or warden and
counsel.

In conclusion, because of reasonable doubt as to their guilt, the accused-appellants must be acquitted. Every
accused is presumed innocent until the contrary is proved; that presumption is solemnly guaranteed by the Bill of
Rights. The contrary requires proof beyond reasonable doubt, or that degree of proof which produces conviction
in an unprejudiced mind. Short of this, it is not only the right of the accused to be freed; it is even the constitutional
duty of the court to acquit him.

US VS. TAN TENG


[23 PHIL 145; G.R. NO. 7081; 7 SEP 1912]

Facts:

The defendant herein raped Oliva Pacomio, a seven-year-old girl. Tan Teng was gambling near the house of the
victim and it was alleged that he entered her home and threw the victim on the floor and place his private parts
over hers. Several days later, Pacomio was suffering from a disease called gonorrhea. Pacomio told her sister
about what had happened and reported it to the police.

Tan Teng was called to appear in a police line-up and the victim identified him. He was then stripped of his
clothing and was examined by a policeman. He was found to have the same symptoms of gonorrhea. The
policeman took a portion of the substance emitting from the body of the defendant and turned it over to the
Bureau of Science. The results showed that the defendant was suffering from gonorrhea.

The lower court held that the results show that the disease that the victim had acquired came from the defendant
herein. Such disease was transferred by the unlawful act of carnal knowledge by the latter. The defendant alleged
that the said evidence should be inadmissible because it was taken in violation of his right against self-
incrimination.

Issue:

Whether or Not the physical examination conducted was a violation of the defendant’s rights against self-
incrimination.

Held:

The court held that the taking of a substance from his body was not a violation of the said right. He was neither
compelled to make any admissions or to answer any questions. The substance was taken from his body without
his objection and was examined by competent medical authority.

The prohibition of self-incrimination in the Bill of Rights is a prohibition of the use of physical or moral compulsion
to extort communications from him, and not an exclusion of his body as evidence, when it may be material. It
would be the same as if the offender apprehended was a thief and the object stolen by him may be used as
evidence against him.

VILLAFLOR VS. SUMMERS


[41 PHIL 62; G.R. NO. 16444; 8 SEP 1920]

Facts:

Petitioner Villaflor was charged with the crime of adultery. The trial judge ordered the petitioner to subject herself
into physical examination to test whether or not she was pregnant to prove the determine the crime of adultery
being charged to her. Herein petitioner refused to such physical examination interposing the defense that such
examination was a violation of her constitutional rights against self-incrimination.

Issue:

Page 150
Section 1-C, SY ’06-‘07
San Beda College of Law – Alabang
Constitutional Law 2 Case Digests

Whether or Not the physical examination was a violation of the petitioner’s constitutional rights against self-
incrimination.

Held:

No. It is not a violation of her constitutional rights. The rule that the constitutional guaranty, that no person shall be
compelled in any criminal case to be a witness against himself, is limited to a prohibition against compulsory
testimonial self-incrimination. The corollary to the proposition is that, an ocular inspection of the body of the
accused is permissible.

BELTRAN VS. SAMSON


[53 PHIL 570; G.R. NO. 32025; 23 SEPT 1929]

Facts:

Beltran, as a defendant for the crime of Falsification, refused to write a sample of his handwriting as ordered by
the respondent Judge. The petitioner in this case contended that such order would be a violation of his
constitutional right against self-incrimination because such examination would give the prosecution evidence
against him, which the latter should have gotten in the first place. He also argued that such an act will make him
furnish evidence against himself.

Issue:

Whether or not the writing from the fiscal's dictation by the petitioner for the purpose of comparing the latter's
handwriting and determining whether he wrote certain documents supposed to be falsified, constitutes evidence
against himself within the scope and meaning of the constitutional provision under examination.

Held:

The court ordered the respondents and those under their orders desist and abstain absolutely and forever from
compelling the petitioner to take down dictation in his handwriting for the purpose of submitting the latter for
comparison. Writing is something more than moving the body, or the hands, or the fingers; writing is not a purely
mechanical act, because it requires the application of intelligence and attention; and in the case at bar writing
means that the petitioner herein is to furnish a means to determine whether or not he is the falsifier, as the petition
of the respondent fiscal clearly states. Except that it is more serious, we believe the present case is similar to that
of producing documents or chattels in one's possession. We say that, for the purposes of the constitutional
privilege, there is a similarity between one who is compelled to produce a document, and one who is compelled to
furnish a specimen of his handwriting, for in both cases, the witness is required to furnish evidence against
himself. It cannot be contended in the present case that if permission to obtain a specimen of the petitioner's
handwriting is not granted, the crime would go unpunished. Considering the circumstance that the petitioner is a
municipal treasurer, it should not be a difficult matter for the fiscal to obtained genuine specimens of his
handwriting. But even supposing it is impossible to obtain specimen or specimens without resorting to the means
complained herein, that is no reason for trampling upon a personal right guaranteed by the constitution. It might
be true that in some cases criminals may succeed in evading the hand of justice, but such cases are accidental
and do not constitute the raison d' etre of the privilege. This constitutional privilege exists for the protection of
innocent persons.

PASCUAL VS. BME


[28 SCRA 345; G.R. NO. 25018; 26 MAY 1969]

Facts:

Petitioner Arsenio Pascual, Jr. filed an action for prohibition against the Board of Medical Examiners. It was
alleged therein that at the initial hearing of an administrative case for alleged immorality, counsel for complainants
announced that he would present as his first witness the petitioner. Thereupon, petitioner, through counsel, made
of record his objection, relying on the constitutional right to be exempt from being a witness against himself.
Petitioner then alleged that to compel him to take the witness stand, the Board of Examiners was guilty, at the
very least, of grave abuse of discretion for failure to respect the constitutional right against self-incrimination.

The answer of respondent Board, while admitting the facts stressed that it could call petitioner to the witness
stand and interrogate him, the right against self-incrimination being available only when a question calling for an
incriminating answer is asked of a witness. They likewise alleged that the right against self-incrimination cannot
be availed of in an administrative hearing.

Page 151
Section 1-C, SY ’06-‘07
San Beda College of Law – Alabang
Constitutional Law 2 Case Digests

Petitioner was sustained by the lower court in his plea that he could not be compelled to be the first witness of the
complainants, he being the party proceeded against in an administrative charge for malpractice. Hence, this
appeal by respondent Board.

Issue:

Whether or Not compelling petitioner to be the first witness of the complainants violates the Self-Incrimination
Clause.

Held:

The Supreme Court held that in an administrative hearing against a medical practitioner for alleged malpractice,
respondent Board of Medical Examiners cannot, consistently with the self-incrimination clause, compel the person
proceeded against to take the witness stand without his consent. The Court found for the petitioner in accordance
with the well-settled principle that "the accused in a criminal case may refuse, not only to answer incriminatory
questions, but, also, to take the witness stand." If petitioner would be compelled to testify against himself, he
could suffer not the forfeiture of property but the revocation of his license as a medical practitioner. The
constitutional guarantee protects as well the right to silence: "The accused has a perfect right to remain silent and
his silence cannot be used as a presumption of his guilt." It is the right of a defendant "to forego testimony, to
remain silent, unless he chooses to take the witness stand — with undiluted, unfettered exercise of his own free
genuine will."

The reason for this constitutional guarantee, along with other rights granted an accused, stands for a belief that
while crime should not go unpunished and that the truth must be revealed, such desirable objectives should not
be accomplished according to means or methods offensive to the high sense of respect accorded the human
personality. More and more in line with the democratic creed, the deference accorded an individual even those
suspected of the most heinous crimes is given due weight. The constitutional foundation underlying the privilege
is the respect a government ... must accord to the dignity and integrity of its citizens.

PEOPLE VS. BALISACAN


[17 SCRA 1119; G.R. NO. L-26376; 31 AUG 1966]

Facts:

Aurelio Balisacan was charged with homicide in the CFI of Ilocos Norte. Upon being arraigned, he entered into a
plea of guilty. In doing so, he was assisted y counsel. At his counsel de officio, he was allowed to present
evidence and consequently testified that he stabbed the deceased in self-defense. In addition, he stated that he
surrendered himself voluntarily to the police authorities. On the basis of the testimony of the accused, he was
acquitted. Thus, the prosecution appealed.

Issue:

Whether or Not the appeal placed the accused in double jeopardy.

Held:

The Supreme Court held that it is settled that the existence of plea is an essential requisite to double jeopardy.
The accused had first entered a plea of guilty but however testified that he acted in complete self-defense. Said
testimony had the effect of vacating his plea of guilty and the court a quo should have required him to plead a
new charge, or at least direct that a new plea of not guilty be entered for him. This was not done. Therefore, there
has been no standing of plea during the judgment of acquittal, so there can be no double jeopardy with respect to
the appeal herein.

PEOPLE VS. OBSANIA


[23 SCRA 1249; G.R. L-24447; 29 JUN 1968]

Facts:

The accused was charged with Robbery with Rape before the Municipal Court of Balungao, Pangasinan. He
pleaded not guilty. His counsel moved for the dismissal of the charge for failure to allege vivid designs in the info.
Said motion was granted. From this order of dismissal the prosecution appealed.

Issue:

Page 152
Section 1-C, SY ’06-‘07
San Beda College of Law – Alabang
Constitutional Law 2 Case Digests

Whether or Not the present appeal places the accused in Double Jeopardy.

Held:

In order that the accused may invoke double jeopardy, the following requisites must have obtained in the original
prosecution, a) valid complaint, b) competent court, c) the defendant had pleaded to the charge, d) defendant was
acquitted or convicted or the case against him was dismissed or otherwise terminated without his express
consent.

In the case at bar, the converted dismissal was ordered by the Trial Judge upon the defendant's motion to
dismiss. The “doctrine of double jeopardy” as enunciated in P.vs. Salico applies to wit when the case is
dismissed with the express consent of the defendant, the dismissal will not be a bar to another prosecution for the
same offense because his action in having the case is dismissed constitutes a waiver of his constitutional
right/privilege for the reason that he thereby prevents the Court from proceeding to the trial on the merits and
rendering a judgment of conviction against him.

In essence, where a criminal case is dismissed provisionally not only with the express consent of the accused but
even upon the urging of his counsel there can be no double jeopardy under Sect. 9 Rule 113, if the indictment
against him is revived by the fiscal.

PAULIN VS. GIMENEZ


[217 SCRA 386; G.R. NO. 103323; 21 JAN 1993]

Facts:

Respondent and Brgy Capt. Mabuyo, while in a jeep, were smothered with dust when they were overtaken by the
vehicle owned by Petitioner Spouses. Irked by such, Mabuyo followed the vehicle until the latter entered the gate
of an establishment. He inquired the nearby security guard for the identity of the owner of the vehicle. Later that
day, while engaged in his duties, petitioners allegedly pointed their guns at him. Thus, he immediately ordered his
subordinate to call the police and block road to prevent the petitioners’ escape. Upon the arrival of the police,
petitioners put their guns down and were immediately apprehended.

A complaint “grave threats” was filed against the petitioners (Criminal Case No. 5204). It was dismissed by the
court acting on the motion of the petitioners. Mabuyo filed a MOR thus the dismissal was reversed. Thereafter,
petitioners filed for “certiorari, prohibition, damages, with relief of preliminary injunction and the issuance of a
TRO” (CEB-9207). Petition is dismissed for lack of merit and for being a prohibited pleading and ordered to
proceed with the trial of the case. Hence, this instant petition.

Issue:

Whether or Not the dismissal of 5204 was a judgment of acquittal.

Whether or Not the judge ignored petitioner’s right against double jeopardy by dismissing CEB-9207.

Held:

For double jeopardy to attach, the dismissal of the case must be without the express consent of the accused.
Where the dismissal was ordered upon motion or with the express assent of the accused, he has deemed to have
waived his protection against double jeopardy. In the case at bar, the dismissal was granted upon motion of the
petitioners. Double jeopardy thus did not attach.

Furthermore, such dismissal is not considered as an acquittal. The latter is always based on merit that shows that
the defendant is beyond reasonable doubt not guilty. While the former, in the case at bar, terminated the
proceedings because no finding was made as to the guilt or innocence of the petitioners.

The lower court did not violate the rule when it set aside the order of dismissal for the reception of further
evidence by the prosecution because it merely corrected its error when it prematurely terminated and dismissed
the case without giving the prosecution the right to complete the presentation of its evidence. The rule on
summary procedure was correctly applied.

PEOPLE VS. COURT OF SILAY


[74 SCRA 248; G.R. NO. L-43790; 9 DEC 1976]

Facts:

Page 153
Section 1-C, SY ’06-‘07
San Beda College of Law – Alabang
Constitutional Law 2 Case Digests

That sometime on January 4,1974, accused Pacifico Sensio, Romeo Millan and Wilfredo Jochico who were then
scalers at the Hawaiian-Philippine Company, weighed cane cars No.1743,1686 and 1022 loaded with sugar
canes which were placed in tarjetas (weight report cards), Apparently, it was proven and shown that there was
padding of the weight of the sugar canes and that the information on the tarjetas were to be false making it
appear to be heavier than its actual weight. The three accused then were charged with “Falsification by private
individuals and use of falsified document”. After the prosecution had presented, the respondent moved to dismiss
the charge against them on the ground that the evidences presented were not sufficient to establish their guilt
beyond reasonable doubt. Acting on the motion, respondent court issued its order dismissing the case on the
ground that the acts committed by the accused do not constituted the crime of falsification as strictly enumerated
in the revised penal code defining the crime of falsification which was charged earlier and that their case be
dismissed. People asserts that the plea of double jeopardy is not tenable even if the case at bar was dismissed
because according to them, it was done with the consent of the accused therefore waiving there defense of
double jeopardy. The accused on the other hand, reiterated the fact that the dismissal was due to lack of merits
of the prosecution which would have the same effect as an acquittal which will bar the prosecution from
prosecuting the accused for it will be unjust and unconstitutional for the accused due to double jeopardy rule thus
the appeal of the plaintiff.

Issue:

Whether or Not the grant of petition by the court would place the accused Sensio, Millan and Jochico in double
jeopardy

Held:

Yes the revival of the case will put the accused in double jeopardy for the very reason that the case has been
dismissed earlier due to lack of merits. It is true that the criminal case of falsification was dismissed on a motion
of the accused however this was a motion filed after the prosecution had rested its case, calling for the evidence
beyond reasonable ground which the prosecution had not been able to do which would be tantamount to acquittal
therefore will bar the prosecution of another case. As it was stated on the requirements of a valid defense of
double jeopardy it says: That there should be a valid complaint, second would be that such complaint be filed
before a competent court and to which the accused has pleaded and that defendant was previously acquitted,
convicted or dismissed or otherwise terminated without express consent of the accused in which were all present
in the case at bar. There was indeed a valid, legitimate complaint and concern against the accused Sensio, Millan
and Jochico which was filed at a competent court with jurisdiction on the said case. It was also mentioned that
the accused pleaded not guilty and during the time of trial, it was proven that the case used against the accused
were not sufficient to prove them guilty beyond reasonable doubt therefore dismissing the case which translates
to acquittal. It explained further that there are two instances when we can conclude that there is jeopardy when
first is that the ground for the dismissal of the case was due to insufficiency of evidence and second, when the
proceedings have been reasonably prolonged as to violate the right of the accused to a speedy trial. In the 2
requisites given, it was the first on that is very much applicable to our case at bar where there was dismissal of
the case due to insufficiency of evidence which will bar the approval of the petition in the case at bar for it will
constitute double jeopardy on the part of the accused which the law despises.

PEOPLE VS. RELOVA


[149 SCRA 292; G.R. NO.L-45129; 6 MAR 1987]

FACTS: In this petition for certiorari and mandamus, People of the Philippines seeks to set aside the orders of
Respondent Judge Hon. Relova quashing an information for theft filed against Mr. Opulencia on the ground of
double jeopardy and denying the petitioner’s motion for reconsideration.. On Feb.1 1975, Batangas police
together with personnel of Batangas Electric Light System, equipped with a search warrant issued by a city judge
of Batangas to search and examine the premises of the Opulencia Carpena Ice Plant owned by one Manuel
Opulencia. They discovered electric wiring devices have been installed without authority from the city
government and architecturally concealed inside the walls of the building. Said devices are designed purposely to
lower or decrease the readings of electric current consumption in the plant’s electric meter. The case was
dismissed on the ground of prescription for the complaint was filed nine months prior to discovery when it should
be 2months prior to discovery that the act being a light felony and prescribed the right to file in court. On Nov 24,
1975, another case was filed against Mr. Opulencia by the Assistant City Fiscal of Batangas for a violation of a
Batangas Ordinance regarding unauthorized electrical installations with resulting damage and prejudice to City of
Batangas in the amount of P41,062.16. Before arraignment, Opulencia filed a motion to quash on the ground of
double jeopardy. The Assistant fiscal’s claim is that it is not double jeopardy because the first offense charged
against the accused was unauthorized installation of electrical devices without the approval and necessary
authority from the City Government which was punishable by an ordinance, where in the case was dismissed, as
opposed to the second offense which is theft of electricity which is punishable by the Revised Penal Code making
it a different crime charged against the 1st complaint against Mr.Opulencia.

Issue:

Page 154
Section 1-C, SY ’06-‘07
San Beda College of Law – Alabang
Constitutional Law 2 Case Digests

Whether or Not the accused Mr. Opulencia can invoke double jeopardy as defense to the second offense charged
against him by the assistant fiscal of Batangas on the ground of theft of electricity punishable by a statute against
the Revised Penal Code.

Held:

Yes, Mr. Opulencia can invoke double jeopardy as defense for the second offense because as tediously
explained in the case of Yap vs Lutero, the bill of rights give two instances or kinds of double jeopardy. The first
would be that “No person shall be twice put in jeopardy of punishment for the same offense and the second
sentence states that “If an act is punishable by a law or an ordinance, the conviction or acquittal shall bar to
another prosecution for the same act”. In the case at bar, it was very evident that the charges filed against Mr.
Opulencia will fall on the 2nd kind or definition of double jeopardy wherein it contemplates double jeopardy of
punishment for the same act. It further explains that even if the offenses charged are not the same, owing that
the first charge constitutes a violation of an ordinance and the second charge was a violation against the revised
penal code, the fact that the two charges sprung from one and the same act of conviction or acquittal under either
the law or the ordinance shall bar a prosecution under the other thus making it against the logic of double
jeopardy. The fact that Mr. Opulencia was acquitted on the first offense should bar the 2 nd complaint against him
coming from the same identity as that of the 1st offense charged against Mr.Opulencia.

ESMENA VS. POGOY


[102 SCRA 861; G.R. NO. L-54110; 20 FEB 1981]

Facts:

Petitioners Esmeña and Alba were charged with grave coercion in the Court of Cebu City for allegedly forcing Fr.
Thomas Tibudan to withdraw a sum of money worth P5000 from the bank to be given to them because the priest
lost in a game of chance. During arraignment, petitioners pleaded “Not Guilty”. No trial came in after the
arraignment due to the priest’s request to move it on another date. Sometime later Judge Pogoy issued an order
setting the trial Aug.16,1979 but the fiscal informed the court that it received a telegram stating that the
complainant was sick. The accused invoked their right to speedy trial. Respondent judge dismissed the case
because the trial was already dragging the accused and that the priest’s telegram did not have a medical
certificate attached to it in order for the court to recognize the complainant’s reason to be valid in order to
reschedule again another hearing. After 27 days the fiscal filed a motion to revive the case and attached the
medical certificate of the priest proving the fact that the priest was indeed sick of influenza. On Oct.24,1979,
accused Esmeña and Alba filed a motion to dismiss the case on the ground of double jeopardy.

Issue:

Whether or Not the revival of grave coercion case, which was dismissed earlier due to complainant’s failure to
appear at the trial, would place the accused in double jeopardy

Held:

Yes, revival of the case will put the accused in double jeopardy for the very reason that the case has been
dismissed already without the consent of the accused which would have an effect of an acquittal on the case filed.
The dismissal was due to complainant’s incapability to present its evidence due to non appearance of the
witnesses and complainant himself which would bar further prosecution of the defendant for the same offense.
For double jeopardy to exist these three requisites should be present, that one, there is a valid complaint or
information filed second, that it is done before a court of competent jurisdiction and third, that the accused has
been arraigned and has pleaded to the complaint or information. In the case at bar, all three conditions were
present, as the case filed was grave coercion, filed in a court of competent jurisdiction as to where the coercion
took place and last the accused were arraigned and has pleaded to the complaint or the information. When these
three conditions are present then the acquittal, conviction of the accused, and the dismissal or termination of the
case without his express consent constitutes res judicata and is a bar to another prosecution for the offense
charged. In the case, it was evidently shown that the accused invoked their right to a speedy trial and asked for
the trial of the case and not its termination which would mean that respondents had no expressed consent to the
dismissal of the case which would make the case filed res judicata and has been dismissed by the competent
court in order to protect the respondents as well for their right to speedy trial which will be equivalent to acquittal
of the respondents which would be a bar to further prosecution.

PEOPLE VS. DE LA TORRE


[380 SCRA 586; G.R. NOS. 137953-58; 11 MAR 2002]

Facts:

Page 155
Section 1-C, SY ’06-‘07
San Beda College of Law – Alabang
Constitutional Law 2 Case Digests

Wilfredo dela Torre, appellee, has three children from a common-law relationship, the eldest of which is Mary
Rose. When Mary Rose was 7 yearsold, her mother left them together with her youngest brother so she and her
other brother were left to the care of her father.

Mary Rose was the brightest in her class despite their poverty. However, in January 1997, a sudden change in
Mary Rose’s behavior behavior was noticed. She was twelve years old at that time. She appeared sleepy,
snobbish and she also urinated on her panty. When confronted by her head teacher, Mary Rose admitted that
she was abused repeatedly by her father. Her father, however, denied vehemently the charges being imputed to
him by her daughter.

The RTC convicted appellee of two counts of acts of lasciviousness and four counts of murder. However, the
RTC refused to impose the supreme penalty of death on appellee. It maintained that there were circumstances
that mitigated the gravity of the offenses such as the absence of any actual physical violence or intimidation on
the commission of the acts, that after the mother of Mary Rose left the conjugal home, for more than five years,
Wilfredo, Mary Rose and her brother were living together as a family and Mary Rose was never molested by her
father.

The prosecution seeks to modify the RTC Decision by imposing the supreme penalty of death of the accused. It
argues that it has proven that the victim is the daughter of the accused, and that she was below eighteen years
old when the rapes took place. As a consequence, the trial court should have been imposed the penalty of death
pursuant to Section 11 of R.A. 7659. .

Issue:

Whether or Not the Court erred in penalizing the appellee with reclusion perpetua in each of the four indictments
of rape, instead of imposing the supreme penalty of death as mandated by R.A. 7659.

Held:

Under Section 1, Rule 122 of the 2000 Rules of Criminal Procedure, any party may appeal from a judgment or
final order unless the accused will be put in double jeopardy. In People vs. Leones, it declared that:

“while it is true that this Court is the Court of last resort, there are allegations of error committed
by a lower court which we ought not to look into to uphold the right of the accused. Such is the
case in an appeal by the prosecution seeking to increase the penalty imposed upon the accused
for this runs afoul of the right of the accused against double jeopardy…When the accused after
conviction by the trial court did not appeal his decision, an appeal by the government seeking to
increase the penalty imposed by the trial court places the accused in double jeopardy and should
therefore be dismissed.”

The ban on double jeopardy primarily prevents the State from using its criminal processes as an instrument of
harassment to wear out the accused by a multitude of cases with accumulated trials. It also serves as a deterrent
from successively retrying the defendant in the hope of securing a conviction. And finally, it prevents the State,
following conviction, from retrying the defendant again in the hope of securing a greater penalty.

Being violative of the right against double jeopardy, the appeal of the prosecution cannot prosper.

Page 156
Section 1-C, SY ’06-‘07
San Beda College of Law – Alabang
Constitutional Law 2 Case Digests

CITIZENSHIP

Page 157
Section 1-C, SY ’06-‘07
San Beda College of Law – Alabang
Constitutional Law 2 Case Digests

CITIZENSHIP

Art. 4

Sec. 1. The following are citizens of the Philippines:


(1) Those who are citizens of the Philippines at the time of the adoption of this Constitution;
(2) Those whose fathers or mothers are citizens of the Philippines;
(3) Those born before January 17, 1973, of Filipino mothers, who elect Philippine citizenship upon
reaching the age of majority; and
(4) Those who are naturalized in accordance with law.

Sec. 2. Natural-born citizens are those who are citizens of the Philippines from birth without having to perform
any act to acquire or perfect their Philippine citizenship. Those who elect Philippine citizenship in accordance with
paragraph (3), Section 1 hereof shall be deemed natural-born citizens.

Sec. 3. Philippine citizenship may be lost or reacquired in the manner provided by law.

Sec. 4. Citizens of the Philippines who marry aliens shall retain their citizenship, unless by their act or omission
they are deemed, under the law, to have renounced it.

Sec. 5. Dual allegiance of citizens is inimical to the national interest and shall be dealt with by law.

FRIVALDO VS. COMELEC


[174 SCRA 245; G.R. NO. 87193; 23 JUN 1989]

Facts:

Petitioner Juan G. Frivaldo was proclaimed governor-elect of the province of Sorsogon on January 22, 1988, and
assumed office in due time. On October 27, 1988, the League of Municipalities, Sorsogon Chapter, represented
by its President, Estuye, who was also suing in his personal capacity, filed with the COMELEC a petition for the
annulment of Frivaldo; election and proclamation on the ground that he was not a Filipino citizen, having been
naturalized in the United States on January 20, 1983. In his answer dated May 22, 1988, Frivaldo admitted that
he was naturalized in the United States as alleged but pleaded the special and affirmative defenses that he had
sought American citizenship only to protect himself against President Marcos. His naturalization, he said, was
"merely forced upon himself as a means of survival against the unrelenting persecution by the Martial Law
Dictator's agents abroad." He added that he had returned to the Philippines after the EDSA revolution to help in
the restoration of democracy. In their Comment, the private respondents reiterated their assertion that Frivaldo
was a naturalized American citizen and had not reacquired Philippine citizenship on the day of the election on
January 18, 1988. He was therefore not qualified to run for and be elected governor. They also argued that their
petition in the Commission on Elections was not really for quo warranto under Section 253 of the Omnibus
Election Code. The ultimate purpose was to prevent Frivaldo from continuing as governor, his candidacy and
election being null and void ab initio because of his alienage. Speaking for the public respondent, the Solicitor
General supported the contention that Frivaldo was not a citizen of the Philippines and had not repatriated himself
after his naturalization as an American citizen. As an alien, he was disqualified from public office in the
Philippines. His election did not cure this defect because the electorate of Sorsogon could not amend the
Constitution, the Local Government Code, and the Omnibus Election Code. He also joined in the private
respondent's argument that Section 253 of the Omnibus Election Code was not applicable because what the
League and Estuye were seeking was not only the annulment of the proclamation and election of Frivaldo. He
agreed that they were also asking for the termination of Frivaldo's incumbency as governor of Sorsogon on the
ground that he was not a Filipino.

Issue:

Whether or Not petitioner Juan G. Frivaldo was a citizen of the Philippines at the time of his election on January
18, 1988, as provincial governor of Sorsogon.

Held:

The reason for this inquiry is the provision in Article XI, Section 9, of the Constitution that all public officials and
employees owe the State and the Constitution "allegiance at all times" and the specific requirement in Section 42
of the Local Government Code that a candidate for local elective office must be inter alia a citizen of the
Philippines and a qualified voter of the constituency where he is running. Section 117 of the Omnibus Election
Code provides that a qualified voter must be, among other qualifications, a citizen of the Philippines, this being an
indispensable requirement for suffrage under Article V, Section 1, of the Constitution.

Page 158
Section 1-C, SY ’06-‘07
San Beda College of Law – Alabang
Constitutional Law 2 Case Digests

In the certificate of candidacy he filed on November 19, 1987, Frivaldo described himself as a "natural-born"
citizen of the Philippines, omitting mention of any subsequent loss of such status. The evidence shows, however,
that he was naturalized as a citizen of the United States in 1983 per the following certification from the United
States District Court, Northern District of California, as duly authenticated by Vice Consul Amado P. Cortez of the
Philippine Consulate General in San Francisco, California, U.S.A.

The Court sees no reason not to believe that the petitioner was one of the enemies of the Marcos dictatorship.
Even so, it cannot agree that as a consequence thereof he was coerced into embracing American citizenship. His
feeble suggestion that his naturalization was not the result of his own free and voluntary choice is totally
unacceptable and must be rejected outright.

This Court will not permit the anomaly of a person sitting as provincial governor in this country while owing
exclusive allegiance to another country. The fact that he was elected by the people of Sorsogon does not excuse
this patent violation of the salutary rule limiting public office and employment only to the citizens of this country.
The qualifications prescribed for elective office cannot be erased by the electorate alone. The will of the people as
expressed through the ballot cannot cure the vice of ineligibility, especially if they mistakenly believed, as in this
case, that the candidate was qualified. Obviously, this rule requires strict application when the deficiency is lack of
citizenship. If a person seeks to serve in the Republic of the Philippines, he must owe his total loyalty to this
country only, abjuring and renouncing all fealty and fidelity to any other state.

It is true as the petitioner points out that the status of the natural-born citizen is favored by the Constitution and
our laws, which is all the more reason why it should be treasured like a pearl of great price. But once it is
surrendered and renounced, the gift is gone and cannot be lightly restored. This country of ours, for all its
difficulties and limitations, is like a jealous and possessive mother. Once rejected, it is not quick to welcome back
with eager arms its prodigal if repentant children. The returning renegade must show, by an express and
unequivocal act, the renewal of his loyalty and love.

Petition Dismissed. Petitioner JUAN G. FRIVALDO is hereby declared not a citizen of the Philippines and
therefore disqualified from serving as Governor of the Province of Sorsogon. Accordingly, he is ordered to vacate
his office and surrender the same to the duly elected Vice-Governor of the said province once this decision
becomes final and executory.

MERCADO VS. M ANZANO


[307 SCRA 630; G.R. NO. 135083; 26 MAY 1999]

Facts:

Petitioner Ernesto Mercado and Private respondent Eduardo Manzano are candidates for the position of Vice-
Mayor of Makati City in the May, 1998 elections. Private respondent was the winner of the said election but the
proclamation was suspended due to the petition of Ernesto Mamaril regarding the citizenship of private
respondent. Mamaril alleged that the private respondent is not a citizen of the Philippines but of the United States.
COMELEC granted the petition and disqualified the private respondent for being a dual citizen, pursuant to the
Local Government code that provides that persons who possess dual citizenship are disqualified from running any
public position. Private respondent filed a motion for reconsideration which remained pending until after election.
Petitioner sought to intervene in the case for disqualification. COMELEC reversed the decision and declared
private respondent qualified to run for the position. Pursuant to the ruling of the COMELEC, the board of
canvassers proclaimed private respondent as vice mayor. This petition sought the reversal of the resolution of
the COMELEC and to declare the private respondent disqualified to hold the office of the vice mayor of Makati.

Issue:

Whether or Not private respondent is qualified to hold office as Vice-Mayor.

Held:

Dual citizenship is different from dual allegiance. The former arises when, as a result of the concurrent
application of the different laws of two or more states, a person is simultaneously considered a national by the
said states. For instance, such a situation may arise when a person whose parents are citizens of a state which
adheres to the principle of jus sanguinis is born in a state which follows the doctrine of jus soli. Private respondent
is considered as a dual citizen because he is born of Filipino parents but was born in San Francisco, USA. Such
a person, ipso facto and without any voluntary act on his part, is concurrently considered a citizen of both states.
Considering the citizenship clause (Art. IV) of our Constitution, it is possible for the following classes of citizens of
the Philippines to posses dual citizenship: (1) Those born of Filipino fathers and/or mothers in foreign countries
which follow the principle of jus soli; (2) Those born in the Philippines of Filipino mothers and alien fathers if by the
laws of their fathers’ country such children are citizens of that country; (3) Those who marry aliens if by the laws
of the latter’s country the former are considered citizens, unless by their act or omission they are deemed to have

Page 159
Section 1-C, SY ’06-‘07
San Beda College of Law – Alabang
Constitutional Law 2 Case Digests

renounced Philippine citizenship. Dual allegiance, on the other hand, refers to the situation in which a person
simultaneously owes, by some positive act, loyalty to two or more states. While dual citizenship is involuntary,
dual allegiance is the result of an individual’s volition.

By filing a certificate of candidacy when he ran for his present post, private respondent elected Philippine
citizenship and in effect renounced his American citizenship. The filing of such certificate of candidacy sufficed to
renounce his American citizenship, effectively removing any disqualification he might have as a dual citizen.

By declaring in his certificate of candidacy that he is a Filipino citizen; that he is not a permanent resident or
immigrant of another country; that he will defend and support the Constitution of the Philippines and bear true
faith and allegiance thereto and that he does so without mental reservation, private respondent has, as far as the
laws of this country are concerned, effectively repudiated his American citizenship and anything which he may
have said before as a dual citizen. On the other hand, private respondent’s oath of allegiance to the Philippine,
when considered with the fact that he has spent his youth and adulthood, received his education, practiced his
profession as an artist, and taken part in past elections in this country, leaves no doubt of his election of Philippine
citizenship.

TECSON VS. COMELEC


[424 SCRA 277; G.R. No. 161434; 3 Mar 2004]

Facts:

Victorino X. Fornier, petitioner initiated a petition before the COMELEC to disqualify FPJ and to deny due course
or to cancel his certificate of candidacy upon the thesis that FPJ made a material misrepresentation in his
certificate of candidacy by claiming to be a natural-born Filipino citizen when in truth, according to Fornier, his
parents were foreigners; his mother, Bessie Kelley Poe, was an American, and his father, Allan Poe, was a
Spanish national, being the son of Lorenzo Pou, a Spanish subject. Granting, petitioner asseverated, that Allan F.
Poe was a Filipino citizen, he could not have transmitted his Filipino citizenship to FPJ, the latter being an
illegitimate child of an alien mother. Petitioner based the allegation of the illegitimate birth of respondent on two
assertions - first, Allan F. Poe contracted a prior marriage to a certain Paulita Gomez before his marriage to
Bessie Kelley and, second, even if no such prior marriage had existed, Allan F. Poe, married Bessie Kelly only a
year after the birth of respondent.

Issue:

Whether or Not FPJ is a natural born Filipino citizen.

Held:

It is necessary to take on the matter of whether or not respondent FPJ is a natural-born citizen, which, in turn,
depended on whether or not the father of respondent, Allan F. Poe, would have himself been a Filipino citizen
and, in the affirmative, whether or not the alleged illegitimacy of respondent prevents him from taking after the
Filipino citizenship of his putative father. Any conclusion on the Filipino citizenship of Lorenzo Pou could only be
drawn from the presumption that having died in 1954 at 84 years old, Lorenzo would have been born sometime in
the year 1870, when the Philippines was under Spanish rule, and that San Carlos, Pangasinan, his place of
residence upon his death in 1954, in the absence of any other evidence, could have well been his place of
residence before death, such that Lorenzo Pou would have benefited from the "en masse Filipinization" that the
Philippine Bill had effected in 1902. That citizenship (of Lorenzo Pou), if acquired, would thereby extend to his
son, Allan F. Poe, father of respondent FPJ. The 1935 Constitution, during which regime respondent FPJ has
seen first light, confers citizenship to all persons whose fathers are Filipino citizens regardless of whether such
children are legitimate or illegitimate.

But while the totality of the evidence may not establish conclusively that respondent FPJ is a natural-born citizen
of the Philippines, the evidence on hand still would preponderate in his favor enough to hold that he cannot be
held guilty of having made a material misrepresentation in his certificate of candidacy in violation of Section 78, in
relation to Section 74, of the Omnibus Election Code.

BENGZON VS. HRET


[357 SCRA 545; G. R. No. 142840; 7 May 2001]

Facts:

Page 160
Section 1-C, SY ’06-‘07
San Beda College of Law – Alabang
Constitutional Law 2 Case Digests

Respondent Teodoro Cruz was a natural-born citizen of the Philippines. He was born in San Clemente, Tarlac, on
April 27, 1960, of Filipino parents. The fundamental law then applicable was the 1935 Constitution. On November
5, 1985, however, respondent Cruz enlisted in the United States Marine Corps and without the consent of the
Republic of the Philippines, took an oath of allegiance to the United States. As a Consequence, he lost his Filipino
citizenship for under Commonwealth Act No. 63, section 1(4), a Filipino citizen may lose his citizenship by, among
other, "rendering service to or accepting commission in the armed forces of a foreign country.” He was naturalized
in US in 1990. On March 17, 1994, respondent Cruz reacquired his Philippine citizenship through repatriation
under Republic Act No. 2630. He ran for and was elected as the Representative of the Second District of
Pangasinan in the May 11, 1998 elections. He won over petitioner Antonio Bengson III, who was then running for
reelection.

Issue:

Whether or Not respondent Cruz is a natural born citizen of the Philippines in view of the constitutional
requirement that "no person shall be a Member of the House of Representative unless he is a natural-born
citizen.”

Held:

Respondent is a natural born citizen of the Philippines. As distinguished from the lengthy process of
naturalization, repatriation simply consists of the taking of an oath of allegiance to the Republic of the Philippine
and registering said oath in the Local Civil Registry of the place where the person concerned resides or last
resided. This means that a naturalized Filipino who lost his citizenship will be restored to his prior status as a
naturalized Filipino citizen. On the other hand, if he was originally a natural-born citizen before he lost his
Philippine citizenship, he will be restored to his former status as a natural-born Filipino.

Page 161
Section 1-C, SY ’06-‘07

You might also like